Anda di halaman 1dari 616

2012

ISBN: 978-960-337-110-6
Copyright 2012
, , ( 3 . 2121/1993).


, ,


.
,
,
.
, 1956 ,
. , .


. 1981 2000 . -.
,
.
,

.
,
,

, .
. 61228/5031, 9 1966, ,
.
,
: )
30 (1967 1979). ) ( ) 54 (1979 2001).
,
,

, .
,
,
,
.
. . 2111. 1/2/99/28-05-1999 ( 1168/14-6-99) , ...
. 3611.2/05/05/16-12-2005 ( 1942 /30-12-2005
169 /13-02-2006),
....

27
.
,

.
,
.
,
,
.


, o. , .
, . .
, .. (..).
, .., / . . & .
. . , . .
.
E ,
. .

. (1955-1959) , . (1957-1970) , . (1955-1983)
, . (1955-1984) . . , . (1970-2003) . ,
. (1958) /, . (1959-1966) /, / . ., . . (19671969) , / . ., . (1967-1969) , . (1969-1971) ., / . ., . (1971-1972) .., / . ., .
(1972-1974) .., / . ., . (1974-1977) .., / . ., . (1977) .., / . ., .
(1977-1979) .., / . ., . (1979-1981) . ., /
. ., . (1981-1982) .., / . ., . (1982-1984)
.., / . ., . . (-2008) . (1984-1986)
.., / . ., . (1986-1988) .., / . ., . (1988-1989) .., / . ., . (1989) .., /
. ., . (1989-1992) .., / . ., . (1992-1993)
.., / . ., . (1993-1994) .., / . ., . (1994-1995) .., . (1995-1996) .., / . ., .
(1996-1998) .., / . ., . (1998-2000) .., / .
., . (2000-2001) .., / . ., . (2001) .., /
. ., . (2001-2003) .., / . ., . (20032004) .., / . ., . (2003-2004) .., / . ., . (2004-2005) .., / . ., . (2005-2008) ..,
/ . ., . (2008-2009) .., / . ., . (20092011) .., / . ., . (2011-2012) .., / . ..

.
/ -

.
- ..

.
-

2012


o
(...).
, , ... .
,
. , ,
,
a. , , , ,
.
: , . ,
(, , ) ,
. , , ,

.
( ), (/ ),
.

...,
, . , , , ,
.
, :
' : ,
' : ,
' : ,
' : .
', ' ' . . . ,
/ '
. . . , / . . . . ,
.
... . ,
, -

, ,
.
. , 7 8

.


, .
( ) ,
, ,
.
, 2012
. . ,
. . ,
. .


( )

1:
2:


3:
4:

lx im
x0

,
,
. , , ,
.

1.1  . .
.
1.2 
.
1.3 .

1.4 .

1.5 .
1.6 .
1.7 .

lx im
x0

12

1.1 . . .
, ,
, , , , . ,
:
1 : 8000 , 5000 6000
.

2 : 6000 , 4000 2000


.
3 : 3000 , 2000 3000
.
4 : 2000 , 1000 1000
.
, , :

8000

5000

6000

6000

4000

2000

3000

2000

3000

2000

1000

1000

, :

1
8 1000 = 8000 , 5 1000 = 5000
6 1000 = 6000 , 4
1 1000 = 1000 , ... 4 (
), 1 ...

,

13

8
6

5
4
2
1

6
2
3

43 43.
:
, , .
,
' . , , .., . ,
i- j- , ij
j

a
a
a1 j ! a1
!
11 12

a21 a22 ! a2 j ! a2
! ! ! ! ! !
m i 

ai1 ai2 ! aij ! ai

! ! ! ! ! !
a1 a2 ! aj ! a

.
=[ij], i=1,2,..., j=1,2,...,
=[ij] , , =[ij].
,
=4 =3 , 43
5
6
8
6
4
2

A=
.
3
2
3

1
1
2
22 4, 41 2 ...

.
, , .
. ,
:
, , = , (
) .

14

.
,
x 2 1 / w
9 5
A= 3
B = 8 6
z

y
x2=9, 1/w=5, y3=8, z=6 x=3, w=1/5, y=2 z=6.
, , :
) ,
. v ,
0 0
0 0

. ,
0 0
0
0

2 2 3 2 .
0 0
) ,
. A.
1 2 3
1 2 3
B = A =
, A =

2 5 4
2 5 4
, . .
) 1 , [4 2 5 4 1].
3

) 1, 1 .
2
) 1 1, .. [10].
)
(. ). . 1 5 10
0 0

, , A =
B = 2 3 9 2 3 0
0

3 7 8
, 2 3 .
,
11

21
...

12
22
...
2

... 1
... 2
... ...

...

11, 22, 33,...,


.
) ,
,

15

a11 0
0 a
22
A=
... ...

0
0

... 0
... 0

... ...

0 a

4
3 0 0
0
x + 1 0


, 0 1 0
0
0
5

0 0 5
2, 3 4 .

0
2

0
0
0
0

0
0
7
0

0
0
0

11 = 22=
= 33=...= =1,
1 0 0 ... 0
0 1 0 ... 0

0 0 1 ... 0

... ... ... ... 0


0 0 0 ... 1
o
.
( ).
) ,
( ) ( ). ..:

0
5 6
0
6
5 ,

0
0 2

9
0

1
4
0
0 5
2

0
1
5

0
0 3

1 0 0
x 2 2 0

,
5 y 3

1
0

0
0
x 0
1 x3
0

0 5 2

) ,
. . ,
.

11

21
...
A
i1

...

12

... 1 j

22

... 2 j

...

...

...

i2

...

ij

...
2

...
...

...
j

... 1

... 2
... ...

... i

... ...
...

a11

a12
...
AT =
a1 j

...
a1

a21

... ai 1

a22

... ai 2

...

...

...

a2 j

...

aij

...
a2

... ...
... ai

... a 1

... a 2
... ...

... aj

... ...
...

16

= [ji] .
2
1
1 5 6

T
, A =
4 .
A = 5
2 4 2
6 2
() = .
) A = [ij]
ij = ji i = 1,2,..., j = 1,2,..., . ,

1
2
x
5

x2
2
y

y
3

3. , = .
1.1.1.

---. ( ..) -, -, - 80, 20 10 .. .


,
. .
.


, :

80

80

20

20

10

10

( ) :
0 80 100 110
80 0 20 30
.
A=
100 20 0
10

0
110 30 10
4, . ij = ji i = 1, 2,
3, 4 j = 1, 2, 3, 4 = .

17

1.1.2.

= [ij]23, ij = i j, i = 1, 2, j = 1, 2, 3
) .
) AT. ;
) x
x 2 1 x 2 2
=
.
3
x 1 0 x 7

)
a11 = 1 1 = 0 , a12 = 1 2 = 1, a13 = 1 3 = 2 ,
a21 = 2 1 = 1, a22 = 2 2 = 0 , a13 = 2 3 = 1

0 1 2
A=
.
1 0 1

0 1
A = 1 0 .
2 1
, .
)
x 2 1 x 2 2 0 1 2

3
x 1 0 x 7 1 0 1

T

x2=0,

x22=2,

x1=1,

x37=1.

x = 2 .
.

1.1.1.
 () ()()
:

1
x
3
5y

3x
2
z2

[0 0 0 0 0]

5y

z2
3

1
0

3

y x 2 + 1 8

1
1
0
0
0
2

[10]

1 0 0
4 2 0


5 6 3

1 0 0
0 2 0


0 0 1

1 0 0
0 2 0

1 0 1

18

1.1.2.
 4 4 = [ij], ij = 2i 3j i =
1, 2, 3, 4 j = 1, 2, 3, 4.
, .
1.1.3.
 1.1 1,
2, .
..
1 3 .
1.1.
1.1.4.  1, 2, 3, 4, 5 12 1, 2,...,12, 1.1. = [ij]44 ,
aij
i j (.. a13 2,
1 3).

1
3
1

5
2

. 1.1.
7
3

6
5
2

1
9

1
11

1.1.5.
 x, y,

12

4
10

. 1.1.

0 y(4 x + 1) (3 y 1)(7 x 3)
1
.
2 x 1 8 x 3 =
0
4x2

1
1.1.6. x 2
x 5 x + 6

x 2 4
.
2

1.1.7. x, y,
1

A = 1 x 2

x2 1
x
y2

( x + 1)2 3
x 1

 . =3.
1.1.8. 3
a
A [ aij ] x'

y'

y
z .

z'
x

=[ij]
1
ij = (ij + ji ), i = 1, 2, 3 j=1, 2, 3 ,
2

19

 .
;

1.2 .
,

,
, :
5
5
5
6
10
8

2
3
4
2
9
6
B=
A=
.

7
8
1
3
2
3

5
2
1
1
1
2

, , , ( ) ,
,
8 + 10

6 + 9

3 + 7

2 + 1

5+5
4+2
2+8
1+5

6 + 5 18

2 + 3 15
=
3 + 1 10

1 + 2 3

10
6
10
6

11

5
.
4

. :
, =[ij] B=[ij] ,
, . +, +=[ij+ij] .
, .
, ,
, ,
.
0
1
8 10 5 5 6 5 2

2
1
6 9 4 2 2 3 3

=
.
2
6
3 7 2 8 3 1 4

4
1
2 1 1 5 1 2 1
(
). :

20

= [ij] B = [ij] ,
, .
, = [ij ij] .
,

=+().

,
.
,
1 4
2 4 6
3 5 7
, B=
= 2 5 ,
A=

4 6 8
8 6 4
3 6

3 5 7 2 4 6 5 9 13
A+ B =
+
=
,
8 6 4 4 6 8 12 12 12
3 5 7 2 4 6 1 1 1
A B = A + ( B) =
+
=
,
8 6 4 4 6 8 4 0 4
3 5 7 1 2 3 4 7 10
A + T =
+
=

8 6 4 4 5 6 12 11 10
1 4 3 8 2 4
A = + ( A ) = 2 5 5 6 = 3 1 ,

3 6 7 4 4
2
T

, , ,
.

. , , , ( ), :
1. +=+

( )

2. +=+=

( )

3. +(+)=(+)+

( )

4. +()=()+=

( )

+(+) , (+)+,
++. , , , , ,
(+ (+))+,

21

((+)+)+, (A+B)+(+,) +((+)+), +(+(+)),


, , , +++.
k (A1, A2,..., Ak, k3 ),
1+2+...+k.

+= =.
+=, +=, =.
=, +=+ , +=.
, , :
5. (+)=+

( )

6. () =

( )

1.2.1.

, (15 ) .

13

11

10

)  , , , ,
.
)  3 , 2
1 , :
.

.
)  ,
, ,
. (
) , .
.

) , , , ,

22

13 11
2 1
0 3

A = 10 9 , B = 2 2 , = 3 4 .
8 6
4 4
3 5

) ,
=3+2+,
13 11
2 1 0 3 39 33 4 2 0 3 43 38

X = 3 10 9 + 2 2 2 + 3 4 = 30 27 + 4 4 + 3 4 = 37 35 .
8 6
4 4 3 5 24 18 8 8 3 5 35 31
1 - , 2
-
.

38
X 2 = 35 .

31

43
X1 = 37 ,

35
13 2 0
= 10 2 3 ,

8 4 3

11 1 3
= 9 2 4

6 4 5

( ) ,

13 2 0 11 1 3 13 + 11 2 + 1 0 + 3 24 3 3
+ E = 10 2 3 + 9 2 4 = 10 + 9 2 + 2 3 + 4 = 19 4 7 .

8 4 3 6 4 5 8 + 6 4 + 4 3 + 5 14 8 8

.
,
2. ,
2 8

2 6

2 3

2 2

2 5
2 4
2 2
2 1

2 6 16 10 12


2 2 12 8 4

.
=
2 3 6 4 6

2 1 4 2 2

2 .
:

23

.
, , =[ij] =[ij] .
, , , ,
:
1. (+)=+
2. ()=()
3. (+)=+
4. lA=Al=A
5. = =0 =

1.2.2.

0 3 4
A = 2 2 5 . , 53=2 3.

4 1 1
( , ,
.
).
.

(,
) ,
.

1
5 X I3 = 2 X 3 A 5 X 2 X = I3 3 A 3 X = I3 3 A X = ( I3 3 A)
3
:

1 0 0 0 3 4
1 0 0 0
1
1

X = 0 1 0 3 2 2 5 = 0 1 0 + 6


3
3 0 0 1 12

0
0
1
4
1
1

9
6
3

12 1 / 3 9
12

15 = 6 19 / 3 15 .

3 12 3 10 / 3

1.2.1.
1 2 0
1 2 3
4 0
2 3 3

= 2 0 1 , = 2 2 0 , =
, = 3 1 .

0
1
2

3 0 3
0 1 1
2 5
, .
) ,
) ,
) ,
) ,
) ,
) , .

24

1.2.2.

4 3
1 4
2 4
A=
, B=
, =

,
1 2
3 2
5 1

 ( ) + (), ( + ), + ( + ), ( + ) + , ( )
+ . ;

1.2.3.

1 2 0
1 2 3
1 1 1

A = 2 3 1 , B = 2 2 0 , = 2 2 1
0 1 1
3 0 3
2 2 1

3B,

1
1
A, A + 2 , 2 B , (2 3 A) .
5
4

1.2.4.
 , .
, , , DVD,
:

DVD
.

20
55
29
90

30
40
15
80

33
39
48
65

25
15
15
30

18
23
50
66

75
29
18
85

)  ,

.
)
 .
)

.
)

( ) ,
-
.

1.2.5.
 , ( )
1.2.4,
:

DVD
.

400
80
290
90

380
100
250
120

390
90
310
100

25

) 
.
)  10%, .

1.2.6.
 1, 2, 3, 1 2.


1 , ,2 3
1, ,2 3
1

30 40 50
E1 =

20 10 30

36 48 60
E2 =

30 10 32 .

1
( E1 + E2 ) .
2
)
 20%,
.
( ) .

1.2.7. , 52=6+3
1 2
A = 1 3 ,
0
1

3 4
B = 2 2 .
4 1

1.2.8. ,
2+3=44 3+2=34+
= [ij] 4 aij = i j , i = 1, 2, 3, 4 j = 1, 2, 3, 4.
3
1 1
3 0 4

1
1
1

B
8
2 4
4 , 6( X B) = 12 X + A 2 B.
1.2.9. A = 2

2
4
3
3 1 2
1
1 1

1.3 .
1.1. ,
8

6
A=
3

5
4
2
1

( ) , , , .
( ),

26

.
:
) 5 ,
6
8 .
) 2 ,
1
3 .
,
( , )
5 2
B = 6 1 .
8 3

1 , :



1





+

1
1


+
1

( )

8 ^ 5 + 5 ^ 6 + 6 ^ 8 = 118
1
1 .
, 1 2
8 ^ 2 + 5 ^ 1 + 6 ^ 3 = 39

1 .

(
) (
) 2, 3 4 .
, :
) 1 : 118 ( ), 39 ( ).
) 2 : 70 ( ), 39 ( ).
) 3 : 51 ( ), 17 ( ).

27

) 4 : 24 ( ), 8 ( ).
:

118

39

70

22

51

17

24

,

118 39
70 22
.
=
51 17

24 8
.
11 1
1 , 12 1
2 ....
:
=[ij] =[ij] ,
, , ij
i
j . , ij = i1 1 j + i 2 2j + i 3 3 j + ... + ijj.

.
j

... ...
... ...

i1 ai 2

... ...

...
...
ai 3
...

... ...
... ...
... ai

... ...

... 1 j
...
2j

... 3 j

... ...
... j

...
... ...
... ... ...

...  = ...
ij

... ... ...

...

= ,
(. 1.3).

...
...
...

...

. 1.3

28

1.3.1.

2.

=
,

0 1
E
E=
.
1 0

, .
.

0 1
AE =

=
,
1 0

0 1
EA =

1 0

,
, , .
2
( ).
, , , ( ), :
1. (kA) (k) = (kk) (A)
2. () = ()
3. ( + ) = + , ( + ) = +

4. ==

( )
( )
( )

( ) , (), .
, , , , (()), (()), ()(), (()), (()),
, , , .
k 1, 2, 3,..., k
k, 1 2 3... k. A1=2=3=...=Ak=A, 1 2 3... k = AAA ... A k.

Al=A, A0=I.

Ik=I k.
k,r , :
5. AkAr = Ak+r
6. (Ak)r=Akr
7. ()k=kAk, R.

29

, :
8. (AB)T=BTAT.
,
= , ,
, . , ,
,
:
) . :
1 2 1
2 5 1 7

A = 2 1 1 , B = 7 1 3 4 ,

5 2 1
3 2 1 2
( 3 4),
( 3 4 3 3).
) , . ,
:
1 4
1 2 3

A=
, B = 2 5
4
5
6

3 6
. 2 2 , 3 3
=.
) ,
.
2
1
5 2
A=
, B=

3 2
3 1
,
1
AB =
3

2 5 2 11 4
5 2 1
=
, BA =

2 3 1 9 4
3 1 3

2 11 6
=
.
2 6 4

) , .
,
1 2
5 3
A=
, B=

2 1
3 5
:
1 2 5 3 11 13
5 3 1 2 11 13
, BA =
AB =
=

2 1 3 5 13 11
3 5 2 1 13 11
=.
. . :

30

) A , .
) H =0, =0
=0 , ..
1 1
0 1
A=
B =

0 0
0 1

0 0
AB = BA =
,
0 0
= =. ,
(. = ), ,
,
.
1.3.2.

, ,
(+)2, (+)3.
, =.
(+)2, (+)3.
.

:
( A + B)2 = ( A + B)( A + B) = ( A + B) A + ( A + B) B = A2 + BA + AB + B 2
( A + B)3 = ( A + B)2 ( A + B) = ( A2 + BA + AB + B 2 )( A + B) =
= A3 + A2 B + BA2 + BAB + ABA + AB 2 + B 2 A + B3 .
(.
=), .
=,
( A + B)2 = A2 + BA + AB + B 2 = A2 + AB + AB + B 2 = A2 + 2 AB + B 2

.
,
BA2 = ( BA) A = ( AB) A = A( BA) = A( AB) = ( AA) B = A2 B,
BAB = ( BA) B = ( AB) B = A( BB) = AB 2
ABA = A( BA) = A( AB) = ( AA) B = A2 B
B 2 A = ( BA) A = ( AB) A = A( BA) = A( AB) = ( AA) B = A2 B

31

( A + B)3 = A3 + A2 B + BA2 + BAB + ABA + AB 2 + B 2 A + B3 =


= A3 + A2 B + A2 B + AB 2 + A2 B + AB 2 + AB 2 + B3 =
= A3 + 3 AB 2 + 3 AB 2 + B3

.
(+)2, (+)3
==.
( A + I )2 = A2 + 2 AI + I 2 = A2 + 2 A + I .
( A + I )3 = A3 + 3 A2 I + 3 AI 2 + I = A3 + 3 A2 + 3 A + I .
1.3.3.

n- (n )
a11 0 ... 0
0 a
... 0
22

A=
... ... ...

0 0 a
0
n-
,
n
a11
0 ... 0

n
0 a22 ... 0
n
A =
.
... ... ...
n
0
0 0 a

.
. n=1
. n=k,
k
a11
0 ... 0

k
0 a22 ... 0
k
A =
.
... ... ...
k
0
0 0 a



k +1
a11
0

k +1
a22
0
...
...

0
0

n=k+1, Ak+1
0

...
0
...
k +1
0 a

...

32

k
a11
0 ... 0

k
0 a22 ... 0
k +1
k
A = A A=

... ... ...


k
0
0 0 a

a11 0 ... 0
0 a
... 0
22

... ... ...

0 0 a
0

:
k +1
a11
0
...
0

k +1
a22
...
0
k +1 0
A =
.

...
...
...

k +1
0
a
0
0

,
.
.

1 2 0
1 2 3
4 0
2 3 3

, = 3 1 .
1.3.1. A = 2 0 1 , B = 2 2 0 , =

0 1 2
3 0 3
0 1 1
2 5
 , , , , , , , ,
, .
1 2 1
2 5 1 7
3 6 0 6

1.3.2. A = 2 1 3 , = 2 1 3 4 , = 1 2 4 5 .

5 2 3
3 2 1 2
4 3 2 3
) , , , .
) , .
) ();
3 2 0
1 0
2 1 2

.
1.3.3. A = 1 0 2 , B = 0 1 , =
1 2 1

0 1 3
2 0
: + 2 + 3, 32 2, , 2 3:
1.3.4. M 1 2, 1, 2 3.
1, 2, 3 .
1 2 3
1 2
1 1, 5 2, 5 1
A 1
2
3 2 

1, 5 2
4 3

10 16 1
B 15 17 2 

13 14 3

33

) .
) 
2 1, 3 2 5
3 ;
1.3.5.  : . .

1
2
3

60
30
10

75
60
70

20
50

15
40

,
.
1.3.6. .
1 0 0 0 1 2 0
1 0 0 = 2 2 2

0 1 0 0 2 0 2 3
1.3.7.  = [x y z], B = [ ] =. x2+y2+z2;
x y
1
1.3.8.  A =
B =

, ,,x,y,R y0,
y x

AB=BA a=1.
x y
1.3.9.
 A=
A1(x+w)A+(xwyz)I2=.

z w
3 1
2
1.3.10.
 A=
, A +5A 1

2.

0

1.3.11. A A = 0 2+2+2=1, A3=A.


0
1.3.12.
 , (+)2=2+2
.
1.3.13.
 , (+3)(2+)=(2+)(+3) .

34

1.3.14.
 ,
= = .
4 3 3
0 1 1

1.3.15.
 A = 3 2 3 B = 2 1 2 .

3 3 2
2 2 3
 A2=B2=I3, AB+BA=2I3, (AB)2=O. : A2=B2, A=B A=B;

1.4 .
a a0 ,
1
a a1, aa1=a1a=1. :
, AB=BA=I;.
AB BA
. :
. AB=BA=I,
.
, .
, , A1. :
AA1=A1A=I.
,

1 2
5 2
A=
, B=

3 5
3 1
1 2 5 2 1 0
AB =

=
= I2
3 5 3 1 0 1

5 2 1 2 1 0
BA =

=
= I2 .
3 1 3 5 0 1
, .
,
.
.
, AB=I BA=I, A,B
, .
, , :

35

, ,

AX=B X=A1B XA=B X=BA1.

(1.4.1)

:
) AX=B, , A1 B = A1(AX )=(A1 A)X = IX = X .
1
1
) X=A1B, , AX = A(A B) = (AA )B = IB = B .
.
,
(. AB=O), , .
AB=O , ,
. , , :
AB = O A1 AB = A1O IB = O B = O .
, 2 .
.

) O A =
, , D = 0.


) A =
, ,

A1 =

1
.
D

(1.4.2)

A =

2. ,
x

X =
= ,
z

x y 1 0
x + z y + 1 0
z = 0 1 x + z y + = 0 1

x, y, z,
x + z = 1

x + z = 0

(1)

y + = 0

y + = 1

(2).

, (1) (2) . D =
:

) D 0, (x, z) x = z =
, D
D

=
(1), (y, ) y =
D
D


(2).
36

D
X =

D

A1 =

1
.

D


D D
X =
(


D
D
D 0) = = .
) D =0, (1) (2) ,
. :
 0 = 0 0 0, (1)
(2) .
= = = = 0, .
,
1 2
A =
, D=1523=56=10
3 5
5 2 5 2
1 5 2
A1 =
= (1)

=
.
1 3 1
3 1 3 1
1 2
, A =
, D=1623=66=0.
3 6
1.4.1.

, ,

()1=11.
.

()(11)= (11)()= . ,

( AB)( B 1 A1 ) = (( AB) B 1 ) A1 = ( A( BB 1 )) A1 = ( AI ) A1 = AA1 = I ,


( B 1 A1 ) ( AB) = (( B 1 A1) A) B = ( B 1( A1 A)) B = ( B 1 I ) B = B 1 B = I .
1.4.2.

0 1 1
4 3 3

A = 3 2 3 , B = 2 1 2 .

2 2 3
3 3 2

37

) 2, 2.
) .
) = =.
.

0 1 1 0 1 1 1 0 0
A = 3 2 3 3 2 3 = 0 1 0 = I3
2 2 3 2 2 3 0 0 1
2

4 3 3 4 3 3 1 0 0
B = 2 1 2 2 1 2 = 0 1 0 = I3

3 3 2 3 3 2 0 0 1

) 2=2=3 ==3 ==3


1= 1=.
) 1=, :

0 1 1 4 3 3 5 4 4
5 4 4

AX = B X = A B X = AB = 3 2 3 2 1 2 = 1 2 1 X = 1 2 1 .
2 2 3 3 3 2 5 5 4
5 5 4
1

0 1 1 4 3 3 5 4 4
5 4 4
3 2 3 2 1 2 = 1 2 1 X = 1 2 1 .

2 2 3 3 3 2 5 5 4
5 5 4
, 1=, :
4 3 3 0 1 1 3 4 4
BY = A X = B A X = BA = 2 1 2 3 2 3 = 1 0 1 .

3 3 2 2 2 3 5 5 6
1

1.4.1.
 , ,
2.
1 2

6 3
6 3
A1 =
, A2 =
, A3 =
, A4 =

.
4 3

4 2
4 2
4 7
1 0
1.4.2. A =
B =
.

3 5
2 1
) .
) 1=.
) 1=.

38

1.4.3.
 , .
1 1
1 2 3
0
1

) A = 4 1 1 , B = 3 2 11
3
1 1 1
3
1 7
1
1
) A =
1

1 1 1
1 2 3
,
2 1 3

3 3 1

2 1 2
1 1 2

) A = 1 1 0 , B = 1 2 2
2 0 1
2 2 3

15 / 8 1 / 4 1 / 4 3 / 8
1 / 4 1 / 2 1 / 2 1 / 4

B=
1 / 4 1 / 2 1 / 2 1 / 4

3 / 8 1 / 4 1 / 4 1 / 8

1 3 x

1.4.4. A = 0 1
0 0

9x2

6x .
1

1 3 x

) B = 0 1
0 0
1 6 36

) 0 1 12 .
0 0 1

9x2

6 x .
1

3 4
1 2
1.4.5. A =
B =

.
2 3
0 1
) .
) (AB)1 .

1.4.6.
 x,y,z , A,B
3 .
2 1 1
A = 1 2 1 ,

1 1 1

x
B = y
3

0 1
x 3 .
1 z

1.5 .

A=[aij] , B=[ij] .
.

A = B aij = ij (i = 1, 2,..., j = 1, 2,..., )


,
.
a11 0
0 a
22

... ...

0
0

... 0
... 0

... ...

0 a

39

1
0

I = I = 0

...
0

0
1
0
...
0

...
...
...
...
0 ...

0
0
1
...

0
0

0
1

,
.

ij = ji
i = 1, 2, ..., j = 1, 2, ..., = A.
A + B = [ aij + ij ]

A=[aij] =[ij] ,
:
,
,
R.
A=[aij]
=[ij] .

A B = [ aij ij ]
A = [ aij ]
AB = [ ij ] ,

ij = a i1 1 j + a i 2 2 j +... + a i j
1=1=

2

A=
D = 0

A1 =

1.6 .
,
, .
1.

A=[aij] + .

2.

(2 + 2) = , = .

3.

A 3( ) = , =.

4.

, 34, .

5.

0 0 0

0 1 0 .
0 0 0

40

6.

2=, = =.

7.

=
, =.

8.

=5, , , ,
.

9.

()2=, , A = I.

10.

, ,
()2=22.

3 2
11.

2 1
12.

3 2
.
2
1

= , ,
.

13. = .

14.

,
=, =1.

15.

, ,
.

.
1.

, ,
, :
) , 1

2.

3.

4.

) 1,

1

2

1
:
0

) 1, 1

a1

a2
1
)
2

a1
a1
)
) a
1
2
, 410, :

a1
)
2

) . ) . ) . ) .
a2 3a + 3

0
0

0
0
a2 a + 1
:

2
0
0
a 4a + 4

) = 1.

) = 2.

) = 0.

) .

41

5.

= =, :
) =

6.

) =

) =

) =

x0 . ' x :
1 1
1
) A
) xA
) xA1
) A
x
x
,, , :

7.

) ( + ) = + .

) ( ) = ().

) ( + ) = + .

) .

1 4

:
2 8
8.

3 2
)
.
2
1

3 2
)
.
1
2

) .

3 2
.
)
1
2

9.

1 0 0
4 2 0 :

6 5 3
) . ) . ) . ) .

10.

2
0

0

5 4 0
3 5 2
:
0
1 3

0 0 0

) . ) . ) . ) .

11.

1 0 0

4 2 0 :
6 5 3
) . ) . ) . ) .
, , , :

12.

) + ( + ) = ( + ) + .
) + = + .
) ( ) = ( ) + .
) .

42

13.

, , :
) 5(A+B)=5A+B
) = =0 =
) =
) (+)2=2+2+2

1.7 .
1.7.1.  x12 + x22 + ... + x2 =[x1,x2,...,x].
1.7.2. :
) 
.
)  .
)  .
1.7.3. . :
) .
) .
1.7.4. , . :
) .
) + .
1.7.5.
 =. , . :
) .
) + .

, , ... .
, . , , ,
. Cramer
, .

2.1 .
2.2 .

2.3 I .

2.4 
.
2.5 .
2.6 .

2.7 
.

2.8 .
2.9 .

lx im

2.10 .
2.11 .

x0

44

2.1 .
A x1, x2
11 x1 + 12 x2 = 1
21 x1 + 22 x2 = 2 .

()


11 x1 + 12 x2 1

= .
21 x1 + 22 x2 2
11 12
(, 22 A =
21 22

x
) 21 X = 1 ( x2
).

11 12 x1 1

= AX = B
21 22 x2 2

(2.1.1)


B = 1 .
2

,
22 12, :

1122 x1  12 22 x2
2112 x1  22 12 x2

22 1

12 2

(1122  2112 ) x1

22 1  12 2 .

(2.1.2)

,
D=112221120
x1

x1 =

22 1 12 2

= 22 1 12 2 .
1122 2112
D

(2.1.3)

x2

(11a22  2112 ) x2

11 2  21 1 

(2.1.4)

, D 0,

x2 =

11 2 21 1
21 1
= 11 2
.
1122 2112
D

(2.1.5)

D = 11 21 21 22 ,

45

12

A = 11
2 ( 21 22
11 12
.
2 2 )
21 22
:
2
11 12
A=
.
21 22

,
a11 a12
A=
= a11a22 a21a12 .
a21 a22

(2.1.6)

:
4 3
4 3
|
A
|
=
= 4( 2) 5( 3) = 7 ,
A =

5 2
5 2
1 0
1 0
| I| =
I =
= 1 0 = 1

0 1
0 1
0 0
0 0
O =
O=
=00=0 .

0 0
0 0
(2.1.3), (2.1.5) . ,
D
D
Dxxx1 112222 221212
11

11 1212
22 2222

,,

D
Dxxx2

22

111122 212111

1111 11
2121 22

, D 0 ,

11 1
12
1212
DD
DxDx x
x
x1x1x1 111 1 2 2 2 222222, , ,
D
DDD
11
1111
12
1212
212121 222222

x 2x x2 2

DDxD
Dx x

11
1111 1
11

212121 2 2 2
D
DD
D
11
1111
12
1212
212121 222222
2 x222

(2.1.7)

( Cramer).
Dx1, Dx2 D =
x1, x2 .
() (2.1.2), (2.1.4),

D ^ x1 = Dx1, D ^ x2 = Dx2

:

46

11 x1 + 12 x2 = 1

12

D = 11
21 22
21 x1 + 22 x2 = 2 .
.
D 0
1111

12
12
12
12
D
DD
D
D
xxxxx
2222 22
2222
22
11111
,, ,,
xx1x1x11
D

D
D
D
D
11
11
11
11 12
12
12
12
21
2121
22
2222
21
22

11
11
11
11 1
111
D
D
D
D
x
x
x
x
Dx22222 21
2121
21
2
222
xx2x2x22
.

D
DD
D
11
11
11
11 12
12
12
12
21
2121
22
2222
21
22

(2.1.8)


 D = 0 Dx 0 Dx 0, 1
2
.
 D = Dx = Dx = 0, ,
1

11 = 21 = 12 = 22 = 0 1 0 2 0,
.
2 3 ,
3 3 .
a11 a12 a13
A = a21 a22 a23 .
a31 a32 a33
11

22

23

32

33

12

21 23
31

33

+ 13

21 22
31

32

11 12
21 22
31 32

13
23 .
33

3
a11 a12
A = a21 a22
a31 a32

a13
a23 .
a33

, :
a11 a12
A = a21 a22
a31 a32

a13
a22
a23 = a11
a32
a33

a23
a21 a23
a21 a22
a12
+ a13
.
a33
a31 a33
a31 a32

47

, , ()
. ()
ij 2 ,
ij. (1)i+j.
2 ,

A = a11(a22 a33 a32 a23 ) a12 (a21a33 a31a23 ) + a13 (a21a32 a31a22 )

A = a21(a12 a33 a32 a13 ) + a22 (a11a33 a31a13 ) a23 (a11a32 a31a12 )
= a21

a12
a32

a13
a11 a13
a11 a12
+ a22
a23
.
a33
a31 a33
a31 a32

.
,
A

a11

22

a23

a32

a33

 a21

a12

a13

32

a33

 a31

a12

13

22

a23

.
, () ,
.

1+1
(1)1+ 2
+ + (1)

+ = (1)2+1 (1)2+ 2


+ + (1)3+1 (1)3+ 2

(1)1+3

(1)2+3 .

(1)3+3

1 2 1
A = 3 2 5 ,
2 0 0

,
, :
A =2

2 1
1 1
1 2
0
+ 0
= 2 (10 + 2) 0 (5 3) + 0 (2 + 6) = 16.
2 5
3 5
3 2

, , .

48

33

11
21
31

12
22
32

13
23
33

11 (22 33  32 23 )  12 (2133  3123 )  13 (2132  3122 ) 

112233 123123 132132 113223 122133 133122

,
Sarrus. (+)
() + + +

D11 D12 D13 D11 D12


(. 2.1).
D 21 D 22 D 23 D 21 D 22 

,
D 31 D 32 D 33 D 31 D 32
( Sarrus

. 2.1.
Sarrus.
).

2.1.1.

2
3
2
A = 1
2
1 .
2
2 1

) =2 .
) .
.

0 2 3
) =2 A = 1 0 1
2 2 1
0 2 3
0 1
2 3
2 3
A = 1 0 1 =0
1
+2
=
2 1
2 1
0 1
2 2 1
= [ 2 (1) 3 (2)] + 2[ 2 1 0 3] = 4 + 4 = 0.
)

49

2
2
3
2
1
1
1
1 2
A= 1
2
1 = ( 2 )
2
+3
=
2 1
2 1
2 2
2
2 1
= (2 )[(2 ) (1 ) (2) 1] 2[1 (1 ) 1 2] + 3[(1 (2) 2 (1 )] =
= 3 + 5 2 2 8.
=2, , .
2.1.2.

B(x2 , y2)

, , xOy (x1, y1), (x2, y2), (x3, y3) , (. 2.1)


1 x3
1
( ) 1 x2
2
1 x1

(x3 , y3)

y3
y2 .
y1

(x1 , y1)
x

. 2.1.

, , , ,
Ox, 2.1,

,

B(x2 , y2)

() = () + () ().

(x3 , y3)

, (. 2.1)
( ABE)
2
A=y1, BE=y2, =x2x1 :

1
( ABE) ( y1 y2 ) ( x2 x1 ) .
2

( )

1

( y2 y3 ) ( x3 x2 )
2
2

( )


1
( y1 y3 ) ( x3 x1 ).
2
2

(x1 , y1)
O

. 2.1.

B(x2 , y2)

y2

,
1
1
( AB ) ( y1 y2 ) ( x2 x1 ) ( y2 y3 ) ( x3 x2 )
2
2
1
( y1 y3 ) ( x3 x1 )
2

y1
O

(x1 , y1)
(x2 - x1)

. 2.1.

50


1
1
( AB ) ( y1 x2 y1 x3 y2 x3 y2 x1 y3 x2 y3 x1 ) y1 ( x2 x3 ) y2 ( x3 x1 ) y3 ( x1 x2 ).
2
2

1 x3
1 x2
1 x1
1 x3

y3

1 x2

y2 y3

1 x1

y1

1 x2
1 x1

y2

y3

y2 3
y1

1 x3
1 x1

y1

1 x3

y3 ( x1 x2 ) y2 ( x1 x3 ) y1 ( x2 x3 )

1 x2

1 x3
1
( AB ) 1 x2
2
1 x1

y3
y2 .
y1

, , , ,
(..
, ... ), , ,
. ,
.
(4,5), (5,2) (1,2)
1
( AB ) =
2

1 4 5
1 5 2
1 1 2

1
= 12 = 6 .
2

2.1.1. (2.1.8).
)

2x + 3y = 5
3x + 2 y = 5

2x + 3y = 5
3x + 6 y = 3

2x 5y =1
4 x + 10 y = 2

2.1.2. 
, 2 .
)

1 0 5
2 2 4
3 0 1

) 1
0

1 0 5
) 0 2 0
3 0 4

0
1

9 3 2
) 6 2 4
0 0 1
1 0
) 1 1
1 1 0

51

2.1.3.
x 1
x 1
x 1

x =0
3

x
3

x
x
=0
)
5x 2x + 3

x2
x 1
x 3

x +1 1
x +1 2 = 0
x
3

x2

x2

1 = x

x2

x 1
1 1

2.1.4.  ,
, , (1,5), (2,3), (2,1) . 2.1.2,
.
2.1.5.
a

a2 2 2 = (a )( )( a) .
1 1
1
2.1.6.
a 1 1
1 1 1
1 a 1 = a 1 a 1 .
1 1 a
1 1 a
2.1.7.
 , 0.
a
x y
x y

z ,
x
y
z
a+ x + y

a
z , x y
+ z x y

z , a+ x + y
z
x
y

+z .
z

2.1.8.  , .


x y z ,
k m n
k m n
x y z
1 1
2.1.9. A = 1 1 .
1 1
) =2 .

52

) .
)
1 1
1 1
1

 1.

2.2 .
,
a11 a12
A = a21 a22
a31 a32

a13
a23
a33

3, ,
A = a21

a12
a32

a13
a11 a13
a11 a12
+ a22
a23
,
a33
a31 a33
a31 a32

A = a11

a22
a32

a23
a21 a23
a21 a22
a12
+ a13
,
a33
a31 a33
a31 a32

A = a11

a22
a32

a23
a12
a21
a33
a32

a13
a12
+ a31
a33
a22

a13
.
a23

, , ij 2 Mij
, ij.
ij mij =Mij.
, + ,
(1)i+j. (1)i+j mij ij cij,
cij = (1)i + j mij = (1)i + j M ij .
,
A = a21c21 + a22 c22 + a23c23 ,

A = a11c11 + a12 c12 + a13c13 ,

(2.2.1)

A = a11c11 + a21c21 + a31c31 .


,

A = a31c31 + a32 c32 + a33c33 ,

A = a12 c12 + a22 c22 + a32 c32 ,

(2.2.2)

53

A = a13c13 + a23c23 + a33c33 .


3
( 33 ) 2 .
, , A = [ij] .
3 1 (
).
A = [ij] .

11 12 1
21 22 2

1 2
= 11c11+12c12++1c1 cij = (1)i+j mij= (1)i+jMij Mij 1 ,
ij.
3 , mij =Mij
ij cij = (1)i+j mij ij.
11c11+12c12++1c1, ,
1 .
3 ,

1. :

11 12

21 22
A=

1 2

1
2
.


A = ai1ci1 + ai 2 ci 2 + + ai ci ,

A = a1 j c1 j + a2 j c2 j + + aj cj

i j 1,2,,.
i ,
, j .
, ,
.
, , = 0.

54

2.2.1.

a
xa
a
a

a
a
0 a x
0
x
a
a

a
0
a
x

=0

.
.

2 (
)
a
a
a
x a 0 a x
a
0
x
a

a
0
= (1)2+1( x a) M 21 + (1)2+3 (a x ) M 23 = ( x a)( M 21 + M 23 )
a
x
a a a
M 21 = 0 x a ,

a a

a a a
M 23 = a 0 a .

a a

21 1 23 2
,
a
M 21 = 0a
M 21 = a0
a
a
M 23 = aa
M 23 = aa
a

a
xa
ax
a
a
0a
a0

a
x
aa = (1)1+1 a x
1+1 a
xa = (1) a a
x
a
a
aa = (1)1+ 2 a a
1+ 2 a
xa = (1) a a
a x

a
a
+ (1)1+3 a a
a
x + (1)1+3 a x
x
x

a
= a( x 2 a2 ) + a(a2 ax ) = ax( x a),
aa = a( x 2 a2 ) + a(a2 ax ) = ax( x a),
a

a
a
+ (1)3+ 2 a a
a
x + (1)3+ 2 a a
x
a

a
= a(ax a2 ) = a2 ( x a).
a
a = a(ax a2 ) = a2 ( x a).
a


a
a
x
x a 0 a x
a
x
0
a
a
0

a
0
= ( x a)[ ax( x a) a2 ( x a)] = a( x a)2 ( x + a)
a
x

, a(xa)2(x+a) x=a x=a.

55

2.2.2.


11
0
, A =

1122.

12 1
22 2

11 0

21 22
, A =

1 2

0
0

, .
.

33
11 0

A = 21 22

31 32

0 .

( 2.2.1) :
A = 11

A = 11

22

32

33

22

32

33

+0

21

31

33

+0

21

22

31

32

= 11 (22 33 0 32 ) = 11 22 32 .

, .
2.2.3.

12
11 + a11
+
22
21 21
: A =

1 + 1 2

:
11 12

22
|A|= 21

1 2

12
1 11
22
2 21
+

1 2

1
2
.

1
2

56

.
1

A=

11 + a11

21 + 21

12 1
22 2

1 + 1


2 v

) c11 + (21 + 21
) c21 + + (1 + 1 c1 )
= (11 + 11

ci1 (1)i 1 mi1 (1)i 1 Mi1 i1, i = 1, 2, ... .

a11
11
cc11c1111aa21
a21
21
cc21
AAA(((11
1111 cc11c1111
cc21
c2121 ...
cc11)1))((a(a11
c2121...aaa111ccc11)1))
21
21
21
111c


11

12 1

21 22 2


1 2
1 ,

11

21

12
22

1
2

1 .
.

2.2.1.
a 1 2 3
0 5 6

4
7

a 1
0 0

0 0 8 9 ,
0 0 0 10
0 0 0 0

2 3
8

4
9

3 1
8 0

0 0 0 0 ,
0 0 0 10
0 5 6 7

2 a
0

4
9

0 0 0 0 .
0 0 0 10
6 5 0 7

;
2.2.2.
1
1
1
1

0
2
2
2

0
0
3
3

0
0
,
0
4

1
1
1
1

0
2
2
2

0
0
0
3

0
0
,
0
4

4
1
1
1

0
3
2
2

0
0
2
3

0
0
,
0
1

1
0
0
11
2
0
111 22 3
1111 222 33

0
0
.
0
4

57

2.2.3.
x 1

x2 +1

x2

x2 +1

x 3

x2 1

x2 +1

x4

x 4

x 1

x +1 x 5

x +1

x2 +1

x+2

x +3

x +1

x2 1

=0 .

x2 +1 x + 4

2.2.4.
1
0
0
0
0

2
1
0
0
0

3
2
1
0
0

4
3
2
1
0

5
4
3 ,
2
1

1
0
0
0

1
1
0
0

1
1
1
0

1
1
,
1
1

1 1 1 1 1
0 111 1 1 1
0 0 0 111 1 .
0 0 0 111 1
0 0 0 0 1

1 1 1 1
0 1 1 1
,
0 0 1 1
0 0 0 1

2.2.5.

y
w
z

p 1

q 1 0 y
=
r 1 w
s 1 z

p 1

p 1

q 1 y
+
r 1 0 w
s 1 0 z

q 1
r 1
s 1

 2.2.3,

.
2.2.6.

y
w
z

p 1

q 1 0 y
=
r 1 0 w
s 1 0 z

p 1

q 1 y
+
r 1 0 w
s 1 0 z

p 1

p 1

p 1

q 1 0 y
+
r 1 0 w
s 1 z

q 1
r 1
s 1

q 1 0 y
+
r 1 w
s 1 0 z

2.2.3,
.
2.2.7.  2.2.2 .
11 12

21 22

31 32

41 0

13
23
0
0

14 0

0 0
,
0 0

0 41

23

32

33

42

43

14

24
.
34

44

58

2.3 I .
, , , ,
.
2,
(2.1.6). ,
,
, .
,
.
2.2.2 :
D1
 ,
.
,

x2

x3

0 2 ex

0 0

x2 1

0 0
0 0

0
0

4
0

x4
x
1
= 1 2 3 4 5 = 120.
4
x +2
xe x
5

D1 :
D2
 ,
,

11
0
#
0

"

22 "
#

0
#

1122 " .

"

D3 1, = 1.
, :
D4
 ( ) ,
.
, , .
D5
 ( ) , .
D6
 ( ) ,
.

59


( ).
D7
 (
) ,
=.
D8

( ) ( ) ,
= .
D9
 (
) '
( ) ( )
, =.
, ,
' .
D10
 ,
=.
D11

, =.
 = 2=2 ,
Ak = Ak , k = 2, 3,...
D12

, = .
D10, D11, D12
=2. ,

A=
,

a
AT =
,

x y
B=
A = , B = x yz
z
x y x + z y +
a
AB =
A =

=
.

z x + z y +

) AT = a = a = A ,
) A = ( )( ) ( )( ) = 2 ( ) = 2 A ,
) AB = (ax + z)( y + ) (ay + )( x + z) = axy + ax + zy + z
yx yz x z = (x x ) (yz zy)
= ( )x ( ) yz = ( )( x yz) = A B .
,
.

60

2.3.1.

1 3 5

A = 2 4 6 9 7 10
, .
.

(2)

1
3
5
0 2 4
9
7 10
D9 ,
1 3 5
1
A= 2 4 6 = 0
9 7 10
9

3
2
7

5
4 .
10

, (9)
1
3
5
A = 0 2 4 .
0 20 35
(10)

1
3 5
A = 0 2 4
0
0 5
, D1
,
1
0
0

3 5
2 4 = 1 (2) 5 = 10 .
0 5

= 10.
2.3.2.

0,1,2 3 (x)=0
x 1
A( x ) =

x2 1 1

x3 1 1

26

x 4 1 1 15 80

61

x=0, x=1, x=2 x=3,


1
1
A(0) =
1
1

1 1 2
1 3 8
, A(1) =
1 7 26
1 15 80

0 1
0 1
0 1

1
3
7

2
0 1 1 2
8
0 1 3 8
, A(2) =
, A(3) =
26
0 1 7 26
0 1 15 80
0 1 15 80

0
0
0
0

1 1 2
1 3 8
1 7 26
1 15 80

. A(0) [ 2
1 (1)], ,
D6 A(0)=0. , 0 A(x)=0.
A(1) , (
D4 ) A(1)=0. , 1 A(x)=0. ,
A(2),A(3) , ( D5 ) A(2)=0 A(3)=0. 2 3 A(x)=0.
2.3.3.


x
5

A = 5

5
5

5 5 5 5
x 5 5 5
5 x 5 5

5 5 x 5
5 5 5 x

(.
).
.

, , 1

x + 20

x + 20
x + 20

x + 20
x + 20

5
x
5
5
5

5
5
x
5
5

5
5
5
x
5

5
5
5

5
x

D9 ,
x 5 5 5 5
5 x 5 5 5

x + 20 5 5 5 5
x + 20 x 5 5 5

A = 5 5 x 5 5 = x + 20 5 x 5 5 .
5 5 5 x 5
x + 20 5 5 x 5
5 5 5 5 x
x + 20 5 5 5 x

62

D8,
x + 20 5 5 5 5
x + 20 x 5 5 5

1 5 5 5 5
1 x 5 5 5

A = x + 20 5 x 5 5 = ( x + 20) 1 5 x 5 5 .
x + 20 5 5 x 5
1 5 5 x 5
x + 20 5 5 5 x
1 5 5 5 x
(1) , ,

1 5 5 5 5
1 x 5 5 5

1
0

5
x 5

5
0

5
0

5
0

0
0
0

x 5
0
0

0
x 5
0

0
0
x 5

A = ( x + 20) 1 5 x 5 5 = ( x + 20) 0
1 5 5 x 5
0
1 5 5 5 x
0

, D1
,
1
0
0
0
0

5
x 5

5
0
x 5

0
0
0

0
0

5
0
0
x 5
0

5
0
0 = 1 ( x 5)4 = ( x 5)4 .
0
x 5

= (x5)4.
2.3.4.

, xOy (x1, y1), (x2, y2), ,


,

1 x
1 x1
1 x2

y
y1 = 0 .
y2

(2.3.1)

1
1

x1
x2

y1
1 y1
1 x1
x
+ y
=0
y2
1 y2
1 x2

x+y= =

1 y1
1 y2

1 x1
1 x2

x1

y1

x2

y2

63

, (2.3.1) . , (2.3.1) x=x1, y=y1 x=x2, y=y2,


1 x1
1 x1
1 x2

1 x2
1 x1
1 x2

y1
y1 = 0 ,
y2

y2
y1 = 0
y2

( D5). , ,

y
1 x
1 x
y1 = 0 .
1

1 x2 y2
, (1,2) (2,1)
1 x y
1 1 2 = 0 1 1 2 x 1 2 + y 1 1 = 0 x + y = 3 .

2 1
1 1
1 2
1 2 1
2.3.5.

1+ x
1+ y
1+ w
1+ z

p
q
r
s

1
x
1
y
=
1
w
1
z

p
q
r
s

1
1
1
1

.
.

2.2.3,
1+ x
1+ y
1+ w
1+ z

p
q
r
s

1
1 x
1
1 y
=
1
1 w
1
1 z

p
q
r
s

1
x
1
y
+
1
w
1
z

p
q
r
s

1
1
1
1

( ),
1+ x
1+ y
1+ w
1+ z

p
q
r
s

1
x
1
y
=
1
w
1
z

p
q
r
s

1
1
.
1
1

64

2.3.1.  1 (0,0)
(5, 4). 2 (3, 1) (4,5).
)  2.3.4,

1 x + 1 y = 1 , 2 x + 2 y = 2
)  (2.1.8),
.
2.3.2.  2.2.2
D7 ( ).
2.3.3.
 D12 D5 ( , D6)
.
1 2 3 4

0 1 0 1 ,
1 0 1 0

5 10 15 20

1
1

2
3

1 2
3 4
4 2 3 1
5 6 6 2 ,

3 3 6 3
1 1 9 1

x
2x
2.3.4.
2
x
2
2

1 1 2 2
1
3
3 4 4 4

1
2
3 4 5

2 2 2
1 1
2
2 3 3 3
1
2
3 4 5

2x
2
= 0 x = ( + ).
x

2.3.5.  (.
)
=1.
1 1 1
1 1 ,
1 1

1 1
1 1 ,
1 1

1 1
1 1 1 ,
1 1

1
1
1 1

1
1 .
1

2.3.6. 1, 2 3
x 1
( x 1)

1 2
2

1 4 =0 .

1 8

( x 1)

2.3.7.  ,,
.
A4 B3 T , ( AT )3 ( B 4 )T T , A3 B 4 , (3 A) B 2 ( 2 T ), 6 B 2 BT , T B3 A4 , B3 A4 , B 2 (6 ) AT

65

2.3.8.  , , = 1, = 2, = 3,

AB , AT B , A3 B 2 , 3 AB 2 T , 3B 2 A .
2.3.9. :
) =

) 2=

) 3=

2.3.10.
x 2
A = 1
1

1
x2
1

1
1
x 2

 (.
).
2.3.11. )
 ,

x x 2 1

A = a a2 1 .

2
1
)
x

x2 1

1
2

1
4

1 + 3
1
2

x2 1
9
4

x2 1

1 =3 3 9 1 .
1
4 16 1

2.3.12.
a3

8a3

27 a3

64a3

a2
a
1

4a2
2a
1

9a2
3a
1

16a2
= 12a6 .
4a
1

2.3.13.  ,
, .
0
1 1 1 1
2 3 4
2
3 4 2 , 2 2 3 3 , 1

2 2 2 2
4 2 3
6
1 2 3 4 2

1 1 2 2
3 4 5 6

2 3 2 1

5 4 3 2
2 1 1 0

2.3.14.  xOy (x1, y1), (x2, y2), (x3, y3), .


2.3.4,

66

1 x1
1 x2
1 x3

y1
y2 = 0.
y3

2.3.15. )  ,


1 +
1+


1+

1 +

1++++.
) 4,
2
1

2 3 4
3

2
3 3 4
A=
2
2 4 4

2 3 5
2

3 4 5
4 4 5
.
3 5 5

3 4 6

.
2.3.16. ) ,
1 1
1
1
1 1+ x
1
1
= xyz .
1
1 1+ y
1
1
1
1 1+ z
) 4,
1
1

1 1 1
1

1
2 1 1
A=
1
1 3 1

1 1 4
1

1 1 1
4 1 1
.
1 3 1

1 1 2

.
2.3.17.  D13 , .
11
00

00

11
11

11
00 00 11 11
11
11 00 11 22
00
00 11 11 33,,
00
11 11 11 44
00
22 33 44 55
00

11 22
22 00
00 22
00 33
00 44

22 33
00 11
00 00
44 00
55 66

00

66

55

88

33 11

11 11

00 33
,,
00 44
00 55

11 66

11 22
22 00
44 22
55 00
66 00

22 33
00 11
33 44
44 55
00 66

77

00

00

00

33

11

55

66
22

88

67

2.4 .
3
a11 a12
A = a21 a22
a31 a32

a13
a23
a33

mij = ij aij cji = (1)i+j mji =


(1)i+jij aij i = 1, 2, 3 j = 1, 2, 3. C
,
c11 c12 c13
C = c21 c22 c23 .
c31 c32 c33
CT (i, j) cji = (1)i+j mji = (1)i+jij,

c11 c21 c31


T
C = c12 c22 c32 .
(2.4.1)
c13 c23 c33


( ).
( ),
CT=A Iv.
.
, C = [cij] cij = (1)i+j mij =
(1)i+jij ij i = 1, 2,..., j = 1, 2,...,
CT C,
. :
) A CT = A Iv.
)  A 0.

1
A1 =
CT
(2.4.2)
A

D11 D3 : AA1= A A1= I= 1.
:
,

1
A1 =
.
(2.4.3)
A

68

, =2

=
.

(), A= 0.
, (2.4.2)



T
C=
, C =
.





A1 =

1 T
1
.
C =

A
A

(1.4.2), 1
.

2.4.1.

3 1 0

A = 2 4 1 .
0 1 3
.
,
3 1 0
A = 2 4 1 = 3(12 1) 1(6 0) = 27 0 .
0 1 3

mij = ij ij
cij = (1)i + j mij = (1)i + j M ij
i = 1, 2, 3 j = 1, 2, 3
c11 (1)11

4 1
1 3

12  1 11, c12

(1)1 2

2 1
0 3

(6  0) 6, c13

(1)13

2 4
0 1

2  0 2, 

c21 = (1)2+1

1 0
3 0
3 1
= (3 0) = 3, c22 = (1)2+ 2
= 9 0 = 9, c23 = (1)2+3
= (3 0) = 3,
4 3
0 3
0 1

c31 = (1)3+1

1 0
3 0
3 1
= 1 0 = 1, c32 = (1)3+ 2
= (3 0) = 3, c33 = (1)3+3
= 12 2 = 10,
4 1
2 1
2 4

69

11 6 2
C = 3 9 3 ,
1 3 10
T

11 6 2
11 3 1

T
C = 3 9 3 = 6 9 3 .
1 3 10
2 3 10
, (2.4.2)
11 3 1 11 / 27 3 / 27 1 / 27
1 T 1
A =
C =
6 9 3 = 6 / 27 9 / 27 3 / 27 .

A
27
2 3 10 2 / 27 3 / 27 10 / 27
1

2.4.2.


a11 0
A = 0 a22
0
0

3, ( ).

0
0 ,
a33

D2 , , A = 11 22 33.
11, 22, 33 , A= 11 22 33 =
0, . 11 22 33 0, (. 11,
22, 33 ) A= 11 22 33 0
(2.4.2).
11, 22, 33
m11 =

a22

a33

m22 =

a11

a33

m33 =

a11 0
0 a22

D2
m11 = a22 a33 ,

m22 = a11a33 ,

m33 = a11a22 .


( ), C=[cij]

0
0
a22 a33

0 .
C= 0
a11a33
0
0
a11a22

70

, , (2.4.2)
a22 a33
1 T
1
1
0
A =
C =
C =
A
A
a11a22 a33
0
1

0
a11a33
0

0
0
a11a22

0
0
1 / a11

1 / a22
0 .
A = 0
0
0
1 / a33
1

. , 11 22 ... 0,
a11 0
0 a
22
A=
... ...

0
0

... 0
... 0
... ...

0 a

0
1 / a11
0
1 / a22
A1 =
...
...

0
0

0
0
.
...

0 1 / a

...
...
...

(2.4.4)

11=22=33=...=a=1

1 0 0 ... 0
0 1 0 ... 0

I = 0 0 1 ... 0

... ... ... ... 0


0 0 0 ... 1
I 1 = I .
2.4.3.

, A= 2, = 4,
.
A1 B, AB 1 , ( AT B)1 A1 , ( A3 )1 BT A2 B 1 , ( AT BT )1 AB 2 .
.

(2.4.4) D10, D11 (. 2.3),

71

A1 B = A1 B =

1
1
B = 4 = 2,
A
2

AB 1 = A B 1 = A

1
1 1
= 2 = ,
B
4 2

( AT B)1 A1 = ( AT B)1 A1 =

3 1

(A ) B A B

T 1

3 1

= (A )

( A B ) AB = ( A B)
T

1
T

A B

B A B

AB =
2

1
1
1
1
1
1
= T

=
A
A B A 16
A B A

1
AT BT

B A
1
1
= 3 B A
= 3
=
B
A
A B 2
1

AB =
2

AB

AT BT

AB
=
= B = 4.
A B

2.4.1.  . ,
,

2 2 6 2 1 1 1 1 1 1 1 1
1 2 1 1 1 2 1 1 2 2 3 3

,
,
.
1 1 3 1 1 1 3 1 2 2 2 2

1 1 1 4 1 1 1 4 1 2 3 4
2.4.2.
2 4 1
1 1 1 ,

3 9 1

1
1

2 4 8 1
3 9 27 1
,
1 1 1 1

4 16 64 1

1
0
1
1

1
1
0
1

1
1
,
1

1
1

1
2
1
1

1
1
2
1

1
1
.
1

2.4.3.  ,, A=== 2,
.
A1 B T , ( AT B 1 )1 , A3 BT 1 , A1 B 1 1 , B 2 1 A1 .
2.4.4.  ,
1 = 2.
2.4.5.  ,
1.
2.4.6.  x , .
1
x 1 1 x 1 1 x 1
1 x 1 , 1 1 1 , 1 x
1 .



1 1 x 1 1 x 1 1
x

72

2.5 .
2, , , ..
.
.

.
2.5 Ohm (),
mper () volt (V).

8
+
38 V _

I1

I3
I2

10
_ +
40 V

. 2.5.

Kirchhoff 1, 2, 3,
I1 I2 + I3 = 0
8 I1 + 10 I2 = 38

10 I2 + 5 I3 = 40

(2.5.1)

4 , , , .
1, 2, 3, , .
1 20 , 40 , 120 , 20 . 2 40 , 60 , 40 60 . , 3 60 ,
60 , 40 , 40 .
400 , 500 , 520
380 ,
1, 2, 3,
.
II, x1,x2,x3
1, 2, 3
, .
x1 1 20 x1 , x2
2 40 x2 x3 3 60 x3 . , :
20x1+40x2+60x3

73

400 , ( )
20x1+40x2+60x3=400.

(2.5.2)

, 500, 520, 380 , , ,


40 x1 + 60 x2 + 60 x3 = 500
120 x1 + 40 x2 + 40 x3 = 520

(2.5.3)

20 x1 + 60 x2 + 40 x3 = 380.

(2.5.2), (2.5.3)
.
, a1x1+a2x2+a3x3+...+ax=, a1, a2, a3,...,
a, x1, x2, x3,..., x, ,
. - (1, 2, 3,..., ),
.
:
x1 = 1, x2 = 2, x3 = 3,..., x = .
, (2, 3, 4) 20 x1 + 40 x2 +
60x3=400, 202+403+604=400.
, x1, x2, x3, (2.5.2)
(2.5.3). x1, x2, x3 4
20 x1 + 40 x2 + 60 x3 = 400
40 x1 + 60 x2 + 60 x3 = 500
120 x1 + 40 x2 + 40 x3 = 520

(2.5.4)

20 x1 + 60 x2 + 40 x3 = 380.

43. , (2.5.1)
33 I1,I2,I3.
,
, :
11 x1 + 12 x2 + ... + 1 x = 1
21 x1 + 22 x2 + ... + 2 x = 2

..................................

1 x1 + 2 x2 + ... + x = .

(2.5.5)

74

ij,i=1,2,3,..., j = 1, 2, 3,...
, 1,2,...,
. aii (. 11,22,33 ...)
. - .
, .
(2.5.1)
I1 I2 + I3 = 0
8 I1 + 10 I2 + 0 I3 = 38
0 I1 + 10 I2 + 5 I3 = 40
(1,3,2).
, . ,
.
. , :
2x + y = 4

3 x y = 1
( )
()
3x + y=5

3 x + 2 y = 7

x + y = 1
, (1,2).
()().
,
. , . :


()

()


( )

1,2,..., ,
11 x1 + 12 x2 + ... + 1 x = 0
21 x1 + 22 x2 + ... + 2 x = 0
.........................
1 x1 + 2 x2 + ... + x = 0

75

.
- (0,0,0,,0) , . ,
( ), ,
.

. ,
, (2.5.4)

20 x1

40 x1

120 x1

20 x1

+ 40 x2 + 60 x3 400

+ 60 x2 + 60 x3 500
=
.
+ 40 x2 + 40 x3 520

+ 60 x2 + 40 x3 380

,
20
40
A=
120

20

40
60
40
60

60
60
,
40

40

x1
X = x2 ,
x3

400

500
B=

520

380

41 (2.5.6) ,
20
40
AX =
120

20

(2.5.6)

20 x1
40 60
x
1
60 60 40 x1
x2 =
40 40 120 x1
x3
60 40 20 x
1

20
40

120

20

40
60
40
60

+ 40 x2 + 60 x3

+ 60 x2 + 60 x3

+ 40 x2 + 40 x3

+ 60 x2 + 40 x3

400
60

x
1
60 500
x2 =
40 520
x3
40 380

= . , (2.5.1) =
I1
1 1 1
0

A = 8 10 0 , X = I2 , B = 38 .
I3
0 10 5
40
, (2.5.1)

(2.5.6)

76


=,

11 12 ... 1 x1 1

... 2 x2 2
21 22
= .

... ...


1 2 ... x

(2.5.7)

11 12 ... 1

22 ... 2
21

A=

1 2 ...


x1
x
2
X = ,
...

x

1

2
B=
...

() () .
, , .
. , , Gauss,

.
2.7 , Cramer, ,
, (.
).
, .
.

. ,
. (

) :
) .
,
, .
) . . , (
). ,
.

77

) .
. ( ).
, (), ,
, .
(2.5.1), , 1 3
I1 =

38 10 I2
,
8

I3 =

40 10 I2
5

'
38 10 I2
40 10 I2
I2 +
= 0.
8
5
2=3
I1 =

38 10 3
= 1,
8

I3 =

40 10 3
=2.
5

1=1, 2=3, 3=2.

2.5.1.


2 x1 x2 + x3 = 3
x1 x2 x3 = 1
.

x1 3 x2 + 2 x3 = 7 .

x1 (
, ),
x1=1+x2+x3

x2 + 3 x3 = 5
2(1 + x2 + x3 ) x2 + x3 = 3

2 x2 + 3 x3 = 8.
(1 + x2 + x3 ) 3 x2 + 2 x3 = 7
x2 x2=53x3
2(53x3)+3x3=8.
9x3=18 x3=2,

78

x2=53x3=532=1,
x1=1+x2+x3=1+(1)+2=0.
(x1,x2,x3)=(0,1,2).
2.5.2.


x1 x2 + x4 = 3
2 x1 + x2 x3 = 2
3 x2 + x3 x4 = 3

x1, x1=x2x4+3 ( x1)



2( x2 x4 + 3) + x2 x3 = 2
3 x2 x3 2 x4 = 4

3 x2 + x3 x4 = 3
3 x2 + x3 x4 = 3.
, x3, x3=3x22x4+4
3x2+(3x22x4+4)x4=3 6x23x4=1.
, . , x4=
R
3 x 1 3 1
=
x2 = 4
,
6
6
x3 = 3

3 1
21 3 7
2 + 4 =
=
,
6
6
2

x1 = x2 x4 + 3 =

3 1
17 3
+3=
.
6
6


17 3 3 1 7
( x1 , x2 , x3 , x4 ) =
,
,
, , R .
6
2
6

2.5.3.


x1 + x2 + x3 = 6
x1 x2 + 2 x3 = 5
x1 + 5 x2 2 x3 = 1
x1 + 2 x2 x3 = 10

79

x1, x1=x2x3+6

( x2 x3 + 6) x2 + 2 x3 = 5
( x2 x3 + 6) + 5 x2 2 x3 = 1

2 x2 + x3 = 1
4 x2 3 x3 = 5

( x2 x3 + 6) + 2 x2 x3 = 10

x2 2 x3 = 4.

, x3, x3=2x21

4 x2 3 (2 x2 1) = 5

x2 2 (2 x2 1) = 4

2 x2 = 8
3 x2 = 2.

x2=4, .
.
.

2.5.1.  1000 kg, ,


, , .
. .
,
, 2.5 2.5.
)  x,y,z ,
x,y,z.
)  , .
B

A
B

10 cm

120 cm

20 cm

.
2.5.

10 cm

B
50 cm

120 cm

20 cm

A
20 cm
30 cm
20 cm
30 cm

50 cm

. 2.5.

80

2.5.2. 
(. 2.5)
(1,0)
y = ax2 + x + .


(x,y)=(2,8), (5,20).

20
15
10
5

2.5.3.  (2.5.5)
.
1 1 x1 3
2
1 1 1 x = 1 ,

2
1 3 2 x3 7

1
1

10

. 2.5.

1 1
0
x1

1 1 0
x = ,
1 1 2 0
x
1 1 3 0

x1
1 1 1 2 2
2 1 1 1 x2 = 1 ,

x
3 1 1 0 3 4
x4

1
1

1
0

2 x1 0

=
2 x2 0


2
0

x1
x1
1 1 1 2
2 1 1 1 x2 = 2 x
2 .

x3
x3
3 1 1 0
x4

2.5.4.  AX=B,
, .
) x1  2 x2  3 x3

3 x1  5 x2  6 x3
x1  2 x2  8 x3

9

) x + y + z = 5

1

x y+z=2

3

x 3 = 0
x += 0

3 x + y + 3 z = 12

3 x 2 = 0
x y+ z =6

) x + 2 y + 3 z = 7

) 5 x1 + 2 x2 + 3 x3 = 0

) 2 x + 3 y + 4 + 5 z = 6

x + 4 y + 9z = 3

2 x1 + 3 x2 + 3 x3 = 2

x + y =1

x 4 y + 3z = 0

2 x1 + 5 x2 + 2 x3 = 3

y+=2

x + 7 y + 3z = 4

2 x1 + 3 x2 + 2 x3 = 1

y+ z =3
x+ y+z=4

2.5.5. .
) x + y + z = 5

) x y z 10

) x + 2 y + z = 7

) x + y + 3 z = 6

x y+z=2

x y 3

x + 3 y + 9 z = 13

x + 2y + z = 4

3 x + y + 3 z = 12

y5

x y+z=0

2x + y + z = 4

yz6

x+ y+z=4

x + y + 2z = 4
x+ y+ z =3

81

2.5.6.  120 m. 40 m ,
.
)  x,y,z ,
x,y,z.
)  , .
2.5.7.  , ,
, . , .
2 , 1 3 .
3 2 , , 4 4 . ,
90 , 30 70 , .
)
 x,y,z , , ,
x,y,z.
)  , , , .
2.5.8.  II , . 1 , 4 3 ,
2 , 3 1 .
15 , 35 20
,
)  x,y
II , ,
.
) 
x, y .

2.6 .

,
. , , '
. , ....
' ,
,
.
,
.
,
' :

82

1.  i j (
i j
i j)
2.
 i ( i)

(0)

i i

3.
 i ( i)

j '
(0).

i i+j

,
( ).
,
, , . ,

,
. , .
(2.5.4) II 2.5.
i i,
20,
( , ). ,

1
i i , i = 1, 2, 3, 4
20
:
x1 + 2 x2 + 3 x3 = 20

2 x1 + 3 x2 + 3 x3 = 25

(1)

6 x1 + 2 x2 + 2 x3 = 26

x1 + 3 x2 + 2 x3 = 19.

, 1 (1) 2
2 (1). 2 x1 .
,
:

2 2+(2) 1
x1 + 2 x2 + 3 x3 = 20

x2 3 x3 = 15

6 x1 + 2 x2 + 2 x3 = 26

x1 + 3 x2 + 2 x3 = 19.

(2)

, 1 (2) 6

83

3 [ 3 3+(6) 1]. 3
x1 :
x1 + 2 x2 + 3 x3 = 20

x2 3 x3 = 15


(3)

10 x2 16 x3 = 94

x1 + 3 x2 + 2 x3 = 19.

1 (3) 1
4 [ 4 4+(1) 1]. x1
4 , :

x1 + 2 x2 + 3 x3 = 20

x2 3 x3 = 15

10 x2 16 x3 = 94

x2 x3 = 1.

(4)

(4) x1 .
x2 .
3 3+(10) 2 :

x1 + 2 x2 + 3 x3 = 20
x2 3 x3 = 15
14 x3 = 56

(5)

x2 x3 = 1.

4 4+1 2 :
x1 + 2 x2 + 3 x3 = 20

x2 3 x3 = 15


(6)

14 x3 = 56

4 x3 = 16


' , x1

x2 .
5 4 + ( 144 )3 , :

x1 + 2 x2 + 3 x3 =

20

x2 3 x3 = 15
14 x3 = 56
0 x3 = 0.

(7)

84

, ( ), (, :
0x1+0x2+...+0x=0, ).
, (7) ,

x1 + 2 x2 + 3 x3 = 20

x2 3 x3 = 15
(8)

14 x3 = 56.


x3=4 x3=4
(8), x2=3. x2=3 x3=4
(8), x1=2.
(8) (2.5.3), (2.5.3) (2,3,4),
x1=2, x2=3 x3=4.
,
, :
1: , i j, ,
x1 . - .
2:  x1 , i i+j.
3:  ,
1 2 1 1 .
2, 3, 4, , 2 ,
.
, , :
) N , ( ,
i i),
) N 1 ( i j i i).
, (2.5.3) ( (4) ) :
2 (1)2 (4), x2 1,
:
x1 + 2 x2 + 3 x3 = 20

x2 + 3 x3 = 15


(5)

10
x
16
x
=
94

2
3

x2 x3 = 1.

85

' 3 3 +10 ^ 2 :
x1 + 2 x2 + 3 x3 = 20

x2 + 3 x3 = 15

14
x
=
56

x2 x3 = 1

(6)

x1 + 2 x2 + 3 x3 = 20

x2 + 3 x3 = 15

14 x3 = 56

4 x3 = 16.

(7)


, 4 4 + (1) 2, :

x3 1 33
( 141 )33, :
x1 + 2 x2 + 3 x3 = 20

x2 + 3 x3 = 15

x3 =
4

4 x3 = 16.

(8)

, 4 4 + 43 :
x1 + 2 x2 + 3 x3 = 20

x2 + 3 x3 = 15

x3 = 4

0x3 = 0.

(9)

2.6.1.

x1 x2 x4 3

2 x1 x2 x3 2 ( ,

3 x2 x3 x4 3
2.5.1 ).
.

x1 x2 x4 3

2 2 + (2) 1, 3 x2 x3 2 x4 4

3 x2 x3 x4 3.

86

1
x2 1, 2 2 ,
3

x1 x2 + x4 = 3

1
2
4

x2 x3 x4 =
3
3
3

3 x + x x = 3.
3
4
2
x1 x2 + x4 = 3

1
2
4

3 3 32 x2 x3 x4 = .
3
3
3

2 x3 + x4 = 7

x4 = , R 2 x3 = 7 x3 =

7
.
2

1 7 2
4 7 + 4 8 3 1
=
H x2 =
.
+ =
3 2 3
3
6
6
, x2,x3
3 1
18 + 3 1 6 17 3
=
=
.
6
6
6

x1 = 3 + x2 x4 = 3 +

17 3 3 1 7
( x1 , x2 , x3 , x4 ) =
,
,
, , R .
6
2
6

2.6.2.

x1 3 x2 + 2 x3 + x4 = 1

3 x1 8 x2 + 2 x3 + 5 x4 = 11

6 x1 5 x2 + 2 x3 4 x4 = 4

x2 4 x3 + 2 x4 = 5

2 2 31
x1 3 x2 + 2 x3 + x4 = 1

x2 4 x3 + 2 x4 = 8

6 x1 5 x2 + 2 x3 4 x4 = 4

x2 4 x3 + 2 x4 = 5.

87

3 3 +(6)1
x1 3 x2 + 2 x3 + x4

x2 4 x3 + 2 x4

13 x2 10 x3 10 x4

x2 4 x3 + 2 x4

=1
=8
= 10
= 5.

x2 1,
3 3 +( 13)2
x1 3 x2 + 2 x3 + x4

x2 4 x3 + 2 x4

42 x3 36 x4

x2 4 x3 + 2 x4

=1
=8
= 114
= 5.

4 4 + (1)2
x1 3 x2 + 2 x3 + x4 = 1

x2 4 x3 + 2 x4 = 8

42 x3 36 x4 = 114

0 x3 + 0 x4 = 3

x3, x4, .
,
0x1+0x2+...+0x= 0,
. .
.

2.6.1. .
x1 +2 x2

) x1 2 x2
2 x +4 x
2
1

x3
+ x3
2 x3

x 2y + z = 0

2 x y + 5 z = 3
)

4 x 5 y + 7 z = 3

3 x + y + 2 z = 1

+5 x4
3 x4
+6 x4

+6 x5
2 x5
+3 x5

= 1
= 2
= 3

2 x1 3 x2 + x3 = 5

) x1 + x2 x3 = 3

x1 3 x2 + 2 x3 = 7
x1 + x2 + x3 = 6

x1 x2 + 2 x3 = 5

) x1 + 5 x2 x3 = 1

x1 + 2 x2 + 3 x3 = 3

x1 + 4 x2 3 x3 = 2

88

2.6.2. .
y + 2z = 5

) x + z = 8

= 4
x + y

x1 x2 + x4 = 3

) 2 x1 + x2 x3 = 2

3 x2 + x3 x4 = 3

x3 x4 = 1

6 x3 + 5 x4 = 2

)

x1 3 x2 + 2 x3 + x4 = 12

x2 4 x3 + 2 x4 = 4

x2 4 x3 + 2 x4

x1 3 x2 + 2 x3 + x4

8 x2 + 2 x3 + 5 x4
)

6 x1 5 x2 + 2 x3 4 x4

x3 8 x4

=8
=1
= 11
=4
= 17

2.7 .
, . ,
(. 1.4.1)
= =1.

(2.7.1)

( ).
11 x1 + 12 x2 + ... + 1 x = 1
21 x1 + 22 x2 + ... + 2 x = 2
.....................................

()

1 x1 + 2 x2 + ... + x = .

2.5, ()
(. 2.5.5 =)
11 12 ... 1 x1 1

22 ... 2 x2 2
21
.
#
# % # # #


1 2 ... x
=,
11 12 ... 1

... 2
21 22

A=
,

1 2 ...

x1
x
2
X = ,
...

x

1

2
B=
...

, ()
() .
() . ,

89

(2.7.1),
=1.
A 0.
(2.4.2).
,
. ,
. ,
.

2.7.1.

3 x1 + x2 = 0

3 x1 + x2 = 1

) 2 x1 + 4 x2 + x3 = 0

) 2 x1 + 4 x2 + x3 = 1

x2 + 3 x3 = 1

x2 + 3 x3 = 1

3 x1 + x2 = 1
) 2 x1 + 4 x2 + x3 = 0
x2 + 3 x3 = 1.

) =,
3 1 0

A = 2 4 1 ,

0 1 3

B = 1 .

, 2.4.1,

11 / 27 3 / 27 1 / 27
1
6 / 27 9 / 27 3 / 27 
1
T
C
A

A
2 / 27 3 / 27 10 / 27

11 / 27 3 / 27 1 / 27 1 1 / 3

X = A1 B = 6 / 27 9 / 27 3 / 27 1 = 0 .
2 / 27 3 / 27 10 / 27 1 1 / 3

x1=1/3, x2=0, x3=1/3.
) =1,

0
B1 = 0 .

1

90

11 / 27 3 / 27 1 / 27 0 1 / 27
X = A B1 = 6 / 27 9 / 27 3 / 27 0 = 3 / 37 .
2 / 27 3 / 27 10 / 27 1 10 / 27
1

) AX=B2,
1
B2 = 0

0

11 / 27 3 / 27 1 / 27 1 11 / 27

X = A1 B2 = 6 / 27 9 / 27 3 / 27 0 = 6 / 27

2 / 27 3 / 27 10 / 27 0 2 / 27

A
x1, x2, x3
11 x1 + 12 x2 + 13 x3 = 1
21 x1 + 22 x2 + a23 x3 = 2
31 x1 + 32 x2 + a33 x3 = 3
=1,
a11 a12
A = a21 a22
a31 a32

a13
a23
a33

. (2.4.2), ,

c11 c21 c31 1


1
1
c
X = A1 B =
CT B =
c22 c32 2
12

A
A

c13 c23 c33


3

c11 1 + c21 2 + c31 3
x1
c
c 1
c

1 11 21 31 1
c
c
c

=
+
+

,
x2 =

12
22
32
2
12
1
22
2
32
3

A c

13 c23 c33 3
x3
c13 1 + c23 2 + c33 3
cij = (1)i + j mij = (1)i + j M ij ij i = 1, 2, 3
j = 1, 2, 3. ,
x1

c11 1  c21 2  c31 3


,
A

x2

c12 1  c22 2  c32 3


,
A

x3

c13 1  c23 2  c33 3


.
A

91

x1
c11 = (1)1+1 M22 =

22
32

23
,
33

1
x1 =

c21 = (1)2+1 M21 =

22
32

23
12
2
33
32

12
32

13
,
33

13
12
+ 3
33
22

c31 = (1)3+1 M31 =

13
23

12
22

13
23

2
3

13
23
33

12
22
32

1 ,

x1

1
2
3
11
21
31

12 13
22 23
32 33
12 13
22 23
32 33

Dx

D
1
Dxx11 = 2
3

a12
a22
a32

a13
a23
a33

D
x1 . x2, x3,
Dx
Dx
3
2
x2
, x3
D
D

DDx xx

2 22

1111 11
1313
2121 2 2 2323, ,
3131 3 3 3333

D
DDxxx

333

1111
1212 11
2121 2222 2 2, ,
3131 3232 3 3

D
x2 x3
.
(

92

). , . '
Cramer1.

Cramer.
= .
) 0, (x1,x2,...,x)
x1 =

Dx

x2 =

Dx

,..., x =

Dx

,
(2.7.2)
D
D
D

D Dxi , i = 1, 2, 3, ...,
D i xi .
) = 0, .
1

= 0,
= Gauss.
, 2.1, Cramer
2 2 (. 2.1.8).
Cramer, ,
' ,
. , , Gauss
Cramer. Cramer
.

2.7.2.


x + 2 y + 3z = 5

) x y z = 0

4 x + 5 y + 6 z = 11
.

x + 2 y + 3z = 1

) x y z = 0
4 x + 5 y + 6 z = 0

) D = 1 1 1 = 6 0
4 5 6

1.  C. MacLaurin 1729.
, Cramer, ,
5 , ' 5 5 .
(1750).

93

Dx = 0 1 1 = 6,
11 5 6

D y = 1 0 1 = 6, Dz = 1 1 0 = 12 .
4 11 6
4 5 11

Cramer,
D D y Dz
( x , y, z) = x ,
,
= (2, 3, 2) .
D D D
(2, 3, 2).
) , D=60.
, .

1 2 3
Dx = 0 1 1 = 1,
0 5 6

1 1 3
1 2 1
D y = 1 0 1 = 10, Dz = 1 1 0 = 9
4 0 6
4 5 0

Cramer,
D D y Dz
1
10 9
( x , y, z) = x ,
,
= ( , , ).
6
6 6
D D D
2.7.3.

2 x
+y
z = 2

2
z = 2
x + y

+ y + 2 z = 2.
x

2
D

1 1

1
1

1

2 ( 4  1)  1( 2  1)  1(1  2 )

2 ( 2  1)( 2  1)

2 (  1)(  1)( 2  1) 

1 2

D 0 2 ( 2  1)( 2  1) 0 2 (  1)(  1) 0 0 1 1. 



2
Dx

2 2
2

Dy

1 1
1

2( 4  1)  2( 2  1)  2(1  2 ) 2( 2  1)( 2  1)

1 2

2 2 1
1 2 1

2 2 ( 2  1)  2( 2  1)  2(1  1) 2( 2  1)( 2  1) 

1 2 2
2
Dz

2
2

1
1 1

2 2 ( 2  1)  2(1  1)  2(1  2 ) 2( 2  1)( 2  1).

2 2 1
1 2 1

Dy

94

1 2
2 2 ( 2  1)  2( 2  1)  2(1  1) 2( 2  1)( 2  1) 

1 2 2
2

2
2

1
1 1

Dz

2 2 ( 2  1)  2(1  1)  2(1  2 ) 2( 2  1)( 2  1).

D 0,1,1,
:
) 0 1 1 D 0
x=

Dy
Dx
2( 2 1)( 2 + 1)
2
2( 2 1)( 2 + 1)
2
y
= 2 2
=
=
=
= 2 ,
2
2 ,
2 2
2
D ( 1)( + 1)
D ( 1)( + 1)
z=

Dz
2( 2 1)( 2 + 1)
2
= 2 2
= 2.
2
D ( 1)( + 1)

1 1 1
2 , 2 , 2 .

y z = 2

z = 2 .
) = 0, x
x + y
= 2

y z = 2

x z = 2
y z = 4,

) = 1,
x + y z = 2
x + y z = 2
x = z
x + y z = 2



.
x + y z = 2
=
=
2
y
4
y
2
.

+
+
=
x
y
z
2

x + y + z = 2

, (k, 2, k), kR.


) = 1, ()
(k, 2, k), kR.

2.7.1. .
3 x1 + 2 x2 = 1

)
x1 + x2 = 0

3 x1 + 2 x2 = 0
)

x1 + x2 = 1

3 x1 + 2 x2 = 1
)
x1 + x2 = 1

95

=5
3 x1 + 2 x2

x2 2 x3 = 2
)

x2 + x3 = 3

=1
3 x1 + 2 x2

x2 2 x3 = 1
)

x2 + x3 = 1

x1 + 2 x2 x3 = 1

) x1 + x2 2 x3 = 2

2 x1 x2 + x3 = 3

x1 + 2 x2 x3 = 1

) x1 + x2 2 x3 = 3

2 x1 x2 + x3 = 2

x1 + 2 x2 x3 = 2

) x1 + x2 2 x3 = 1

2 x1 x2 + x3 = 3

x1 + 2 x2 x3 = 2

) x1 + x2 2 x3 = 3

2 x1 x2 + x3 = 1

x1 + 2 x2 x3 = 3

) x1 + x2 2 x3 = 2

2 x1 x2 + x3 = 1

x1 + 2 x2 x3 = 3

) x1 + x2 2 x3 = 1

2 x1 x2 + x3 = 2

1
4
1
2

1 3
2.7.2.  A = 1 1 2 A =

4
2 1 1
1

4
.

1
4
1

4
3

3

4

1
4
1

4

2.7.3. Cramer.
x+ y + z =3

) x + 2 y + 2 z = 1
4 x + 3 y + 4 z = 5

1
1

x + 6 y + 6 z =1

1
1
) x + 2 y + z = 1
6
6
1
1
6 x + 6 y + 3z = 1

x+ y + z =6

= 1
) x y
yz
= 1

x + 2y + 3 z = 6

) x 2 y 4 z = 1
7 x 10 y 21z = 0

x+ y+ z =3

) x + 2 y + 4 z = 7
x + 3 y + 9 z = 13

2.7.4.  Cramer
.
1
=
x+ y+z

) 2 x + 2 y + z = 2 + 1

= 1
x + y + z

x + 2 y + 2 z = 1

) 2 x + 2 y z = 0
2
=1
x + y + z

x + 2y z =

) 3 x y + 2 z = 0
7 x + 7 y 2 z =

96

x + y + z = 1

) x + y + z = 1
x + y + z = 1

x + 2 y + 2 z + 2 = 1
2 x + y + 2 z + 2 = 1

)
2 x + 2 y + z + 2 = 1
2 x + 2 y + 2 z + = 1

2.8 .
2.5,
, AX=O
11 x1 + 12 x2 + ... + 1 x = 0
21 x1 + 22 x2 + ... + 2 x = 0
.........................
1 x1 + 2 x2 + ... + x = 0.
- (0,0,0,,0) ,
.
, ,
.
( = ),
Cramer (2.7.2). , D = 0, Dx1 = Dx2 =

... = Dx = 0 ( ),
. D = = 0,
( ).
= ,
) 0
) ( ) = 0.

2.8.1.

R
Cramer.
x  y  2 z 0

x  2 y  z 0
2 x  y  z 0.

97

1
1

1 2
2 1 (2  1)  (1  2 )  2 (1  (2 )2 ) 8 3  6  2 

1
1.  ,
2
1
1 ( 2
1
) = = 1 .
2
1
= x+y+z=0 x=yz
2
(k m, k, m) k, m R .

D 0 8 3  6  2 0 2(2  1)2 (  1) 0

=1
x + y 2z = 0 x + y 2z = 0
x + y 2z = 0

y = z

x 2 y + z = 0 3 y + 3z = 0 3 y + 3z = 0
x = z ,
2 x + y + z = 0

y
z
z
3
3
0
0
0

=
=

(k,k,k) kR.

2.8.1.  ,
.
( 2 3) x
+y
+z = 0

x +( 2 3) y
+z = 0
)

x
+ y +( 2 3) z = 0

x +y + z = 0

) x + y +2 z = 0
2
x + y +4 z = 0

x +
y + z = 0

) ( + 1) x + ( + 1) y + z = 0
( + 1) x +
2 y +2 z = 0

x + y + z = 0

) (1 / ) x +3 y +2 z = 0

2
(1 / ) x +9 y +4 z = 0

2.8.2.  .

98

x  y 

x 2 y 
x  y 

0
0
0

0
0

x  y 

x  y 
x  y 

x  y 

x  y 
3 x 3 y 

1
1 
1

2.8.3. R Cramer.
x + y +z = 0
2
) x + 4 y + z = 0
4
x +16 y + z = 0

x + y +z = 0
y +
= 0
( + 1) x +

= 0 )
( 1) y
= 0
) 2 x + y

2
y +( 1) = 0
= 0

4 x + y

2.8.4. 
Cramer.
x
+y + z

+ y +3 z
x
2
2
x + y +9 z

,R
=
=

0
0

= 0.

2.9 .

a11 a12
A=

a21 a22 .
.

A=

a11 a12
= a11a22 a21a12
a21 a22


( ) ( ).

= 0

=, = , =

= , k = 2, 3,...
A1 =

1
( 0)
A

1 T
A1 =
C C = [cij], cij = (1)i+j Mij
A
0.
ij.

99

11 x1 + 12 x2 + ... + 1 x = 1
21 x1 + 22 x2 + ... + 2 x = 2
..................................
1 x1 + 2 x2 + ... + x =

, =
.
11 12 ... 1
1
x1

22 ... 2
x2
21
2

A=
, X=
, B=

1 2 ...
x

:

( ).

Cramer ( ).

= .

x1

Dx

x2

Dx

,..., x

Dx

,
, .

2.10 .
,
, .

1.

11 12
A =
, = 11 12 21 22.
21 22

2.

3.

1,

.

4.

5.

1,

.

6.

, .

7.

3 = , = 1.

100

8.

' : i

.
.

9.

' : i
j.
.

. 10. -
= , .
x y = 0
.

11.
x + y = 0

12. , .

13. - (0,0,,0) .

14. .

1.

0
2 0

0 :
A = 3 1
2 0 1 / 2
) = 0

2.

3.

) = 1

) = 6

) = 1/2

) 2 = 12

) 2 =

22 , :
) 2 = 4

) 2 = 2

, = 3, = 4
) =12

) = 34

) = 43

) = 7

( )
, :
4.
) += 0

) =

) =

) B =

1
A

101

5.


)
( ) , :
) += 0

) =

) =

) B =

1
A

x, y, z ;
6.

7.

) x 3 y + 2 z = 1

9.

) x 3 y + 2 z = 0 )

( );
) = 2

8.

) x y 3 + 2 z = 1

) =

) =

x + y = 1
,

+
=

x
y
1

) = 0
) = 1
) = 1

) .

) 0, 1, 1

;
) 1 2+53

) 1 1+52

) 152

) 1(1/5) 2

= 0 CT
, :
) A CT = I.
1 T
1
10. ) A = A C .
1
.
) A1 =
A
) .
11.

T = , :
) =
) =
) =

) =

= . = 0, :
) .
12. ) .
) .
) .

102

4 4 :
) 4 .
13. ) .
) .
) .

2.11 .
2.11.1.  4x3y+23=0, 3x2y+5=0, 4x2y+1=0,
R.
)  ,
.
)

.
2.11.2.  0
A4=A.
2.11.3. 
AT= A .
2.11.4.  0
ATA3AT=AAT.
2.11.5.  , P
B=P1AP.
) =P1().
) .
2.11.6.  ,
= 0. .
) 
=P 1AP .
) .
1
2 2 1
1 3 1 ,


1 1 2

0
2

0
0
3

0
0
,
0

2
0

0
4
0
0

0
0
6
0

0
0
.
0

2.11.7. .
0
0
) A =
0

0
0
4
8

0 1
2 3

5 6

9 10

0 0 0 0 x
0 0 0 y a

) A = 0 0 z k

0 w m b
u n c s

,
0
0
0
0

0
0
A=


0 a 1,2

a1 a 2

0
a1
a2, 1
a2
a2, 1
a3


a 1, 1 a 1,

a , 1
a

0
0
0

a 1,3
a 3

103

2.11.8. )
 ,
( )
a
a
a
a a + a
a
1

a
a
a + a2

a
a
a

a
a
a

a + a 1

a
a
a

a + a

(12 ... ).
) 5,
5
5

A 5

5
5

5
4
5
5
5

5
5
3
5
5

5
5
5
2
5

5 2
5 2
5 = 2

5 2
1 2

2
4
2
2
2

2
2
5
2
2

2
2
2
6
2

2
2
2 .

2
7

.
(: . ).
2.11.9.


x a a
a x a

a a x
A=

a a a

a a a

a a
a a
a a


x a

a x

=(x+(1)a) (xa)1.
2.11.10.

104

( x + a)2

x 2 + a2

ax

( y + )

y +

z +

w 2 + 2

( z + )

(w + )2

=0.

(: 2
).
2.11.11.  , , xOy (x1, y1), (x2,
y2), (x3, y3) . 2.3.14,

1 x1 y1
1 x2 y2 = 0.
1 x3 y3

x 2 + y2

x12 + y12

x1

y1 1

x22 + y22

x2

y2 1

x32 + y32

x3

y3 1

1
=0

.
)  (x1, y1), (x2, y2),
(x3, y3).
) (1,1), (3,4) (4,2).

.
.
, fungor, , , Leibniz 1673. 1755, Euler
, ,
: ,
, .
, .
x , x
, x.
,
.
, Bolzano
.

3.1  . .
3.2 , .
3.3 .
3.4  . .

3.5  ' x0R.


.
3.6  + . .

3.7 .

lx im

3.8  Bolzano .
3.9 .

3.10 .
3.11 .

x0

106

3.1 . .
. ,
, . ,
, ' . :
) (...)
. , x ... 19%,
... y = 0,19x.
) . ,
x,
y = x2.
, y, , x. ,
, x,
y. x.
, :
' (R), f, xA y y = f(x).
f
(domain) f D(f).
f, (range) f R(f),
R(f) = {f(x)|x D(f)}.
,
x, y,
. , , . ,
,
( ).
, , R, <, , (, ) = {x R| <x<}, [, ]={x R| x }, [, )={xR| x< }
(, ]={x R| < x }. , , .
, R,
(. 3.1):
(, + x R x }
[, + x R x }
(_ x R x < }
(_ x R x }
. 3.1.
.

107

R (,+). R,
(, +),
. , ,
1 .
f,
= D( f) x.
x , .
y, x = D( f), f
x f(x).
f :
f : AR

y=f(x), xA,
x y
f BD( f).

f ( B) = { y y = f ( x ) x B} = { f ( x ) x B}.
f f( A).
, f,
:
a)
) , f(x), x ( x
f(x)).
:
) A = [0, +), x f(x) = 0,19 x.
:
f : [0, ) R

y=0,19x, x[0,+).
x 0,19 x
) A= [0,+), x
f(x) =x2.
, f f(x). f
x (. R),
f(x) . , , :
f:[3,+)R f ( x ) = x 3
:

f f ( x ) = x 3

:
f ( x ) = x 3 y = x 3 .
, x30 x3
A=[3,+).
1. , ( ) R,
, ( , + ) .

108

3.1.1.

' .
, ( ) 33, 1
15.
)  f, o o
xm.
) 198 m;
.

) , xm ,
x

+ 1 15 .
33
o f x

x
15
y = 15 + 1 = x + 15
33 33

x
15
f ( x ) = 15 + 1 = x + 15 .
33 33

) x = 198 m :
198
y = f (198) = 15
+ 1 = 15 (6 + 1) = 105 ( ).
33

3.1.2.

(). , ,
1
x f f ( x ) = x 3 x 2 + 30 x .
9
)
 1 ..,
7 ..;
) 10 .. 1 ..;
.

) 7 x = 0
f(0) = 0 ( , , ).
1 . 137=6 ,
1
f (6) = 63 62 + 30 6 = 168
9
( , . 16.800 ).
) 10 .. 1 .. f(6)f(3) 10:00
3 1 6 .

1
1
f (6) f (3) = [ 6 3 6 2 + 30 6] [ 3 3 3 2 + 30 3] = 168 90 = 78
9
9
( , . 7.800 ).

109

3.1.3.

.
) f ( x ) = 9 x 2 16

) f ( x ) =

x2 5x + 6
x 1

) f x,
9 x 2 16 , xR 9x2160. :
9 x 2 16 0 9 x 2 16 x 2

16
4
4
4
x x x
9
3
3
3


4
4
A = (, ] [ , +) .
3
3
) H f , , , x25x+60 x10.
x25x+6 2 3, x25x+60 , , , x2 x3.
, f
A=(,1) (1,2][3,+).
3.1.4.

x , , f
f(x)=x43x2+100x+80.

) 3 .
) f(0).
.

) 3 f x = 3,
f(x)=34332+1003+80=434.
) x = 0
f(0)=04302+1000+80=80.
f(0)=80.
, .
, , ...
, , , x . , ' ,
,
( )
. , x
0, 1, 2, ( ) f
A=[(0,+).

110
y
1,2
1,1
1
0,9
0,8
0,7
0,6
0,5
0,4
0,3
0,2
0,1
_ 10

_8

_6

_4

_2

_ 0,1
_ 0,2

10

. 3.1.
.

f xy .
M(x, y) y = f(x), M(x,f(x)),
xA, f Cf (. 3.1)1.
f () y = f(x)
Cf . , y = f(x)
f.

' , , , ,
, x = a , x a ...
, f , xA y = f(x)R , f
x. , f,
3.1.
y
, 4 3.1 () () Cf
,

.
Cf
x
f,
O
A
Cf . , x0
. 3.1.

1.  Cf R2 ={(x,f(x))xD(f)] , f. , f. , , () Cf f .

111

f, x = x0
Cf (. 3.1).
, f f(A) Cf.
f, y =
Cf Cf (. 3.1).
, :
) Cf x (. Cf )
(. 3.1).
) Cf y (. Cf )
4
8

6
2
4
1
2
_1

_ 0,5

0,5

1,5

2
2

()

10

()
1

0,5

_1

0,5

_ 0,5

0,5

10

15

_ 0,5

_ 0,5

_1

_1

()

()

. 3.1.
y

x=x0

x0

y=

. 3.1.

Cf

. 3.1.

. 3.1.

112

y
10

8
6

f(A)

Cf

2
_4

. 3.1.

_3

_2

_1

. 3.1.

(. 3.1).
, f ( x ) = 9 x 2 16
3.1,
4
4
A = (, ] [ , +) .
3
3
( 3.1.3), =[0,+).
3.1.5.

3 x 1 x < 1
f f ( x ) =
6 x 7 x 1.
) f.
) x f(x)>5.
.

) f R, y=f(x) x. x<1
Cf y=3x1 3
(1,4) Cf . x1 Cf
y=6x7 6 (1, 1),
Cf.
3.1, .
10
8
6

4
2
_4

_3

_2

_1

1
_2
_4

. 3.1.

113

) y = 5 Cf A(x1, 5) B(x1, 5).


: 3x11=5 6x27=5, x1=2 x2=2. x
5 Cf y=5. , x(,2)(2,+).
3.1.6.
f f(x) = x2 + x + , ,, R.
) , (1, 2) Cf ;
) , Cf y'y (0, 3);
)  f
() ().
.

) (1, 2) f, f(x) = x2 + x + ,
f(1)=2, 1++=2 +=1.
) Cf y'y (0,3) f(0) =3,
= 3.
) ,

f f(x)=x22x+3.

+ = 1
= 2

= 3 = 3

3.1.1. f, g h
2 x 2 , x < 0
,
f ( x) =
,
x
x
0

x , x< 0
g( x) =
x , x 0.

) f(1), f(0), f(1) f.


) g(1), g(0), g(1), g(1/2), g(1/2) g. ;

3.1.2. f g f(x)=x2+2 g(x)=5x4.


) f g.
)
 f(1), f(0), f(1), f(1/2), f(1/2) f g(0), g(4/5),
g(4) g.
) x f(x)=g(x).
3.1.3. .
) f(x)=2x35x2+3
) f ( x ) =

1
1
1

+
+ 2
x x 1 x 4

) f ( x ) =

x2 3x + 7
( x 1)( x + 7)

) f ( x ) =

x4
x2 + 9

3.1.4. .
) f ( x ) =

3 x

) f ( x ) = x 2 9 + x 2 16

114

) f ( x ) =

x2 + 2

x2

) f ( x ) =

x2 3x + 2
x2

3.1.5.  t=0 .
( m), t sec, f,
1
f ( t) = (9t 2 + 144) .
4
) 3 sec;
) ;
) ;
) 27 m;
3.1.6.
 (oC) ,
(oF) : 32 5/9.
) f oF oC.
)
 oC 10, 0, 10 40 oF.
3.1.7.
 , 2 m 3 m
3.1. ,
x m , ,
.
)  V(x) x.
) V(1), V(2/10), V(3/10).
) ;

2m

3m

. 3.1.

3.1.8.
 3.1 .
3.1.9.

.
2 x 1
) f(x)=x29x+20
) f(x)=(3x1)5
) f ( x ) = 2
x +1
3.1.10.
 f, g f(x)=e2x+1 g(x)=3ex1. .
3.1.11. .
) f(x)=2x3

2x + 1, 0<x 2
) f ( x ) =
x>2
x ,

xd0
2 x,

) f ( x ) 5x , 0<x d 1 

x !1
2 x  3,

x 2 ,
) f ( x ) =
4 x 1,

x <3
x 3

=ae x

115
y

_3

_2

()
()

()
y

y
3
2

1
_1

_2

_1

()

x
()

()

. 3.1.
y

10

y
6

y= x

y=ax

y=ax

2
_2

_2

()

. 3.1.

()

3.1.12.
 ,
(. 3.1).

3.2 , .
4 x5 + 8 x
g(x)=4x,
x4 + 2
. , xR, :
f g f ( x ) =

f ( x) =

4 x 5 + 8 x 4 x( x 4 + 2)
=
= 4 x = g ( x ).
x4 + 2
x2 + 2

f g . :
f g
f(x)=g(x) xA. f=g.

116

y
f, g , ,
f(x)=g(x) (.
3.2). f g
.

x
O

(, , ,

). ,
. 3.2.
,
, . x
f f(x)=5x2+4x+1, x
g g(x)=5x+7. , x
:

f ( x ) + g ( x ) = (5 x 2 + 4 x + 1) + (5 x + 7) = 5 x 2 + 9 x + 8.
h , h(x) = f(x) +
g(x), xA. f g
f + g. .
, f g , :
) f+g f g : (f+g)(x)=f(x)+g(x), xA.
) fg f g : (fg)(x)=f(x)g(x), xA.
) cf c f : (cf)(x)=cf(x), xA.
) fg f g : (fg)(x)=f(x)g(x), xA.
f
f ( x)

( x) =
, x A f g : f ( x ) = f ( x ) , x A g(x)0.
)

g( x)
g( x)
g
g
. f f (
), f (x)=(f(x)).
f/g f
g g(x) , :
{xxA g(x)0}.
, , ,
.
3.2.1.

x ( )
10x+100 ,

117

x 15x .
)  f x
.
) g x .
) N f g .
)  x ,
x .
.

) , x (
) f(x)=10x+100.
) T x g(x)=15x.
) H f g
3.2.
500
) x
g

400
f
x
300
.
200
h=gf x.
:
100
h(x)=g(x)f(x)=15x(10x+100)=5x100
3.2. Ch
x (20, 0),
,
20 .
(20, 0)
3.2.
3.2
.

10

f ( x ), f ( x ) 0
g( x) =
f ( x ), f ( x ) < 0
) g=f+c ( c ),

30

40

. 3.2.
500
400
300
200

h=g _ f

100

3.2.2.


f, Cf .
:
) g=f,
g(x)=f(x).
) g=f, :

20

_ 100

10

20

30

40

_ 100

. 3.2.
500

400

300
200

h=g _ f

100
10
_ 100

20

. 3.2.

30

40

118

g(x)=f(x)+c.
) g=fc( c ),
g(x)=f(x)c.

y
Cf

) Cf Cf xx (.
3.2).
) Cf Cf , f(x)
0, Cf f(x)<0 (.
3.2).
) g g(x)=f(x)+c,
c>0 Cf c
(. 3.2).
) g g(x)=f(x)c,
c>0 Cf c
(. 3.2).
y

C_ f

. 3.2.

y
Cf

C f

. 3.2.
y
y

Cf+c

Cf

Cf

c
Cf_ c

. 3.2.

. 3.2.

. , f
3.2, A
B (. f , x
), B
(. , x )
. f
[, ] [, ]
[, ].

y
f()
f()

f()

O
f()

. 3.2.

119

, f:AR
( ). :
)  f , x1, x2
x1<x2 f(x1)<f(x2) (. 3.2).
)
 f , x1, x2
x1 < x2 f(x1) f(x2) (. 3.2).
)  f , x1, x2
x1<x2 f(x1)>f(x2) (. 3.2).
)  f , x1, x2
x1 < x2 f(x1) f(x2) (. 3.2).
, f(x)=x2 (0,+),
0x1<x2 x12 x22 , f(x1)<f(x2).
(,0], x1<x20 0x2<x1, 0 x22 x12 , f(x1)>f(x2).
f ' ,
. , f , .
1

0,8

f(x2)

0,6

f(x1)

0,4

x1

x2

0,2

0,5

1,5

. 3.2.

. 3.2.

f(x1)

f(x2)

2
1

x2
x1

. 3.2.

_2

_1

1
_1

. 3.2.

120


.
,

y
1

f(x)=x

f(x)=x, x[0,2]

3.2,
[0, 2 ] [ 32 , 2],

3/2

/2

_1

. 3.2.

[ 2 , 32 ]

.
3.2
:
) ()
f .
) ()
g .
f y
f(x)=y x .
:
f:AR 1, x1, x2A x1x2 f(x1) f(x2). , f
x1, x2A :
f(x1)=f(x2) x1= x2.
:
) f(x) = x+, 0 ,
f(x1)=f(x2), (. 3.2)
x1 x2 x1 x2 x1 x2 .
) f(x)=5x2 , f(x)=f(x)=5x2
x (.. f(2)=f(2)=20 22) (. 3.2).
f:AR , x1, x2 x1 x2, f(x1) f(x2). ,

(
).
, , .
f:AR x1, x2 A
x1 x2. x1<x2, x1>x2. , f , f(x1)< f(x2), f(x1) f(x2). , x1 > x2 f(x1)> f(x2),

1. 11.

121

f(x1) f(x2). , x1 x2, f(x1) f(x2) f


. .
, , , .
3.2.
1

0,5

2
_ 1,5

_1

_ 0,5

0,5

_3

1,5

_2

_1

_2

_ 0,5

_4
_6

_1

()

()

. 3.2.
y

f(x2)

f(x1)
f(x2)

f(x1)
x2

x1

y=x+, >0

x1

x2

y=x+, <0
. 3.2.

20

y 120
100

10

80
_2

60

_1
1

40
20
_5

_4

_3

_2

_1

_ 10
1

. 3.2.

. 3.2.

122

3.2.3.

.
x+2

) f(x) = x.
) f(x) = (x1) (x3)
) f ( x ) =
x +1
.
) f(1)=f(3)=0, 13. f .
) f A=R{1}. x1,x2A f(x1) = f(x2), :
f ( x1 ) = f ( x2 )

x1 + 2 x2 + 2
=
( x1 + 2)( x2 + 1) = ( x2 + 2)( x1 + 1)
x1 + 1 x2 + 1

x1 x2 + x1 + 2 x2 + 2 = x2 x1 + x2 + 2 x1 + 2 x2 = x1 .

) (x + 2) = x x. , f(0) = f (2) = 0,
0 2, f .
3.2.4.

.
x+2
, x 1

2
) f ( x ) = x + 1
) f(x) = 5x 7
0, x = 1

.

=

f ( x2 ) f ( x1 )
, x1x2.
x2 x1

f ( x2 ) f ( x1 )
>0
x2 x1

x1x2 ' , f .
,
f ( x2 ) f ( x1 )
=
<0
x2 x1
x1 x2, ' , f .
) x1,x2R x1 x2 :
f ( x2 ) f ( x1 ) = (5 x22 7) (5 x12 7) = 5 x22 5 x12

f ( x2 ) f ( x1 ) 5 x22 5 x12 5( x2 x1 )( x2 + x1 )
=
=
= 5( x2 + x1 ) .
x2 x1
x2 x1
x2 x1

123

:
 x10 x20 ( x1x2), =5(x2+x1)>0. f
1=[0,+).
 x10 x20 ( x1x2), =5(x2+x1)<0. f
2=(, 0].
) x1, x2, =R{1} x1x2 :
f ( x2 ) f ( x1 ) =

x2 + 2 x1 + 2 ( x2 + 2)( x1 + 1) ( x1 + 2)( x2 + 1)
x1 x2

=
==
x2 + 1 x1 + 1
( x2 + 1)( x1 + 1)
( x2 + 1)( x1 + 1)
=

f ( x2 ) f ( x1 )
1
=
.
x2 x1
( x1 + 1)( x2 + 1)

 x1>1 x2>1, (x1+1) (x2+1)>0, <0. f


1=(1,).
 x1<1 x2<1 (x1+1) (x2+1)>0, <0. f
2=(,1).
 x1>1 x2<1 ( ),
(x1+1) (x2+1)<0, >0. f
A=R.

. f(x)=x2, g(x)=x3 h(x)=x (
R),
:
) f(x)=(x)2 =x2=f(x), x.

y 25
20
15
10
5

) g(x)=(x)3 =x3 =g(x), x.


) h(x+2)=(x+2)=(x)=h(x), x.

, .
:

_5

_4

_3

_2

_1

. 3.2.

f(x)=x2.

)  f:AR , xA xA
f(x)=f(x).
)  f:AR , xA xA
f(x)=f(x).
)  f:AR ,
xA x+TA f(x+T)=f(x).
:
)
( y'y) (. 3.2).

124
y 120

100
80
60
40
20
_5

_4

_3

_2

_1_
20
_ 40
_ 60
_ 80
_ 100
_ 120

_ 10 _ 8

. 3.2.
f(x) = x3 .

_6 _4

1
0,8
0,6
0,4
0,2

_2
_ 0,2
_ 0,4
_ 0,6
_ 0,8
_1

10

. 3.2.
f(x) = x.

) (. 3.2).
) ( ) (. 3.2).

3.2.5.

.
) f(x) = x4+3x2+2 ) f ( x ) =

1
+ 3x
x

) f(x) = x2+2x.

) f = R. x A x A.
:
f ( x ) = ( x )4 + 3( x )2 + 2 = x 4 + 3 x 2 + 2 = f ( x )
f .
) f A=R*=R{0}. x A
x A . :
1
1

f ( x ) =
+ 3( x ) = + 3 x = f ( x )
x
x

f .
) f = R. x A x A.
:
2
2
f ( x ) = ( x )2 + 2( x ) = x 2 2 x , f ( x ) = ( x + 2 x ) = x 2 x
f(x)=f(x) x (.. f(1)=13=f(1)),
f(x)=f(x) x (.. f(1)=13=f(1)).
f .
.

3.2.1.
 f=g.

125

fg, R, f(x)=g(x).

) f ( x ) = x 2 , g ( x ) = ( x )2
x + 3 , x < 0
(
)
=
f
x
)
,

x0
x + 3 ,

) f ( x ) = x 2 , g ( x ) = x
g( x) =

x +3

) f ( x ) =

x
x
+ 3 , g( x) = + 3
x
x

3.2.2.
 f(x)=x2, g(x)=x+2. h=f+g

.
3.2.3. f, g
3, x <x 0< 0
5x5x 3,
f ( fx() x=) =
g ( x )
2 2
x x+ 2+ ,2 , x x 0 0

x0
2 x+3,


2
3 x  x  2 , x t 0

h=f+g.

3.2.4.

f
, 3f + 2 g .
f + g , f g , fg ,
g
1
) f ( x ) = x, g ( x ) =
) f ( x ) = x 3, g ( x ) = x + 3
x
1
) f(x)=x+2, g(x)=x2
) f ( x ) = x 2 1, g ( x ) =
x 1
3.2.5.  (. 3.2) f
g=f
g=f.
3
4

2,5
3
2
2,5

24
_2

_1

_2

_1

2
_2

1,5
2
1
1,5
0,5
1

_ 24

0,5

_4

10

10

4
5

42
_1
_1

2
_2

_1

_1

_2
4
_4
6
_6

5
_5
_
_ 105

. 3.2.

_ 10

126

300

300

200
300
200 f 300
3.2.6. 
g 3.2.

_ 0,4

_ 0,2

_ 0,4

_ 0,2

_ 0,4
_ 0,4

_ 0,2
_ 0,2

f
f
f
f

300
200
100
300
200
100
200
100
_ 100
100

f
f
f

0,2

_ 100
200
_ 100
__ 200
300
100
_ 200
__
300
200
_
300
_ 1
300

1
0,5
1
0,51

f
f
f

0,2

0,4

0,2
0,2

0,4
0,4

_5
_5

_ 0,2

_ 0,4

_ 0,2

_ 0,4
_ 0,4

_ 0,2
_ 0,2

5
5

_5
_5
_
5

_4

_2

_6

_4

_2

_6
_6

_4
_4

_2
_2

4
43

3.2.7.

4
32
4
3
21
3
2
1
2
_1
1
1
_2
1
_
_ 21
_
_ 12

_2

3
2
3
32
1
2
21

1
1
_1
_1
_1
_1

_6

_4

_2

2
2

4
4

6
6

_6
_6
_6

_4
_4
_4

_2
_2
_2

f
f
f
1

1,5

0,2

0,4

0,2

0,4

0,2
0,2

0,4
0,4

g
g
g
g

5
5
5
5

3
3
2
3
32
1
2
21
1
1
_1
_1
_1
_1

2
2

4
4

6
6

34

0,5

g
g
g
g

f
f
f2

g
g
g

0,81
1
0,61
0,8
0,8
0,6
0,4
0,8
0,6
0,4
0,2
0,6
0,4
0,2
0,4
0,2
0,2

_5

_1
_ 13

_ 100
200
_ 100
__ 200
300
100
_ 200
__
300
200
_
300
_ 1
300

_ 0,5
_
_ 0,51
_ 0,5
_1

_6

g
g
g

0,5
0,5
_ 0,5

_5

_ 0,4

0,4

300
200
100
300
200
100
200
100
_ 100
100

2,5

0,5

1,5

2,5

0,5
0,5

1
1

1,5
1,5

2
2

2,5
2,5

3
3 .

3.2.

4
23
4
3
12
3
2
1
2
_1
1
1
_
21

g
0,5

0,5

0,5
0,5

1
1

_
_1
_ 12
_
2
.
_2

g1,5
g
1,5
g
1,5
1,5

2,5

2,5

2
2

2,5
2,5

3
3


x+2
2
) f(x) = x
) f(x)=x5+x2+15
) f(x)=(x5)(x7)+1 ) f ( x ) =
x +1
3.2.8. 
.
2x + 3
) f(x)=8x3+x+1
) f ( x ) =
) f(x)=(x2)2 +4x
) f(x)=3x5
2x + 5

127

3.2.9. .
1
2
) f ( x ) = + 3 x
) f(x) = 3x3 + 2x2 x
) f(x) = x6 +3x4x+2x
x
x
) f ( x ) = + 1
) f(x)=x3.
) f(x) = 2x3 + 2xx
x
3.2.10.  (. 3.2) ,
; ;

__ 1
_ 11

__ 0,5
_ 0,5
0,5

__ 4
_ 44

2,5
2,5
2,5
22
2
1,5
1,5
1,5
0,5
0,5
0,5

__ 6
_ 66

11
1

__ 2
_ 22

__ 4
_ 44

__
2
_2
2

__
0,5
_ 0,5
__ 1
0,5
_ 11
__ 1,5
1,5
_ 1,5

0,6
0,6
0,6
0,4
0,4
0,4
0,2
0,2
0,2
__ 3
_ 33

__ 2
_ 22

__ 1
_ 11

__
2
_2
2

__
1
_1
1

__ 2
_2
2
__
4
_ 44

11
1

33
3

22
2

44
4
22
2
22
2

__ 2
_ 22

44
4

__ 1
_ 11

11
1

__
2
_ 22
_4
_4
_
4

22
2

0,4
0,4
0,4

44
4
22
2
__
3
_3
3

66
6

0,8
0,8
0,8

__ 2
_ 22
__ 4
_ 44
__ 6
_ 66

1,5
1,5
1,51
1
1
0,5
0,5
0,5

44
4

22
2
11
1

66
6
44
4
22
2

__ 2
_ 22

__ 4
_4
4

33
3

11
1
0,8
0,8
0,8
0,6
0,6
0,6
0,4
0,4
0,4
0,2
0,2
0,2

0,2
0,2
0,2
11
1

22
2

33
3

__ 3
_ 33

. 4..

__ 2
_ 22

__ 1
_ 11

__ 0,2
_ 0,2
0,2
__
0,4
_ 0,4
0,4

11
1

22
2

33
3

128

3.2.11. :
)
 f c, d
c > 0, cf + d .
)
 f , c < 0, cf + d .
3.2.12. :
)  f, g ( )
, f + g ( )
.
)  f ' g
, f g .
)
 f, g ' f(x) > 0
g(x) >0 x , fg .
)  f, g ' f(x) >0
g(x) >0 x , fg .
3.2.13. :
)  f:AR g:AR , f+g f g .
)  f:AR g:AR , f g
.
)
 f:AR g:AR , f g .

3.3 .

.
) f(x) = x + , 0.
A=R f(A)=R.

xx, yy A( , 0) B(0, ) . >0 f(x)

(. 3.3), <0 (. 3.3).


) f(x) = x2, 0. ( yy) A=R.

B(0, )

B(0, )

(_ ,0)

(_ ,0)

>0

>0

. 3.3.

B(0, )

B(0, )

(_ ,0)

<0

. 3.3.

<0

x
(_ ,0)

>0

. 3.3.

129

>0, f(x) (,0]


[0,+). f(A)=[0,+). f(x) = x2 >0 3.3.
< 0, f(x) (,0]
[0,+). f(A)= [,0). f(x) = x2 <0 3.3.
) f(x) = x3, 0. (
) A= R
f(A)= R.
>0, f(x) (. 3.3), <0
(. 3.3).

) f ( x )
, z0.
0  A =
x
= R* = R {0} f(A) = R* = R {0}.
. >0 f(x)
(,0) (0,+) (
), <0 (,0) (0,+)
( ). 3.3
f(x) = /x >0, 3.3 <0.
y

>0

<0

<0

. 3.3.

. 3.3.

<0

>0

. 3.3.

>0

>0

<0

<0

. 3.3.

. 3.3.

130

) : , . f(x) = x
= R f(A)=[1,1].
T = 2,
(. 3.3).
f(x) = x.
x

/2

3/2

f(x) = x A=R
f(A)=[1,1]. T = 2,
(. 3.3).
f(x) = x.
x

/2

3/2

f(x) = x R,

+ , Z f(A) = R. T = ,
2

( , + ), Z (. 3.3).
2
2
x

/4

/4

) a: f(x)= ax, 0<a 1. f(x)=ax,


0< a1 = R f(A)=(0,+). 0<a<1, f(x) (. 3.3), . a > 1 f(x)
(. 3.3) .
.
Euler e .
) a: f(x)=log a x, 0<a1.
f(x)=log a x, 0<a1 A=(0,+) f(A)=R. 0<a<1,
f(x) (. 3.3), a>1 (. 3.3). H Euler e
lnx .
, , :
) logx=y y=x

) log =1 log 1=0

) log(x1x2)= logx1+logx2

) logx=logx

) logx=x, x=exln logx=x




x
) log 1 = log x1 log x2
x2

)  >1, logx1< logx2 x1<x2,


0<<1, logx1< logx2 x1>x2.

131
y

_
2

3
2

_1

_
2

_1

. 3.3.

3
2

. 3.3.

y
y
_
2

3
2

y=log a x, 0<a<1

x
y=x, 0<<1

(0, 1)

(0, 1)

. 3.3.

y=x, >1
x

. 3.3.

. 3.3.

y
y=log a x, 0<a<1

y=log a x, a>1

. 3.3.

. 3.3.

3.3.1.

2 x + e 2, x < 1
x
f ( x) =
x
x 1.
e ,

a x, a>1 f.
) y=log

)  Cf x y.

) f
y=2x+(e2) (,1) -

132

y = e x [1,+ ). 3.3.
) Cf x y = 0
x. x 1 y = ex > 0, x y = 0
, x<1. 2x+e2=0,

2e
,
x=
2
,
2e
< 1.
2
Cf y x=0, f(0)=y
2e
, 0) (0,e2).
y=e2 . Cf A(
2
3.3.2.

f
f ( x) =

1 .
x 3

(. 3.3)
1
f1( x ) =
x

1
f2 ( x ) = = f1( x ) .
x
3.2.2, Cf Cf f(x)0 (.
), Cf, f(x)<0 (.
). , 3.2.2, Cf Cf
xx ( x<0).
(. 3.3)
f2 ( x ) =

1
.
x

20

15

10

y=
_1

. 3.3.

1
x

. 3.3.

y=

1
x

133
x

,
f ( x) =

1
= f2 ( x 3) ,
x 3

h , f2 ( . 3.3) ( . 3.3).
y

y=

1
x

y=

1
x

1
y= _
x 3

. 3.3.

. 3.3.

.
1

y=
3.3.1.

y= _ .
x
x 3

) f ( x ) =

x2 4

) f(x)=ex3
) f(x)=ln(x3)+2
) f(x)=lnx
x2 4
x ;

3.3.2. .
0<x e

2
2 x , x < 1
) f ( x ) =

x 3 + 1, x 1

ln x ,
) f ( x ) =
2
( x / e) ,

e x + 1,
x0

) f ( x ) = 3x + 2, 0<x 1

x >1
5,

xd0
x  1,

) f ( x ) x , 0<x d 

x!
1,

x>e

3.3.3.  f, g
.
) f(x)=2ex 3 g(x)=3ex+2
) f(x)=ex g(x)=ex
1
) f(x)=lnx g ( x ) = ln 2
) f(x)=2x g(x)=x2
x
3.3.4.
 f, g , f+g

134

.
5x ,

f ( x) = x 3 ,
3
x ,

x<0

x <1
x+3,
.
0 x < 2 g ( x ) =
3
x 3 , x 1
x2

3.4 . .
15 y + 1 , y
( ).
,
:
g ( y) = 15 y + 1 , y>0.

(3.4.1)


f(x)=20e0,01 x, x0, x 20e0,01x [ f(0)=20 ].
t,
: x y=20e0,01x (3.4.1),
:
z 15 y  1 15 20e 0,01 x  1 ,
x,
h( x ) 15 20e 0,01 x  1.

y=20e0,01x = f(x) ,

h( x ) 15 20e 0,01 x  1 15 y  1 
(3.4.1) h( x ) 15 20e 0,01 x  1 15 y  1

g ( y)

g ( f ( x )) .

3.4.
z = g(y) = g( f(x))
x

y = 20e 0,01x = f(x)

z = h(x) = 15 20e 0,01x + 1 = g(f(x))

H x g(f(x)) f g gf. , g(f(x))


gf, f(x) g.
(. 3.4):
f, g , . x
f(x), gf (gf)(x)=g(f(x)) f g.

135

, gf
, f, x f(x)
g,
A1 = { x A f ( x ) B}
(. 3.4). f(A)B= f g, f(A)B .

. , f, g, h h(gf),
(hg)f
h(gf)=(hg)f.
:

f, g h hgf ( hgf)(x)=h(g(f(x))).
.
f(A)

gf

f(x)
g

f
x

y = f(x)

z = g( f(x))

g(f(x))

gf

A1

. 3.4.

. 3.4.

3.4.1.

,
. f(t)=5+3t, t .
.
.

r,
E=g(r)=r2.
,
t
r=f(t)=5+3t.
, t
f g,

( g D f )( t)

g ( f ( t))

g (5  3t) (5  3t)2

25  30t  9 t 2.

136

3.4.2.

f g f(x)=x+1 g(x)=x2+1. gf
fg.
.

f g R, fg gf
R. fg :
( f g )( x ) = f ( g ( x )) = g ( x ) + 1 = ( x 2 + 1) + 1 = x 2 + 2 ,
gf :
( g f )( x ) = g ( f ( x )) = [ f ( x )]2 + 1 = ( x + 1)2 + 1 = x 2 + 2 x + 2 .
, fg gf ( R),
(fg)(x)=(gf)(x) x,
gf fg,
.
3.4.3.

f g f ( x ) = x 2 g(x)=x2+2. gf
fg.
.

f A=[2,+). g g(x)=x2+2
, =R. gf A1={xA:f(x)B}=[2,+)
= xA1 :
( g f )( x ) = g ( f ( x )) = g ( x 2 ) = ( x 2 )2 + 2 = ( x 2) + 2 = x .
H fg
A2={xB:g(x)A}= {xR:x2+20}= R

( fog)
g )( x ) = f ( g ( x )) = f ( x 2 + 2) = ( x 2 + 2) 2 = x 2 = x .
(f
gf fg ,
gf fg. :
(fg)(x)=(gf)(x)=x
x , .

( cm)
x ( sec) f(x)=3x+5.
6 min.
x (0x600), sec

137

cm
. 3.4.1 . , (x = 0)
5 cm , x=5sec 20 cm,
1805 cm .
. 305 cm ,
100 sec. ( , ),
sec
, ,
f, f1. f1(y) ( sec)
.
f1 3.4.1 ,
, 3.4.2.
f f1 , , , . x=20sec y=65cm :
f(20)=65 f1(65)=20.
f f1 .
. f
t , 0x600
f1. f y
, , 5x1805
f1.
f, f1
.
. (Alan Shepard, 1961), 186 km ,
. 15 min. 3.4 y ( km),
3.4.1

.

x
( sec)
0
1
2
5
10
20
30
100
500
600

f(x)
( cm)
5
8
11
20
35
65
95
305
1505
1805

3.4.2
X
.

y
( sec)
5
8
11
20
35
65
95
305
1505
1805

f 1(y)
( sec)
0
1
2
5
10
20
30
100
500
600

y(km)
186
160

x1

x2

. 3.4.

15

x(min)

138

x . f(x)
. , f
160 km.
:
160 km .
y x,
f .
, , ,
. , 3.2,
. :
f:Af(A) ,
:
f1:f(A)A,
y f (y f(A))
x, y=f(x). f1 f.
, f x y,
f1, , y x,
f(x)=y f1(y)=x.
,

f1(f(x))=x, xA, f(f1(y))=y, yf(A).
g(f(x))=x, f(g(y))=y f, g , f1=g g1=f.
, (x,y) f
(y,x) f1. C 1 (, ), =f() (,
f
).
Cf y = x (. 3.4).
:
y


(.
y = x).
, f(x)=ex.
(. . 3.3 a=e)
11 R (0,+).
f1 f. (. 3.4),

M(,)

Cf

y=x

M(,)
Cf _1

y=x

. 3.4.

139

(0,+), R y(0,+)
x ex=y. ex=y x=lny :
f1(y)=x=lny.
x
y, f:Af(A) f1(x),
xf(A) ( f1(y), yf(A) ). ,
f(x) =ex
f1(x)=lnx.
( 3.4 f, f1 ).
, : f(f1(y))=y, yf(A), f1(f(x))=x,
xA lnex=x, xR elnx=x, x(0,+).
20
2,5

15

10

1,5

f
y=x

f 1

1
_5
0,5
_2

_ 1,5

_1

_ 0,5

10

15

20

_5
0,5

_ 10

. 3.4.

. 3.4.

3.4.4.

f ( x ) = 2 x 1
.
.

3
2,5
2

f,
1
2x10, x . 2

1
f ( x ) = 2 x 1 A = [ , + ) .
2
,
x1,x2A f(x1)=f(x2)
2 x1 1 = 2 x2 1
x1=x2. ,
f(A)=[0,+) (. 3.4).

1,5
1
0,5
0

0,5

. 3.4.

f ( x) = 2 x 1 .

140

, f f1:f(A)A 1
yf(A)=[0,+) f1(y)=xA, f(x)=y. x
2
1 2
2
y = f ( x ) y = 2 x 1 y = 2 x 1 x = ( y + 1)
2
1
f 1( y) = x = ( y 2 + 1) ,
2

1 2
1
f ( y) = ( y + 1) , y[0,+).
2
, f
1
f1:[0,+)A f 1( x ) = ( x 2 + 1) .
2
3.4 f1,
3.4 f, f1 . , , f
f1 y=x.
8

3
2,5

y=x

2
1,5

f 1

1
2

0,5
0

0,5

1,5

2,5

. 3.4.

1
f ( x ) = ( x 2 + 1) .
2
1

3
0

. 3.4.
f, f1.

3.4.5.

)  f g ,
gf .
)  h(x)=5e2x3+2 .
.

) x1, x2 gf gf (x1)=(gf) (x2).


g(f(x1))=g(f(x2)) g
f(x1)=f(x2).

141

, f , x1=x2.
(gf) (x1)=(gf) (x2) x1= x2 gf .
) h(x)=5e2x3+2 f1(x)=2x3, f2(x)=ex
f3(x)=5x+2 ( 3.3
R). ( ) (), h .
h y=h(x) x.
h( x ) = y 5e 2 x 3 + 2 = y e 2 x 3 =

y2
y2
2 x 3 = ln
,
5
5

1 y2 3
+ , y>2.
y>2 x = ln
2
4
2
,

1 y2 3
f 1( y) = x = ln
+ , y>2
2
4
2

f
1 x2 3
+ , x>2.
f 1( x ) = ln
2
4
2

3.4.1.
 1 4 lt 100 km. lt
1,1 .
)  f .
)
 g o
.
)

.
3.4.2.
 , , 3 x + 2 x
. ()
f(t)=ae 0,05t, t0, f(t) t .
10.000 .
) a.
)  g
.
)  h
.

1. .

142

) h .

3.4.3. .
) f(x)=ln(e3x+2)

) f(x)=ln(x2+1)

) f(x)=ln(3x)

) f ( x )

) f ( x ) = ln( x 2 + 2)

) f ( x ) = ln x 2 + 2

( x 3  1)  2 

) f(x) = 32(5x)2
) f(x)=(x3+1)

3.4.4. ff fff :
) f(x)=x

) f(x)=2x

) f(x)=x/2

) f(x)=x+2

) f(x)=x2

) f ( x ) = x 2 x + 1 ,0x1

3.4.5. fg gf .
x ) f(x)=x
= x 2 , 2, g ( x ) = x 2
) f(x)=x2, g(x)=x3 f ()
) f(x)=2x2+3, g(x)=x+2
3.4.6. fg gf :
) f(x)=ex, g(x)=lnx

f ()
x )f(x)=x
= x 2 , 2 , g( x) = x

) f(x)=ex, g(x)=ln(1/x)

3.4.7.
 f , (fg)(x)=x22x+3, g
g(x)=x1.
3
2x +1
3.4.8. f ( x ) = 1 +
, g( x) =
.
x 1
x2
:
) f(f(x))=x x1

) g(g(x))=x x2.

3.4.9.
 f(x)=ax+1 g(x)=x+3 fg =
gf f(f(1))=1. .
3.4.10.
 C ( ) x ( )
f f(x)=3+5x. f1
f1(y).
3.4.11.

.
2e x + 2
2
) f(x)=5x+2
) f(x)=ln(1x )
) f ( x ) = x
3e + 1
) f(x)=x21

) f(x)=5ln(3e2x+1)

) f ( x ) = ln x 2 + 4

2x
, g(x)=ln(x2).
x 1
) f, g.
) h= gf.
)  f, g, h .
) h1=g1f1.

3.4.12. f, g f ( x ) =

3.4.13.
 (. 3.4) .
.

143

1
0,8

0,5

0,6
_1

_ 0,5

0,5

0,4

_ 0,5

0,2

_1

_1

_ 0,8

()

_ 0,6

_ 0,4

_ 0,2

()

1
0,8

0,5

0,6
_1

0,4

_ 0,5

0,2

0,4

0,6

0,8

_1

()

_ 0,5

0,2

0,5

0,2

0,4

0,6

()

0,8

_ 0,2

0,8

_ 0,4

0,6

_ 0,6

0,4

_ 0,8

0,2

_1

0,2

0,4

0,6

()

()

. 3.4.

0,8

144

3.4.14. (. 3.4) .

.
3

2,5

2,5

1,5

1,5

0,5

0,5

_1

0,5

1,5

2,5

3
2
_1

1
_3

_2

_1

_ 0,5

_1
_1

_2
_3

_3

_2

_1

_3

_2

_1

_1

_1

_2

_2

_3

_3
4

3
0,5
_ 1,5

_1

_ 0,5

2
0,5

1,5

_ 0,5

_1

_1

_2

. 3.4.

0,5

1,5

2,5

145

3.5 ' x0 R. .

. ,
, . ,

, .
:
2 + x 3 ,
x<0

f ( x ) = 0,
(3.5.1)

x =0

x
, x >0 ,

1 1 + x
3.5.
f x 0,
3.5.1.
2

1
_1

_2

_1
_2
_3
_4
_

. 3.5.
(4.5.1).

3.5.1
f.
x 0

x
0
0,3
1,973000

x
f(x)

0,2
1,992000

0,15
1,996630

0,1
1,999000

0,01
1,999999

x 0

0
x
x
f(x)

0
?

0,0001
2,0004998

0,001
2,0004999

0,01
2,0049876

0,1
2,0488088

0
?

146

3.5.1
:
) x, xx, 0 (x <0),
f(x) 2.
) x 0 (x>0), f(x)
2.
, x 1.
, (. 3.5) ( 3.5 )
3.5.2, x, xx, 1 , 1, f(x)
f 1. ,
( 1) x, f(x)
1.
2
1,5
1
0,5
_ 1,2

_1

_ 0,8

_ 0,6

_ 0,5
_1
_ 1,5

. 3.5.

3.5.2
f.

x 1 1 x 1
x

1,1

1,01

1,001

1,0001 1 0,9999 0,999

0,99

0,9

f(x)

0,6690

0,9697

0,9970

0,9997

1,0297

1,2710

1,0003

1,0030

f ,
x. , , ,
:
) f ' (,x0).
l1, x x0 (x<x0),
f x0 l1 1 (. 3.5)

1. lim limes .

147

lim f ( x ) A1 

xo x 
o

( : f(x), x
x0 , l1).
) f '
(x0, ). l2, x x0 (x> x0), f x0 l2 (. 3.5)

C
2
1

lim f ( x ) = 2

x x +

. 3.5.

( : f(x), x
x0 , l2).
) f,
(, x0) (x0, ).
l, x
x0 (
x0), f x x0 l
(. 3.5)
lim f ( x ) A 
xo x

x0

y
C

( : f(x), x
O
x0, l f(x) x0 l).
, f (3.5.1),

lim f ( x ) 2, lim f ( x ) 2, lim f ( x ) 1. 
x o0

x o0

x0

. 3.5.

x o1


f x0, ' (a, x0),
f x0,
' (x0 , ). , x0,
x0, ' (a, x0) (x0 , ).
, ,
f x0, .
:
f,
(, x0)(x0 , ). f x x0
, .
:
lim f(x)= l , , lim+ f(x)= lim f(x)= l.

x x0

x x

x x

f (a, x0),
(x0 , ), x x0
( x<x0). lim f(x) lim f(x). ,
x x
x x
0
0

148

f ' (x0 , ) (a, x0), x x0 ( x > x0).


, lim f(x) lim+ f(x).
x x0
x x
0

, lim f(x)=, x x0
:
) x0 [. 3.5() ()].
) x0 [. 3.5()].
) f x0, f(x0) x0,
[. 3.5()] [. 3.5()].
y

= f(x0)
f(x0)
O

x0

x0

()

()

x
()

. 3.5.

, f x0, ( ). , , (, x0)
(x0 , ), f
x0 ( )1.
(
x0) ,
.
,
f ( x)

2x  6

x 3

x0=1, (0,1)(1,2) ,
( x<3 2x6<0)
f ( x) =

(2 x 6)
= 2 .
x 3

, lim f ( x ) = 2 ( x 1,
x 1
2).

1.  , , f x0 (a, x0) (x0, ), f x (a, x0) (x0, ).

149
y

3.5.1.

y=x _ 2

f
x2 5x + 6
.
x 3
f ( )
x0 = 3.
f ( x) =

1
O

f = (,3)(3,+)
x 3
. 3.5.
( x 2)( x 3)
= x2 .
f ( x) =
x 3
f, x 3 3.5
. x xx (
) 3, 1.
x 3 3 x 3
2,9

2,99

2,999

2,9999 3

3,0001

3,001

3,01

3,1

f(x) 0,9

0,99

0,999

0,9999

1,0001

1,001

1,01

1,1

f
x0 = 3, .

.
.
L1. lim f ( x ) A lim ( f ( x )  A) 0 
xo x

xo x

L2. lim f ( x ) A lim f ( x0  h) A 


xo x

h o0

L3. lim f ( x ) A lim f ( x )


xo x

xo x

lim f ( x ) A 

xo x
0

L4. )  lim f ( x ) ! 0, f(x)>0 1


xo x

x0

x0 (. 3.5).

)  lim f ( x )  0 , f(x)<0 x0.


xo x

. 3.5.

1.  x0 x0 > 0 o ,
xD( f ) |xx0 | f(x)>0.

150

, x0
, , x0 .
L5.  f,g f(x)g(x) x0
x0, lim f ( x ) d lim g ( x ) .
xo x

xo x

l1, lim
L6.  f
f ( x )f =
d
g ( x )g ( x )=
limg(
f ( x )x
g ( x ))lim lim
( x )lim

0
xo x

xo x xo x

xo x xo x
0

= l2 l1, l2 R f+g x0
:
lim ( f ( x )  g ( x )) lim f ( x )  lim g ( x ) = l1 + l2
xo x

xo x

xo x

( ).

L7.  f x0 c ,
cf x0 :
lim (cf ( x )) c lim f ( x ) .

xo x

xo x

L8.  f
l1, lim
f ( x )f =
g ( x )g ( x ) =
limg (
f ( xx)0,
g ( x ))lim lim
(dx )lim
xo x

xo x xo x

xo xx o x
0

= l2 l1, l2 R f g x0 :
lim ( f ( x ) g ( x ))

xo x

lim f ( x ) lim g ( x ) 

xo x

xo x

( ).

l1, lim
L9.  f
f ( x )f =
d
g ( x )g( x ) =
limg(
f ( x )x0, g
( x ))lim lim
( x )lim
xo x

xo x xo x
0

xo x xo x
0

l2 l2 0, f/g x0
:
lim f ( x )
f ( x ) x o x0
l
 1 .
lim
x o x g( x)
lim g ( x ) l2
0

xo x

L10.  f x0, w f x0 :
lim f ( x ) .

lim f ( x )

xo x

xo x

L11.  f x0 f(x)0
x0, f x0 ( ) :

lim [ f ( x )] lim f ( x ) .
xo x
x o x

L12.  f, g, h. h(x)f(x)g(x) x0
lim h( x ) lim g ( x ) A ,
xo x

xo x

lim f ( x ) A .

xo x

151

(
,
f , x0,
h g
, x x0, ,
3.5, f .


.
x0

. 3.5

3.5.2.
lim f(x) = 1 lim g(x) = 7 :
x x0

x x0

lim [3 f ( x )  5 g ( x )] 
) xlim
x
x o x0
0

3 f ( x)  5 g( x)

x o x0 [ f ( x )  ( g ( x ))2 ]3
0

lim [ f ( x )  ( g ( x ))2 ]3 
) xlim
x

) xlim
lim
x

x o x0
0

) L6 L7
:

lim [3 f ( x)  5 g ( x)]

x o x0

3 lim f ( x)  5 lim g ( x)
x o x0

x o x0

3 (1)  5(7)

38.

) L6, L7 L11
:
lim [ f ( x ) + ( g ( x ))2 ]3 = [ lim [ f ( x ) + ( g ( x ))2 ]]3 = [ lim f ( x) + lim ( g ( x)) 2 ]3 =

x x0

x x0

x x0

x x0

= [ lim f ( x ) + ( lim g ( x ))2 ]3 = [ 1 + ( 7)2 ]3 = 483.


x x0

x x0

)
lim [3 f ( x ) + g ( x )] = 10,

x x0

lim [ f ( x ) + ( g ( x ))2 ]3 = 8 0

x x0

L9 :
lim (3 f ( x ) + 5 g ( x ))
x x0
3 f ( x) + 5 g( x)
38
lim
=
= 3.
2
3
2
3
x x0 [ f ( x ) + ( g ( x )) ]
lim [ f ( x ) + ( g ( x )) ]
48
x x0

L1L13, , . 3.3.

152

1. lim x = x0 ( f(x) = x, x
x x
 0
),
lim
lim x

x0  .

xx
oxx0

0
2.  f(x)=c, x , lim f ( x )
xo x

lim c c (

xo x

, x0).
3. lim x x00,
xxo

oxx0

00

4. lim

x 0

lim x x00 
lim

o
xxx
oxxx0

00

x
= 1 (. 3.5)
x

x 1
= 0 (. 3.5)
x 0
x

5. lim

P ( x ) n x n  n1 x n1  ...  1 x  0 .


:

lim
lim
lim
lim
lim(((nnnxxxnnn nnn111xxxnnn111"
"
"000))) lim
lim
lim(((nnnxxxnnn))) lim
lim(((nnn111xxxnnn111)))"
"
" lim
lim
lim 000
lim PPP(((xxx))) lim
lim

xxxo
o
xo
xxxx0

xxxo
o
oxxx0

000

xxxo
o
oxxx0

000

xxxo
o
oxxx0

000

1
nnnlim
lim
xxx xnnn
lim
xxxnnn111xn"
"
""
lim
lim
lim
000
lim
lim

lim
n111lim
n lim
nlim
1 lim
xxxo
xo
o
xxxxx o x
xxo
x
o
o
xx xx o x
o
xo
o

xxx
xx0xxx o x
0
0
nnn

000

000

000

xxxo
o
oxxx0

000

000

nnn
1
x0xnn0n0n1111x0n"
"
""
000 PP0P(((xx0x0P0))()x 0 ).
0nnnxx0x00nx0nnn
n11x
1

P(x) : lim P(x) = P(x0).


x x0

,
lim (5 x 3 2 x 2 + 3 x + 10) = 5(1)3 2(1)2 + 3(1) + 10 = 0 .

x 1

0,8
0,5

0,6
0,4
_5

0,2
_6

_4

_2

_ 0,2

_ 0,4

. 3.5.
x
.
f ( x )
x

5
_ 0,5
_1

. 3.5..
x  1 .
f ( x )

x

153


R( x ) =

P( x)
,
Q( x )

(x), Q(x) x ( x).


x0R Q(x0)0. :
lim P ( x )
P ( x00 )
P ( x ) x x0
.
=
=
lim R( x ) = lim
x x0
x x0 Q( x )
lim Q( x ) Q( x00 )
x x0
:
R( x ) =

P( x)

Q( x )

P ( x ) P ( x0 )
=
,. Q(x0)0.
x x0 Q( x )
Q( x0 )

lim R( x ) = lim

x x0

5 x 3 2 x 2 + 3 x + 13 5(1)3 2(1)2 + 3(1) + 13 3


lim
=
= = 1.
x 1
3
x2 + 2
(1)2 + 2

3.5.3.
.

) lim x 2 3 
x o0
x

) lim ( x 3 + 1)
x 1

) lim[( x  1)2 x 2  1 ] 
x o2

) x0

x5 5
x 1 x 2 + 1

) lim

x 5x + 4

x 4
x4
2

) lim

2 x x 2 + 12
x 2
x2

) lim

11 2 2 2 2
1
2 2 11
2 2

d dx x x x . ,
1 , x x 3 3 x x
3
xx
x 3x 3
x
1

x
lim2 x 2 3 x20, 
x o0
x
lim( x 2 ) = lim x 2 = 0 ,

x 0

x 0

,
1

lim x 2 3 0 .
x o0
x

) P(x)=x3+1
lim ( x 3 + 1) = lim P ( x ) = P (1) = 13 + 1 = 2 .

x 1

x 1

154

) R( x ) =

x 5 5 P( x)
Q(1)=12+1=20
=
2
x + 1 Q( x )

x5 5
15 5
4
lim
(
)
(
)
=
R
x
=
R
1
=
= 2.
2
x 1 x 2 + 1 x 1
3
1 +1

lim
)

lim[( x + 1)2 x 2 3 ] = lim( x + 1)2 lim x 2 3 = [ lim( x + 1)]2 lim( x 2 3) = 32 22 3 = 9


x 2

x 2

x 2

x 2

x 2

) lim( x 4) = 0, L9.
x 2

x=4 ,
f ( x) =

x2 5x + 4
, x4
x4

. , x4, :
x 2 5 x + 4 ( x 4)( x 1)
=
= x 1 .
x4
x4

f ( x) =

, lim f ( x ) = lim( x 1) = 3
x 4

x 4

) x=2 . :
2 x + x 2 + 12 :
f ( x) =

2 x x 2 + 12 (2 x x 2 + 12 )(2 x + x 2 + 12 )
(2 x )2 ( x 2 + 12 )2
=
=
x2
( x 2)(2 x + x 2 + 12 )
( x 2)(2 x + x 2 + 12 )

4 x 2 ( x 2 + 12)
( x 2)(2 x + x 2 + 12 )
3( x + 2)

3 x 2 12
( x 2)(2 x + x 2 + 12 )

3( x 2)( x + 2)
( x 2)(2 x + x 2 + 12 )

2 x + x 2 + 12

,
lim f ( x ) = lim
x 2

x 2

3( x + 2)
2 x + x + 12
2

3 lim( x + 2)
x 2

lim(2 x + x + 12 )
2

3 4
3
= .
4 + 16 2

x 2

3.5.4.

x 2 2, x<2

, , x0=2 f f ( x ) = 8
3 + 1, x 2.
x

x < 2 lim f ( x ) = lim ( x 2 2) = 22 2 = 2,


x 2

x 2

x > 2 lim f ( x ) = lim (


x 2+

x 2+

8
8
+ 1) = 3 + 1 = 2 .
3
2
x

155

lim f ( x ) = lim+ f ( x ) = 2 , lim f ( x ) = 2 .


x 2

x 2

x 2

3.5.5.
3
x 25, x<3
f f ( x ) =
x 3a , x 3.
) f x0 = 3
) f x0 = 3
)  , f
x0 = 3.

.
) x<3 lim f ( x )

lim ( x 3  25) 33  25 2. 

x o3

x o3

lim ( x  3a) 3  3a. 

) x>3 lim f ( x )

x o3

x o3

) f x0 = 3, :

lim f ( x ) 

lim f ( x )

x o3

1
, 33=2 = .
3

x o3

3.5.1.
 3.5
f. ( )
:
) lim  f ( x ) 
) lim  f ( x ), lim  f ( x ), lim f ( x ) 
x o4

x o2

) lim f ( x )

x o2

y
3
4

x o2

) lim f ( x ), lim f ( x ), lim f ( x ) 

x 0

x o2

x o2

x o2

_4

_2
2 3

) lim [ f ( x ) + ( g ( x )) ] = [ lim [ f ( x ) + ( g ( x )) ]] = [ lim f ( x) + lim ( g ( x)) ] =


2 3

x x0

x x0

x x0

3.5.2. f f ( x ) =
x

0,9

0,99

x x0

0,999

1,001

1,01

( )

lim f ( x ),

. 3.5.

3x 3
. :
2x 2

f(x)

x o1

lim f ( x )

x o1

lim f ( x ). 
x o1

1,1

156

3.5.3. N
 lim f ( x )  f(x0), , xo x

(. 3.5).

y
y
y

y
y
y

3
3
3

_2
_2
_2

y=f(x)
y=f(x)
y=f(x)
O
O
O

1
1
1

2
2
2

3
3
3

x0=2 x0=4
x0=2 x0=4
x0=4
x0=2()
()
()

y
y
y

y=__ 2
y=_ 2
y= 2

4
4
4

2
2
2

x
x
x

x0=0 x0=2
x0=0 x0=2
x0=2
x0=0 ()
()
()
y
y
y

y=f(x)
y=f(x)
y=f(x)

8
8
8

O
O
O

y=2
y=2
y=2

y=f(x)
y=f(x)
y=f(x)

10
10
10

4
4
4

5
5
5

O
O
O

4
4
4

8
8
8

x0=4 x0=8
x0=4 x0=8
x0=4 x0=8
()
()
()

. 3.5.

3.5.4.
 f f ( x ) =
:
x
f(x)

5,1

5,01

O
O
O

x
x
x

x
x
x
x0=5, x0=10 x0=15
x0=5, x0=10 x0=15
x0=5, x0=10 x0=15
()
()
()
5
5
5

10
10
10

15
15
15

x 2 + 3 x 10 .
x +5

5,001

4,999

4,99

4,9

( ) f(x) x0=5.

x  2, x  0

x =0 
3.5.5. f f ( x ) = 0 ,
2
x  2 , x ! 0.
)  3.5
f(x) x0=0.
)
 , .

_2

. 3.5.

157

3.5.6.  f, , lim f ( x ) , x0 .
xo x

) f ( x ) =

x 5x + 4
, x0=2
x2 4
4

) f ( x ) = 5 x +

) f ( x ) =

2
2 x ,
) f ( x ) =
2 x + 3,

( x )2
, x0=0
x

2 + x 3 ,
) f ( x ) =
8 2 x ,

x >3
x <3

x4 5x2 + 4
, x0=2
x2 4

, x0=3

x2
,

) f ( x ) = 4
1 + 1 ,
2 x

x0
, x0=0
x >0
x2
, x0 = 2
x>2

3.5.7.  f lim f ( x ) 4 . x0 .
xo x

1
) g ( x ) =
( f ( x ))2 + 1
) g ( x ) =

f ( x) 1

( f ( x ))3 61

) g(x)=5(f(x))24

) g ( x ) =

) g(x)=(f(x)3) (f(x)10)100

) g ( x ) =

3 f ( x) 1
( f ( x ))2 + 1
f ( x) 1
f ( x) 1

3.5.8. lim f ( x ) 1  lim g ( x ) 2  x0 .


xo x

xo x

) h( x ) =

f ( x) + g( x)

) h( x ) =

) h( x ) =

( f ( x )) + ( g ( x ))
2

f ( x) + g( x)

) h(x)=(2f(x)+g(x))10

 x,
2
3.5.9. f ( x ) x  1,
7  x ,

x0
0 d x d 3 . , , :
x ! 3.

) lim f ( x ) 
x o0

) lim f ( x ) 
x o0

) lim f ( x )

) lim f ( x ) 
x o3

) lim f ( x )

) lim f ( x )

x 3

x 0

x 1

) lim f ( x ) 
x o3

) lim f ( x )
x 4

3.5.10. N a x0=1
3 x 2 a, x < 1

f ( x) = 1
x 1.
3 ,
x

158

3.5.11. :
5
2
2
) lim(3 x 4 x + 2 x + 1)
) lim(2 x 3 x + 1)
x 4
x 1


5
2
3x 4 x + 2 x + 1
3x2  2 x 1
) lim
) lim

2
x 1
x +1
x o1
x 1


3.5.12. :
2 x

) lim
x o0 x (1  x )

x
x 0 x

3.5.13. :
) lim

4 x
4 x
3.5.14. :
) lim
x 4

2 x 1
x 0
x

x x
) lim

x 0
x

x2 x 2
x 1
x +1

) lim

) lim

) lim

x
) lim 3

x 0 x + 2 x

1 1 + x3
) lim
x 0
3x3

) lim

x4 1
x 1 x 3 1

x o3

( h 1)2 1
h 0
h2 + 1

) lim

x(x 1)
x 0
x2

) lim

x + x2
x 0
x

) lim

3x5 4 x 2 + 2 x + 1
x 1
x2 +1

) lim

) lim
x 1

x 1
x 5x + 4
2

x
x 0 1 1 + x

) lim

2 x2 + 3x + 1
x 1
1 x2

) lim

x x 1

x 3 2

( x + 2)2 4
x 0
x

) lim

3.6 + . .
f, x ,
( ) .
(+ ).
.
. ' 1 2.
1
0 C , . ,
x , , f
f ( x) =

2x2 +1
,
x2 +1

(3.6.1)

x0.
, ,
(x) .
,
. , x ,
y=f(x) 2.

159

x
x

10

20

f(x)

1,96

1,99

50

100

y=2

1,9999 1,9996 1,9999

3.6, f,
x ,
y =2 . ,
y =2
f +. : f +
2 : f + 2 :
lim f ( x) 2 .

2x2+1
x2 +1

f(x)=
1

. 3.6.

y=

(. 3.6):
f , x
, f
+ lim f ( x) .. -

f(x)

. 3.6.

y=
f +.

+,
(,+).
(3.6.1)
,
xR.
f , x,
A=R xR
2( x ) + 1 2 x + 1
f ( x ) =
= 2
= f ( x) .
x +1
( x )2 + 1
2

y
y=2
f(x)=

_3

_x

_2

2x2+1
x2 +1

_1

. 3.6.

, x 10, 20, 50, 100,.,



2 (. 3.6).
f
2 f 2 :
lim f ( x ) = 2 .
x
y=2
f .
(. 3.6):

y
O

f(x)
y=

. 3.6.

160

f ,
x ( ), f :
lim f ( x)=.

y= f .
,
(,).
f1, f2, f3, :
1
1
1
,
f1( x ) = , f2 ( x ) = 2 , f3 ( x ) =
x
x
x
3.6.
3.6.1
x , .
o:
1
1
1
= 0.
lim = 0, lim 2 = 0, lim
x + x
x + x
x+ x
:

1
0,
x x
lim

1
0 .
x x 2
lim

:
r , :
1
)  lim r =0
x+ x
1
) lim r =0, x r .
x x
1000

10

800

600

40

400

20

200

100
80
60

0,1

0,2

0,3

f1

0,4

0,5

0,1

0,2

0,3

f2

. 3.6.

0,4

0,5

0,1

0,2

0,3

f3

0,4

0,5

161

,
1
lim
0,
x of x 2 / 3

3.6.1

lim

x5

x of

0,

lim

x of

1
x1/ 2

f1

:
f4(x)=x2, f5(x)=x3,

f2

f6(x)= x


3.6.
,
f7(x)=x2,

x
1
x
1
x2
1
x

0.

f8(x)=x3,

f3

102

103

104

106

102 103 104 106


104 106 108 1012
10 103/2 102 103

f9(x)= x ,

3.6.
3.6 3.6.2, 3.6.3,
, x (x+):
) f4, f5, f6 ,
) f7, f8, f9 .
f4, f5, f6 + +, f7, f8, f9 + , :
30

7. _ 108
6. _ 108
5. _ 108
4. _ 108
3. _ 108
2. _ 108
1. _ 108

800.000
600.000
400.000
200.000
200

400

600

25
20
15
10
5

800

200

f4

400

600

800

200

f5

400

600

800

f6

. 3.6.
200

400

600

800

_ 200.000
_ 400.000
_ 600.000
_ 800.000

200
_ 1. _ 108
_ 2. _ 108
_ 3. _ 108
_ 4. _ 108
_ 5. _ 108
_ 6. _ 108
_ 7. _ 108

f7

400

600

200

800

400

600

_5
_ 10
_ 15
_ 20
_ 25
_ 30

f8

f9

. 3.6.

3.6.2

3.6.3

102

103

104

106

102

103

104

f4

x2

104

106

108

1012

f5

x3

106

109

1012

f6

10

103

102

106

f7

x2

104

106

108 1012

1018

f8

x3

106

109 1012 1018

103

f9

10 103 102 103

800

162

lim x 2=+,

lim  x = +
lim
x +
xof

lim x3=+

xof

xof

lim (x
 3)= lim
lim ( x ) = .
x +
xof
xof

lim (x
 2)=

xof

, 3.6.4, 3.6.5 3.6, , x (x):


) f4, f8 ,
) f5, f7 .
f4, f8 +,
f5, f7 , :
lim x 2=+, lim x 3=

xof

xof

lim (x
 2)=, lim (x
 3)=+,

xof

xof

.
3.5 +
.
x :

3.6.4

3.6.5

102

103

104

106

f4

x2

104

106

108

1012

f5

x3

106

109 1012 1018

_ 100

_ 80

_ 60

_ 100

_ 80

_ 60

f4

_ 40

_ 20

_ 40

_ 20

_ 80

_ 60

_ 40

_ 100

_ 80

_ 60

_ 40

f5
f5

102

103

104

f7

x2

104

106

108 1012

f8

x3

106

109

1012

10 000

1._ 106

108 000

800
1._ 000
106

68 000

600
800 000
000

46 000

400 000
000
600

24 000
000

200 000
000
400

2 000

f4
_ 100

_ 20
_
_ 20 200 000
__ 400 000
200 000
__ 600 000
400 000
__ 800 000
600 000

_ 80

_ 60

_ 40

_ 20 200 000

_ 100

_ 80

_ 60

_ 40

_ 20

_ 100

_ 80

_ 60

_ 40

_ 20

_ 100

_ 80

_ 60

_ 40

_ 20

f8
f8

_ 2 000
_ 4 000
2
__ 6 000
4 000
_ 8 000
6

__800
1._000
106
_ 1._ 106

_ 100

f7

. 3.6.

f7

_ _1 0 000
8 000
_ 1 0 000

106
1018

163

n , :
+ , n
lim x n =
x
, n .
+ 3.3
:
>1, (. 3.6) :
lim x 0 ,

lim x ,

lim log x

lim log x ,

x 0

0<<1, (. 3.6) :
lim x ,

lim x 0 ,

lim log x

lim log x ,
x 0

, = e > 1 :
x

lim e x 0,

a=1/e<1 :

lim e x , lim ln x ,

lim ln x ,

x 0

lim e x ,

lim e x 0 .

y
y=x
y=x

0<<1

y=log x
O

y=log x

>1

. 3.6.

. 3.6.

3.6.1.

)limlim
(5
x(5 x 7x7x3).
3)
)

)xlim
) )xlim
(lim
5 (x53 x(53 7x3x37
x73)x3) 3)
x

x
)
xx7
x37)x3) 3)
lim
(xlim
5
(x53 x(357
))xlim
x
x

6 x622 x22 42 4
) )limlim6 x36 x364x 4 4

xx362x
)xlim
lim
24
) )xlim
x3244 4
364
x

x6

x
x
)
lim
x x 4x3 4 4
) )limlim
3 3 x 4
x
x
x
x
4x7 7
6 x633x
x34x
) )limlim6 x63x633x3x7x7 7

3xx6333x
x36 x3x1x 1 7
)xlim
lim
) )xlim
x6

x
)
lim
3
x3 x33 xx33xx17x71 1
) )xlim
lim
x
x x1x1 1
x
x
3 x33 x33x
3 3

) )limlim
(5 x(533 x3373x7x3) 3)

)xlim
) )xlim
(lim
5 (x5 x(57xx37
x73)x3) 3)
x
x
x
)
xx7
x37)x3) 3)
) )
lim
lim
(xlim
5
(x53 x(357
)
x
x

3 x322 x22 62x6x9 9
) )limlim3 x33x336xx6
x69x9 9

6xx3263x
x23365xx23x65xx916x91 9
)xlim
lim
) )xlim
3
x6

x
)
lim
x6 x65xx35
x15x1 1
) )xlim
lim
x
xx5
x15x1 1
x
x
6 x633x3635
6 x63 x3 x3x7 7
))limlim6 x63x63xx7x7 7

3xx6333x
x36 x3x1x1 7
)xlim
lim
) )xlim
x6

x
)
lim
3
x3 x33 xx33xx17x71 1
))xlim
lim
x
x x1x1 1
x
x
3 x33 x33x

6 x62 x2 4 4
) )limlim6 x362 x362 4x2 4 4
2

xx362x

)xlim
lim
) )xlim
x32644x
4x344444
x6

x
x
lim
)
x
x

) )limlim
3 x3 4
x
x
x
x33xx
24
32 x22 642x6x9 9
))limlim3 x33x336xx6
x69x9 9

xlim

6xx3263x
x23365xx23x65xx916x91 9
lim
))x)
lim
)
3
x6

x
)
lim
x6 x65xx35
x15x1 1
))xlim
lim
x
xx5
x15x1 1
x
x
6 x63 x365

164

) x0 f(x)=5x3+7x3 :

7
3

f ( x ) 5 x 3 1 2 3 ,
5x
5x

7
3

lim 1 2 3 1 0 0 1
x
5x
5x

lim (5 x 3 )

lim f ( x) lim (5 x 3 ) .
x

7
3

) lim 1 2 3 1 0 0 1 , lim (5 x 3 )
x
x
5x
5x
lim f ( x) lim (5 x 3 ) .

.
P(x)

P ( x ) x n  x n1 "  ,

0,

lim P ( x ) lim ( x n ) 

x of

x of

lim P ( x ) lim ( x n ) .

x of

x of

P( x)

1
1
1
1
x n (   ...  n ) lim (   ...  n ) ,
x
x
x
x
x of

) x0 f ( x )

1
1
lim (   ...  n ) . 
x
x
x of

6x2 4

x3 4

4
6 x 2 1 2
2 1
6x 4
6x 6x 6x2 ,
f ( x) 3

x 4
x3 1 4
x 3 1 3
x3
x
2

4
lim 1 2
2
6 x x 6 x 1 0 1
lim
4
4 1 0
x

1 3
lim 1 3
x
x
x
1

6x2
.
x
x x 3
)

lim f ( x) 0 lim

6x2
6
lim 0
x x 3
x x
lim

165

4
lim 1 2
2
x

6 x 1 0 1
6x
lim
4
4 1 0
x

1 3
lim 1 3
x
x
x
1

6x2
6
lim 0
3
x x
x x
lim

6x2
lim f ( x) 0 lim 3 .
x
x x
3x2 6 x 9
) x0 f ( x ) 3

6x 5x 2
2 3
1 2
3x 2 6 x 9
3x 2
x x

3,
f ( x) 3
6 x 5x 1 1 5 1 6 x
6 x 2 6 x3

2 3
1 2
3x 2
1
x x
lim
1 lim
lim
0
3
5
1
x
x 6 x
x 2 x
1 2 3
6x
6x
3x2
.
x 6 x 3

lim f ( x ) 0 lim
x

2 3
2
3x3
1
x
x
lim
1 lim 3 lim
0
5
1
x
x 6 x
x 2 x
1 2 3
6x
6x
1

3x 2
0.
x 6 x 3

lim f ( x) lim
x

) x0 f ( x )

6 x3 x 7

3x3 x 1

1
7

1
7
6 x 3 1 3 3
3
3 1
3
x
6 x3 x 7
6
6x
6x
6x ,
f ( x) 3

6x
3
1
1
1
1

3 x x 1 3 x 3 1
3 3x 1 2 3

2
3x
3x
3x
3x

1
7
3
3
6x
6 x 1
lim
1
1
x
1 2 3
3x
3x
1

6 x3
lim 2 2
x 3 x 3
x
lim

6 x3
.
x 3 x 3

lim f ( x ) 2 lim
x

6 x3 .
x 3 x 3

) () lim f ( x ) 2 lim
x

166

.
:
R( x )

lim R( x )

x of

P( x )
x n x n1

,
Q( x ) x m x m1

lim x n m
x of

n<m lim R( x )
x of

0, 0 , n 0 , m 0

lim R( x )

x of

lim x n m . 
x of

lim R( x ) 0, n=m :

x of

lim R( x )

x of

lim R( x )

x of

.

, n > m + .

3.6.2.

, ( ). t
,
f(t)=7040e0,5t
(. 3.6). :
) ,
)
) .
.

) ( t=0)
f(0)=7040e0,50=7040=30.
) 6
f (6) 70 40e 0,56 70 40 e3 68 .
70
) t
(t).
60
lim ( f ) t = lim= (7040e0,5t)=70400=70,

t+

t+

()
f(t) 70.
y=70
f (. 3.6)
70 .

50
40
30
20
10
0

. 3.6.

10

167

3.6.3.

y = x + Cf f
+
lim [ f ( x )  ( x  )] 0 ,
x of

f ,

lim [ f ( x )  ( x  )] 0 1.

x of

f ( x)

x 4e x 3
.

20 2e x 3

x
1 Cf .
20
x
) y
2 Cf +.
20
.
x
) , y
1 20
Cf , :
) y

lim f ( x ) ( 1) 0 .
x
20

x
6e x
60

lim f ( x ) ( 1) lim

0.
x
x
20
x 2e 3 2 0 3
x
) , y
2 Cf +,
20

lim f ( x) ( 2) 0 .
x
20

6
lim f ( x) ( 2) lim x
0.
x
20
x 2e 3

3.6, x +, f

x
y
2,
20
x ,
f,

x
y
1 .
20

2
1,5
1
0,5
_6

_4

_2

_ 0,5

_1

. 3.6.

1.  , =0 . 0, y=x+
.

168

, f ,
x ,
( ) . f
, x ' x0.
x 1
, f f ( x )
,
x 1
x(,1)(1,+). 3.6.6 , x
1.
3.6.6
f.
x 1 1 x 1
x
f(x)

0,9
19

0,99
199

0,999
1999

0,9999
19999

1
?

1,0001
20001

1,001
2001

1,01
201

1,1
21

3.6.6,
3.6 : x 1,
f . ,
x, f(x) f
. , 3.6.6
:
) x 1 0,001 (. 1<x<1,001),
f(x) f 2001 [.
f(x)>2001].
) x 1 0,0001 (. 1<x<1, 0001),
f(x) f 20001 (.
f(x) > 20001) ... f 1 ,
+ :

lim f ( x ) f .

x o1

3.6.5 ,
x 1,
f

M(>0). f 1 , :
lim f ( x ) f .
x o1

3.6 x 1, x =1,
xx.
x = 1
f f.

y
M

y=1

x+1
f(x)= _
x 1

_1

x=1
_M

. 3.6.

169

xoR :
) lim ( x ) f 
xo x

) lim f ( x ) f 
xo x

lim (f x()x ) f
) lim
f 
x ox x
xo
0
0

lim ff ((xx)) f
) lim
) lim f ( x ) f 
) lim f ( x ) f 
f 
oxx 
xo x
xo x
xxo
0


0
00
3.6 ( x0 =1).
f ()(),
10

4
2
2
1
_ 0,5

x0

0,5

_2

_ 0,5

_4

) lim f ( x ) f 
xo x

2,5

1
_ 0,5

x0

0,5
_1

x0

0,5

_2

1,5

2,5

_2

_4

_3

_6
_8

_4

_ 10

_5

) lim f ( x ) f 
xo x

lim f ( x ) f 
) lim

x oxx
xo
0
0

10

8
_ 0,5

6
4

_2

x0
0,5

1,5

_4

_2

oxx
xxo
0

_ 0,5

1,5

) lim f ( x ) f 

_ 0,5

x0

0,5
_1

_6
0,5

x0

1,5

_8
_ 10

) lim f ( x ) f 

) lim f ( x ) f 
xo x

xo x

. 3.6.

170

x = x0 f
f.
()() x x0,
) ( ) f x0,
' (, x0),
) ( ) f x0,
(x0 , ),
) ( ) x0, x0, (, x0 ) (x0 , ).
( ) ,
, ( ) f x x0
, :
 .
) lim f ( x ) f  lim f ( x ) f  lim( fx() x ) ff
xo x

oxx
xxo
0

oxx
xxo
0

) lim f ( x ) f  lim ff((xx)) f


 .
f  lim( fx() x ) ff
xo x

oxx
xxo

oxx
xxo

00

00

:
M1 )  lim f ( x ) f , f(x) (f(x)>0) x0.
xo x

)
 lim f ( x ) f , f(x) (f(x)<0) x0.
xo x

M2 ) lim f ( x ) f , lim ( f ( x )) f .
xo x

xo x

) lim f ( x ) f , lim ( f ( x )) f .
xo x

xo x

M3 lim f ( x ) f  , lim
xo x

xo x

1
f ( x)

0 .

1
f.
xo x f ( x)
xo x
0
0
1
) lim f ( x ) 0  f(x)<0 x0, lim
f .
x
o
x
xo x
f
x
(
)
0
0

M4 ) lim f ( x ) 0  f(x)>0 x0, lim


M5 lim f ( x ) f  , lim f ( x )
xo x

xo x

M6 lim f ( x ) f , lim
xo x

xo x

f.

f ( x)

f ( ).

:
n , :
1
1
1
lim
= + , lim
lim+
lim
= + , lim
=
lim
2 n 1
2
n
2
n

1
x x ( x x )
xx
xx0 ( x x )
xx
xx0 ( x x0 )
0
0
0
0
0
+

171

1
1
1 1
1 1
= +
=2 +
= +
, , limlim
,limlim4 =4 +
, lim 3 f  lim 3 f ,
2
x o0 x
x o0 x
x x0
x x2
x0 x
( x2 (x2) 2)
1 .
0 f ( x ) = 3
x

, .
:
)
) x0R + .

f
1
1

g
2

f+g
1 + 2

+
+

0
0

+
+

+
+

f
1

g
2

+
+

0
0

1
1

2 0

1>0
1>0
+
0

+
2 >0
2 >0
0,(g(x)>0
0,(g(x)<0
+
0

f ^ g
1 2

+, 1 >0
, 1 <0

f
g
1/2

0
+

,
, f g. .

172

,
:
(+)+() 0()
. fg=f+(g),
:
(+)(+) ()().
f
1
, = f ,
g
g
0 rf
,
.
0 rf
, 1
1
f ( x ) = 2 + a ( ) g ( x ) = 2 ,
x
x
:
1
1

lim g ( x ) = lim 2 = + .
lim f ( x ) = lim 2 + a = ,
x

0
0
x 0
x 0 x
x

1
1
f ( x ) + g ( x ) = 2 + a + 2 = a lim( f ( x ) + g ( x )) = lim a = a
x 0
x 0
x
x
() .
:
1
1
) f ( x ) = 2 + 1 g ( x ) = 2 , :
x
x
lim f ( x ) = , lim g ( x ) = + lim( f ( x ) + g ( x )) = 1
x 0

x 0

) f ( x ) =

x 0

1
1
+ 2 g ( x ) = 2 , :
2
x
x

lim f ( x ) = , lim g ( x ) = + lim( f ( x ) + g ( x )) = 2


x 0

x 0

) f ( x ) =

x 0

1
1
g ( x ) = 2 , :
2
x
x

lim f ( x ) = , lim g ( x ) = + lim( f ( x ) + g ( x )) = 0 ..


x 0

x 0

x 0

3.6.4.
.

2x2 2x +1
x 5 x 4 10 x 3 + 25 x 2

) lim

( x 2) x + 3
+
x 1
x 1

) lim

f ( x) =

) lim
x o2

2x2 2x +1
x 4 10 x 3 + 25 x 2

2 x2  3x  1
2 x2  3x  1
)


lim
x o 2 ( x  2)3
( x  2)3

173


f ( x) =

2x2 2x +1
2x2 2x +1
2x2 2x +1 1
.
=
=
x 4 10 x 3 + 25 x 2 x 2 ( x 2 10 x + 25)
x2
( x 5)2


2 x 2 2 x + 1 2 52 2 5 + 1 41
=
=
> 0,
x 5
25
x2
52

lim

lim( x 5)2 = 0, ( x 5)2 > 0 ,


x 5

lim f ( x ) = +.
x 5

) lim ( x  2) x  3
x o1

lim

x 1+

1
= + ( x>1 (x1)3>0),
3
+
x 1 ( x 1)

2  lim

( x 2) x + 3
= .
x 1

) lim( x 2) = 0 x2>0 x>2, lim


x o2

x 2

lim (2 x 2  3 x  1)
x o2

lim
x o2

2 x2  3x  1
( x  2)3

1
( x  2)3

f .

lim(2 x 2  3 x  1) 3 , x o2

2


f .
lim
(
2
x
3
x
1
)

x o2 ( x  2)3

) lim( x 2) = 0 x2<0 x<2,


x 2

lim

x o2

1
( x  2)3

lim (2 x 2  3 x  1)

x o 2

f .

lim(2 x 2  3 x  1) 3,
x o2

:
lim

x o2

2 x2  3x  1
( x  2)3

lim
(2 x 2  3 x  1) f .
3
x o 2 ( x  2)

3.6.5.

5000 ,
20 .
) (x) x .
) :
K ( x)
.
K ( x) =
x
lim K ( x ) lim K ( x ). ;
x +

x o0

174

) K(x).
.

) (x) x K(x)=20x+5000, x0.


)
K ( x ) 20 x + 5000
5000
K ( x) =
=
= 20 +
x
x
x
5000
5000
500
= 20 .
) = lim 20 + lim
= 0 , : lim K ( x ) = lim ( 20 +
lim
x +
x +
x +
x + x
x
x + x
y = 20 (x) +.
, ,
, .
0 :
400
5000
lim K ( x ) lim (20 
) f .
x o0
x o0
x
x = 0 300

(x). , , - 300
.
) y = 20 x = 0 100


100
300
300
400
500

. 3.6.
3.6.
.

3.6.1. :
2
1 4
5
2

) lim 3 + 5
) lim 2 3 + 4
x +
x
x x
x
x
3
) lim ( x + 2 x 7)
x

x 4 3x3 + x 4
x +
x3 3x + 4

) lim

3
) lim ( x + 2 x 7)
x +

2 x 3
x x 4 + 3

2 x3
x + x 2 + 5

) lim

2 x3 + x 1
x + 3 x 3 2 x 2 + 2

) lim

) lim

) lim

x 3 27
x x 3 + 27

) lim ( x 2 + 9 x )
x +

3.6.2. + .
1
3x2 + 1
1 2x
(
)
f
x
=
)
) lim
) f ( x ) =
2
x + 2 x + 3
2+ x
3 + 5x

175

3.6.3.

. 1000 x
1200
, P ( x ) = 600 +
.
x +3
) 6 .
) 800 .
) ;
3.6.4.
 x
, ( ) :
2000 x 2
1000 x 4
S2 ( x ) = 4
S1( x ) = 2
2 x + 200
2 x + 400
.
) 
5000 ;
)  ,
;
3.6.5. :
) lim e

2 x +3

) lim

x of

x2
e x 1 

) lim ln( x 2 + 2)
x +

2
3
) lim
) lim
x of
x 3
x+ 2

3.6.6. f, 3.6.
f ( x ) lim + f(x), lim f(x), lim + f(x),
) N
 ( ) lim+ f(x), lim f(x), lim f(x),
x 1
x 1
x 1
x 1
x 1
x 1
lim f(x), lim f(x).
) lim e 2 x

3 x

x+

)
 , , + , f.
3.6.7. f, 3.6.
f ( x ) lim + f(x), lim f(x), lim f(x),
f ( x)
) N ( ) lim+ f(x), lim f(x), lim f(x),
x 2
x 2
x 2
x 2
x 2
x 2
lim f(x), lim f(x).
x

x+

4
3
5

2
1
_4

_2

_1

_4

_2

_1
_2

_5

_3

. 3.6.

. 3.6.

176

)  , , + , f.
3.6.8.  10.000 ,
200 .
)
 K(x) ' x
.
)
 ;
) K(x).

3.6.9. :
3
6

) lim
) lim

x o1
x o2 2 x  4
1 x
1
x2

) lim 2
) lim 

11
6
x
o
x o2 x  4
x

(
6
)

) lim ln(x 2 )
x 0

) lim

1
+ 3)40
1

) lim 
x o6 ( x  6)11

) lim

x 3 ( x

1

ln x

x o0

3.6.10. 
x=x0.
1
1 3x2
1 3x
1
, x0 = 2 ) f ( x ) =
) f ( x ) =
) f ( x ) =
, x0 = 1, x0 = 1
, x0 =
4
2+ x
3 + 6x
2
1 x

3.6.11. ( ) f x0 .
) f ( x ) =

x +3
, x0=0
x + 3x2
3

) f ( x ) =

1
, x0=0
x

) f ( x ) =

x x
, x0=0
xx

3.7 .
f .
, , . , ,
. .
,

.
, 3.7.
() () xo
, () . ,
() x0, ()
() xo.
, f ' x0
, (x0, f(x0)). ,
f .

177
y

f(x0)

f(x0)
f(x0)

x0

x0

()

x0

()

()

. 3.7.

'
, f,
1
x + 2 , x ( , 2) ( 2, 2) ( 2, 4) ( 4, 6)

f ( x ) = 1, x = 4

1, x [6, +)

f 3.7 () ,
2, 2, 4, 6.
f , :
x0
4
2

lim f(x)

x x0

1/2

f(x0)

1/2

y
1

1/2

1/2

1/4

1/6

_6

_4

_2

_1
2
_1

. 3.7.

:
) 2, 2 Cf , f
.
) x0 = 4, Cf , f lim f ( x ) f (4) .
) x0 = 6, Cf , f. x 4
, ,
4, 0, 8,
lim f(x)=f(x0).
x x0

, -

178

. :
f x0 ,
lim f(x)
x x0
lim f(x) = f(x0)1.
x x0

, f f(x)=x32 x0=1,
lim f ( x ) = lim( x 3 2) = 13 2 = 1 = f (1) .
x 1

x 1

, f x0
, 2:
) lim f(x).
x x0

) lim f(x) lim f(x) = f(x0).


x x0

x x0

:
x 2  1,
f ( x)
3  2 x ,

x d1
x !1

x2 1
,

g( x) x  1
0,

x z1
x 1

1,
lim f ( x ) = lim( x 2 1) = 0,

x 1

x 1

lim f ( x ) = lim(3 2 x ) = 1

x 1+

x 1

( f 1)
( x 1)( x + 1)
= lim( x + 1) = 2 , g(1)=0
x 1
x 1
x 1

lim g ( x ) = lim
x 1

[ g 1, lim g ( x ) = g (1) ].
x 1

, ,

.
f g x0,
f+g, f ( R), fg,

f ( g(x )0)
f,
0
g

f ( f(x0)0)

x0.

1.  f x0 ( ) :
> 0 > 0 , x |x x0|< |f (x) f (x0)|<.
2.  x0 ( ). x0 , (x0 , x0+ ), >0, , (x0 , x0+ ) D(f) = {x0}, f
x0. , , .

179

, f g x0 :
lim f(x)=f(x0)

x x0

lim g(x)=g(x0).

x x0

, L6 (. . 3.5), f g x0, f + g x0
lim (f(x)+g(x))= lim f(x)+ lim g(x)=f(x0)+g(x0)=(f+g)(x0),

x x0

x x0

x x0

f + g x0.
.
,
, :
f x0 g f(x0), gf x0.
' , .

( ). :

f :
)
 =(,), ,
x0 .
)
 =[,], ,
x0 (, )
lim f ( x ) = f (a) lim f ( x )

x a+

xo 

f ( )

f .
, f f(x)=x32 ,
x0R
lim (x32)=x032=f(x0).
x x0

x 2 ,
f ( x) =
0,

0 x <1
x =1

[0,1], (
lim f ( x ) = 1 0 = f (1)).
x 1

,
,
. :

180

f g ' = (, ),
,
f+g,
cf ( cR),
fg,
f ( g(x)0),
g
.

f,

f ( f(x)0),

:
f = (, ), g f(), gf .

. :
) , xR (. . 3.5)
lim P(x) = P(x0).

x x0

) , xR Q(x0)0 (. . 3.5)
lim PP((xx))
P ( x0 )
PP((xx)) x x0
lim
==
=
lim
.
xx
xx00 Q
Q((xx)) lim Q( x ) Q( x0 )
0

xx
xx00

) x x , x0R (.
3 . 3.5)
lim x = x0 lim x = x0.
x x0

x x0

f, g f(x) = x g(x)=log x 0<1 .


f(x) = ex, f(x) =10x, g(x) =lnx, g(x)=logx.
x
x
g
( x=
) =xx==
g(x)
x
x
( R) .

h(x) = (x3 + 2x + 1) g(x) = x f(x)=x3+2x+1.
,
( ), .

f(x) = x =

g, h x = x0, g(x0) 0 h(x0) = 0,


g
f =
h
lim f(x)=+ lim f(x)=,
x x0

x x0

f x = x0.

181

, g, h
(, x0] [x0, ).
3.7.1.

, f :
1 3
x , x 2
f ( x) = 2
22 x 3, x > 2.

f x> 2 , x <2 .
x 0 = 2 , :
lim f ( x ) = f (2) = 4a .
x 2

1
lim f ( x ) = lim ( ax 3 ) = 4a , lim+ f ( x ) = lim ( 2a2 x 3) = 4a2 3 .

x 2
2
x 2
x 2
x 2+

1
4a2 3 = 4a 4a2 + 4a 3 = 0 = = 3 .
2
2
3.7.2.
f f ( x ) =

x+2
.
x( x + 3)

f A=(,3)(3,0)(0,+)
g(x)=x+2 h(x)=x(x+3) ( R, ).
,
h(0)=0(0+1)=0, g(0)=0+2=20,
f x = 0.
h(3)=3(3+3)=0, g(3)=3+2=10,
f x=3.
3.7.
10

5
_1
_4

_3

_2

1
_5

_ 10

. 3.7.

182

3.7.1.  3.7 . ,
.
y

y
3

2
2
1
1

10

x
_1

_1

()

()

. 3.7.

3.7.2.  x0 .
x2 +1
,

x +3
) f ( x ) =

x0 = 3 0

1 x ,

) f ( x ) = 0,

x0 = 1 x 2 2 x + 1,

x 3

x = 3

x2 2x 8
,

) f ( x ) = x 4
x0 = 4 6

x4
x=4

x 2 6,
=
f
x
(
)
)

x0 = 3 6 + x ,

x <1
x =1
x >1

x <3
x 3

3.7.3. .
2 x 1,
) f ( x ) =
8 x,

x <3
x 3

x 1, x<1
) f ( x ) =
ln x , x 1

x<0
x,
2
) f ( x ) = x , 0 x 1
3
x >1
x ,

ln x ,
) f ( x ) =
2 x + 2,

2 x 2 + 1,

) f ( x ) = x 1
,

x 1

2 x
x0
,

f
x
(
)
)
x
1  x , x t 0

x 1
x >1

xe
x<e


3.7.4.  , f R.
ax10 1, x 1
+

x
3
,
x
3

) f ( x ) = 2
) f ( x ) = 1
, x <1
x 2, x > 3
2
x +3
3.7.5. .
) f(x)=(x)
) f(x)=ln(x2+x+1)+ex

) f(x)=ex
1
) f ( x ) 2 
e x 1

) f(x)=ln(x2+4)
) f(x)=ln(ln(ex+x))

183

3.7.6. .
) f ( x ) =

x +3

( x 1)( x + 4)

) f ( x ) =

x4 +1
( x 3)( x 2 4)

3.8 Bolzano .
.
3.8, f
[,], f()< 0 f ()> 0. (, f())
B(, f()) f (
f), x'x
x0. f(x0)=0, x0 (, )
y
f(x)=0 (,).
3.8,
f ()
(, f())
f x'x
.
Cf x'x.
x

O
,
f()>0 f()<0,
f ()
Bolzano.
(, f ())
, :
. 3.8

Bolzano.
f, [, ]. f() f()< 0, ,
, x0 (, ) , f(x)=0,
f(x) = 0
(, ).

f
Bolzano, f ,
, f() f( ) < 0
(. 3.8).
, (
) f f()f()>0 f (, ).
:
) f(x)=x22
Bolzano [1,2] ,
f(1) f(2)=(1)2=2<0.
f(x) = 0 -

(, f())

f()

f()

x0
x0

x0

(, f ())

. 3.8
y
(, f())

f()

O
f()

(, f ())

. 3.8

184

[1,2] ( x0 = 2 ). ,
Bolzano [2,2], , , f(2)
f(2)=22=4>0., f(x) = 0 [2,2], xx`11 = 2 , x2 = 2 .
) f(x)=x3x Bolzano [2,2]
, f(2)f(2)=(10)6=60<0. Bolzano
f(x) = 0 [2,2]. , [2,2], x1=1, x2=0, x3=1.
Bolzano f ' , x,
x. :
, , .
f f(x)0,
x f(x)>0 x [. 3.8()] f(x)<0 x [. 3.8()].
f
, f
(. 3.8).
f x. , :
1. f.
2.  ,
f().
3.  f() > 0, f . f()<0,
f .
y

f(x)>0

O
O

x
f(x)<0

()

()

. 3.8.
y
+
_

+
1

+
_

. 3.8.

5
_

185

3.8.1.

f(x) = x x [0,2].
.

f(x)=0 [0,2].
5

x=
.
4
4
, f
x x = 0 x = x x = 1 x =

5
5

0, 4 , 4 , 4 4 , 2 ,
f .
f .
f .

0, 4

5
,
4 4

, 2
4

f ()

1
3

<0
2 2

1
3

<0
2 2

11
6

, 0, , , 2 (f(x)<0),

4 4
5 (f(x)>0).
,

4 4
3.8.2.

ex
x3
+
= 0 (1,2).
x 2 x 1

f(x)=(x1)ex+(x2)x3 :
2
f (1) = + 3 > 0 , f(0)=1<0 f(2)=e2>0.
e
f . f
[1,0] f(1) f(0) < 0, Bolzano , ,
x0(1,0) , f(x0)=0.
, x01,0

186

f ( x0 ) 0 ( x0  1)e

0  ( x  2) x 3
0
0

( x  1)e 0  ( x0  2) x03
0 0
( x0  1)( x0  2)


0

x0  2

x03
x0  2

0 ,

(1,0).
, f [0,2] f(0) f(2) < 0, (0,2).

f,
[,], f() < f(). f() f(),
f() < < f() (. 3.8).
g(x)=f(x), x[,], Bolzano. g
[,] f [, ], g()g() < 0, g()=f() <0
g()=f() >0.
, olzano x0 (, ) ,
g(x0)=f(x0)=0 f(x0)=.
y
olzano . f()
(, f())

:
.
f, '
[, ]. f [,
] f() f() ,
f() f() , x0 (, ) ,
f(x0) = .

f()
O

(, f ())
x0

. 3.8.
y
f()


, ' , f
()
f() f( ) .
, , f()
f(), y = f x0.
3.8
y = f
. ,
f [, ], ,
3.8,
.
(. 3.8):

(, f())

y=
f ()
O

(, f ())
x0

. 3.8.
y
f()

y=
f ()
O

. 3.8.

187

f()
f .
f '
, f ()
( ) (
).
:
A f ' (,
) , (, ) [. 3.8()],

lim f ( x) lim f ( x ) .
x

, , f (, ),
y
y (, ) [. 3.8(].
f [, ], [, )
(, ] (
,
).
x
O
O
,
f (
,

f, , ),
()
()
.
y

yy

()
y

()
y

()
()

()
()

()

y
y

O
O

O
O

xx

O O

O x2

()

. 3.8.

x2

() ()

()

x O
x

()

x2 x
()

x O

()

. 3.8.

188

, f(x) = lnx, x(0, e), , (, 1) [. 3.8()],

lim f ( x ) f  lim f ( x ) ln e 1.

x o0

x oe

f 1(x) = e x f,
(, 1) (0, e) [. 3.8()].
, f
[, ] , {f(x)| x[, ]} f(), f() (. [ f(), f()] f [f(), f()] f
).
1

2,5

0,5

f 1 (x) = e x, x (_ ,1)
_

0,5

1,5

2,5

1,5

_ 0,5
_1

f(x) = lnx, x (0,e)

0,5

_ 1,5
_2

_3

()

_2

_1

()

. 3.8.

f ' [, ]
, {f(x)| x[, ]}
f(), f().
( ).
f [, ], x1, x2 [, ] f( x1) f ( x) f (x2) x[, ].
, f x1
( f( x1) f( x) x[, ]), x2
( f( x) f(x2) x[, ]). .
f
[, ] [m, M], m .

3.8.3.


f ( x ) ln x  e x , x [1, 3] g ( x )

1 1
 , x (0, 2]. 
x 2

189

f ( ) (. 3.2.11).

lim f ( x ) = lim+ (ln x + e x ) = lim(ln x + e x ) = ln1 + e1 = 0 + e = e

x 1+

x 1

x 1

lim f ( x ) = lim (ln x + e x ) = lim(ln x + e x ) = ln 3 + e3

x 3

x 3

x 3

f [e,ln3+e3].
g ( ) .
x
0 0
x
1 1 x
1 1
1 1 1 1
lim
,
lim
1
lim
(
)
lim
=
f
x
+lim
+ = + .
1 +
+ = +
= = = =
= + , x 0lim
x 0x
x 0
x 0 + x 2
x20 2 2 2
0x x

x
1 1 x
1 1 x
1 1 3
1
1
lim f ( x ) = lim + = lim + = lim + lim = + =
x 2
x 2 x 2 x 2 x 2 x 2 x x 2 2
2 4 4

3
f [ , + ] .
4

3.8.1. f .
) f(x) =x + 2 x 3, =[0, ]

) f(x) =x + x x +1, =[0, ]

) f ( x )

x 3  3 x  2, [2, 0]

x
x
x
) f ( x ) 4  3  2  2, [0, 1]

) f ( x )

x 5  2 x 2  2, [1, 0]

) f ( x )

x 3  3 x  2, [2, 0]

3.8.2.  f(x)=x33x2+3
[1,0], [0,2] [2,3].
3.8.3. :
x3 + 1 x 2 + 1
e 2 x ln( x / 2)  1
+
=0
)
)

0
x+2 x2
x2
x2

x 2
x2

x3  1
0
x2

(2,2).
3.8.4.
 f x
.
) f(x)=x316x

) f(x)=x42x2+1

) f ( x ) = ( x 1)( x 4 16)

) f(x)=x44x2+3

) f(x)=(x21)(x24)

) f(x)=x32x2x+2

3.8.5.
 f x
[0,2].
) f(x)=x+x
) f(x)=2x+2x
) f(x)=x+x
x
) f ( x ) 2  1
) f(x)=22x1
) f(x)=22x1
2

190

3.8.6. .
) f(x)=ex, (1,1]

) f(x)=3x+2, (2,2)

) f(x)=2lnx+1, [1,e2]

) f ( x ) = x , (1,4]

) f(x)=ex+1, (,0)

) f(x)=2x+1 x [0, /3)

) f(x)=2ex3, (1,+)

) f(x)=2x+1 x [/2, )

) f ( x ) =

1
, (1,10]
x3

3.9 .
f=g.

xA
f(x)=g(x).
(f+g)(x)=f(x)+g(x), xA
(fg)(x)=f(x)g(x), xA

- (cf)(x)= cf(x), xA
.
(fg)(x)=f(x)g(x), xA
f
f ( x)
A
( )( x ) =
, x
A g(x) 0.
g
g( x)

f .
f .

f(x1)<f(x2) x1, x2 x1<x2


f(x1)>f(x2) x1, x2 x1<x2.

f:A R.
( 11).

x1, x2 A : x1 x2,
f(x1)f(x2) f(x1)=f(x2), x1=x2.

f:A R.

x A xA
f(x)=f(x).

f:A R.

x A xA
f(x)=f(x).

f:A R xA x+TA
.
f(x+T)=f(x).
f:A R g:B R.

(gf)(x)=g(f(x))
xA, f(x)

A y f1:f(A)A - f (yf(A)) x
y=f(x).
f.
f1(y)=x f(x)=y

x0R.

lim ( f ( x ) + g ( x )) = lim f ( x ) + lim g ( x )

x x0

x x0

x x0

lim (cf ( x )) = c lim f ( x )

x x0

x x0

lim ( f ( x ) g ( x )) = lim f ( x ) lim g ( x )

x x0

x x0

x x0

lim f ( x )
f ( x ) x x0
lim
( lim g ( x ) 0)
=
x x0 g ( x )
x x0
lim g ( x )
x x

lim ( f ( x ) + g ( x )) = lim f ( x ) + lim g ( x )

x x0

x x0

x x0

lim (cf ( x )) = c lim f ( x )

x x0

191

x x0

lim ( f ( x ) g ( x )) = lim f ( x ) lim g ( x )

x x0

x x0

x x0

lim f ( x )
f ( x ) x x0
lim
( lim g ( x ) 0)
=
x x0 g ( x )
x x0
lim g ( x )
x x0


x0R.

lim f ( x ) = lim f ( x )

x x0

x x0

lim [ f ( x )] = lim f ( x )
x x0
x x0

g(x)f(x)h(x) x0
.

lim h( x ) = lim g ( x ) = , lim f ( x ) =

x x0

x x0

x x0

lim P(x)=P(x0)


P(x) x0.
P( x)
R( x ) =
Q( x )
x0 ( Q(x ) 0):
0

x x0

PP((xx)) PP((xx00))
xxo

oxx000Q
Q((xx)) QQ((xx00))

lim
limRR((xx)) lim
lim
lim

xxo
xx0x00
xo

lim x =x0

x x0

lim x =x0

x x0

x0.

ax
x
=a
= 1 , lim
x 0 x
x 0 x

lim
f +.

y=, lim f ( x ) = .

f .

y=, lim f ( x ) = .

x +
x


P(x)=xn+xn1 +...+, 0, + .

lim P ( x ) lim ( x n ) 

x orf

x orf


R( x )

P( x)
Q( x )

x n  x n1 " 

' x m  ' x m1 "  '

lim P ( x ) lim ( x n ) 

x orf

x orf

+ ( 0, 0).
f.
x0 .

x = x0,
lim+ f(x), lim f(x), +
x x
x x
0
0
lim f(x)=f(x0)

x x0

192

(, ). f (, ).
f x0 (, )

[, ].
lim f ( x ) = f (), lim f ( x ) f ( )
x +

xo 

f g
x0, x0
.

f+g, cf ( cR), fg, f


f
( g(x0)0),
g

f ( f(x0)0).

f
=(, ),
g f(), g f
.

.
.
.
f(x)=ax.
g(x)=logax, 0<1.

Bolzano.

f [,] f() ^ f() < 0,


, , x0 (,) ,
f(x0)=0.

f [,] f()f() ,
f() f( ) ,
x0(a,) , f(x0)=.

3.10 .
,
, .
y

1.

3.10

O
. 3.10.

2.

f f(x) = x3. h = f f h(x) = x6.

3.

R y = a Cf.

193

x2 + 5
. lim f ( x ) = 5 / 2 .
x +
x2 + 2

4.

f ( x ) =

5.

f, g f(x) g(x) x0
x0, lim f ( x ) d lim g ( x ) .
xo x

xo x

x  1 . f 0.
x 1

6.

f ( x )

7.

f(x) = ln x g(x) = ex, ( f g )(x) = x.

8.

f ( x ) =

9.

1
1
1
lim x 3
= (lim x ) lim 3
= 0 lim 3
= 0.

x 0 x + 2 x
x 0
x 0 x + 2 x
x 0 x + 2 x

1
, lim f ( x ) f , f 4.
x o4
( x 4)3

2
2
f ( x) = 0 .
10. 4 f ( x ) 2 , x(1,+), xlim
+
x
x

f(x)= lim f(x), f x0.


11. f xlim
x+
x x

12.

13.

lim f ( x )
xo x

0 , lim f ( x ) 0 .
xo x

lim f ( x ) l , lim g ( x ) m , l,m R x0 f(x) < g(x),


xo x

xo x

l m.

(1 3 x )5
.
14. lim
x + x 3 + 1

.
f ( x ) = lim g(x)=+
f ( x ) = , lim (f(x)+g(x))=0
f ( x ) =
f(x)=
15. lim
x 0
x 0
x 0

f,
16. 3.10. f
x1 = 0 x2 = 1.

. 3.10

17.

f ' = [, ],
, x0 (, ).

18.

f, ' [, ]. f() f () < 0,


, , x0 (, ), , f(x0)=0.

19.

' , , .

194

20.

f ,
f .

21.

f [1,1] f(1) = 4, f(1) = 2, x0(1,1) , f(x0)=e.

22.

f R ,
f(x3) = f(x) 0.

23. olzano .
24.

f (,),
.


f ( x ) =
1.

2.
3.

4.

) R

) R {1,1}}

) R {1}

) R {2}

f, g ,h f(x)=3x, g(x)=x33, h(x)=3(x31). :


) h = f g
) h = g f
) f = h g
) g = h f
f f(x) = x 2 . h = f f f :
) h(x) = x8
) h(x) = x
) h(x) = x2
) h(x) = x6
1
lim 
x of x
) +

, , :

)
) 0
10

5.

x2 2
:
x2 +1

) 1

f(x)=x +x 1000. :
) lim f ( x ) =
) lim f ( x ) = +
x

) lim f ( x ) = 0

) lim f ( x ) = 0

) lim f ( x ) = 0

) lim f ( x ) = 0

) lim f ( x ) = 5 / 2

) lim f ( x ) = 5 / 2

x 0

6.

f(x)=e , :
) lim f ( x ) =
x

) lim f ( x ) = +

f f ( x ) =
7.

) lim f ( x ) = 5 / 2
x +

f ( x ) =
8.

) lim f ( x ) = 1
x 0

x 0

x2 + 5
. :
x2 + 2

) lim f ( x ) = 5 / 2
x

x 0

x 1

(x / 2)
. :
x
) lim f ( x ) = 2
x 0

) lim f ( x ) =
x 0

1
2

) lim f ( x ) = 0
x 0

195

2x  4
x0 = 0 :
5( x  2)

f ( x )
9.
) 2

10.

5
2

5
2

) 2

>10, , x0, :
lim f ( x ) =
x 0

) f(x) > 0

) f(x) < 0

) f(x) = 0

) f(x) = x0

lim f(x)=1 lim g(x)=1, lim [3f(x)2g(x)] :


11.

x x0

x x0

x x0

) 5

) 1

) 2

) 5

lim f ( x ) = 1, y = 1 :
x +

12.

13.

) Cf .

) f .

) f +.

) .

f(x)=lnx10, x>0. :
) lim f ( x ) 10
) lim f ( x ) f .
x o0

) lim f ( x ) f .
x o0

14.

x o0

) lim f ( x ) = 0 .

x 1

P(x) P(x)=x+1x1+...+0, 0,
:
) lim P ( x ) = 0
) lim P ( x ) lim ( x )
) lim P ( x )
) lim P ( x )
x of

x of

x of

x o0

lim f(x)=+ :
x x0

) f(x) (f(x) > 0) x0.


15.

) f(x) (f(x) < 0) x0.


) lim (f(x))=+.
x x0

) lim
xo x

1
f ( x)

f .

lim f(x)=0 f(x) < 0 x0, :


x x0

16.

) lim
xo x

17.

1
f ( x)

f

) lim

xo x

1
f ( x)

f

) lim

xo x

1
f ( x)

) lim
xo x

1
f ( x)

f

f , x1, x2
x1<x2 :
) f(x1) < f(x2),

) f(x1) > f(x2),

) f(x1) = f(x2),

) f(x1) < f(x2) f(x1) > f(x2).

196

f x0 g f(x0), :
18.

19.

) g f x0.
) g + f x0.

) f g o x0.
) g f o x0.

f R, . f(x2) = f(x):
) 0.
) R.

) 0 1.
) .

3.11 .
3.11.1. f f(x)=6x3+ax22x+.
)  , , (2, 25) (1,0) Cf.
) f .
) .
) x, f(x)>0.
) o f;
3.11.2.
 ,
5/3.
)  y ,
x.
) lt 10 lt;
)
 f(x)=1005x, x lt
.
, 3 lt .
3.11.3.
 , .
, t, ( min) t
t2

f2 ( t) = 15 + .
C, f1 f2 f1( t) = 30
10
50
)
 .
)

;
) ;
) ;
)
P
10C.
100
3.11.4.
 3.11
f(t),
t .
100%
. t2

50

t1

t2 t3

. 3.11

t4

197

, .
t4
.
)
 ( ) lim f ( t) , lim f ( t), lim f ( t) .
tot

tot

tot

)  f ;
)  lim f ( t) ;
tot

2
3.11.5.
 f (,x0) (x0,), lim f ( x ) lim f ( x ) 6  5.
xo x

xo x

, lim f ( x ).
xo x

3.11.6.
 x ,
f(x) , : f(x)=+(1ex), x0
(, , ).
) ;
)

;
3.11.7.
 .
, 40 ,
8 kg 30 , 8 .
)  x kg
.
) 8 kg;
3.11.8.
x2 4
x2 9
x2 +1
(
)
(
)
(
)
f
x
=
f
x
=
f
x
=

)
)
)
x2 5x + 6
x2 5x + 6
x2 5x + 6
3.11.9. f g 4x23x+2f(x)x23x+2 xR
2 + x3 f ( x) 1 +

1

x , lim f ( x ) = lim g ( x ) .
2
x 0
x 0
2 x
8 8

3.11.10.
 f f(x)=(x2)(x25x2+1)

f ( x)
,

h( x ) x  1
x  5,

x z1
x 1.

) f xx, yy.
) N , h .
) f g.
3.11.11.  , ' y
x .
y = f(x)
:
) , f(0) = 0.

198

)  x 0, .
)
 , , lim f ( x ) = 0 .
x +

)  y 83, 1
57%, 4 h 18%.
f
f ( x) =

12 x
, x0
15 x + 5 x + 1
2

.
1
.
x2
) f.
) : lim ( f ( x ) x( x + 1)) lim ( f ( x ) x( x + 1)) .

3.11.12. f ( x ) = x 2 + x

x +

) : lim ( x 2)( f ( x ) x( x + 1)) lim ( x 2)( f ( x ) x( x + 1)) .


x +

3.11.13.
 (x, f (x))
(x, g (x)) , f g

f ( x) =

x3 + 2
,
x2 + 2

g( x) =

x4 +1
,
x+2

x 0.

,
1 2 (1x2).
3.11.14.  x , :

50 x 2
, x 0.
5 x 2 + 50
f(x) x.
) .
) lim f ( x ) lim f ( x ) .
x 0
x +
50 x 3
) f ( x ) = 10 + 2
, x0
5 x + 50
x ; .
f ( x ) = 10 +

, , , ..
, ,
,
... 17 ,
, . Newton
(16421727) Leibniz (16461716),
.
(
)
.

4.1 .

4.2  . .
4.3 
.

4.4  .
4.5 L Hospital.

4.6 .
4.7 .

4.8 .

lx im

4.9 .

4.10 .

x0

200

4.1 .
, . : . f
x0 , f(x) f(x0),
x x0 . Cf (. 4.1), A (x0, f(x0)) (x,f(x)) , x x0.
h=xx00 (
x=x0+h),
f ( x ) f ( x 0 ) f ( x 0 + h) f ( x 0 )
=
x x0
h
f
f ( , f ). To
x x0 (, h0 f x0.
:
f A (x0, f(x0)) Cf.
f ( x )  f ( x0 )
lim
 , f xo x
x  x0
0
x0 . f x0 f(x0) , :
f ( x0 ) = lim

x x0

f ( x ) f ( x0 )
f ( x 0 + h) f ( x 0 )
= lim
.
h0
x x0
h

( 3),
, , .
xx0=x f(x0+h)f(x0)=f(x0),
:
f c( x0 )

f ( x0 ) .

x o0 x

lim

Cf

B(x0+h, f(x0+h))

f(x0+h) _ f(x0)
A

A(x0 , f(x0))
O

x0

. 4.1.

x0+h

x0

. 4.1.

201

x0 f '(x0) Lagrange, Leibniz, , x0 :


df ( x0 )
df ( x )

dx
dx

x=x

, , .
,
. '
f ' (x0, f(x0))
(' ) ,
B(x, f(x)), f, (. 4.1).
f x0, Cf A(x0,f(x0))
, A(x0,f(x0)) f(x0). :
f x0, Cf A(x0,f(x0))
yf(x0)= f(x0)(xx0).
Cf A(x0,f(x0))

Cf f x0 f x0. A ( )
2
Cf A(x0,f(x0)) , = f(x0).
, f ( x ) =

x2
x0=1 :
2

(1 + h)2 1

f ( x0 + h) f ( x0 ) f (1 + h) f (1)
2h + h 2
h
2
2
=
=
=
=1+
h
h
h
2h
2

lim

h 0

f ( x 0 + h) f ( x 0 )
h
= lim(1 + ) = 1 .
h 0
h
2

f x0=1
f(1)=1. , f x0=1

1= ,
4
Cf A(x0,f(x0))

= = 45 (. 4.1). 4
Cf (1, f(1)),
y f (1) = f (1)( x 1) y

1
1
= 1 ( x 1) y = x .
2
2

Cf

y
4
3

2
1
_1

45o
1

_1

. 4.1.

202

x0 = 1

x 2 + 2,
f ( x) =
3 x ,

x 1

x< 1.

(4.1.1)

x > 1, x =1 + h, h > 0 :
f (1 + h) f (1) [(1 + h)2 + 2] 3 2h + h 2
=
=
=2+h,
h
h
h

lim

h o1

f (1  h)  f (1)
h

lim (2  h) 2 .

h o1

x < 1, x =1 + h, h < 0 :
lim

h1


lim

h o1

f (1 h) f (1)
3(1 h) 3
lim
3.
h

1
h
h

f (1  h)  f (1)
h

3z 2

lim

h o1

f (1  h)  f (1)

h

f x0=1.
4 , x0 f,
:
f x0,
f ( x0 ) lim
h0

f ( x0 h) f ( x0 )
,
h

f ( x0 ) lim
h0

f ( x0 h) f ( x0 )
h

.
f(x0), f+(x0) f x0
.
x0 ( f), (x0, f(x0) .
f, ' x0. xx0
:
f ( x ) f ( x0 )
f ( x ) f ( x0 ) =
( x x0 )
x x0

ff((xx)) ff((xx00))
ffc(c(xx00),), lim
lim
lim
lim((xxxx00)) 00, 
xo
xxx0
xx
xxo
o
ox0x
xxxx00
00
00

lim ( f ( x )  f ( x0 ))

x0
xo

f c( x0 ) 0 0.

203

,
lim
lim ff((xx))

xxo
oxx0

00

lim
lim [(
[( ff((xx))  ff((xx00))
))  ff((xx00)]
)] lim
lim (( ff((xx))  ff((xx00))
))  lim
lim ff((xx00)) 00  ff((xx00)) ff((xx00).).

xxo
oxx0

xxo
oxx0

00

xx
o
oxx0

00

00

f x0. :
f ' x0,
.
, f
x0, x0.
x0=0 ' . ,
(4.1.1), x0=1,
.
f ( x ) = x . f 0,
, h>0 :

lim

h o0

f ( x 0  h)  f ( x 0 )
h

lim

h o0

h 0
h

lim

h o0


Cf, (0, 0),
B(x,f(x)), x >0 Cf.
4.1, f,
x . ,
(0, 0) , yy.
, Cf,
(0, 0)
x=0.
:

1
h

f .

y= x
B(x, f (x))

O
y

. 4.1.

f x0 :
) lim

h 0

f ( x 0 + h) f ( x 0 )
= +
h

( ),

) lim

f ( x 0  h)  f ( x 0 )
h

f  lim

f ( x 0  h)  f ( x 0 )
h

f ,

) lim

f ( x 0  h)  f ( x 0 )
h

f  lim

f ( x 0  h)  f ( x 0 )
h

f ,

h o0

h o0

h o0

h o0

Cf A(x0,f(x0)) ()
x=x0.

204

f ( x )  f ( x0 )
lim
lim 
x  x0
0

f ,

xxo
o xx0

f + (. 4.1).

f ( x )  f ( x0 )
lim
f ,
limx 
xxo
o x0

x
x
0
0

f .

x0

. 4.1.

4.1.1.

xd0
x  5,
f f ( x )
, R. 
2
x !0
x  5,
) f x0=0 .

) f x0=0;
.

) f x0=0, lim f ( x ) = f (0).


x 0

lim f ( x )

x o0

lim (x  5)

lim(x  5) lim x  5 5 

x o0

x o0

x o0

lim f ( x )

x o0

lim ( x 2  5)

x o0

lim( x 2  5) 5 ,
x o0

lim f ( x ) = 5
x 0

f(0)=0+5=5 R, lim f ( x ) = f (0).


x 0
x0=0 R.
) h0 :
lim

h o0

f (0  h)  f (0)
h

lim

h o0

[ (0  h)  5]  5
h

lim

h o0

h
h

, 

lim

h o0

f (0  h)  f (0)
h

lim

h o0

( h 2  5)  5
h

lim

h o0

h2
h

lim h 0. 

h o0

f x0=0, = 0.
, f x0=0 R,
=0 .

205

4.1.1. x0 :
) f(x)=x2+2, x0=4
3
) f ( x ) = , x0 = 1
x

) f(x)=3x+2, x0=1
) f(x)=3x2+2, x0=0

) f(x)=2, x0=1
) f(x)=x, x0=0

4.1.2. N x0
.
4 x 2
) f ( x ) =
2

2 x

x 1
x >1

3 x 2
) f ( x ) =
2

2 x

, x0=1

) f ( x )

x  1 , x0=1

) f ( x ) = x , x0=1

) f ( x )

x  1  x  1 , x0=1

) f ( x )

x 1
x >1

, x0=1

x  1  x  1 , x0=0.

, Cf
A(x0,f(x0)).
4.1.3. f f(x)=3x+2x2.
) f x0 R.
)
 f x0 =1
A(1,5).
x 2 x d 1

f
(
x
)
4.1.4.

x  x ! 1.
, f x0 =1.
4.1.5. f x0 :
lim

x
o x0

f ( x)
x  x0

a R .

f x0 f(x0)=a.
4.1.6. f x2+2x+1f(x)5x2+2x+1, xR.
) f(0)=1 x(x+2)f(x)f(0)x(5x2+2) x R.
) f x0=0.
) Cf x0=0.
4.1.7.
 g x0=0, f(x)=x2g(x)
f(0)=g(0). f(x) = x2 + x3 x
x0=0 f(0)=1.

4.2 . .
f(x)=x2 ( R) x0. f(x0)
x0 , h0 :

206

f ( x0 + h) f ( x0 ) ( x0 + h)2 x02 ( x02 + 2 x0 h + h 2 ) x02 2 x0 h + h 2


=
=
=
= 2 x0 + h
h
h
h
h
,
lim

h 0

f ( x 0 + h) f ( x 0 )
= lim(2 x0 + h) = 2 x0 .
h 0
h

f(x0)=2x0.
f(x0)=2x0 x0, ,
x0 f(x0)=2x0 , , x
2x. f f. , f(0)=20 x' , .. f(1)=21=2, f(0)=20=0, f(2)=2(2)=4 ..
, f, x0,
. f: x f(x)
f f :
dy
df
.
y
dx
dx
df ( x )
f(x)
.
dx
, . f , f,
f :
f

d2 y
d2 f
f(x).

dx 2
dx 2

f () f f ,

c
f ( )
f ( 1) , t 3 .

( )

d f
( x) 
dx

()

d y

.
dx

.
,
3
.
. 4.2.1, .
4.2.1
.

c
()

f~

x1

1 , x0
x

1
x2

x , x0
1
2 x

, x >0

ex

lnx , x>0

ex

1
x

207

.
f(x)=x, 0, 1. x0
, xx0 :
f ( x ) f ( x0 ) x x0 ( x x0 )( x 1 + x 2 x0 + + x0 1 )
=
=
= x 1 + x 2 x0 + + x0 1 ,
x x0
x x0
x x0

lim
lim

o
oxx0
xx

00

f ( x )  f ( x00)
x  x00

lim ( x 11  x 22 x00 "  x0011) x0011  x0011 "  x0011 x0011.
lim

o
oxx0
xx

00

, x f, R, (x)=x1. .
f ( x ) = x , x>0
1
f ( x ) =
. , x0 , h0 :
2 x
x + h x0 ( x0 + h x0 )( x0 + h + x0 )
f ( x 0 + h) f ( x 0 )
= 0
=
h
h
h ( x0 + h + x0 )
=
,
lim

h 0

x0 + h x0
=
h ( x0 + h + x0 )

1
.
x0 + h + x0

f ( x 0 + h) f ( x 0 )
1
1
= lim
=
.
h 0 x + h + x
h
2 x0
0
0

, 4.1, f ( x ) = x
0, .
f(x) = x. x0R h0 :
f ( x 0  h)  f ( x 0 )
h

( x0  h)  x0
h

x0 h  x0 h  x0
h

x0

(h  1)
h 
 x0
h
h

(. 3, 4, 5 . 3.5)
h
h 1
= 1 , lim
= 0,
h 0 h
h 0
h
lim

:
lim

h o0

f ( x 0  h)  f ( x 0 )
h

x0 0  x0 1 x0 .

(x)' = x x R.
4.2.1.

A(x0,f(x0)) f(x)=lnx,
:
) H
 Cf A(x0,f(x0))
45.

208

) H 1/4.
)  . N
.
.

f ( x ) = (ln x ) =

1
,
x

1
( x x0 ) .
x0
) , Cf A(x0,f(x0))
, = f(x0). =/4, :

Cf A(x0,f(x0)) y ln x0 =

f c( x0 )

1
1
4
x0

1 x0 1 

(1,f(1))= (1,0)
1
y ln1 = ( x 1) y = x 1 .
1
) Cf A(x0,f(x0)) f(x0).
:
1
1 1
f ( x0 ) =
= x0 = 4
x0 4
4

1
x
y ln 4 = ( x 4) y = + (ln 4 1) .
4
4
) O(0,0),
0 ln x0 =

1
(0 x0 ) ln x0 = 1 x0 = e .
x0

, A(e,1)
1
x
y ln e = ( x e) y = .
e
e
,
, ,
.
,

.
f, g x0,
f+g, cf cR, fg, f/g x0 :
1. (f+g)(x0)= f(x0)+ g(x0)

209

2. (cf)(x0)= cf(x0)
3. (fg)(x0)= f(x0)g(x0)+f(x)g(x0)

f
f ( x0 ) g ( x0 ) f ( x0 ) g ( x0 )
, ( g(x0)0).
4. ( x0 ) =
[ g ( x0 )]2
g

( f ( x ) + g ( x )) = f ( x ) + g ( x ) , (cf ( x )) = cf ( x )

f ( x ) f ( x ) g ( x ) f ( x ) g ( x )
( f ( x ) g ( x )) = f ( x ) g ( x ) + f ( x ) g ( x ) ,
.
=
[ g ( x )]2
g( x)
14 .
, 1 xx0, :
( f + g )( x ) ( f + g )( x0 ) f ( x ) + g ( x ) f ( x0 ) g ( x0 ) f ( x ) f ( x0 ) g ( x ) g ( x0 )
=
=
+
x x0
x x0
x x0
x x0

, f, g x0, :
lim f ( x )  f ( x0 )
x  x0

f c( x0 ),

x x0

lim g ( x )  g ( x0 )
x  x0

x x0

g c( x0 ). 

, x0 f+g, :
((ff  gg))cc((xx00))

ff((xx)) ff((xx00)) lim gg((xx)) gg((xx00))


lim
ffcc((xx00)) ggcc((xx00).).
lim
lim
 lim
lim
xxx0
xxxo
o
x
x
x
x
x
x
x
x




0
0
0
0
00
00

xxxo
oxxx0

14, :
2
) f ( x ) = x + 2 x ,

x > 0 , :

f ( x ) = ( x 2 ) + (2 x ) = 2 x + 2
) f ( x ) = xe x x 3 , x R , :

1
2 x

= 2x +

1
,
x

x>0 .

f ( x ) = ( xe x x 3 ) = ( xe x ) ( x 3 ) = x e x + x (e x ) 3 x 2 = e x + xe x 3 x 2 .

f ( x ) x , x z +

g( x)

x , x z , Z .

c
x

f c( x ) (x )c

(x )cx  x(x )c

2 x

x x  x(x )
2 x

2 x  2 x
2 x

1
2 x

x x  x(x )
2 x

2 x  2 x
1  2 x 
2
x

2 x  2 x
2 x

1
2 x

210

c
x
g c( x ) (x )c

(x )cx  x(x )c
2 x

-( 2 x  2 x )
2 x

(-x )x  xx
2 x

2 x  2 x
2 x

1 
=
2 x

(1  2 x ). 

4.2.2.
, ,
x (rad) .
4.2.2
.

f~

1
1  2 x 
2
x

x


1
2 x

(1  2 x ) 

1 . , f1, f2, ..., fk,


, :
( f1 + f2 + + fk )( x ) = f1( x ) + f2( x ) + + fk ( x ) .
, 3
. , , :
( f ( x ) g ( x )h( x ))c [( f ( x ) g ( x )) h( x )]c

( f ( x ) g ( x ))c h( x )  ( f ( x ) g ( x )) hc( x ) 

= [ f ( x ) g ( x ) + f ( x ) g ( x )]h( x ) + f ( x ) g ( x )h( x )
= f ( x ) g ( x )h( x ) + f ( x ) g ( x )h( x ) + f ( x ) g ( x )h( x ) .
:
2
x
2
x
x
) (x  x  e  2 x )c (x )c  ( x )c  (e )c  (2 x )c x  2 x  e  2 
4
x
4
x
4
x
4 x
) ( x e ln x ) = ( x )e ln x + x (e ) ln x + x e (ln x ) =

1
= 3 x 3e x ln x + x 4 e x ln x + x 4 e x , x > 0 .
x
4.2.2.

.
ex
x2 4
) f ( x ) =
, x 3
) f ( x ) =
) g ( x )
x +3
1+ x
.

) :

x x 

ln x
, x1
x 1

211

f ( x ) =

( x 2 4)( x + 3) ( x 2 4)( x + 3) 2 x( x + 3) ( x 2 4) 1 x 2 + 6 x + 4
=
=
.
( x + 3)2
( x + 3)2
( x + 3)2

) :

1
e x (1 + x ) e x
(e )(1 + x ) e (1 + x )
2 x
f ( x ) =
=
2
2
(1 + x )
(1 + x )
x

= e x 2 x + 2 x 1
.
2 x (1 + x )2

) :

c
(ln x )c( x  1)  ( x  1)c ln x
ln x
g c( x ) ( x x )c 

( x )cx  x (x )c 
( x  1)2
x 1
x 1
ln x
x + 2 xx x 1 x ln x
1
.
x + x x + x
=
=
+
2
2 x
2 x
( x 1)
x( x 1)2

4.2.3.

f f(x)=x42x312x2+1. x
) f(x)=0
) f(x)>0
) f(x)<0

.
f ( x ) = ( x 4 ) (2 x 3 ) (12 x 2 ) + (1) = 4 x 3 6 x 2 24 x

f ( x ) = (4 x 3 6 x 2 24 x ) = (4 x 3 ) (6 x 2 ) (24 x ) = 12 x 2 12 x 24 = 12( x 2 x 2) .
:
) f cc( x ) 0 x 2  x  2 0 x

2 x

1.

) f ( x ) > 0 x (, 1) (2, +) .
) f ( x ) < 0 x (1, 2) .

h(x)=2x. 2x=xx, 3 :

hc( x ) ( 2 x )c (x x )c (x )cx  x(x )c x x  x x ,


h(x)=(2x)=2xx. h(x) = 2x 2x, (x2)=2x.
h(x) = 2x = (x)2 f(x)=x2 g(x) = x (. h=fg),
f(x)=2x, g(x)=x, h h
:
hc( x ) 2x x [ 2 g ( x )] g c( x ) f c( g ( x )) g c( x ) .
( ) , :

212
(gf)(x)=

g ' f g(),
fg :
(fg)(x)=f(g(x))g(x).

dy dy . du
=
dx du dx

gf
x

g(x)=u

f(u)=y

M Leibniz, u=g(x)

y=f(u),
dy
du
dy dy du
f(u)=
g(x)=
=

(.
du
dx
dx du dx
. 4.2.
dy dy du
=

4.2).
dx du dx
, ( ).
4.2.1 4.2.2
4.2.3,
(g ).
4.2.3
.
f

f'

(g(x))

(g(x))1 g(x)

(g(x))

g ( x )
2 g( x)

g ( x )
2 ( g ( x ))
g ( x )
2
( g ( x ))

(g(x))

e g(x)

(g(x))g(x)

e g(x)g(x)

(g(x))

ln(g(x)), g(x)>0

(g(x))g(x)

g ( x )
g( x)

(g(x))
g ( x ) , g(x)>0

4.2.4.

:
4

3
2 x 1
) h( x ) =
) h(x)=ex +2x

x +1
.
) f g

) h(x)=ln(x4+2x2+2)

f(x)=x4 g ( x ) =

2 x 1
x +1

. , h(x)=[g(x)]4=f(g(x))
4.2.3 h(x)=4(g(x))3g(x).

213

g ( x ) =

(2 x 1)( x + 1) (2 x 1)( x + 1) 2( x + 1) (2 x 1)
3
,
=
=
2
2
( x + 1)
( x + 1)
( x + 1)2
3

3
12(2 x 1)3
2 x 1
.
h( x ) = 4

( x + 1)5
x + 1 ( x + 1)2

) h(x)=ex +2x =eg(x) g(x)=x3+2x.


3

(eg(x))= eg(x)g(x)=ex +2x (x3+2x)=(3x2+2)ex +2x.


) u=x4+2x2+2 y=h(x)=ln(x4+2x2+2)=lnu

dy dy du
=

y =
dx du dx

1
1
1
4 x( x 2  1)
4
2
3

h( x ) = y = (ln u) u = u = 4

(
x
+
2
x
+
2
)
=
(
x
x
)


4
4
.
u
x + 2x2 + 2
x4  2x2  2
x4  2x2  2
4.2.5.

:
) f(x)=ax, xR, 0<a1
) f(x)=x, x > 0 aR*

) f(x)=lnx, xR*

) , x>0, x=elnx, f(x)= ea^lnx = eg(x), g(x) = alnx. ,


11
ffc(c(xx)) eegg( (xx) )ggc(c(xx)) eeaalnlnxx((aaln
lnxx))c c eeaalnlnxxaa
xx

11
xxaaaa
ax
axaa11 
xx

(xa)=axa1 aR*(x>0),
4.2.1.
) f(x)=ax=exlna=eg(x) g(x)=xlna :

f(x)=eg(x)g(x)=exlna (xlna)' =exlna ^ lna = ax lna,


(ax)=axlna xR, 0<a1
(ln x )c (ln x )c

) x>0,

1
x

1
.
x

x<0
ln x


lim

xo x

ln(  x ) ln( g ( x ))  g(x)=x


f ( x )  f ( x0 )
x  x0

, xR*, (ln x )c

1.

x

f c( x0 ),

lim ( x  .x0 ) 0 

xo x

214

4.2.1. f x0.
1
1
) f(x)=x5, x0=1
) f(x)=lnx, x0 =
) f ( x ) = , x0=3
x
2
) f ( x ) = x , x0=4

) f(x)=ex, x0=0

) f(x) = 3x, x0 =

4.2.2.
 (. 4.2).
(
).
y

f(x)=2

f(x)=2x

f(x)=x 2

x
x

O
()

()

()

y
y

_2

x+2, _ 2 x 0
1,
0x
f(x)=
x+8 , 3< x 4
()

2
1

x, x>0
f(x)= x, x0
()

_O
1

2, 0 x 1
f (x)= 1, 1 x
, 2< x 4
()

. 4.2.

4.2.3.
 f f(x)=x4. x f :
) .

) .

) 24.

4.2.4.
 f f(x)=x3. Cf ,
A(1,1) B(1,1).
4.2.5. f f(x)=x10. :
(1 + h)10 1
,
h 0
h
lim

(1 + h)10 1
,
h 0
h
lim

(2 + h)10 210
.
h 0
h
lim

4.2.6. N , , :
x 3 , xxx133,, xx11
f ( x )) ff((xx))

x , x t 1xx,, xxtt11

x ,
) f ( x ) ff(2(xx))
x ,

e x ,
f ( x ) ff((xx))
ln x ,

e x , xdeex0x,, xxdd00
f ( x ) ff((xx))
f ( x)
! 0xx,, xx !!00

x , x

xdee1xx,, xxdd11

x!ln
ln1xx,, xx !!11

 x0x,, xx00
x
x 2 ,
f ( x ) )ff((xx))
22
xxxt ,0, xxtt00
2 x ,

xxx122,, xx11

xt221xx,, xxtt11

e x , xdeex0x,, xxdd00

ff((xx))
x !11,0, xx !!00
1,



x 3, x 3x,  1xx13 , x  1
f ( x )f ( x) f ( x )
x, x ,t 1x t 1x , x t 1
e x , e xx, d 1x de1x , x d 1
f ( x )f ( x) f ( x )
)
, xx ,! 1x!ln1 x , x ! 1
ln x ln

, xx,  0x
 0x ,
x
f ( x )f ( x) f ( x )
2
2
2
x , x , x t 0x xt 0,

x  0 x 2, x 2x,  1xx12 ,
f ( x )f ( x) f ( x )
x t 0 2 x, 2 xx, t 1xt21x ,

e x , e xx, d 0xde0x , x d 0 e x , e xx, d 0xde0x , x d 0


 
f ( x )f ( x))f ( x )
) f ( x )f ( x) f ( x )
!
!
!
!
!
!
x
x
x
x
x
x
x
x
x

,
,
0

0
,
0
1
,
1
,
0
1
0
,
0


x 1
x t 1215


4.2.7. N :
1 6 1 5 1 2
) f ( x ) = x + x + x + 1
) f(x)=(x101)(x92)
6
5
2

) f ( x ) =

x5 x 4 x3

+ x2
4
3
2

) f(t)=(t2+1)(t2+3)

4.2.8. N .
x2 x+2

x+2 x2
1
) f ( r ) = r 3 + 3
r

) g ( x ) =

) f ( x ) =

x +1
+
x 1

x 1
x +1

) f(t)=2t2+t2 ( )

4.2.9. N :
5
) f ( x ) = 3 x + 5 ln x 3 x
2
) x x + 6 x

) f ( x ) 2x  3x  ln 3 
) x 2 x + ( x 2 + 1) x

4.2.10. N .
x ex
3
f
(
x
)
=
f
(
x
)
x

x

)
)

ln x
x x + x
x x
) f ( x ) = x
) f ( x ) =
1 + x x
e

4.2.11.
 f
f(x)=2x+x3, xx /4.

 f f(x) = (x ) (x ) (x ) ,, 4.2.12.
. x,,, :
f c( x )
f ( x)

1
1
1


.
x x x

 f(x)=5x2
4.2.13.
24 x + 1
A(1,1), .
g( x) =
5x
4.2.14.
 f(x)=x39x+3,
y=10x5.
4.2.15. P(x) , P=1 P()(0)=2 =1,2,3.
4.2.16.
 h(x)=g(x)e2g(x). g(1)=1 g(1)=1,
h 1.
4.2.17.
 f(x) = x+x, , ,
f(x)+2f(x)=0 xR.
4.2.18.
 f(x) = eax, aR, f(x)7f(x)10f(x)=0
xR.

216

4.2.19. N
 A(0,f(0))

f f ( x ) (  x 3 ) .
3
4.2.20. N :
) f(x)=(x3+2x2+x)8
) f(x)=(x63x2+1)15
) f(x)=x55x
1
) f ( x )
) f ( x ) 3 x 
) f(x)=(x3+2x+1)

1 x2
4.2.21. N .
) f(x)=ln(2x+3)+e2x1
) f(x)=xex
) f ( x ) (ln x )  x x 2  4 
) f ( x ) x 2 (2 x )  2 x 
4.2.22. N .
) f(x)=(3x)1/4+(5x)1/3
) f(x)=5x+2x
) f(x)=(3x4+4x3)2
) f(x)=25x3
4.2.23. N :
1

) f ( x ) (3 x )1/ 4  (5 x )1/ 3  ) f ( x ) = ln x
x

1
5x
x
) f ( x ) = (ln x ) , x>1
) f ( x ) = ln e + 2

x +1

) f ( x ) =

1
1 + x 2

4
3 x +1
)
) f ( x ) = ln( x + 2
2

4.3 .
'
(...).

.
4.2, f ,
. :
, f(x)=0, f
(' ); ' ,
, , ....
( ).
.
f [, ]
(,) , , (,) , :
f c()

f ( )  f () .



4.3,
(, f()) Cf , ,
(, f()) B (, f()) ( ).
AB

f ( )  f () .



217

:
f ( )  f ()


4.3.
f f() = f(), f'() = 0, (, f()) Cf
x. Rolle
4.3 4.3.
f c()

Rolle.
f [, ] (,). f() = f(), ,
, (,) ,
f'() = 0.
, :
f(x)=x39x2+8x+1, x[0,8].
Rolle , f [0,8], f
(0, 8) f(x)=3x218x+8 f(0)=f(8)=1,
(0,8) , f'() = 0, 3218+8=0. , 3x218x+8=0,
1 =

18 2 57
18 + 2 57
= 0, 48 2 =
= 5, 52
6
6

( f'() = 0).
y
f ()
f ()
f (a)

B
y
(,f())

Cf
(,f())

(,f())

A
(,f())

. 4.3.

. 4.3.
y

f(1)=0

f()=0
f()=f()
f ()=f()
O

f(2)=0

. 4.3.

. 4.3.

218

4.3.1.

f(x)=x, x[,].
) ... f [,].
) .
) xx x.
.

) f(x) = x [,]. f (,)


f '(x) = x. ....
) ..., (,) ,
:

f ( )  f ()
.
f c()
 =


d 1  (,) :

d1



d 1  d  . 


) x=0, ( ). x>0,
() =0, =x :
x 0 x 0 x x .
, x<0, () =x, =0
:
0 x 0 x x x x x .
x.

f,
x f(x)=0. x1,x2 x1<x2 ... ( f ,
(x1,x2), [x1,x2] )
(x1,x2) , :
f c()

f ( x2 )  f ( x1 ) .

x2  x1

f'(x) = 0, , f'() = 0
:
f ( x2 ) f ( x1 )
= 0 f ( x2 ) = f ( x1 ) .
x2 x1
, f x1, x2 . , f
.
x1,x2 x1,>x2.

219

, :
f . f
f'(x) = 0 x , f
.
f, g ,
x f '(x) = g '(x). fg,
, x ,
( f g )( x ) = f ( x ) g ( x ) = 0
, , fg . , c , x f (x) g (x)=c.
:
f, g . f, g
f '(x) = g '(x) x ,
c , x :
f (x) = g (x)+c.

, f
x. , ' 3.8, f
. f '(x) > 0
x x1, x2 x1<x2. , f
... [x1,x2], (x1,x2) , :
f c()

f ( x2 )  f ( x1 )

x2  x1

f ( x2 )  f ( x1 )

f c()( x2  x1 ) ! 0 

( f '() > 0 x1x2>0). f(x1) f(x2)>0 f (x1) < f(x2) f


. x f(x)<0
, f .
,
y

' .
f >0
f,
.
f'(x) >0 x ,
f (. 4.3).

. 4.3.

220

f'(x) <0 x ,
f (. 4.3).

y
f <0


f ' ' ( ) .
f '(x) 0
f ( ) . , f'(x) 0 f (
) .

. 4.3.

4.3.2.

f f'(x) =2f(x) xR.


f ( x)
) g ( x ) = 2 x R.
e
) f, f(0)=3.
.

) xR :

2x
2x
( f ( x ) 2 f ( x ))
f ( x ) f ( x )e f ( x )2e

g ( x) = 2 x =
=
2x 2
(e )
e2 x
e

f f '(x)=2 f (x), g '(x)=0 xR. ,


g R.
) g , cR , g (x)=c xR , ,
f ( x)
= c xR.
e2 x
f (x)=ce2x xR. f(0)=3, ce20 =3, c=3 f (x)=3e2x xR.
4.3.3.

N , f(x)=x39x2+24x+1
(, ) .
.

f f ( x ) = 3 x 2 18 x + 24 = 3( x 2 6 x + 8) = 3( x 2)( x 4)) .
f :
x
f(x)

2
0

4
0

+
+

221

E, f:
) (,2], (,2] f(x)>0
(,2).
) [2,4], [2,4] f(x)<0 (2,4).
) [4,+), [4,+) f(x)>0
(4,+).
f f (,2], [2,4] [4,+)
.
x
f '(x)

2
+
0
f(2) =1

f(x)

4
0

+
+

f(4) = 125

4.3.4.

f(x)=3lnx+2x10, x>0.
) f .
) .
) 3lnx+2x10=0 (0, +) R.

) f (0,+) f ( x ) =

3
+ 2 . x

3
+ 2 > 0 x(0,+)
x
f (0,+).
)
lim f ( x ) = lim (3 ln x + 2 x 10) = + ,
f ( x ) =

x +

x +

lim f ( x )

x o0

lim (3 ln x  2 x  10) f ,

x o0

0
+

f(x)
f(x)

) f R = (, +) , (. . 3.8), f(x)=0
x0. E f (0,+), .

222

, , f(x)=c, cR.
f [, ],
(,) f (,), f(x)=c, c f() f(),
(,) .

4.3.1. f(x)=3x4x3+2x23x+1.
) N
 f Rolle
[0,1].
) N
 l2x33x2+4x3=0 , ,
(0,1).
4.3.2.
 ,
, ,
f ( )  f ()
.
(,) f c()

2
) f ( x ) = , [1, 3]
) f(x)=xx2, [1,1]
) f(x)=x, [2,2]
x

x + 1, x 1
, [1,1]
) f ( x ) = x , [1, 9]
) f ( x ) = 2
3 x x , x > 1
4.3.3.
 f, [,], f() = 0
mf(x)M x (, ) (m, M ).
()mf() ().
4.3.4.
 <, f(x)=ex ... [,] < . , < :
e <

e e
< e .

4.3.5. f(x)=2x49x2+4x+2.
)  f(x)=0 , , (1,0) , , (0,1) , , (1,2).
)
 8x318x+4=0 , , (1,1).
4.3.6. f f(x)=3f(x) xR.
f ( x)
xR.
e3 x
) f, f(0)=1.
) g ( x ) =

4.3.7.
 f, g R. f, g

223

f(x)=2g(x), g(x)=2f(x) x.
) f(x)+4f(x)=0 g(x)+4g(x)=0 xR.
)  h h(x)=(f(x))2+(g(x))2 ' R.
)  f(x) = 2x, g(x) = 2x,
. '
;
2
x<0
x ,
2
4.3.8. f g f ( x ) = 3 +
g ( x ) =
x
1 + 2 , x > 0.
x
) f(x)=g(x) x0.
) fg .

 f f(x)2 xR. f(x)=2x


4.3.9.
.

4.3.10. N
(, ) .
1
) f(x)=x33x+5
) f ( x ) = x
) f(x)=x5+3x3+12x1
x
1  x 3 ,
x d1
2

) f(x)=x33x2+3x2
) f ( x )
) f ( x ) x  9 
2
2 x  1, x ! 1
4.3.11. 
.
x2
x
3
x
f
(
x
)
=
)
) f(x)=lnx +e
) f(x)=2e 3+ln(x+2)
e x
1

3
5
) f ( x ) =
) f ( x ) x  x , x [0, ]  ) f(x)=3x+5x+7x
2
16 x 2
4.3.12. f(x)=x33x2+6x+3 g(x)=ex+2x52.
) o f, g .
) .
) x3=3x26x3 ex+2x5=2 .

4.4 .
H , . :
) .
) , ,
.
) ,
, .

( ).
f, 4.4.

224

x=x1
x1 ( , M1 (x1,f(x1)) ,
x x1). f
x=x1.
:
f, ,
A = ( , + ), >0 ,

f(x)f() x A .
, f() f.
4.4 M3(x3,f(x3)) . , x x3,
. f
x3 ( ), f(x3) f.
, x= x2
x2 ( , M2 (x2,f(x2))
, x x2).
f M2 (x2,f(x2)).
:
f, , A, = ( , + ), >0, ,
:
f(x)f () x A .
, f() f.
f(x)f () xA, f A
, f() f. f 4.4, () M4(x4,f(x4)).
f ,
f .
f . , ,
,
.
. f 4.4. f
x1 x5. x4, x6 (
x1) , x3, x7 ( x5)
.
, x5, x6, ,

225
y
f(x3)

1(x1, f(x1))

f(x1)

x1 x2
O x3

f(x2)

2
f(x4)
x1

3(x3, f(x3))

x2

x4

x6
x5

x7

4
x3

x4

. 4.4.

. 4.4.

xx, :

f c( x5 ) 0 f c( x6 ) 0.
, x2 ,
f(x2)=0. x3,x4 , f .
, f
:
) ( )
) , f ( ),
.
() f.
( ) ,
Fermat.

Fermat.
f ' x0
f(x0), : f(x0)=0.
, f, f
, .
.
, x2=0 4.4 f(x2)=0, x2
.
, . ,
.
f 4.4 x3 :
) f ' (,x3], , f(x)f(x3)
x(, x3],
) f ' [x3, ), , f(x)f(x3)
x [x3, ).

226

' f(x)f(x3) x = (, ) ( f
x3). x5.
, f x4 :
) H f (,x4] f(x)f(x4)
x (, x4].
) H f [x4, ), f(x)f(x4)
x [x4, ).
, x =(, ) f(x)f(x4), f x4. x6.
( ):
f , (,) x 0[, )
f.
)
 f(x)>0 <x<x0 f(x)<0 x0 < x < , f x0 (. 4.4).
)
 f(x)<0 <x<x0 f(x)<0 x0 < x < , f x0 (. 4.4).
y

y
f <0

f >0

f <0

f >0
f(x0)

f(x0)
O

x0

x0

. 4.4.

. 4.4.


, f ,
f . f
.
, f(x)=x33x+2 (. 4.4), R. f
f(x)=3x23. :

f c( x ) 0 3 x 2  3 0 x 1 x

1 ,

f :

f c( x ) ! 0 3( x  1)( x  1) ! 0 x  1 x ! 1
f c( x )  0 3( x  1)( x  1)  0 1  x  1.

f (,1] [1,1],
f(1)=4 . f [1,1]
[1,+], f(1)=0 (. 4.4).

227
y
4

f(x)=x3 _ 3x+2
2

O
_1

. 4.4.

. 4.4.

4.4.1.
4.4.1
x
f(x)
f(x)

1
1
+ 0 0
4

x0=1

+
+

0

x0=1

f x0 ( f ).
, (. 4.4):
2 x 3 ,
x 1

f ( x)
2( x  2)2 , x t 1
2
x 1
6 x ,

R R{1} : f c( x )
4( x  2), x ! 1
x0=1 f. , f(x)>0 x<1 f(x)<0
1<x<2, f x0=1
,
,
( ) , .
' , f x0 x0 .
.

f ' x0
, f(x)=0.
)
 f(x0) f(x0)<0, f
x0.
)  f(x0) f(x0)>0, f
x0.

228

f(x0)=0
f(x0)=0,
x0 ( ).
, f(x) = 2x
(. 4.4) R. f R :

f c( x ) 22x ,
f cc( x ) 4 2 x. 

x  , .
Z. 
x x=,
,
f(x)=0
2
f()=42=4<0, , f 0, , 2,... .
,

f cc(  ) 4 2(  ) 4 4 ! 0, 
2
2

3
, ... .
f ,
2
2
, ,
3.8, f
[,], x1, x2 [a, ] :
f ( x1 ) f ( x ) f ( x2 ) x [a, ].
, , f ()
() [a, ]. , f (a, ), :
) f.
) f , .
) f f.
, f(x)=x44x3+4x21, R f(x)=4x(x23x+2)=4x(x1) (x2) x = 0, x = 1, x = 2.
:
3
) [ 1, ] , (. 4.4) f 2
3
7
, f(1)=8, ,
f(1)=8, f(0)=1, f(1)=0, f ( ) =
2
16
3
7
f (1) = 8, f (0) = 1, f (1) = 0, f ( ) = , f(0)=1.
2
16
8

6
1
4
_4

_2

4
2

_1

. 4.4.

0,5
_

0,5

. 4.4.

1,5

229

1 5
) [ , ] ,
2 2
(. 4.4) f
1
7
5
9
f ( ) = , f (1) = 0, f (2) = 1 , f ( ) =
,
2
16
2 16
5
9
, , f ( ) =
2 16
1
7
f ( ) = , f (1) = 0, f (2) = 1, f (4) = 63 ,
2
16

1
0,5
0,5

1,5

2,5

_ 0,5
_1

. 4.4.

f(2)=1.

4.4.1.

x.

24 m2 ,
.

y
x

y ,
:
=2xy+2xy+2x2=4xy+2x2.
24 m2, E=24,
4 xy + 2 x 2 = 24 4 xy = 24 2 x 2 y =

24 2 x 2
.
4x

:
2
V=(xy)x = x

24 2 x 2 1
= (12 x x 3 ) .
4x
2

E :
1
f ( x ) = (12 x x 3 ),
2

x >0

1
2
f ( x ) = (12 3 x ) . f(x)=0
2
x=2. f f 4.4.2
4.4.2.
x
0
2
+
f(2)=0 f 2, f x=2
+
0

f' (x)
, :
f(x)
1
f (2) = (12 2 23 ) = 8.
2

230

,
(8 m3) :
24 2 22
x = 2m y =
m
=2m
42
( , ).

f(x)=x3, g ( x ) = x [0,+),
4.4.
1
>0
(0,+) f(x)=3x2>0 g ( x ) =
2 x
x>0. [0,+) 0. 4.4
. , f ,
f ,
g , g
. , x
, f(x) Cf , f [0,+), g(x)
Cg , g [0,+).
f [0,+)
g [0,+).
:
f , '
. f :
) , f' .
) , f ' .
, f ' , ,
Cf , (
) .
f ' , ,
Cf , (
) .
y

f(x)=x3

g(x)= x

O
x

. 4.4.

231

4.4.

1


, 

.
'
, , ,
.
f ' , . f :
) , f(x)>0 .
) , f(x)<0 .
, f(x)=x3, R,
4.4. f(x)=3x2>0 xR*, f
R. f(x)=6x>0 x>0 f(x)=6x<0 x<0. , f
(,0], [0,+).
f x0=0
O(0,0) y=0, . Cf
x0.
:
f (, ) x0. (x0, f(x0))
f :
) H f (a, x0) (x0, )
) Cf (x0, f(x0)).
y
y

y=x3
_

Cf
+

. 4.4.
. 4.4.

1.  , ,
, . , ,
. f(x) = x + (
) . , /
/
( f(x) = x + ).

232

f(x)=x3 , O(0,0) f, f(0)=0.


( ):
(x0, f(x0)) f,
f(x0)=0 f x0.

:
) , f
) , f.
1 3
4 x ,
, f ( x ) =
( x 2)2 2,

x2
x>2,

4.4.
3 2
x ,
f ( x ) = 4
2( x 2),

x<2

3
x ,
f ( x ) = 2
2,

x>2

x<2
x > 2.

f(x)=0 0, 2 f.
f 4.4.3.
, 0 2. 0 2 f . (2,2) Cf .
O(0,0) f, A(2,2) .
, f x0
Cf (x0, f(x0)) (. x0),
(x0, f(x0)) f .
y

4.4.3

x
2

_2

f(x)

fx)

. 5.4.

4.4.2.

f(x)=x39x248x+100.

+
+

233

f R :
f(x)= 3x218x48 f(x)= 6x18.
: f(x)=0 3x218x48=0 x=2 x=8 f(x)=0 6x18=0 x=3.
,
(/) 4.4.4 f.
x=2 f(2)=152 x=8 f(8)=348. (3, f(3)),
(3, 98). 4.4 f.

200

4.4.4

x
f
f

2
0

8
0
+

+
+

_5

10

15

_ 200

_ 10

_ 400

. 4.4.

H . ,
:
) f.
) f .
)  f f ( ) .
)  f
f.
)
 f , f .
) 
( , ..).
)  f
f .

4.4.3.

f f(x)=ex.
) f xR.

234

) Cf
A(0,1).
) ex0x0+1,
x0R.
.

y
f(x)=e x

e x0
x0+1

) f f(x)=e
y=x+1
R. , R, f(x)=ex
f(x)=ex. f(x)=ex>0 xR, f , R.
x0
x
O
) f(0)=e0=1,
. 4.4.
Cf A(0,1) y1=1(x0), y=x+1 (.
4.4).
) f R,
, (x0, f(x0)). , x0 R :
:

f(x0)y0=x0+1
e 0 x0 +1.
x

4.4.1. ( ) .
) f(x)=2x43x312x2+5
) f(x)=2x34x22x+3

) f(x)=x23x+5
ln x 1
) f ( x ) =
x

) f(x)=2(x24)5
) f(x)=xe2x

4.4.2. .
) f(x)=2x3+4x+1
x

) f(x)=2 +3 +4

) f(x)=ex +lnx1
2 x

) f(x)=x e +3e

) f ( x ) = x (1 + x )
) f(x)=x+3x

4.4.3.
 f,
f(x)=3(x24) (x2)2 (x4)2.
4.4.4.  f(x)=2x3+3x212x+6
x2(2x+3)=12x6.

4.4.4.
 40 cm
.
4.4.6.
 ,
:
) N
 3 cm
( ) 2 cm.
) T
 294 cm2.
x
y (. 4.4).

. 4.4.

235

) x
1176
E( x ) = 6 x +
+ 318 .
x
)
 x,y ,
.
4.4.7.
 ,
4.4. 1m,

;
4.4.8.
 ,
(' )
y=3x2. '
A(2,3).
4.4.9.
 2 m
, 4
, 4.4. x , , .

. 4.4.
x

2m

2m

. 4.4.

4.4.10.
 '
y
. ,
,
4.4, , .
y
20 , x
30 . . 4.4.
1000 , x y , .

4.4.11.  f
f(x)=(x2)(x1)2(x+1). N ,
.
4.4.12. N f .
1
) f ( x ) = 2 x
) f(x)=1+9x+x2
) f(x)=x2ex/2
x
2
3
2
) f(x) = 2x
) f(x) = ex /2
) f(x)=x 5x +x+1
x
) f ( x ) = 2
) f(x) = xex
) f(x) = x3 | x |
x +1

236

4.4.13.  f f(x)=x(x1).
f , .
4.4.14. f(x)=x45x2+4.

4.5 L Hospital.
f1 f2 :
f1( x ) =

ln x
e x 1
f2 ( x ) =
,
1
x
1+
x

A1= (,0)(0,+) A2=(0,+) .


x 0,
e x 1
,
x 0
x

lim f1( x ) = lim


x 0

lim f2 ( x )

x o0

lim

x o0 

ln x
,
1
1
x

,
f1
lim(e x 1) = e 0 1 = 1 1 = 0 lim x = 0
x 0

f2,

x 0

1
lim (1  ) f .
x o0
x o0
x
0

.

0
+

L Hospital , .
lim ln x

f

K L Hospital.
, x0 f, g
{x0}. g(x)0 {x0},
f c( x )
( ) ' :
lim
x o x g c( x )
0
) lim f ( x ) 0 lim g ( x ) 0 
xo x

xo x

) lim f ( x ) f  lim g ( x ) f  .
xo x

xo x

lim
xo x

f ( x)
 :
g( x)
f ( x)
f c( x )
lim
lim
.
xo x g( x)
x o x g c( x )
0
0

237

x x0+ , x x0 , x+
x, ( ).
0
e x 1
f ( x)
= lim
T lim
:
x 0
x 0 g ( x )
0
x
f ( x ) (e x 1) e x
=
=
= ex .

( x)
1
g ( x)

f ( x )
= lim e x = e 0 = 1,
x 0 g ( x )
x 0

lim

L Hospital
f ( x)
f ( x )
= lim
= 1.
x 0 g ( x )
x 0 g ( x )

lim

, :
lim

x o0

1
lim x
1
x o0
 2
x

(ln x )c

c
1
1


lim ( x )

x o0 

lim

x o0

f ( x)
g( x)

lim

x o0

f c( x )
g c( x )

lim( x ) 0 
x o0

0.

, L Hospital
, .
,
e5 x
f ( x)
, f ( x ) = e5 x , g ( x ) = x 3
= lim
3
x + x
x + g ( x )
lim


lim f ( x ) = lim g ( x ) = + , lim f '( x )

x +

x +

x of

lim g '( x ) f, lim f ''( x )

x of

x of

lim g ''( x ) f 

x of

e5 x
5e 5 x
25e5 x
125e5 x
=
=
=
= + .
lim
lim
lim
x + x 3
x + 3 x 2
x + 6 x
x +
6
lim

L Hospital
0 ^ (), () (), ,
.
lim ( x ln x 2 ) , (+)(+) ( lim x = +
x +

x +

lim ln x 2 = +) :

x +

(
x)

x ln x 2 = x 2 ln x = x(1 2

ln x
)
x

238

11
ln x
(ln x )
= lim
= lim xx = 00 
lim
x + x
x + ( x )
x + 11


lim (1 2

x +

ln x
) =1
x

ln x
) = + .
x +
x +
x
, (f(x))g(x) 00, 0, 1. , L Hospital
(f(x))g(x)=eg(x)ln(f(x)).
lim x x  00
lim ( x ln x 2 ) = lim x(1 2

x o0

lim xxln x0. lim x


xx=exlnx lim
xxo
o00

x o0

0  lim ln x
x o0

f 

0()

ln x
.
x ln x =
1
x
1
lim ln x f  lim
f  :
x o0 x
x o0
1
c
ln x
(ln x )
lim x ln x lim
lim
lim x
lim ( x ) 0 
1
1
1 x o0 
x o0
x o0
x o0
x o0
( )c
 2
x
x
x

lim x x

x o0

lim e x ln x

x o0

lim ( x ln x )

e xo0

e 0 1 .

4.5.1.

f f ( x ) =

ln(x 1)
.
x2

A=(1,2)(2,+).
Cf 1, 2 +.
) x=1 :
lim ln(x  1) f lim ( x  2) 1 
x o1

x o1

ln(x  1)
f .
x o1
x2
x=1 f.
) x=2 :
lim( x 2) = 0
lim ln(x 1) = ln1 = 0

lim

x 2

x 2

239

L Hospital :
1
( x 1)

ln(x 1)
(ln(x 1))
1
lim
= lim
= lim x 1
= lim
= 1.
x 2 x 2
x 2 ( x 2)
x 2
x 2 x 1
1
x=1 f.
ln(x 1)
) f +. lim
x + x 2
:
1
(ln(x 1))
lim ( x 2) = + lim
lim ln(x 1) = + ,
= lim x 1 = 0 .
x +
x +
x + ( x 2)
x + 1

ln(x 1)
lim
=0.
x + x 2
y=0 f.

4.5.1. .
x
x2
lim
lim
) x + x
) x + e x
e
2x
) lim
x 0 ln( 2 x + 1)

1 x
x 0
x2

) lim

) lim
x o0

1  x

x  x

e3 x
lim
) x + x

2 
ex
1 e x
lim
) x 0 x

) xlim
of

) xlim
o0

x

x

4.5.2. :
( x 4 ln x )
) xlim
+

2
( x ln(1 + ))
) xlim
+
x
1 2
(1  ) x 
) xlim
of
x

( x 3 ln x )
) xlim
+

) lim

ln x 3
lim
) x + x 5

ln x
) xlim
+ e x
1
1
lim( 2 2 )
) x 0 x
x

x 4 ln x 2
) lim
x 0

x 5e x
) xlim
+

x100 e x
) xlim
+

(3 + 2 x )1/ x
) xlim
+

xe x 2 ln(1 + x )
x 0
x2

4.5.3. :
x2 3x + 2
lim
) x 1 x 2 2 x + 1
) lim
x 0

ln( x + 1)
e x 1

) xlim
2

( x 2)
x2 5x + 6

ln( x 2)
) xlim
2 x 4 x + 3

) lim
x 0

ln( x + 1)
x2 3x

e x 1 1
lim
) x 1 x 2 2 x + 1

240

4.5.4.
 .
ln x 2
x2
ln x
) f ( x ) = x
) f ( x ) = 2
) f ( x ) =
( x 1)( x 2)
x
3

4.6 .
(), t=0
xx (. 4.6). (
) S(t) t (.
).
H S t
.
, t=0 , t0
M0 h, t=t0+h,
(. 4.6). [t0,t]
S=S(t)S(t0)
S=S(t)S(t0) t=tt0=h , :
S S( t) S( t0 ) S( t0 + h) S( t0 ) .
=
=
t
t t0
h

O
x

x
S(t)

. 4.6.

M0

S
M
x

S(t0)
S(t)

. 4.6.

t t0 (t t0 h0),
t0. ,
t t0
S( t + h) S( t0 )
lim 0
= S( t0 ) ,
h 0
h
t0 (t0).
.
, , S/t [t0,t],
S( t0 ) = lim(S( t0 + h) S( t0 )) / h ,
h 0

t0 ( t0). , .
, S(t)=2t2+10t ,
t1=1, t2=2 t3=3 ( S(t)=4t+10)
SSSc(c(1(11)))= 444111+10
444 222+10
444333+310
= 222.
10
10
10= 666,,, SSSc(c(1(2)2))= 
10= 22,2,,SSS(c1c(()33)=) 
10
( , , t0

S( t) S( t0 )
> 0 ,
t t0

241

S( t) S( t0 )
< 0,
(t0)0, , , t0
t t0
(t0)0).
:t(t)=S(t). ,
( t) ( t0 ) ( t0 + h) ( t0 )
=
t t0
h

t t0 (t t0 h 0) , t0.
( t0 + h) ( t0 )
= ( t0 ) = S( t0 )
h 0
h
lim

t0. ,
t0 (t) () t0.
:
S(t) t,
t (t)=S(t) a(t)=(t)=S(t) .
,
1
S( t) s0  0 t  a t 2 ,
2
s0, 0, a ,
1
( t) = S( t) = ( s0 + 0 t + a t 2 ) = 0 + a t a( t) c( t) ( 0  a t)c a ,
2
( ).

y x.
:
x,y y=f(x), f ,
df ( x )
f(x0)=

dx x x0
y x x=x0.
:
) A y x y=f(x)=x3,
x=x0 f(x0)=3x02. 1 ( ) 312=3,
2, ( 322=12).
) A Q(t) -

242

t,
t0 ( , t0),
Q(t0).
) A J(t)
t,
t0, J(t0).

. , x .
x K(x) x
E(x).
:
) T x
P ( x ) = E( x ) K ( x ) .
) T x
K ( x)
.
x
x0 x (
x ),
K ( x) =

P ( x ) P ( x0 ) P ( x0 + h) P ( x0 )
=
, h=xx0
x x0
h

x0 x .
x x0 ( h0), ( )
x0. :
P ( x 0 + h) P ( x 0 )
= P ( x0 ),
lim
h 0
h
x0.
x0 K(x0), E(x0) .
:

K ( x ) K ( x ) x K ( x )
P ( x ) = E( x ) K ( x ) K ( x ) =
=
x2
x
:
P ( x ) = 0 E( x ) K ( x ) = 0 E( x ) = K ( x )

K ( x ) = 0 K ( x ) x K ( x ) = 0 K ( x ) =

K ( x)
K ( x ) = K ( x ) .
x

, , :
) .
)
.

243
y

4.6.1.

(. 4.6).
40 ./h,
50 ./h.
)

.
)
 D(t)
3 .

D(t)

y(t)

x(t)

. 4.6.

) 40 50 ./h,

1
S( t) s0  0 t  a t 2 
2
s0=0, a=0 0=40, 50 . : x(t)=40t, y(t)=50t.
) t :
D( t) = [ x( t)]2 + [ y( t)]2
D(t) t :

Dc( t)

[ x( t)]2  [ y( t)]2
1

2 [ x( t)] + [ y( t)]
2

1
2 [ x( t)]2  [ y( t)]2

(2 x( t) x ( t) + 2 y( t) y( t) ) =
2

[ x(t)]

 [ y( t)]2

x( t) x ( t) + y( t) y( t)
[ x( t)] + [ y( t)]
2

t0=3 x(t0)=40, x(t0)=40t0=403=120, y(t0)=50,


y(t0)=50t0=503=150, :
D( t0 ) =

x( t0 ) x ( t0 ) + y( t0 ) y( t0 )
[ x( t0 )] + [ y( t0 )]
2

120 40 + 150 50
120 + 150
2

12300
= 64, 03 .
36900

64 ./h.
4.6.2.


t ,
f ( t) = 2 e

t
100

t0.

t,
1/10
t0=0.

244

f ( t) = (2 e

t
100 ) =

2e

t
100

t
1
1
) = 2 e 100 (
) = e 100 .
100
100
50


t0 =0 :
0
1 100
1
= .
f (0) = e
50
50
1
t , f ( t) = f (0) ,
10
t

1
1 1
1
t
1
e 100 = e 100 =
= ln = ln10 t = 100 ln10 = 230, 26 .
50
10 50
10
100
10

4.6.3.

- t

3T ( t)
E( t) = 2 out
4Tin ( t)
Tin=Tin(t) Tout= Tout(t) (0C). Tin 6oC/min , E(t)
[. E(t)=E t], Tout
.
.

3T ( t)
E( t) = 2 out
Tout(t) E(t)=E
4Tin ( t)
t, :

E( t ) = 2

3Tout ( t)
3T ( t)
4(2 E)
Tin ( t) .
out = 2 E Tout ( t) =
4Tin ( t)
4Tin ( t)
3

,
( t) =
Tout
Tin ( t) =

4(2 E)
Tin ( t)
3

dTin ( t)
= 6o C / min
dt
dT ( t) 4(2 E)
( t) = out =
Tout
6 = 8(2 E) o C / min .
dt
3

4.6.1.
 ' , t
9
S( t) = 3t 3 t 2 + 2t + 1 . t
2
. ;

245

4.6.2.
 -
t S(t)=32t.
) t.
) , .
;
4.6.3.
 '
S(t)=s0t, t s0, .
) t.
) y.
)  , 0, .
4.6.4.
 f g f(t)=2e3g(t)+1. g(1)=0 g(1)=1,
f t=1.
4.6.5.

3cm2/min.
5cm (
d E = d2).
4.6.6.
 R(t)
R(t) =3t2+5t+1. ,
' t=10sec.
4.6.7.
 10 cm3/sec.
R(t) ,
4 3
5 cm ( r E
r ).
3
4.6.8.  . , , , t ,
V(t)=10(20t)3.
)  3
5 .
) N .
)  1
.
4.6.9. T
 o , K(x), , E(x), x :
K(x)=x380x2+2400x+1 E(x)=300x,
. x ,
.
4.6.10.
 . K(x),
, x :
2
K ( x ) = 200 + 80 x x , x [0, 200] ,
3

246

x
2
E( x ) = 70 x + x .
2
) K(10) .
) N 5 ( ).
) E(10) .
)  P(10), P(x) x .
)
 ,
.
4.6.11.
 . N(t)
t (min) N(t)=104(100t)2.
)  [0, 1] min
[0, 10] min.
) 10 .
) .
4.6.12.
 10 m
5 m. , z
t ( cm) z(t)=0,2t2. V t0=5 sec.

4.7 M .
, . r h
V=hr2.
,
A={(r,h): rR hR} V(r,h)=hr2,
r h . ,
r=1 m h=2 m,
:
V (1, 2) 2 12 V (1, 2) 2 m3.
.
,
.
:
A R2,
A ^( x , y) : x R y R` R 2.

247

, (x,y) z f A.
z f(x,y) (z=f(x,y)) ,
x,y .
z=f(x,y)R (x,y)A.
V(r, h)=
hr 2, V,
r, h.

, ....

, OM M(x,y,z)
O(0,0,0) -

z
(OM)= (x_ 0)2+(y_ 0)2+(z_ 0)2 =
= x 2 +y 2 +z 2
M(x, y, z)
y

O(0,0,0)

xyz (. 4.7).
OM = x 2 + y 2 + z 2

. 4.7.

f ( x , y, z) = x 2 + y 2 + z 2 .
f (1,2,3) M(1,2,3)
O(0,0,0) :
f (1, 2, 3) = 12 + 22 + 32 = 14 , (OM ) = 14 .

25

4.7.1.

20

15
10

f ( x , y) = x 2 + y .

5
.

;
.

_4

_2

. 4.7.

(x,y) f .


x2+y0 yx2.
(4.7.1)
A y=x2 (4.7.1)

y=x2 (. 4.7).

248

f ,
:
z = f ( x , y) = x 2 + y 0
(x,y) yx2 , z0 ( )
(x,y) yx2, :
z = x2 + y x2 + y = z2
( , .. x=0, y=z2).


z = f ( x , y) = 50 x 2 y 2 xy.

(4.7.2)

y
x
dz d
d
=
f ( x , y) = (50 x 2 y 2 xy) = 0 2 x y = 2 x y.
dx dx
dx
d
f ( x , y) z=f(x,y) x
dx
y f x. ,
f x, :

w
wf ( x , y)
f x ( x , y) 
f ( x , y)
wx
wx

f
z
.
fx
x
x

, (4.7.2) :

f ( x , y) = 2 x y .
(4.7.3)
x

, , x (x0,y0).
f ( x , y) = 2 x y (x0,y0)=(1,2) 212=4.
x
:

f ( x , y)
= 4
x
x =1, y = 2

f ( x , y)
= 4 .
x
(1, 2)

, f x (x0,y0)
w
wf ( x , y)
f ( x0 , y0 ) f x ( x0 , y0 )
wx ( x
wx

,y )
0

wz
wx ( x

.
0

,y )
0

249

,
(
, 4.1)
f f((xx, ,yy00))f f((xx00, ,, ,yy00))
f f((xx hh, ,yy00))f f((xx00, ,yy00)) .
ww
lim
lim
lim
lim 00
f f((xx00, ,yy00)) lim


x xo
oxx0
hhoo
00
wwxx
xxxx00
hh
00

, f y, :
w
wf ( x , y)
f ( x , y)
wy
wy

f y ( x , y)

f
z
fy
.
y
y

(4.7.2) :

f ( x , y) = (50 x 2 y 2 xy) = 0 0 2 y x = 2 y x ,
y
y

f y (x0,y0)=(2,3), :

f ( x , y)
= (50 x 2 y 2 xy)
= 2 y x ( 2, 3) = 2 3 2 = 8 .
y
y
( 2, 3)
( 2, 3)
,
n. ,
.
(. . 4.2).
,
( ),
, . ,
(4.7.3) x y :




f ( x , y) = (2 x y) = 1 , f ( x , y) = (2 x y) = 2 .
x x
y x
x
y

2
2 f ( x , y)
f ( x , y) =
yx
yx

2
2 f ( x , y)
.
f
(
x
,
y
)
=
x 2
x 2

:

2

f ( x , y) = f ( x , y) = (2 y x ) = 1 ,
xy
x y
y

2

f ( x , y) = f ( x , y) = (2 y x ) = 2 .
2
y y
y
x

250

4.7.2.


f(x,y)=ex y.
.

f
y . :
x
w
w x
w x
f ( x , y)
(e y)
(e ) y e x y .
wx
wx
wx

f
x ,
y

w
w x
w
f ( x , y)
(e y) e x
( y) e x y .
wy
wy
wy
4.7.3.

f
: f ( x , y) = e x ln( x 2 + y 2 + 1) .
y

x
. :
1

f ( x , y) (e x ln( x 2 y 2 1)) e x
( x 2 y 2 1)
( ln( x 2 y 2 1)) e x 2
2
y
y
y
x y 1 y
ex

ex
1
2 y ex
y
y
0
2
0
2

(
)
x 2 y2 1
x 2 y2 1
1 x 2 y2

4.7.4.

R1, R2, R3, ,


Ro :
1
1
1
1
=
+
+
.
R R1 R2 R3
R
.
R1
R
) ;
R1
.
1
1
1
1
=
+
+
)
, R R1 R2 R3
Ro

R =

R1 R2 R3
R2 R3 + R1 R2 + R1 R3

251

R2, R3 R1 :

wR
wwwRRR
1
wwRR11

w
R1

R1
R2 R3 ww

R
1
R1 R2  R2 R3  R1 R3
RR22RR33 wwR
1

wRR11 RR11RR22  RR22RR33  RR11RR33


w
w
w ( R1 ) ( R1 R2  R2 R3  R1 R3 )  R1 ww ( R1 R2  R2 R3  R1 R3 )
w
wR ( R1 ) ( R1 R2  R2 R3  R1 R3 )  R1 wR ( R1 R2  R2 R3  R1 R3 )
R2 R3 wwRR11( R1 ) ( R1 R2  R2 R3  R1 R3 )  R1 wwRR11( 2R1 R2  R2 R3  R1 R3 )
( R1 R2  R2 R3  R1 R13 )2
RR22RR33 1

((RR11RR22  RR22RR33  RR11RR33))2

1 ( R R  R R  R R )  R ( R  0  R )
R2 R3 11((RR11RR22  RR22RR33  RR11RR33))RR11((RR222 00 RR33))
1 2 (R
2 R3  R
1 3
3
R11 R3 )222
RR22RR33
1 2  2 R3 

R
R
R
R
R
R
(
)
( R11R22  R22R33  R11R33)
R1 R2  R2 R3  R1 R3  R1 R2  R1 R3
R2 R3
R2RR3
R2 R3 RR1RR2  RR2RR3  RR1RR3 RR1RR2 2RR1RR3 R2 R3
R
2
RR22RR33 1 2 ( R12R23  R21R33  R11R32 )22 1 3 RR22RR33 ( R1 R2  R22 R33  R1 R3 )22 .

R
R
(
R
R

R
R
)
(
R
R

R
R

R
R
)
( R11R22  R22R33  R11R33)
( R11R22  R22R33  R11R33)

R
R2 2 R32
.
=
R1 ( R1 R2 + R2 R3 + R1 R3 )2

(4.7.4)

) , 4.6, . , ,

( ),
.
R
, ,
R1
Ro R1, R2 R3 .
Ro R1 R1 = 1 Ohm
R2 = 2 Ohm, (4.7.4) :
R
R
12 22
4
=
=
.
2
R1 ( R1 + 2 + 2 R1 )
R1 (3 R1 + 2)2
R1 1 Ohm , Ro
4/(31+2)2=4/25 ( R2 R3
R2=1 Ohm R2 =2 Ohm).

4.7.1.  , xOy
M0(x0,y0). MM0 M(x,y)
M0(x0,y0).
;

252

4.7.2.  x, y, z (. 4.7).
.
)  ,
.
)
 ,
z
y
.
)  5m3 x
, . 4.7.
.
2
)  10m
,
x y.
4.7.3. :
) f ( x , y) = xy

) f ( x , y) = xy

) f ( x , y, z) e x sin( y z)

) f ( x , y) = x 2 + y 2 2

) f ( x , y) = e x 1

) f ( x , y) =

1
x + y2
2

4.7.4. .
) f ( x , y) = x 2 + y 2

) f ( x , y) = y e x

) f ( x , y) = 4

2
2
) f ( x , y) = ln( x + y )

) f ( x , y) = e x sin y

5
) f ( x , y) = ( xy)

4.7.4.

f f
.
,
y z

2
2
) f ( x , y, z) = x + y + 2 z

x
) f ( x , y, z) = y z e

x
) f ( x , y, z) = e sin( y z)

2
2
2
) f ( x , y, z) = ln( x + y + z )

z
) f ( x , y) = ( xy)

5
) f ( x , y, z) = ( x y z)

f

z
f ( x , y, z) = ( x 2 + y 2 + z 2 )1/ 2 .

4.7.6.

4.7.7.  , , x
t , w( x , t) = cos(5t 2 x ) e 0,2 x .
w w
,

.
t x
4.7.8.
 V

V V V
E =
,
,

y
z
x
V ( x , y, z) = x 2 + y 2 2 z 2
V V V
,
,
)
.
x x z

253

) E (1, 1, 1).
4.7.9.  , W

V 2
W ( P , V , , , g ) P V 
,
2g
, V
, , g
.
wW wW wW
,
,
)
.
wP wV w
)
 W (. ) g
, ;
(
). ; .
4.7.10. x, y r, :
x=rcos y=rsin.

wx wy wy wx

wr w wr w

r.

4.7.11. f ( x , y) =
x

1
x 2 y 2
2

f
f
+ y 2 x = 2 x( x + y 3 ) .
x
y

3 2
3
2
4
4.7.12. f ( x , y) = 3 x y + 2 xy + x + y .

f(f x(,xy, )y) f(f x(,xy, )y)


)
.
,,
xx
yy
)

2 f ( x , y)
,
x 2

2 f ( x , y)
,
y 2

2 f ( x , y)
,
yx

2 f ( x , y)
.
xy

2 f ( x , y) 2 f ( x , y)
=
yx
xy
(. x
y),

) x y

2 f ( x , y)
2 f ( x , y)

.
x 2
y 2

4.8 .
f
x0.

f ( x0 ) = lim

x x0

f ( x ) f ( x0 )
f ( x 0 + h) f ( x 0 )
= lim
h 0
x x0
h

254

f
x0.
Cf A(x0,f(x0)) (x0
f).
.

fc ( x0 )

lim

h o0

f ( x 0  h)  f ( x 0 )
h

f ( x 0  h)  f ( x 0 )
fc ( x0 ) lim
h o0
h

yf(x0)= f(x0)(xx0)
' x0,
x0. To .
( f + g )( x ) = f ( x ) + g ( x )

( f g ).

(.M.T.)
Rolle.

(cf )( x ) = cf ( x )
( f g )( x ) = f ( x ) g ( x ) + f ( x ) g ( x )

f
f ( x ) g ( x ) f ( x ) g ( x )
(g(x)0)
( x) =
[ g ( x )]2
g
( f g )( x ) = f ( g ( x )) g ( x )
f
[, ] (, ),
, , (, ) :
f c()

f ( )  f ()
 (.M.T.)


f()=f(), f( )=0 (
Rolle)
f f .
.
f, g f g c R, :

f(x)=g(x)+c, x
.

)  f'(x) > 0 (f '(x) 0) x


, f () .
)
 f '(x) < 0 (f'(x) 0) x
, f () .

f (x) f (x0) x A
f, , ( ).
x0 A .
f (x) f (x0) x A
f, , x0 A . ( ).

255

Fermat.

f ' x0 f(x0),
f(x0)=0.

) ( ).
)  , f f
( ), .
(
f).
)  f'(x) > 0 < x< x0 f'(x) < 0 x0 < x< ,

f x0.
f o (, ).
)  f'(x) < 0 < x< x0 f '(x) > 0 x0 < x< ,
f x0.
.

)  f'(x0) = 0 f ''(x0) < 0, f


x0.
)  f '(x0) = 0 f ''(x0) > 0, f
x0.

f .

f ' (f ''(x0) > 0


).

f .

f' (f''(x0) < 0


).

A(x0, f(x0)) f.

H f '' x0
Cf A(x0, f(x0)).

lim f ( x ) 0 lim g ( x ) 0 

xo x

L Hospital.

M z = f(x,y)
x.
M z = f(x,y)
y.

xo x

lim f ( x ) rf  lim g ( x ) rf 

xo x

xo x

f c( x )
lim

x o x g c( x )
0
f ( x)
f c( x )
lim
lim

xo x g( x)
x o x g c( x )
0
0
w
wf ( x , y)
fx ( x , y) 
f ( x , y)
wx
wx
w
wf ( x , y)
f ( x , y)
f y ( x , y) 
wy
wy

4.9 .
, , .
1.

f f(0) = 0, f'(0) =1, Cf


(0,0) y=0.

256

2.

f x0 f x0.

3.

x0 f

x0. f x0, f(x0)=0.

4.

f f.

5.

f
x0 , Cf o A(x0, f(x0))
xx.

6.

f'(x) = (x 3)2 (x + 1) xR, f'( 1) f.

7.

f(x) = 23x, f'(x) = 3 ^ 23x.

8.

f'(x) = (x3 1)3, f 0 .

9.

f [, ] f() > f(), x0 (, )


, f'(x0) >0.

f [, ]
10. (, ) , , (, )
f'() =0.

11.

f'(x) > 0 < x< x0 f'(x) < 0


x0 < x< , f x0.

12.

f'(x0) = 0 f''(x0) < 0, f x0.

13.

f(x) = x g(x) = x , f, g :

[(f(x))2+(g(x))2]= 0.

14. f(x) = x3, x (f(2x))' = f(2x).

15. A f(x) = x, g(x) = x f'= f g'' = g.

16.

f lim
h o0
f(x0)=.

f ( x0  h)  f ( x0 )
h

A  1  lim
h o0

f ( x 0  h)  f ( x 0 )
h

A  1,

17. f(x) = x11 + x9 + x7 + 1 (1, 0).

f [, ],
(, ) f'(x)0 x (,), f()=f().

f ,
19. , , ,
.

20. .

18.

21.

f '
f.

257

f, g x0, f / g
x0 :

22.
ff '
ff((xx00))
, ( g(x0)0).
((xx00))==
gg ((xx00))
gg

g ' f
23. g(), fg
(fg)(x)=f(x)g(x).

f f'(x) = 5f(x) x R.
f ( x)
24.
g ( x ) = 5 x g(x)=0 x R .
e

25.

S(t) = 5 + t t3,
.

26.

f(x, y) = 2x + 3y x
f y, x,y.

f ( x , y) f ( x , y)

.
y
x

2 f ( x , y) 2 f ( x , y)
.
=
28. f(x,y)=x y +2,
yx
xy

27. f(x,y)=xy f ( x , y) =
2 3

.
1.

f 0 f(0) = 0, Cf A(x0,f(x0)) :
) y=x
) y=x+f'(0)
) y=xf'(0)
) y=f'(0)

2.

f(x) = x :
) x
) x1
) x1

) ( 1) x

3.

f(x) = ln x :
1
) 2
) ex
) ln x
x

) 1
x

4.

f(x) = 2x :
) f '(x) = 2x
) f'(x) = 2x
) f'(x) = 2x x

) f'(x) = 2x

5.

;
1

) (ex)=ex1
) (xa)=axa1
) (ln x )c
x

6.

f ' x0
f(x0), :
) f(x0)<0
) f(x0)>0
) f(x0)=0
) f(x0)=1

) (ax)'= ax

258

7.

8.

;
) f(x)=ex

) f(x)=x3

) f(x)=x4

) f(x)=3x

f(x)=|x|:
) E x0 = 0, :
) E x0=0.
) E x0=0, .
) E R.
f [, ], (, )
f(x)>0 x(, ). f()=0 f()=10, f(x)=5:

9.

) (, ).
) (, ).
) (, ).
) (, ).

f :
) f.
10. ) f.
) f.
) f.
f (1,1) f(0) =1.
f(x)=0 x(1,1), :
) f(x)=0 x(1,1).
11. ) f(x)=1 x(1,1).
) f(x)>1 x(1,1).
) f(x)<0 x(1,1).
O :
) .
12. ) .
) .
) .
f f'(x) = (x 1) (x + 1), :
) f (,1].
13. ) f (,+1].
) f [1,+1].
) f (,1].

259

S(t) t, t :
) (t)=S(t) a(t)=(t) .
14. ) (t)=S(t) a(t)=S(t) .
) (t)=S(t) a(t)=(t) .
) (t)=S(t) a(t)=(t) .
f f'(x) =(x 1)3 + (x 2)5, :
) f R.
15. ) f .
) f .
) f R.

4.10 .
4.10.1. P(t) ( ) , t , :
P ( t) = 10 + 6

e ( t 3)
.
1 + 2e ( t 3)

) P(0);
) .
) ;
4.10.2.
 '
t
(t)=(t2)2(t3), 0t4.
) t.
) .
) .
) .
ln x
, x z 0.
x
) f .
) .
1
) lnx= 2 x .

4.10.3. f ( x )

) lnx=x .
1
x2
f ( x ) = 3 2e 2

4.10.4.
.
) f .
) f, .
) f.
) f(x)=a, a>3 .

260

)
 f(x)=2 2 .
.
4.10.5. f(x)=x3(2x)3.
) f .
)  f .
f .
) f, .
) f.
) f(x)=a, a>1, .
) f(x)=a, a<1, .
x
+1.
2
) f .
) f, .
) f.
) 2lnxx+2=ln2 .

4.10.6. f ( x ) = ln x

4.10.7. f f(x)=ax+2x, xR 0<a1.


)  f(x).
)  Fermat , f
x0=0, a 1/2.
4.10.8.  , R f f(x) =x4+x3+x2+x+1 x1=0, x2=1 x3=2.
f
.
1
f ( x )f(x)=xe
= xe x , x, g ( x ) = x 2 e x.
4.10.9.
2
) f(x)= f(x)+ex, g(x)= f(x)+g(x).
)  - f
f()(x)=(x+)ex.
) - g.
I
4.10.10.  P P ( I ) =
, I0,
+ I
, . H
.
) .
) .
)  .
) P(I)/[1P(I)]2 .
4.10.11.  , () (), p (0<p<1),
lp.
, ' ,

261

f ( p) = p (1 p) .


) p=/.
)  (p=1/2) 10 ,
;
4.10.12.  (. 4.10)
10 m 0,2 m.
h ,
, 1 m/min.
h 0, 2
) x( t) =
s( t) .
0, 2
)
 h,

.
4.10.13. 
x km/h :
x3
f ( x ) = a(50 x 2 ), 0 x 150 ,
3
a .
)
 f xR.
)  x(0x150) .
4.10.14.
 2 m .
0,2m/sec (. 4.10).
) x(t)=2(t)(t).
x ( t)
x ( t) .
) y( t) =
y( t )
) 
1m.
) 

1 m.
)


0,5 m.
4.10.15.
 AB=10m

20 m/min. t, (t)
(. 4.10).
)
 h(t)
(t).

0,2 m
O

x(t)

. 4.10.

y(t)

2m

(t)

x(t)

. 4.10.

h(t)
(t)

10 m

. 4.10.

262

) (t)=22(t).
)  (t) 10 m .
)  (t) 20m.
4.10.16.
 f, ' x1<x2
.
x1 + x2

)
 ... x1 ,
, 2

1 , :
x x
f 1 2  f ( x1 )
2
.
f c(1 ) 2
x2  x1

x1 + x2

, x2 )
 ...
2

2 , :
x x
f ( x2 )  f 1 2
2 .
f c(2 ) 2
x2  x1

) :
x + x f ( x1 ) + f ( x2 ) .
f 1 2 <
2
2

263


( )

5:
6:


7:
8: Laplace

lx im
x0

MI
16 2 .
,

. ,
1 ( , ) .
16 Gerolamo
Cardano, Euler
.
, , ,
, , ...

5.1  .
5.2  .
5.3  .
5.4 .

5.5  .
5.6 De Moivre.

5.7  .

5.8 .

lx im

5.9 .

5.10 .

x0

266

5.1 .
, = 2 4 x2 + x + = 0,
0, , .
,
x26x+10=0,
=4,
( x 2 6 x + 9) + 1 = 0 ( x 3)2 = 1
, y=x3,
y2=1. y . , .
y2=1, . , ,
C.
R ' , R.
R x2=1 .
i ( ) i,
i2=1 (i)2=1.
,
C, :
) a+ i a, R*=R{0} (
).
) a+0 i aR aR
) i, R* ( ).
( )
z = a + i a, R.
Re(z), Im(z).
, C, a R
z = +0i

( m(z) = 0),

i, R*
z = 0 +i

( Re(z) = 0).

z1 = + i z2 = + i = = ,
z1 = z2 Re( z1 ) = Re( z2 ) Im( z1 ) = Im( z2 ) .
, z = +i = 0
= 0,
a + i = 0 a = 0 = 0

267

:
) A z1=3+4i, z2=34i, z3=3+4i, z4=34i,
,
( ).
) z = ( 2) + (3 5)i = 2
= 5/3.

a + x a, R. , a + i + i
, :
: (a + i) + ( + i) = (a + ) + ( + ) i
: (a + i) ( + i) = (a ) + ( ) i
, , . ,
(7 3i) + (2 + 8i) = (7 + 2) + (3 + 8)i = 9 + 5i ,

5i + (6 2i) = (0 + 6) + (5 2) i = 6 + 3i .

(a + i) + (0 + 0 i) = (0 + 0 i) + ( a + i) = a + i ,


0+0i=0. ,
(a + i) + (( a) + ( ) i) = (a a) + ( ) i = 0 + 0 i = 0 ,
, z = + i () + ()i = i,
z.
, ,
. z1 = + i
z2 = + i , z1 = z2, z1 z2. , ,
z1 z2 ( = = 0,
).

A 5.1.1.

, z1 = ( + 5) + 4i
z2 = (2 3) + ( + )i .
.

z1,z2 ,
Re( z1 ) = Re( z2 ) a + 5 = 2a 3 a + = 2 2 = 6

a+ =4
Im( z1 ) = Im( z2 )
a+ = 4
a + = 4.

= 3 = 1. z1=z2=1+4i.

268

A 5.1.2.

a 2
= = ,
2 3
z1=2(a+)+(a)i z2=5a+2i .

, , R 0

a 2
= = = k ,
2 3
a
= k a = 2k,
2

= k = 3k
3

2
2
=k= .

2
2
z1 = 2(a + ) + ( a) i = 2(2 k + 3k ) + (3k 2 k ) i = 10 k + k i = 10 k + 2i,
k
k
z2 = 5a + 2i = 5 2 k + 2i = 10 k + 2i
.

5.1.1.  , , z1=(a+4)+(a+1)i
z2=(2a4)+(a+3)i :
) , ) ,
) .
5.1.2. , .
) (10 + 3 5) + (20 4 )i = 0
) (a 3) + 3i = 1 + ( 1) i
) ( ) + (a + ) i = 7 5i

) (a2 2 ) + (a + ) i = 9 + 3i

5.1.3. a+i .
) (10a i  3  5i)  (20a i  4  3i) 0 
) (9 + 4i) + (5 7 i)
) (2 7 3 3i) + (3 7 2 3i)

) (4 + 3i) (6 4i) + (2 7 i)

5.1.4. x, y, zR (x+y)(xy)i=5z+zi, 2xy=z.


5.1.5.  z=(k5)+(4)i, k, . k,
:
) z=i
) z=3
) zR Re(z)=2
) Re(z) = 2 Im(z) = 4
a
= = ,
3 4 5
z1 = 3( ) + 4( a)i, z2 = a + i, z3 = a + 2( a)i .

5.1.6.  , , ,R

5.1.7.
 z1=1+2i, z2=2+3i, z3=3+4i. o z
:
) z+z3=z1+z2
) z+ Re(z3)=z1+z2
) z+z3= Re(z1+z2) Im(z1+z2)

269

5.2 .
,
, . z = + i M(, ) 1 (. 5.2).
z M(, ) M(z)
+ i. ,
Gauss. .
5.2 z=3+2i.
H O(0,0) M(, ), z = a + i,
(. 5.2) ( ) z,
z = a + i = a2 + 2 .

, z=3+2i
z = (3)2 + 22 = 9 + 4 = 13.


. RLC,

z = R + i L
,
C

1
, Re(z) = R Im( z) = L
C
L
C. :
1

z = R + L

M(, ) (z)

. 5.2.
y
_ 3 + 2i

.
H , .
z = + i
OM .

_3

. 5.2.
y

OM = a2 + 2 = z ,

OM
z.

M(a, )
z=+i

. 5.2.

1.  , Jean
Robert Argand (1806).

270


. OM1 , OM 2
M1(z1), M2(z2) .
z1+z2 OM

OM1 OM 2 (. 5.2).

, OM
M1(z1
z2)

OM 2 = OM 2
M2(z2) OM1 M1(z1).
5.2 ,
z1 z2

M 2 M1 .
, M1(z1) M2(z2) , z1+z2, z1z2
OM1, OM2, 5.2 5.2.
z1 z2 ,
, M1(z1) M2(z2) .
OMM2 5.2 , :
( M 2 M ) (OM 2 ) < (OM ) < ( M 2 M ) + (OM 2 )

M 2 M = OM1 ,

M(z1+z2)

M1(z1)

M2(z2)

. 5.2.
y

M1(z1 )
O
x
M(z1 _ z2 )
M2 (_ z2 )

. 5.2.

z1 z2 < z1 + z2 < z1 + z2 .

M(z1 + z2)

, M1(z1) M2(z2)

OM1 OM 2 , (. 5.2)
z1 z2 < z1 + z2 = z1 + z2 ,

M2(z2 )

M2(z2)
M1(z1)

, M1(z1) M2(z2)

OM1 OM 2 (. 5.2),

z1 z2 = z1 + z2 < z1 + z2 .

. 5.2.
y
M1(z1)

, z1 z2
,

M(z1 _ z2)

z1 z2 = z1 + z2 = z1 + z2 .
O

,
z1 z2 z1 + z2 z1 + z2

M2(z2)

. 5.2.

271

.
5.2

JJJJG
OM'  M 2 M1 .
:

M(z1 + z2)

M2(z2)

M1(z1)

1 2, z1, z2 ,
z1 z2.

x
M(z1 _ z2)
M2 (_ z2)

, z0 = x0 + y0i,
K(x0, y0)
r, :

. 5.2
y

z1 _ z2

z z0 = r
K(x0,y0) r.

M1(z1)
O

, ( ) K(x0,y0) r (.
5.2),

x
M(z1 _ z2)

z z0 r x y i x0 y0 i r
( x x0 ) ( y y0 ) i r

M2(z2)

. 5.2.
y

( x x0 )2 ( y y0 )2 r ( x x0 )2 ( y y0 )2 r 2 .


O(0,0), x0 = y0 = 0,

x2+y2=r2 z=r.

K(x0 , y0 )
M(x, y)

E
z
z>r z=r,

. 5.2.

z z<r
z=r.

5.2.1.

z=(x1)+(y1)i z=5. (x, y)


K(1,1) r=5.

272

z = 5 ( x 1) + ( y 1)i = 5 ( x + yi) (1 + i) = 5 w (1 + i) = 5 ,

w=x+y i. , (x, y)
K(1,1) ( z0=1+i) r=5.
( x 1) + ( y 1)i = 5 ( x 1)2 + ( y 1)2 = 5 ( x 1)2 + ( y 1)2 = 52 .
5.2.2.

z, z + 4 = z + 2 2i .

y
(x, y)

B(_ 2, 2)

z + 4 = z + 2 2i z (4) = z (2 + 2i) .
A(4,0), B(2,2) 4 + 0i, 2 + 2i,
(x, y) z=x+yi, 5.2.
z (4 + 0i)
(), z (2 + 2i) ().
z (4 + 0i) = z (2 + 2i) ,

A(_ 4, 0)

. 5.2.

(AM) = (MB). , A(4,0), B(2,2)


, .
, M(x, y) (AM)=(MB),
z (4 + 0i) = z (2 + 2i) .
.
z1, z2 z1 z2, z
z z1 = z z2 ,

z1 z2.
5.2.3.

(z) z z 4 + 2i < 3. , (z) z


z 8 2i < 4. :
)

273

) 
.

) , (z) ,
z (4 2i) < 3 ,

z (8 + 2i) < 4 .

(8, 2)

2
O
4
_2

(4, _ 2)

(4, 2), (8, 2) 42i, 8+2i .


z

. 5.2.

z (4 2i) < 3


K(4,2) r=3,

z (8 + 2i) < 4
(8,2)
r=4. , z
z (4 2i) < 3
z (8 + 2i) < 4 5.2.
) , M(z) ,
z (4 2i) < 3 ,

(8, 2)

2
O
4
_2

(4, _ 2)

. 5.2.

z (8 + 2i) 4 .

z
K(42) r=3,
z (8 + 2i) 4 ( )
(8,2) r = 4. , z
5.2.
.

5.2.1.  .
) 3+4i

) 34i

) 3+4i

) 34i

) 2 3 13 i

) 2 3 + 13 i

) 13 + 2 3 i

) 13 + 2 3 i

) (4 + 3i) (5 + 4i) + (2 + 3i)

5.2.2.  z1=x1+y1i, z2=x2+y2i.


.
1
1
) ( z1 + z2 )
) (2 z1 + z2 )
) 2(z1+z2)
2
3

274

5.2.3. z , :
) z=16

) z + 3 = 1 / 2

) z 2 + i = 1

) z i = z 1

5.2.4.  z1=4+3i, z2=5+2i z=x+yi x, y .


z,
z 2 z1 = z 3 z2 .
x, y.
5.2.5.  z1=32i, z2=1+i z=x+yi, x, y .
z,
z z2 = z1 + z2 .
x, y.
5.2.6.  z=x+yi ( x, y ) z=3 w,
w=z+5. w
.
5.2.7. z, :
) z 1 + 3i 5
) z z 1 + 3i
) z 1 + 3i 5 z z 1 + 3i .
) z 1 + 3i < 5 z 1 + 3i 3 .
) z z 1 + 3i z 1 + 3i z + 1 + 3i .
) z 1 + 3i 3 z z 1 + 3i .
5.2.8.
 z1=1+i, z2=1+5i, z3=5+5i, z=x+yi. o
z = x + yi, z1, z2, z3,

z z1 = z z2 = z z3 .
x,y.
5.2.9.  z1=5+3i z2 , z2=8.
,
z1 + z2 .
5.2.10.  , ( )
z0=7+9i. ,
z1=2+3i,
z2=32i.
) ( ).
)

;
5.2.11.
 t :
z1 = t + i t, z2 = t i t, z3 = t + i t z4 = t i t.
)  t, 4 -

275

1.
)  1 3 2
4.
)  1 2 3
4.
)  t, 1 2, 2 4, 4 3, 3
1 .
5.2.12.  z1=1+i, z2=i, z3=1, z=x+yi, x, y .
x, y, z z1 = 3 z z2 = z z3 .

5.3 .

+ x , R, x x2=1. , z1 = + i, z2 = + i,
(a + i)( + i) = a ( + i)+ i ( + i) = a + a i+ i+(i) ( i) =

=  i  i  i 2

( + i) ( + i) = ( ) + ( + ) i .

 i  i  (  )  (  ) i ,

(5i) (3+4i)=5(3+4i)i(3+4i)=15+20i 3i 4i2 = (15+4) + (20 3)i = 19+17i.


3+4i 34i, . :
(3 + 4i)(3 4i) = 3(3 4i) + 4i(3 4i) = 9 12i + 12i 16i 2 = 25 .
, .
+ i i
. , z = + i, , z , z = i. , 7+5i 75i, 8i 8i, 3 3.
z = + i .
(z ) = a i = a + i = z
z + z = (a + i) + (a i) = 2a = 2Re ( z ) ,
z z = (a + i) (a i) = 2 i = 2 i Im ( z)

zz

(a  i)(a  i) (a a  ( ))  (a ( )  a) i a2  2

z .

, 1.

1. A , , z, z z , z + z , (z z )/i ,
.

276

( z ) = z ,
Im( z) =

Re ( z) =
zz
,
2i

z+z
2

z = a2 + 2 = z z .

_ z = _ a+i

:
z z = z .
z z = z .

x
z = a_ i

_ z = _ a_ i

5.3
z

z = a+i

. 5.3.

z= z = z = z .
.
, z = + i, z = () + ()i
z = ( )2 + ()2 = 2+2 = z
.
z1 z2 :
z1 z2 z1 z2
z1 z2 z1 z2

, z1 = + i, z2 = + i, :
z1 + z2 = ( + ) + ( + )i, z1 z2 = ( ) + ( )i
:
z1 z2 ( ) ( ) i ( i) ( i) z1 z2

z1 z2 ( ) ( ) i ( i) ( i) z1 z2

, .
. z1 = + i z2 = + i
z2 0, z=x+yi, zz2=z1. z
1 . zz2=z1 z2 , :
z2
2
zz2 z2 = z1 z2 z z2 = z1 z2 ( x + y i)( 2 + 2 ) = (a + i)( i)
x( 2 + 2 ) + iy( 2 + 2 ) = (a + ) + ( a) i ,

x(2 + 2) = + y(2 + 2) =

277

x=

+

.
2
2 y = 2
+
+ 2

+ i +

= 2
+ 2
i.
2
+ i +
+ 2

z
1 . z2
,
z1 z2 (a + i)( i) (a + ) + ( a) i a +
a
i.i
=
=
= 2
+ 2
2
2
2
z2 z2 ( + i)( i)
+
+
+ 2
,
2 + 3 i (2 + 3 i)(5 + 2 i) 10 + 4 i + 15 i + 6 i 2 4 + 19 i 4 19
=
=
=
=
+
i.
5 2 i (5 2 i)(5 + 2 i)
29
29 29
52 + 2 2
.
, z1 = z, z 2 = zz, z 3 = z 2 z , , 2
z = z 1 z.

z 0

1
.
z
i.
z0=1 z =

i1=i,

i2=1,

i3=i2i=i,

i4=i3i=i2=1,

i5=i4i=i,

i6=i5i=ii=1,

i7=i6i=i,

i8=i7i=(i)i=1 ..

i 4 1, 1, i, i . , i , , 4 + , 0 < 4 (
4), :
i v = i 4+ = i 4 i = (i4) ^ i = i, 0 < 4.
, , i

1

i

4
4  1
4  2

 = 0, 1, 2, .

4  3

i38 = i 49+ 2 = i 49 i 2 = (i 4 )9 i 2 = 1(1) = 1 .


, , z1 z2 :

278

z1z1 zz22 = z1 z2

z1 z1
( z2 0)
=
z2 z2

( z ) = ( z ) , N.

, , z1z1 zz22 = z1 z2 . z1 = + i z2 = + i,
z1 z2 = (a + i)( + i) = (a ) + ( + )i = (a ) (a + )i

z1 z2 = (a i)( i) = (a ) (a + )i .


z1z1 zz22 = z1 z2 .
z=z1/z2. zz2=z1 ,
z
z
, z1 = zz2 = z z2 , 1 = z = 1 .
z2
z2
.
, , z1,z2,...,z,

z1 z2 z = z1 z2 z .

5.3.1.

z RLC z 2 = z , R=Re(z) 1 hm.


.

z 2 = z ,
2

z 2 = z ( z 2)( z 2) = z z ( z 2)( z 2) = z z 2 z 2 z + 4 = 0 .
,

z + z = 2 2Re( z) = 2 Re( z) = 1 .

5.3.2.

z, z2=3+4i.
.

z=x+yi x,y ,
( x + yi)2 = 3 + 4i x 2 + 2 xyi + y 2 i 2 = 3 + 4i x 2 y 2 + 2 xyi = 3 + 4i .

279
2
2
x 2 y 2 = 3 x y = 3

.
2
2 xy = 4
y=
x

y = 2 / x x4 3x2 4 = 0,
x = 2 3 4 = 0. = 4 = 1. ,
, = x2 0. , x2 = 4, x = 2 x = 2.
2

x = 2 y =

2
2
= 1, x = 2 y = = 1 . , z=z1=2+i z=z2=2i.
x
x

z1=2+i z2=2i 3+4i. , +i z=x+yi,


z2=+i.
5.3.3.

z (z)=0, P()
, P ( z) = 0 .
.

P ( z) = a z + a 1 z 1 + ... + a1 z + a0 = 0 ,

0, 1, 2,..., R. P(z) = 0
z + 1 z 1 + ... + 1 z + 0 = 0

P ( z) = 0 a z + a 1 z 1 + + a1 z + a0 = 0 a ( z ) + a 1( z )-1 + + a1 z + a0 = 0 .

( z ) = 0.
:
, .
5.3.4.

z z z = 4i ( z z ) . (z) .
.

z=x+yi x, y , z = x y i ,
zz = x 2 + y 2 z z = 2 y i .

280
y

4i(z  z) x 2  y 2

zz

4i(  2y i) x 2  y 2

8 y i2

x 2  y 2  8 y 0 x 2  ( y  4)2

(x, y)

42 .

, M(x,y)=M(z) K(0,4) r=4 (. 5.3).

K(0, 4)

. 5.3.

5.3.1.  + i (3+5i)(52i), (1 + 3 i)(1 3 i)


(1+i)(1+2i)(1+3i).
5.3.2. + i (1i)2(1+i)2, 3i(i3)2, (1i)47i .
2 + 3i
2 + 5i
3i
.
, z2 =
, z3 =
3i
4+i
i7
z z2 z2 z
5.3.4. z1=3+i, z2=12i, 1 , 12 , 1 , 12 .
z2 z2 z2 z2
5.3.3. + i z1 =

5.3.5. z, 2 i z z z = 7 + 4i .
5.3.6.  z1 = k + 15 i, z2 = 5 + i k, R. k
, z1 = 5 z2 .
5.3.7. z, z z + ( z z ) = 5 + 2 i .
5.3.8. z1 = 5i, z2 = 1 + 2i.
1 1 1 1
5.3.9. i+i2+i3+i4, + 2 + 3 + 4 i5+i10+i15+i20.
i i
i
i
2 3i
(7 i 5)(4 2i)
5.3.10. +i z1 =
, z2 =
.
(2 + 3i)(3 4i)
(8 + 5i)(3 2i)
5.3.11. z=x+yi u = ( z 1)( z i) , x=y+1.
1
3

i
2 2
) z2,z3,z4 1z+z2.
) 1z+z2z3+z4 z5+z6=1.

5.3.12. z =

5.3.13. z=x+yi x,yR, :


) z(2+3i)=4+i,
) (z+1)(2i)=34i.
5.3.14. z z+16=4z+1, z=4.
5.3.15. z 2zi=iz+2, z=1.
x
iy
4
+
=
5.3.16. x, y,
.
2 i 2 + i 1 2i

281

5.3.17.  z = a2 + 1 + a i , R.
w=z2 , , .
5.3.18.  z=x+yi, x, y w
i (i + z)
w =
z i . :
iz
) w =

2x
1 x2 y2
+
i.
x 2 + ( y 1)2
x 2 + ( y 1)2

)  w , z (0, 0)
1.
)  z , w (0, 0)
1.

5.4 .
z = + i M(, ) z . Ox,
, , (. 5.4)
= , = , = 2 + 2 .
,

+ i = + i = ( + i ) .

z = + i. , :
y

z = + i ,
, z

z=(+i),

M(, )

= z

= z = 2 + 2 , = =.

= =
z. , [0,2).
Arg z.
2,
1. , (, )
z.
, z = 3 + i
(. 5.4),

. 5.4.

z= _ 3+i
_ 3

z = ( 3) + 1 = 4 = 2 .
2

1. .

. 5.4.

5
6
2

282


3 = 2

1=2,

3
2

1
.
2

Argz=5/6, =2+5/6 z. ,
z = 3 + i
z

2 5 +i 5 .
6
6

,
( ). ,
z=0.
,
:
2, :
z1 = 1(1 + i 1) z2 = 2(2 + i 2),
z1 = z2 1 = 2 12=2,

, ,
z=zw, zw C. 5.7.
5.4.1

, :

0 < <Arg
Arg
3
z <z < z< >Arg
z <0 < Arg z <
) Arg z =
) zz >< 43
63 3
2
6
6
6

< Arg z <


6
3

) z < 4 < Arg z <


6
3
)

) z < 4 0 < Arg z <

) z < 3

< Arg z < z > 3 0 < Arg z <


6
3
6

) Arg( z 1 2 i) =

< Arg z <


6
3
2

) z 5.4
/6=30 , .
) z 5.4
, /6 = 30
/3=60 , .

283

) z, z<4,
(0,0) r=4. z,

< Arg z < 6


3
, 5.4 /6=30 /3=60 .

5.4.
) z, z<4
(0,0) r=4. z

 Arg z   0  Arg z 
6
3
2
5.4 /6=30
,
5.4 /3=60
. , 5.4.
) z, z>4
O(0,0) r=4. z

 Arg z   0  Arg z 
6
3
2
5.4 /6=30
' 5.4 /3=60 .

5.4.
y

/3

/3
/6

/6
x

. 5.4.

/6

. 5.4.
y

. 5.4.
y

/3

/3
/6

/6
O

. 5.4.

. 5.4.

284

)
z,
5.4.
) M(z), A(z0) z, z0 (. 5.4).
:

OM + MA = OA

AM = OM OA ., z z0 5.4 (
). , z0=1+2i, z12i=zz0 /3
5.4, x
/3, .
y

(z)

z _ z0

z
/3
/6
O

z0
3

. 5.4.

A (z0)

. 5.4.

/3

A(1, 2)

z0 = 1+2i
1

. 5.4.

5.4.2.

) z1 = 3 + i z2 = 3 i .
) +i z,
z = 2 Arg( z) =
.

3
4

) z1 = 3 + i z1 = ( 3 )2 + 12 = 2 .
z1 = 3 + i (+i),
=

=/6. ,

1
3
= ,
2
2

z1 = 3 + i = 2 + i .
6
6

, z2 = 3 i =(+i), iz2=2 ,

3
1
=
= ,
2
2

11
.
6

285

11
11

z2 = 3 i = 2
+ i
.
6
6

) z=(+i)

3
3
2
2

z = ( + i ) = 2
+ i = 2
+i
= 1 + i .
4
4
2

2
,

7i
5.4.1.  z =
. z
3 4i
x.
5.4.2. .
) z1=8i
) z2=5
) z3=1+i
) z4 = 2 3 + 2i
5.4.3. +i .
) z1=3(2+i2)

11

11
) z3 = 5
+ i

6
6

5.4.4. 
z=22i;



) z = 2 + i
) z = 2 2 + i
4
4

4
4

3
3
) z = 2 + i
4
4

5.4.5. , :
3
3

+ i
) z = 2 2
4
4

) z=3 Arg z=
4

) Arg z=

3

2

3
2
Arg z <
2
3

) Arg(z5)=

3

4

5.4.6. N z,
t 5.4, 5.4 5.4.
t

(1, 0) x

/4
O

. 5.4.

(_ 3, 6)

(_ 2, 0)

/3
O

. 5.4.

. 5.4.

286

5.4.7.  ( )
1(z1), z1=3+3i. 2(z2),


z2 = 2 + i .
3
3

.
5.4.8.  ( ' )
(z), z=1+3i. ,
(z), z=4+2i. () .
5.4.9.  z1,z2 ,
1 1 1
z = + . z1=3+4i z2=1+i,
z z1 z2
.
2
.
5.4.10. z
11  i
) z z=x+yi, x, yR.
) z.
)  z (2,0)
r = 2 .
z 3i
.
5.4.11. z, w, w =
1+ i
) w=22i, z.
)  w = 2 2 , z
.
) z = x + yi x, yR, :
x + y 3
x + y 3
, Im (w) =
2
2

) z Arg(w) = .
4
Re(w) =

5.5 .
(, ,
) , o
. , 5.2

z1 z2 z1 + z2 z1 + z2
, , .
z1,z2

287

z1 z2 = z1 z2 ,

z
z1
= 1 ( z2 0).
z2
z2

,
2

z1 z2 = z1 z2 z1 z2 = z1 z2 ( z1 z2 )( z1 z2 ) = z1 z1 z2 z2 z1 z2 z1 z
2

= z1 z2 z1 z2 = z1 z2 ( z1 z2 )( z1 z2 ) = z1 z1 z2 z2 z1 z2 z1 z2 = z1 z1 z2 z2
.

z1 =

z1
z
z2 = 1 z2 .
z2
z2

, z1 z2 z = z1 z2 z z1 = z2 = ... = z = z z = z .

, z z = z ,
z
z
=
=1 .
z
z

5.4.
( ).
z1 = 1 (1 + i 1), z2 = 2 (2 + i 2)
,
z1 ^ z2 = 12 [ (1+2)+i(1+2)]
z1
z2

U1
[(1  2 )  i (1  2 )],  2 0.
U2


z1

2  i z2
3
3

4  i .
6
6

z1 z2 = 8 + + i + = 8 + i = 8i ,
2
2
3 6
3 6

z1 2

3 1

1
= + i = + i =
+i .
z2 4
6
6 4
4
3 6
3 6 2


:
) ,

288
M(z1 z2)

M1(z1 )

y
_

M1(z1)

M2(z2)

M(iz)
M(z)

M2(z2 )

1 2

M(z1/z2 )

. 5.5.
.

1 2

. 5.5.
.

. 5.5.

(
).
) () () , :
Arg (z1 z2) = Arg z1 + Arg z2, Arg (z1 / z2) = Arg z1 Arg z2 .

, z1 = 1 (1 + i 1)
z2 = 2 (2 + i 2) z1
2 (. 5.5).
, z1 z2 z1
2 (. 5.5).
i=(/2)+i(/2) 1 /2, iz z /2 z,
z i z 1 /2 (. 5.5).
:
z i M(z) /2, i2=1
M(z) , i3=i M(z) 3/2.

5.5.1.

1 + + i
(2+1),
1 + i
. z100.

z =

w=1++i z =

w
z=1.
w

1.  , .

289

z z =

w w2
=
:
w ww

(1   i )2
(1   i )(1   i )

(1  )2  2  2i (1  )

(1  )2  2

1 + 2 + 2 2 + 2i (1 + ) 2 2 + 2 + 2i (1 + )
=
=
2 + 2
1 + 2 + 2 + 2

(1 )( i )
i .
1

z . z100
z100 = z

100

= 1100 = 1 .

5.5.1.
2

1+ i 1 i
(1 + i)2 , (1 i)2 ,
,

1 i 1+ i

a+ i

, , a i
2+20.
5.5.2.  z1=1+i z2 = 1 3 i ,

z z2 z
z1 z2 , 1 , 1 , 12 .
z2 z2 z2
5.5.3.
 z z = 1 /3.

1 z
z1 =
.
1+ z
1

5.5.4.
 z = + i .
z
1
1
z + , z .
z
z
5.5.5.

z =1+ i
[0,] .

2
5.5.6. z=x+yi,
z
1
=
z 3 2

290

+ i
z22 =
= 11+i,
+i z1/z2 ,
5.5.7.  z1 = 3 +i

,

.
12
12

5.6 De Moivre.
z z = ( + i ).
5.5 :
z2

UU [(  )  i (  )] U 2 ( 2  i 2) .

z3

z z2

[ U (  i )] [ U 2 (2  i 2)]

U 3 [(  2)  i (  2)] U 3 (3  i 3) 
... , ( ) De Moivre
( ).
De Moivre
:

[ U (  i )]

U [( )  i ( )] .

De Moivre (.
). De Moivre, (1+i)40.
1+i :
1 i
1+i
,

(1 i)40 2 i
4
4

40

2 i
4
4

40
40

( 2)40
i

4
4

220 [(10 )  i (10)] 220 (1  i 0) 220

2048. 

5.6.1.

( 3 i)3
.
( 3 + i)5

z1 = 3 + i , 5.4.2
:

z1 = 3 + i = 2 + i
6
6


5
5

( 3 + i)5 = z13 = 25
+ i .
6
6

291

, z2 = 3 i ,
11
11

z2 = 3 i = 2
+ i

6
6

33
33

z23 = ( 3 i)3 = 23
+ i
.
6
6

, :
33
33

23
+ i

( 3 i)

6
6
28
28

=
= 22
+ i
=
5
5
5

6
6

( 3 + i)
5

2
+ i
6
6

1
2
2
3

= 4
+ i
= 2 + 2i 3 .
= 4 + i
3
3
2

2
3

5.6.2.

z=1+i.
) N z10.
) z2,z3,...,z11.
.

) z=1+i z = 12 + 12 = 2 .
z=1+i ( +i ),
=

=/4. ,

1
2
1
2
=
=
,
=
2
2
2
2

z = 2 + i
4
4

De Moivre
10

10
10



10
+ i
z10 = 2 + i = 2
= 32 i.
4
4
4
4


) z :
2
2
3
3

3
3
z 2 = 22
+ i
+ i
= 2i, z = 2
= 2 + 2i
4
4
4
4

4
4
11
11

11
z 4 = 24
+ i
+ i
= 4,... z = 32 2
= 32 32 i.
4
4
4
4

292

z2,z3,...,z11 z 5.6.
, , (. 5.6).
. ,
, [. 3.6()] .. .
, 5.6().
z11

z10

z9

z4

z3 z2

z
z8

z5
z6

()

()

. 5.6.

z7

. 5.6.
.

5.6.1. +i

z1

1
,
(1 i)5
100

1+ i 3
z4 =
,
2

i
6
6

16



z2 i ,
4
4

z3

9 i 9

100

100

2+ 2 +i 2
1 i 3
z5 =

, z6 =
2+ 2 i 2
2

1+ i
z7 =

2008

5.6.2. +i
5

5
5




i .
[2(10 i 10 )] , i , 3
3
3
4
4

5.6.3. z

( i)10
.
( i )5

600
600
5.6.4. z1 = 3 + i z2 = 3 i , z1600 + z2600 z1 z2 .

293

3
, z3 (1z)8.
2
3
1
3
i = 1.
5.6.6. +
2
2

5.6.5. z = 1 i

5.6.7.  z1 = 3 + i z2 = 3 i , 12
12
z1 + z2 , . z1 + z2 .

5.6.8. ( +i )3 +i:
) De Moivre.
)
 .
3=423, 3=343.

5.7  .
5.3.2 z z2=3+4i z1=2+i z2=2i. , z=x+yi,
z2 = +i + i.
:
z w,
>1, z= w.
, ,
w,
.
w= ( + i ) ,

 2
 2
, =0, 1, 2,..., 1.
 i

(5.7.1)

.
z, =0,1,..., 1 ,
(0,0) r U ( z U 
=0,1,..., 1). , , z0,z1,z2,...,z1 ' ,
, z0 /.
i .
A wa== 3 i
(=3) , .
w = ( 3 )2 + (1)2 = 4 = 2 =

7
. ,
6
7
7

w 2
i

6
6

1
3
, = ,
2
2

294

, wa = 3 i

7
7

+ 2
+ 2

3
6
6
z = 2
+ i
, = 0, 1, 2
3
3


7
7
19
19
31
31

i , z1 3 2
i
i
z0 3 2
, z2 3 2
.

18
18
18
18
18
18

,
(0,0) 3 w 3 2 .
5.7.1.

z=1. ,
1, z1, z12 , z13 ,..., z1 1

2
2
.
i

0,
(1)1.
z1

z=1, (5.7.1), w = 1(0 + i0),


0 + 2
0 + 2

z = 1
+ i
, =0,1,2,..., 1,

2
2
 i
, 0,1, 2,...,  1.

=1
z1 =

2
2
+ i

2
2
2
2

+ i
=
+ i
,

z = z1 ,

=0,1,2,...,1.


z0 = 1,

z1 = z1 ,

z2 = z12 , ... , z 1 = z1 1 .


z0z0++zz11 +
= 1 + z1 + z12 + + z1 1
+ zz22 ++...+
+ zz1
1 ==

295
2

1, z1 , z1 ,..., z1
1=1 =z11,
( z ) 1 1 1
=
= 0.
z0 + z1 + z2 + + z 1 = 1 + z1 + z12 + + z1 1 = 1 1
z1 1
z1 1
,
z0 z1 z2 z 1 = 1 z1 z12 z13 z1 1

z11+ 2+3+ +( 1)

2
2

=
+ i

z0 z1 z2 " z 1

( 1)
2

( 1)
z1 2

2( 1)
2( 1)
+ i
,
2
2

[ (  1)]  i ( (  1)] (  i ) 1 (1) 1 .

z=1 .
,
2
r = 1. =
.

, z3 = 1
:
2
2
z =
+ i
, =0,1,2.
3
3
y
B
2
r=1. =
,
3
5.7.
2/3

z3=1

A
x

z31=0 (z1)(z2+z+1)=0 z 1 = 0
z2+z+1=0 z=1 z2+z+1=0
, z3=1
(. 1),
3=1, 2++1=0.

. 5.7

,
3
z = 3 i z3+3z2+3z+2=0
, z=1 ,
. :
P(z) = 0, , z+1z1 +...+1z +0=0 0,1,...,C, 0
.

296

w P(z)=0, P(z)=zw.
z1,z2,...,z P(z) ( ),

P(z)=(zz1) (zz2) (zz).

C. C

.
z2+z+=0, ,,R 0.
2


2 4

z
+

=0z+

2 = 0,
2
2
2 4
4

=24 z2+z+. ,
) > 0 , , z1

 
, z2
2

 
.
2

.
2
)
 <0, ( >0).
) =0 , , z =

i
i
i

z
+
+

=0z+
z +
= 0.


2 2
2
2
2
2

,
z1

  i 
,
2

z2

  i 
.
2

5 , (z) = 0,

P(z)=z5+2z4z2.
2, .
1,2. z=1 P(1)=0, 1 .
Horner z =1,
1

2
1

0
3

0
3

1 2
3 2
0
2


P ( z) = z 5 + 2 z 4 z 2 = ( z 1)( z 4 + 3 z 3 + 3 z 2 + 3 z + 2) .
,
P(z) ( 1 )
P ( z) = z 5 + 2 z 4 z 2 = ( z 1)( z 4 + az 3 + z 2 + z + )

297

,,,
z.
z 4 + 3 z 3 + 3 z 2 + 3 z + 2 , z=1 Horner

3
1

3
2

3
1

2
2
0

z 4 + 3 z 3 + 3 z 2 + 3 z + 2 = ( z + 1)( z 3 + 2 z 2 + z + 2) .

z=2 Horner
1
2
1
2
2
2 0 2
0
1
0
1

z 3 + 2 z 2 + z + 2 = ( z + 2)( z 2 + 1) .

, P ( z) = z 5 + 2 z 4 z 2

P ( z) = ( z 1)( z + 1)( z + 2)( z 2 + 1) = ( z 1)( z + 1)( z + 2)( z i)( z + i)
1,1,2,i,i.
, z2+z+=0, (,,R, 0) <0
.
, (. . 5.3.3):
+i (0) , i .
' :
K , ,
.
P(z)=z5+2z4z2 1,1,2,i,i

<0, P(z)=(z1)(z+1)(z+2)(z2+1).
:
, .

298

5.7.2.

(x), , P(1)=2
(x) = 0 2 1+i.
.

1+i, 1i.
, 3 2, 1i, 1+i
P ( x ) = a( x 2)( x (1 i))( x (1 + i)),

a R* .

P ( x ) a( x  2)( x 2  x(1  i)  x(1  i)  (12  i2 ))

a( x  2)( x 2  2 x  2) a( x 3  4 x 2  6 x  4) .

P(1)=2,
(13 4 12 + 6 1 4) = 2 = 2 = 2

P ( x ) = 2( x 3 4 x 2 + 6 x 4) = 2 x 3 8 x 2 + 12 x 8 .

5.7.1. .
) z4=16

) z9z5+z41=0

4
) z =

2i
1 i

5.7.2. .
) z4=i
3
) z =

(1 i)

2

8
8

+ i
) z 4 = 16
5
5

4
) z =

1+ i 3

2

) z6=64
4
) z =

1 i 3
2

5.7.3. z = ( +i ) z
) z .
) z9( z )5=1.

3
5 1
+
= .
5.7.4. z7=1, +
7
7
7 2
5.7.5.  , 1,
(1 + 2) (1 + 2).
5.7.6.  .

( z 2  1)2  z 3  z 0 , z 23  z16  1 0 .
5.7.7.  1. w , w1:
) w2w+1=0.

299

) (1+w+w2) (1+ww2).
) w, w, ww .
5.7.8. :
) z 2 2 z = 0
) z34z=0.
;
5.7.9. :
) (1i)z+2i z =5+3i

) z37z2+16z10=0

) z33z2+3z28=0

) z4=4(z2+z+1)+z

) z +4+8i = 2 z +1+2i

2
2
) z = z

) z22z+2=0

) ( z 1)3 (1 i)( z + 1)3 = 0

5.7.10. z 4 ( z + 1)2 + z 2 + 2 z 3 + z 2 + ( z + 1)2 = 0 .


5.7.11.
 z3+3z2+3z+9=0
.
5.7.12.  1+i z3z2+2a=0, R
1 .
5.7.13.  z=3+2i z35 z2+7z+13=0,
,R. ==1 .
5.7.14.  P(x)=x6x3+1
.
5.7.15.  P(z)=(z+1)6k+1z6k+11 k .
) P(z).
) (z) z2 +z+1.
5.7.16.
 z2+6z+18= 0. z1, z2
, w =

z12 + z22 2i
+i, , R.
z1 z2 + 3i

5.7.17. 3+i z38z2+az+=0.


) =22, = 20.
) z38z2+22z20 z26z+10.
) .

5.8 .
i.
z = + i.

z = + i.

z1 = + i, z2 = + i

i2=(i)2=1
Re(z)=a, Im(z)=
z = a2 + 2
= =

300

( + i) + ( + i) = ( + ) + ( + )i

(  i)  (  i) (  )  (  )i
( + i) ( + i) = ( ) + ( + )i

+ i +
=
+
i
+ i 2 + 2 2 + 2
z

.
(z )

2 Re( z ), z  z

2 i Im(z)

z1 + z2 = z1 + z2 , z1 z2 = z1 z2

.
z1 z2

i.

z, zz

a i a i

z
z1 z2 , 1
z2

z1
, ( z ) ( z ) ,
z2

1
i

i 4  i
1
i
, 0, 0<4.

0
1
2
3

z z z z , zz z , z1 z2 z1 z2 ,

z1 z1

, z2 0
z2 z2

z1 z2 z1 + z2 z1 + z2

zz0=r
z0=x0+iy0.

(x0, y0) r.

z z1 = z z2 .


z1 z2.
z= (+i) =z,

z = + i.

z,

z1 = 1(1 +i 1), z2 = 2(2 +i 2)


.

z1 z2 = 1 2 [(1 + 2) + i (1+2)
z1
z2

U1
[( 1  2 )  i ( 1  2 )], U 2 z 0
U2

/2 (
z i.
).

301

3/2 (
z i3=i.
).
z = ( +i ), z = [()+i ()]

De Moivre.

w=( +i ).

 2
 2 , =0,1,2,...,1
 i


- + i ( 0), i
.
.

z+1z1+...+ 1z+0=0
P(z)=0, . 0,1,...,C, 0,
.

5.9 .
,
, .
1.

z , z + z = 0 .

2.

z1,z2 z12 + z22 = 0 ,


z1=z2=0.

3.

z z 2 = z .

4.

z z 2 = z 2.

5.

z=(2+3i)20+(23i)20 .

6.

z + z = 0 , Re(z)=2.

7.

z =

8.

z3+z2+=0 ,,R 0

.

9.

z + z = 0

.

10.

z z = 2i

.

11.

z , z/i


.
2

+ i , z200 1.
4
4

302

12.

z z = z = z = z .

13.

.N
z ( z ) = ( z ) , N

14.
15.

,
.
(z)=0

1i, (z) z22z+2.

16.

1(1 + i 1) = 2(2 + i 2), 1 = 2 1 = 2.

17.

De Moivre [( + i )] = [() + i ()]



.

18.

z i
M(z) /2.

19.

z i3 = i
M(z) 3/2.

20.

, z+1z1+...
...+ 1z+0=0 0,1,...,C, 0, -
.

21.

z40=1024 ,


.
40

22.

z10 = 310 ,
(0,0) r = 3.

23.

z 3 = 7 + i 25 , (0,0)
r= 2 .

24.

3

1 + i, i i.

.
1.

2.

3.

4.

z = +i , :
) = 0 = 0

) = 0

) = 0 = 0

) = 0

z = i, :
1
= z
) z2 = i
) z2 = z
z
z i2 = 1
:
) /2
)
) 3/2
) 3/4
) z2 = 1

z (z1). 1+z+z2 :
) 0
) 1
) 3
) 1

303

5.

6.

7.

4 :
) z=2i

) z=2

) z=2+i

) z=2

z = 3 + 2 :
) z = 2 3

) z = 3 + 2

) z = 3 2

) z = 3 2

z , :
) zz = 1

) z + z = 0

) z z = 0

) zz = 0

z2+3z+4=0 :
8.

) . ) .
) .

9.

) .

z1, z2 1, 2 .
z1
:
z2

) 1+2
) 12
) 1
) 12
2

z1, z2 1, 2 .
z1z2 :
10.

) 1+2
) 12
) 1
) 21
2
P(z)=0 i,
11. P(z) :
) z22z+2
) z2+2
) z2+i
) z2+1

OA
OB

z,
,
JJJG
JJJG
iz, O  i2z O 
iz. :
JJJG
12.
) O  z . ) .

) A, B, , . ) 
.
z1=1, z2=i, z3=1, z4=i :
13. ) 1. ) yy.
) 2. ) xx.
z w, 14. >1, :
) w = z
) z = w
) z = wi
) z = w
= ( + i ) :
15. ) .
) .

) .
) 2 .

304

5.10 .
z z
5.10.1. z z0 , 2 + 2 .
z z
2 + iz
5.10.2. z, w, w =
1 iz
, wR, z .
z + z

5.10.3.  z z z = z = 1 . u =
1 + zz
.
5.10.4. (1+i)4(1i)4, .
5.10.5. (+i)2008=(i)2008 , .
5.10.6. z1=7+8i z2=45i.

 z = z1 z2 z
z4.
z 4i
, z2 , 5.10.7.  z. u =
z2
M(z)
.
5.10.8.
 z=x+yi, zz 2( z + z ) = 0 .
M(x,y)
.
5.10.9.  z=x+yi. u=2iz+1+i u = 4 , M(x,y)
.
2

5.10.10.  z1 z2 z1 + z2 + z1 z2 = 2 z1 + 2 z2 .
2

, i + z + i z
z z=3.
5.10.11. T
z
T ()
, >0
1

R i L

 z R,L,C .

rad/sec.
1
1
z
1
) h()
.

L 
, ! 0, T ()
R 1  i h()
R
C
) h.
) 
1+ih()
5.10.12. O RLC
i
z1 R  i L z2 R 
, R, , L, C .
C
)  z , z1 z2
1 1 1
. = +
.
) z , . z z1 z2

5
. , ,
.
. , ,
.
.
17
. ,
.

. (
) ,
.
, ...

6.1  .
6.2  .
6.3  .
6.4 
.
6.5 .
6.6 . .
6.7 .
6.8 .
6.9 .

lx im
x0

306

6.1 .

. , f
F, F(x)=f(x) .
:
) S(t) t, (t)
, 4, S(t) = '(t).
) P(t) t,
, 4, f(t)=P(t)
P(t).
) N(t) t,
f(t)=N(t) .
f
F, F(x)=f(x) . H F
f.
:
f ' . f F
:
F(x)=f(x), x .
, f F, .
1
, F ( x) = x 4 f(x)=x3 R,
4
1
1
( x 4 ) = x 3. , cR ( x 4 + c) = x 3 , 4
4
1 4
G ( x) = x + c = F ( x) + c , cR, f R.
4
:
f . F f
, :
)  G(x)=F(x)+c, cR, f

)  f G(x)=F(x)+c,
cR.
, F f, F(x)+c, cR,
f, (F(x)+c)= F(x)=f(x).
, G f, x F(x)=f(x) G(x)=f(x),
:
G(x)=F(x), x.

307

, 4.3, cR ,
G(x)=F(x)+c, x .
f f f ( x) dx (
x x). , F f :

f ( x)dx

F ( x)  c , cR.

, (ex)=ex (ln | x |) =

dx

ex  c

1
, :
x
1

x dx

ln x  c  .

, c
.
4.2.1, 4.2.2 4.2.5 6.1.1.
6.1.1
.

adx
1

x dx

x dx

ax  c (
a
( a RR) ) 

e dx
x

ex  c 

ln | x |  c 

xdx

x  c 

x 1
 c ( a z 1) 
a 1

xdx

x  c 

dx

x dx
2

x
a dx

x  c 
x  c 

ax
c
ln a

6.1.1 , x
. .
, :
xD 1
D
x
dx
 c 

D 1
c
x D 1
' (D  1) x D

 0 x D .
 c
1

1
D
D


6.1.1.

N f ,
A(2,4) Cf (x, f(x)) x xR.
.

Cf (x, f(x) f(x). , f(x)=x, :

308

x2
+c .
2
22
+ c = 4 c = 2,
f A(2,4) f(2)=4,
2
x2

f ( x) =
+ 2.
2
f ( x) = xdx =

, F f
, F(x)=f(x), x, :

f ( x)dx

F ( x )  c , cR.

, f(x)=F(x),

F c( x)dx

F ( x )  c .

:
f, ' , :

f c( x)dx

f ( x)  c , cR.

A F , G f, g ' R,
:
( F  G ) c( x ) F c( x )  G c ( x ) f ( x )  g ( x ) 

(O F )c( x) O F c( x) O f ( x) 
x . F +G f+g F, R,
f .
:
f g , ( ) f, R f + g.
:

f ( x)dx f ( x)dx, R .
) ( f ( x)  g ( x))dx f ( x)dx  g ( x)dx.

:
9
5 x dx

(4e

5 x 9 dx

 5x)dx

x10
c
10

x10
 c ,
2

4 e x dx  5 dx

4e x  5x +
 c.

309

4.2.3
. , :
c g c( x)
c
g c( x)

 2 g ( x)
g ( x)
g ( x)
2 g ( x)

g c( x)
dx
g ( x)

g ( x)

2 g ( x)  c , cR.

, 4.2.2, 6.1.2.
6.1.2
.

g c( x)

[ g ( x)]

dx

g c( x)

g ( x) dx

1
c
g ( x)

g c( x)dx

eg ( x)  c 

( g ( x)) g c( x)dx

ln | g ( x) |  c 

a
( g ( x)) g c( x)dx

g ( x)

( g ( x ))a1
 c ( a z 1) 
1

( g ( x)) g c( x)dx

( g ( x))  c 
( g ( x))  c 

, 3x 2 ( x 3 + 1) 4 d x
g(x)=x3+1, g(x)=3x2

3x

( x  1) d x

4
g c( x)( g ( x)) dx

( g ( x)) 41
4 1

( g ( x)) 5
5

( x 3  1) 5
.
5

6.1.2.

m/min t
(t)=t2(2t+3)+5. t = 0 3 m
, t=2 min.
.

4, u(t) = s'(t) S(t) . :


S (t ) = t 2 (2t + 3) + 5 = 2t 3 + 3t 2 + 5

Sc(t)dt (2t
:

 3t 2  5)dt

2t dt  3t
3

S (t ) =

dt  5dt 2 t 3 dt  3 t 2 dt  5dt 2

1 4 3
t + t + 5t + c
2

t4
t3
 3  5t  c. 
4
3

310

S(0)=3, 0+c=3, c=3,


S (t ) =

1 4 3
t + t + 5t + 3 .
2

t=2min
t=2. :
1 4
S(2)
m.
S( t )
2  23  5 2  3 29m
2
6.1.3.

N :
)

3x  5
x 2  4 x  3 dx ,

2x 2  5x  1
x 2  4 x  3 dx .

3x  5
3x 5
R{1,3} f ( x)

( x  1)( x  3)
x 4x + 3
( x1
x3) :
3x  5
a
E

.
( x  1)( x  3) x  1 x  3

) f ( x) =

, , ,

3x  5 D ( x  3)  E ( x  1)  3 x  5 (D  E ) x  (3D  E ) .
xR{1,3} :

D E 3

 3D  E 5

= 1, = 2. :
3x  5
( x  1)( x  3)

1
2


x 1 x  3

:
3x  5

( x  1)( x  3) dx x  1  x  3 dx x  1 dx  2 x  3 dx .

g c( x)

g ( x) dx

ln | g ( x) |  c 

6.1.2 g(x)=x1 g(x)=x3 ,


3x  5

( x  1)( x  3) dx

ln | x  1 | 2 ln | x  3 |  c

ln(| x  1 | ( x  3) 2 )  c .

311

) 2x25x+1 x24x+3,
2 x 2 5 x + 1 = 2( x 2 4 x + 3) + (3x 5)
:
:

2 x 2  5x  1
x 2  4 x  3 dx

2 x 2 5x + 1
3x 5
= 2+ 2
2
x 4x + 3
x 4x + 3

2dx  x

3x  5
dx
 4x  3

2 x  ln(| x  1 | ( x  3) 2 )  c .


,
(.. ,
).

6.1.1.
 N(t), , N(t)=et .
.
6.1.2.
 (t) t, (t)= t.
S(t) t
S(t)=a(t), a(t) .
6.1.3.
 .
) F(x)=x23x+2

) F(x)=ex2

) F(x)=e2x+1

) F(x)=ex+2

) F(x)=x(x3)

) F(x)=e2x

) F(x)=e2x2e

) F(x)=e2x(e+e2x)

) F(x)=x2+3x2

6.1.4. .
3
)
) (( 44 xx 3 
2
2
xx 

xx)) dx
dx

99
39
29
)
) (( xx 99 
4
4 xx 39 
3
3 xx 29 ))dx
dx

)
)

)
)

2
x 22 
 xx 2 dx
x 

e
5

5e  xx 33 dx

x2
4 xx11 )dx
)

) ((33xx22x 
 24 x 1)xdx
1
) )
(3 (3  4 4 ) dx) dx

3  2

x 
x
2
2

2
2

dx
xx dx

x 22  3x  2
2x 2 3 x  2 dx
)
) x x 3xx223x2 2dx
) )
x 2xx 2 dxdx

6.1.5. :
f ( x)

x  x
,
ex

g ( x)

) f(x)=g(x) xR.
)

x x
dx .
ex

dx
xx xx dx
xx 
3
3
)
) x  9 dx
dx
x 9

)
)

3
3

2
2

x 22  x
2 x 2  x dx
)
) x x xx33 x dx
) )
x 3 xx 3dxdx

x .

ex

312

x 3
.
x 6x + 8

+
) , , f ( x ) =
.
x2 x4
x 3
dx
) f ( x )dx = 2
x 6x + 8

6.1.6. f f ( x ) =

6.1.7. , 6.1.2.
2
6
) 2 x( x + 3) dx

4 x3 + 3
dx
dx
) 4
x + 3x + 2

) x e

x
x
) e e dx

x
x
10
) (2e + 1)(2e + x ) dx

x
dx
x

dx

ex
dx
(2 + e x )3

2x

x 2 + 1 dx

1
dx
dx
x ln x

 f, f(x)
6.1.8.
xR, f(0)=f(0)=1 f(1)=4.
 f ,
6.1.9. N
A(1,4) Cf (x,f(x)) 3x2+2x+1
xR.
 E(x), 0x1000 x ,
6.1.10.
x
E( x ) = 500 , x
2
10. 200 ,
.
.
 m/min2 t
6.1.11.
a(t)=et+2t1. t=0 3 m
1m/min.
) t.
) t.
) t=2 min.
 ' ,
6.1.12.
:
t2
f ( t) = 0,1 + 0, 3t
,
60
f(t) t .
2 3 ( f f(0)=0).
2

 , 6te3t , t
6.1.13.
. T0=37oC,
t.
6.1.14. :

313

x2 4x + 3
dx
dx
) 2
x 4x

9 x2 + 3x 1
dx
) 2
9 x + 3x 2

x2 3x + 2
dx
)
x2 4

x x2 + 2
5e 2
dx
)
x 1

x2 3x + 1
dx
)
x2 9

2x
dx
x +9
2

6.2 .
f, g ' ,
( 3 . 3.2) :
( f ( x ) g ( x )) = f ( x ) g ( x ) + f ( x ) g ( x ) , x.

f ( x) g ( x)dx = ( f ( x) g( x))dx f ( x) g( x)dx = f ( x) g( x) f ( x) g( x)dx + c ,

( f ( x) g ( x))dx = f ( x) g( x)dx + c .

c (
) ,
.

f ( x) g ( x)dx = f ( x) g ( x) f ( x) g( x)dx

, f(x) x,
.
, xe 2 x dx , e2x
c
12 e 2 x ', :

xexe

22xx

dx
dx

11

22xx '

xx22ee

11
11 22xx 11 22xx
11
dx xx ee22xx ((xx))cc ee22xxdx
dx
xe
xe  ee dx
dx 
dx
22
22
22

22

xe

2x

dx =

1 2x 1 2x
xe e + c .
2
4



x
P( x)e dx , P( x)( x)dx , P( x)( x)dx , P( x)ln( x)dx ,
P(x) x ,

x
x )exxdx.
dx 
(ax)e dx, ((ax)e

.

314

6.2.1.

N :
) x 3 xdx

2
) ( x + 2 x + 1)ln xdx

2
x
) ( x + x )e dx

) (ex)=ex, :

(x

+ x )e x dx = ( x 2 + x )(e x )dx = ( x 2 + x )e x ( x 2 + x )e x dx == ( x 2 + x )e x (2 x + 1)e x dx .

(2 x + 1)e

dx = (2 x + 1)(e x )dx = (2 x + 1)e x (2 x + 1)e x dx =


= (2 x + 1)e x (2 x + 1)e x dx = (2 x + 1)e x 2e x dx = (2 x + 1)e x 2e x + c

(x

+ x )e x dx = ( x 2 + x )e x ((2 x + 1)e x 2e x + c) = ( x 2 x + 1)e x + c 1 .

. ,
c
.
) x2+2x+1 ( 13 x 3 + x 2 + x ) :

(x

+ 2 x + 1)ln xdx = ( 13 x 3 + x 2 + x ) ln xdx = ( 13 x 3 + x 2 + x )ln x ( 13 x 3 + x 2 + x )(ln x )dx

( 13

x + x + x )(ln x )dx =
3

1
3

x3 + x 2 + x
x

dx = ( 13 x 2 + x + 1)dx =

2
3
2
( x + 2 x + 1)ln xdx = ( 13 x + x + x)ln x (

x3 x 2
+
+ x+c
9
2

x3 x 2
+
+ x + c)
9
2

) 3x ( 13 3x )c :
1

x3xdx = 3 x(3x)dx = 3 x3x + 3 3xdx = 3 x3x + 9 3x + c .


6.2.2.

xdx
N e 2 x 4(4x)dx.
.

xdx
I= e 2 x 4(4x)dx
I

( 12 e

2x

)c(4 x )dx

1 2x
e (4 x )  4 e 2 x (4 x )dx 

1
1
= e 2 x (4 x ) 2 e 2 x (4 x )dx = e 2 x (4 x ) (e 2 x )(4 x )dx =
2
2

315

1 2x
e (4 x ) e 2 x (4 x ) e 2 x ((4 x )) dx =

1
= e 2 x (4 x ) e 2 x (4 x ) 4 e 2 x (4 x )dx .
2
:
1
= e 2 x (4 x ) e 2 x (4 x ) 4
2

1 1

= e 2 x (4 x ) e 2 x (4 x ) .
5 2

, :

2x

(4x )dx

1 2x
1
e (4 x )  e 2 x (4 x )  c , cR.
10
5

f ( g ( x )) g ( x )dx f F.
f, F F=f, F(g(x))=(Fg)(x)
( F ( g ( x ))) = F ( g ( x )) g ( x ) = f ( g ( x )) g ( x ) ,

f ( g ( x)) g ( x)dx = F ( g( x)) g ( x)dx = ( F( g( x)))dx = F( g( x)) + c


u=g(x)

f ( g ( x)) g ( x)dx = F(u) + c = f (u)du .


,
.

f ( g ( x)) g ( x)dx = f (u)du ,

u=g(x) du=g(x)dx.

, (3 x 2  2) (x 3  2 x  1)dx
dx 3
2
3
u=x +2x+1, du = ( x + 2 x + 1)dx = (3 x + 2)dx

(3 x

 2) (x 3  2 x  1) dx

(u)du

 u  c ( x 3  2 x  1)  c .


3
(g (x)) g ( x)dx = (g ( x)) + c 6.1.2 g(x)=x +2x+1.

316

6.2.3.

N :

3
2
19
2
) x( x + )dx
) x( x 2)( x 3 x + 1) dx =
3
.

(1 / x )
dx
x2

, du = ( x 2 + )dx = 2 xdx :
3
3

1
1
1
2
2
x( x + 3 )dx = 2 udu = 2 u + c = 2 ( x + 3 ) + c .

) u = x 2 +

) u = x 3 3 x 2 + 1 , du = ( x 3 3 x 2 + 1)dx = 3 x( x 2)dx ,
:
1 19
1 u20
1
3
2
19
x
(
x

2
)(
x

3
x
+
1
)
dx
=
u
du
=
+ c = ( x 3 3 x 2 + 1)20 + c .

3
3 20
60
1
1
1
) u = , du = ( )dx = 2 dx , :
x
x
x
(1 / x )
x 2 dx = udu + c = u + c = (1 / x) + c .
.

6.2.1. N .
2 2 x
) x e dx

2
3 x
) ( x + x + 1)e dx

11
) x ln xdx

) e 3 x 2 xdx 

6.2.2. N .
2
) 3d

) u5udu

2
) (2t + 3t)ln tdt

6.2.3. N .
x3
2
2
9
dx .
(
x

2
x
+
3
)
(
x

1
)
dx
) x x 2 + 1dx
) x5x dx
)
)
4
1+ x
x+2
2
2
dx
) e d
) x 9 x dx
te  t dt 
) 2
5
)

( x + 4 x)
6.2.4. N .
)

ln x
dx 
x3
3

2 x
) x e dx

e
dx
(e + 2)ln(e x + 2)

) e

2 x +1

(3 x )dx

1

x dx 
)
x2
x
dx
)
x

317

) e 2 x (5 x )dx

) x 5 e x dx
3

x 2 + 1 x 2 + 1 dx

6.2.5. N :
( ) + ( + ) = 2 ,

( ) + ( + ) = 2 ,

( ) ( + ) = 2 .
) (20x )(10x )dx 

) x (3x )dx 

) (3x )(5x )dx 

6.3 .
f [,], ,
f(x)0 x[,]. E()
f, x x = a, x =
:

) [,] , x =
,

=x0<x1<x2<...<x=. [,].
) [x1,x], =1,2,..., . 1,
2,..., .
x f(). (. 6.3) :

f ( ) x 

) (),

E1  E2  ...  E

f (1 ) x  f (2 ) x "  f ( ) x,

R=[f(1)+... +f()]x.
,
, R
y

.
R, ( x=()/
, . )
.
f()
f(2)

f(1)
f , :

f ( x)dx 

f
.

Riemann, (6.3.1)

(6.3.1)

y=f(x)

f()

=x0 1 x1 2 x2 ... x_1 x ... x_1 x=


_
x=

. 6.3.

318
y

Riemann Riemann.
Leibniz.
.

4,
f .

f ( x )dx x

x_1

. 6.3.

, , f ( x )dx , f ( t)dt , f (u)du

( f ( x )dx
6.1, ).
f [,] ,
, [x1,x], =1,2,..., , ,
(
3.8).
(. 6.3):
f ( ) d f ( ) d f ( ) 
E f ( ) x  x f() :
f ( ) x d f ( ) x d f ( ) x f ( ) x d E d f ( ) x 
( E
f, x x = x = .
:

f (1 ) x  f (2 ) x "  f ( ) x , S

f ( 1 ) x  f ( 2 ) x "  f ( ) x 

(6.3.2)

sRS, s()S.
, (
x=()/ ), s, S
. R
( sRS) E() ,
f, x x = , x = ( s()S).
f ,
R.
6.3,
f [, ].
f [, ] :
f(x) 0 x [, ], E()

f ( x)dx t 0 .

319

, f [, ],

f ( x)dx ,

f ( x)dx > 0.

,
[, ] (6.3.1), (6.3.2),
sRS. , s, R, S , ( x=()/ ).
f . ,
,

f ( x)dx

xx
xx (. 6.3 6.3)1.

f ( x )dx

< ( [, ]).
= >, :

f ( x)dx
y

0 

f ( x)dx

 f ( x )dx , >.

s
O

f()

R
1

. . .

. 6.3.

. . .

y=f(x)
y

O =x0 1 x1

x2

. 6.3.

+
x_1

x= x

. 6.3.

1.  , .. [, ]. ,
.

320


,

( ).
f f(x)0
x , , , << (. 6.3).
, 1 2 ,
()= (1)+(2).

E( )

f ( x)dx,

f ( x)dx, E(2 ) f ( x)dx, 

E(1 )

Cf

. 6.3.

f ( x)dx f ( x)dx  f ( x)dx .

, , ( )
f. :
f , , , :

f ( x)dx f ( x)dx  f ( x)dx .



, .
f, g [, ] R. :
)
)

f ( x)dx

f ( x )dx 

[ f ( x )  g ( x )]dx

f ( x )dx  g ( x )dx 

, f (f(x)0 x [a, ])

f ( x )dx
a

E() t 0 . f, g , f(x)g(x)

x [, ], :

g ( x)dx  f ( x)dx a ( g ( x)  f ( x))dx t 0 ,


:

f ( x)dx d g ( x)dx .

, :
f, g [, ],
f(x)g(x) x [a, ]. :

f ( x)dx d g ( x)dx .

321

f [,], x [a, ]
 f ( x ) d f ( x ) d f ( x ) . :

a ( f ( x) )dx d a f ( x)dx d a

f ( x ) dx  

a f ( x)dx d a

f ( x ) dx d f ( x )dx d

f ( x ) dx 

f ( x ) dx .

, :
f [a, ] 1:

a f ( x)dx d a

f ( x ) dx.

6.3.1.

f ( x )dx

2, f ( x )dx 1 
1

)
)

1 g ( x)dx

4 .

1 [3 f ( x)  2 g ( x)]dx .

3 f ( x )  2 g ( x ) dx t 2.

2 f ( x)dx. 

) :
2

1 [3 f ( x)  2 g ( x)]dx 1 3 f ( x)dx  1 (2) g ( x)dx 3 1


2

f ( x )dx  2 g ( x )dx 3 2  (2) 4 2 .


1

) :

[3 f ( x )  2 g ( x )]dx d 3 f ( x )  2 g ( x ) dx ,

) :

2 f ( x)dx 2 f ( x)dx  1

3 f ( x )  2 g ( x ) dx t 2

2.

f ( x ) dx  f ( x )dx  f ( x )dx  1  2 1 .
1

5
11 11
55
6.3.1.


1 555 ln
6.3.1.

ln
x222dx
dx 33511 ln
ln11 dx
dx 55 55 ln
lnxdx
xdx...
5
6.3.1.
x
1
2
2

x
11 ln
55ln
11 ln
3
5
1. 
ln
x
dx

ln
dx
ln
xdx
.

3
3
5
5
6.3.1.
6.3.1. 

ln
x
x
dx
dx


ln
dx
dx
ln
xdx
xdx
..
x
1
5 11 x
1 505 xx
11

0
2
2
6.3.2.

22 xdx
xdx  0 (1
(1
22 xx))dx
dx 00...

6.3.2.


000
6.3.2.
2
2 2  0
2
2. 

xdx

(1


x
)
dx
0
.

6.3.2.
6.3.2. 


xdx
xdx


(1
(1



x
x
)
)
dx
dx
00 ..
10100
001010

10
10
6.3.3. 
 10
dx 55,, 10
dx 22
 10
dx 13
13,
,
10 6.3.3.
dx
1010ff((xx))dx
10 ff((xx))dx
3103 ffff((((xx10xx))))dx
88 ff((xx))dx
3.  6.3.3.
f 6.3.3.
( x )dx
5 , 11 ff((fxx())xdx
)dx 55 2,,,
f ( x )2dx
13,
6.3.3.


dx
dx
2

dx 13
13,,
,

1
11 3
33 33 8
88
88
88

f
(
x
)
dx
,
f
(
x
)
dx
,
f
(
x
)
dx
.
3
8 33 f ( x ) dx8, 88 f ( x ) dx , 88 f ( x ) dx . 



f
(
x
)
dx
,
dx,, 11 ff((xx))dx


((xx,))dx
dx
((xx.))dx
dx.. 

f(11x )ffdx
,, f(33x )ffdx

4.

1
3 11
1 33
11
10
55
88
10
6.3.4.


f
(
x
)
dx

2
,
f
(
x
)
dx
2


f
(
dx 44




5 6.3.4.  55 f 8( x ) dx 2 , 88 f ( x10
)dx 2   10
10 f ( xx))dx
55 ff((xx))dx
88 f

f6.3.4.
( x )dx 
2 ,22 ff((xfx)()dx
x )dx 222,, 
f ( x )22dx
4  

6.3.4.

dx
dx

f((xx))dx
dx 44






2
8
22 5
55
88
10
5
88
10
5
1. 

+ ,

dx
dx
dx ..2 ,
8
10 88 ff((xx))dx

,,5 10
ff((xx))1dx
,2510
ff((1xx))+
dx
10
5
2
2
2 + ... + 1 + 2 +

f (...x+
)dx
,dx ,, f2( xff)((dx
.dx
 ,, 10 ff((xx))dx
, . f (2 xff)((dx
1 +

xx))dx
xx))dx
dx ..
2
2 22 
10 22
10
10




.. 

xdx
xdx  (1
(1

xdx
xx))
dx
dx 0(1
0 ..
6.3.2.
6.3.2. 


xdx

xx))dx
dx 00 ..
00
(1

.. 


10
10

11

00
10
10
10 10

10
10
10
10

10
10

ff6.3.3.
((xx))dx
dx 55
dx 5522,,
 ff((xx))ffdx
((xx))dx
dx
13
13,
,
6.3.3.
,, 33 fff(((xxx)))dx
dx
22 

dx 13
13,
,
88 dx
ff((xx))dx
11

33

88

33

88

88

33

88

88

33

11



dx,, ff((xx))dx
322 

fdx
f((,xx, ))dx
dx
,,ff((xx))dx
fdx
f((.xx. ))dx
dx,, ff((xx))dx
dx.. 
11 ff((xx))dx
33
11
11

55

55 88

10
10

88

10
10

.. 
 ff(6.3.4.
(xx6.3.4.
))dx
dx 
22,, ff(f(fx(x()x)xdx
))dx
dx 22 ,,,
 ff((xx))dx
f((xx))dx
dx
44








6.3.4.

22 


ff((xx))dx
dx 44 








55 dx
88 fdx


22
22

55

88

88

10
10

55

10
10
55



dx ,, ff((xx))ffdx
dx
dx
dx ,, ff((xx))dx


((xx,),,)dx
dx
((xx).).dx
dx ..
22 ff((xx))dx
1010,, ff((xx))ffdx

22

6.3.5. 

0 f ( x)dx



1 

88

22

22

10
10


2, -

0 g ( x)dx

. 3
3
3
f ( x )dx 
3 ) 2
3 ) (33) g ( x ) dx 
3 ) (32 f ( x )  3 g ( x )) dx 
3
3( 2 f 3( x0) 3 3 g ( x )) dx 
) 32 f)
(3x )0dx 2 f ( x )dx  ) 3(3)
)3 g 0( x )(dx
(3x))gdx
( x)dx )
))gdx

(xx))
))
dx 
) )
f
(
x

)
dx

)
)
(

3
)
(
g

(
3
x
)
g
dx
)
)
(2 )
f)
((x2)0f0(((3x22)gff(((xx3x))
(x33))
gg(dx
 dx
0 2 f ( x2)dx
0
0
)

)
0
0

0 0
0 )
0
0 0
0
0
3
)dx 
)
0 ) (02 f ( x )  g ( x ))
0 3 0f ( x ) dx
3  2 g3 ( x ) dx 
)
0
3
)
)
3  2 3 g ( x3) dx
0 
0 f ( x0) dx
) 0(2 )
f 0( x3) (g2(fx())x )dx g ( x ))0dx) 0f ( x )3dx  g3 ( x ))
3 )
dx  )
)0dx
( x)dx 
)
)
) )
2)3f( xg)( x))gdx
( x ))
 dx  0 f ( x0)dx
3 3)
3f ( x)3dx
f(3x)fdx
2(x
2g (x )2gdx
( 0x)gdx
3 ( 2 f ((x

3  3 g ( x3) dx
0 

f
(
x
)
dx

g
(
x
)
dx

3
3
3
3
0
0
3 f ( x)dx

f( x)dx0 g ( x )gdx
( x)dx 

3 3

0 0
0


6.3.6.  f,
R.


3

.
2

f ( x )dx  f ( x )dx
5

10

f ( x )dx  f ( x )dx 

f ( x )dx

1 f ( x)dx  1 f ( x)dx

3 f ( x)dx  3 f ( x)dx  1

f ( x)dx 

f ( x )dx

6.3.7. N :
5

5 ( x

5 ( x

 1)dx t 0

1 ln xdx t 0

 5 x  9)dx t 0

1/ 2 ln xdx d 0

2 3

2 2

1 x dx t 1 x

1 3

1 2

0 x dx d 0 x

dx t xdx
1

dx d xdx
0

6.4  .
, ,
, ( , , ).

, .
f
( ). F :

F( x)

f (t)dt, x .

(6.4.1)

( ) F
f, :
F(x)=f(x), x

(6.4.2)

[ (6.4.1), (6.4.2)]

f ( t)dt

f ( x ),  x .

(6.4.3)

323

(6.4.3) g(x), f g,
:
c
g( x)
'
(
(6.4.4)
f
t
)
dt
ff((gg((xx)))) ggc(c(xx)).


x
x
x      f  




(6.4.2),


F
(
x
)



(6.4.2),

F ( x )F ( xf)( tf)dt
( t)dt



f
f  

 

(6.4.2),
 
f (t
)dt  


 f[,].
 

(6.4.2),


f ( t)dt ,,
:

[ ,[],.
F0(
F)( )F ( f)( tf)(dtt),
F (aF)(a)F
(af)(tf)(dtt)dtf (0t

0 
dt
[].
, ] .
)
dt
f (,
t)dt ,

f (at)dta

F ( )  F ( a) .
f(tf)dt
( t)dt
F ( )F( F)(a)F.(().
f ( t)dt
a) .

 

, ,

f ( t)dt  ,

F f [, ].

.
f [, ]. G
f [, ], :

f (t)dt G( )  G() 

, G f [, ], F ( x )

f (t)dt  -

f [, ], cR , G(x)=F(x)+c.
x = x = :
G() F ()  c
G( ) F ( )  c

f (t)dt

f (t)dt  c

c c G(),

f (t)dt  c f (t)dt

G( )  c.

G( )  G().

G()G() [G( x )] , :

f ( x)dx

[G( x )]  G( x ) = f ( x )dx .


6.4.1 ( c), 6.1.1.
6.4.1
( ) .
f(x)

x, 1

1
x

ex

1
2 x

1
2 x

x +1
+1

ln x 

ex

x
ln

324

,
6.2 ( ) , , .
)
)

f ( x) g c( x)dx

[ f ( x ) g ( x )]  f c( x ) g ( x )dx 

g ( )

f ( g ( x)) g c( x)dx g () f (u)du  u=g(x), du=g(x)dx.

6.4.1.

N cR

c dx

c(  ) .

f(x)=c [, ] F(x)=cx. :

c dx [cx] c  c c(  ) .

6.4.2.

:
2
2 x + 2x 3
9 2
31

dx ,
) x dx ,
dx xdx ,  ) 1
3
1 x

x

x  3 dx 

) 6.1.1
:
9
31
x3
9 2
93 33
dx = [ln x ]13 = ln 3 ln1 = ln 3 ,
,
x
dx

243

9
234


3
1 x
3 3 3 3

 xdx

[ x ] 

  (  ) (1)  (1) 0 .

)
2

2
2
2
21
x2 + 2x 3
3
dx = ( x + 2 )dx == xdx + 2 1dx 3
dx =
1
1
1
1 x
x
x
2

x2
1
1
= + 2[ x]12 3[ln x]12 = 2 3 + 1 3 ln 2 = ln 2 .
2
2
2 1
)

3  x , x d 3

x 3
x  3, x t 3

:
4

x  3 dx

1 (3  x)dx  3 ( x  3)dx

9 5
x2 x2
3
x


  3 x =  2 2 + 4 2 = 5 .
2 1 2

325

6.4.3.

)

)
x
 xdx
 
x)xdx
0
0

3 ln x
3
3
2
2
 )
))
0 x 90 xx dx9  x dx   ) 2  x dx

) :
I

0 x(x)cdx [ xx]  0 ( x)cxdx [ xx]  0 xdx

[ xx]0  [x]0 (  0 0)  (  0) 1.


1
) u=9x2, du=2xdx, xdx = du . x=0 u1=9 x=3
2
u2=0. :
1
1 2
1
1
u ( ) du  [ u3/ 2 ]90  (0  93 )  (27) 9. 
2
3 3
3
3
1
) u=lnx, du = (ln x ) = dx , u1=ln2 u2=ln3. :
x
3

0 x

9  x 2 dx

3 ln x

ln 3

u2
udu

0
2 ln 2
1

dx

(ln 3)2  (ln 2)2 .



2

6.4.1. f(x)=xex F(x)=xexex.


) F f.
)
2

xe x dx ,

xe x dx ,

xe x dx ,

1 xe

dx. 

6.4.2. f(x) = xx:


) f.
) :
/2

6.4.3.

/2

0 (x  xx)dx,  / 2 (x  xx)dx .

(x  xx )dx ,

f ( x ) = ln( x + x 2 + 1) .

) f.
)
1

1
1+ x

dx ,

1
1+ x

dx ,

1
1+ x2

dx .

326

6.4.4. :

1 x
e dx 
0

/2

(2x  3x )dx 

x
1 e dx 
2
2
1
x
+
) 1
dx
x

25

dx 
2

) 1

1
x dx
x

6.4.5. :
x

6.4.6.

x 2

0t

1  t 2 dt 

) F ( x )

) F ( x )

(t 2 )
dt 
x
t

) F ( x )

e  t dt 
2

x 3  2 x 5  xR, f(2).

f ( t)dt

6.4.7. F ( x ) e x  e x f ( t)dt.
2

6.4.8. :
2 2222 1 11 2 22
12 2 dxdxdx
22x x 
)

)
)
)
dx
122 23233233dx
2xxx112122
)

  
)
12
1111

x
x



x
dx
)



)
x
dx
x
x
2
3

x
x

) 111x x2 2x2 x3 3x3dx



1 xx x xx x
1 11 11 2 2
11( x (22x
2 23x 3
11dx
))dx
)

x1)dx
  
))
)dx
)
dx
12

)

))
)1dx
)dx
222((21((x22x2xx2((2xx23333xx3xx3xx11)11)dx
)
  
2
2 2222
1
1 1 1
22x x1
)
)
  
)
dx
dx
02xxx1111
)
11x112dx
dx
2

dx
)
0

0
dx
dx
))
00000xxx111xxxx2222dx
)
)
 
xx22 2
0
2
x

  
 
6.4.9. :
333

2
3(3x32(22x
2 29
)
)
))dx
)
)9)dx
  
3 (3x( 2x
)
)9dx
)
9dx
dx

29

)
xx2(x
03

00(0(
)
9)dx
)9dx
)
)dx

0
) 00( x0 9
  
0 222
2 23
42 x
4 4x

3xx33xxx22
222dx 
dx
444x4x2 2x
23
)
  
)
dx
)
4


x
3
x


x
3
x
2dx

)
4

)
dx
)
dxdx
)
)
11111x111  3xxxxxx 22dx
 
)

1
xx
x
2
2 22 2
2 22 22
3 33 33
2
33x)(
))dx
dx
)
)(
  
2( 2
))
(2222x((x22xxx33
3)()(
x222xx233
3xx33xxx11)
1)3)311dx
)

)(


22
3dx
3dx

)
(
x


)
)dx
dx
222(22(222x(x2x33)()(3x)(x x33xx3x1)1)1dx
)))
  

1( x

 x )dx 

x 2  1 dx 

2

3 3333 2 2
111 dx 
22 2  1 1
332
)
)
)
  
21111
)
dx
11232xx22x222xx
11313x3133dx
)
dx
3


)
dx
1
dx
))
1111122xx2 x111xxx333x3dx
)
 
dx
1
x x
x
20
2020
2020 3 33 33
20
)
x 3x )3dx
xxdx
))dx
)
(
2 x(23 
  
20
)
)dx

)
dx
20

)
)
)
)xdx
)
)dx
2020202020((202020202((222(xx(2xx323xx33333xx3xx))3dx
)
 
 20
3 3333
1
1 1
2
33x 222x
2 2xxx 1 1dx
)

)
dx
)
dx
dx
22x3x22x
)
3

2 xx 1x

)

dx
  
2

dx
))
dx
22222xx xxxxxxx1x111111dx
)
)
2
xxx11 1

x 2  5 x  6 dx 

6.4.10. :
1
1x  11x  1x  1
1 31 43 1 434 4 4
1 )
1x  1
1x  1xdx
1 x3)
1( xx43 1(x1
 1 dx  dx) )
) )
)
)
x4)34
(dx
x14)4dx
1)44 dx 

2
2
2

dx

) )
dx

dx

0
0 x2)
0x
)
x
(
x

1
)
dx

)
x
(
x

1
)
dx
)
x
(
x

1
) dx 
1

1

1

x

x

2
6
x
2
6
x

2
6
20 2
0 x 0x2
0
0
0
x2x26x2 26x  6 1 1 1
2
1
1
1
1 2 11x 2 1 x 2 1
1
11
1
1x2 x
2 x12  1
2 xe
1 xe
1 x)
1 xx2 xe
1 )
1x x11  x1dx
 1dx  dx)
)
(dx
x122)55dx
1)55 dx 
) )
( x122x(x1122)x55
)

)
)
11xdx
1 dx
x 2 1 dx 
)

3
3
3
) )
)
 dx  dx 
x
(
x

1
)
dx

)
xedx
x
(
x

1
)
dx
)
)
x
(
x

1
) dx 
1 xe
0
dx

) )
dx

)
0
1 xe
0
1
dx

0
)
0 x3)
0x3
0 0 0
x32x32x  2
3 x0
1 1 1
0 x 0x3
xx332x32x  2
  
 

6.4.11. :
2
x  332
/2
33222
x x)
33 / 2
3 2 (
22

xdx33
2xx 
32x 2 3xdx
)
//22
3

x  )

/
2
3 2
xx22 dx
)
x

xdx

)
x dx

/
2
33
)
3 

x
3 x

2
3
)
)
)
2
)

xdx
)
dx
3
)
1
)
dx

2
2
)
)

xdx

)
dx

2 )
2

00 )
) )

xdx

)
dx

1
x

x
(
)
6

xdx

)
dx


xdx

dx

2 2 xxx22 xx xx 1 ((xx12 (x61621xx(())x2x62 x)266xdxx))22 dxdx

0 0 00
111
1
22

1 11

1
22 1  11
1

1
2
2
33//

2
11   1
2
x
ln
xdx

)
2
2

12 ee 1 xx2x2dx
x/ x
)
xln
ln
xdx
xln
) )
)
dx

3/ 3/ 33/xx
x

ln
xdx

)
1
x
xdx

)
x

)

dx
x
ln
xdx

)
)
e
dx 

x
)
)
1
)
)
e
e
dx

dx
xdx
x
xdx

)

x  dx 
1
)

) )
dx  dx

1 x
22

1
221 211 22 e
)
1
)
22// )
dx
1
x
1
1

dx

2/ 2/ xxx222//x 2 xx22
x x xx
2
5
2
1
2
x 2
22 x2x 2x
x
2 x ) dx
11 x444e1xxx554dx
22 x ( e2xxx22 
dx
)
22 
)
)
11x 544 xx55
22x2
xdx
dx  xdx
)
x
e
dx
)
x
(
e

2
x
)

)

x
22 dx

dx 
)
1
)
x
e
dx

)
x
(
e

2
x
)
dx

)
)
00 x )
11 x ()
e
dx

)
e

2
x
)
dx

) )
)
)
)
x
e
dx

)
x
(
e

2
x
)
dx

)
x
e
dx

x
(
e

2
x
)
dx

1
9

x

2
2

1 1 11

x 11 x99 xx22 dx 
1 99
1 x9
0 0 00



  
6.4.12. :
1

12

12

t t
1 te
1tt2tedt
)
)
) )
 t012tetedtdt t2 dtdt 
) )
0 te)
0 tedt
2

2
2

2
2

0 xe

2x

dx 

5
5

1 2  x/2

0x

2
2

dx 

0 (2 x

 3 x  1)e  x dx.

327

6.4.13. f ( x ) =

x
x2 + 2x + 3
g ( x ) =
.
x +1
x +1

)  , f :

f ( x) 
.
x 1
1
) f ( x )dx .
0

) , , , g :
.
g ( x ) x  

x 1
)

0 g ( x)dx .

6.4.14. f, g f ( x ) =

1
x 1
.
, g( x) = 2
x 4
x 5x + 6
2

) , , f :

f ( x) =
+
.
x2 x+2
)

3 f ( x)dx .

) , , , g :

g( x) =
+
.
x 2 x 3
)
6.4.15.

4 g ( x)dx. 

2 x 2 ,
f ( x)
3
6 x ,

1 d x  0
0 d x d 2,

f 0
2

1 f ( x)dx .
6.5 .
, ,
,
() /
y
Cf
.
6.3
, () ,
(f(x)0
x[, ]) [,], x = ,
x = x, (. 6.5)
x
O

E( )

f ( x)dx. 

. 6.5.

328

, f(x)=x2, x=1, x=1 xx :


1

1 x

dx

x3
13  (1)3

3
3 1

2
.
3

f [,],
(f(x)0 x[, ]) f (
) f x
= a, x = xx (. 6.5). , xx
, :
E(c)

E( )

 f ( x )dx. 

( f ( x ))dx

f, g, [, ],
f(x)g(x)0 x[, ]. () ,
f, g x = a x = ,
6.5 .
:
E( )

f ( x )dx  g ( x )dx

( f ( x)  g ( x))dx. 

f, g, [, ], f(x)g(x) x[, ].
f(x)g(x) [, ], 6.5,
, f, g x =
x = 1 2, :

E() E(1 )  E(2 )

( f ( x )  g ( x ))dx  ( g ( x )  f ( x ))dx. 

:
E( )

f ( x )  g ( x ) dx , 

() ,
f, g
x = x = .
y
y

Cg

Cf

Cg

Cf
O

. 6.5.

Cf

C_ f

. 6.5.

. 6.5.

329

6.5.1.

f(x)=x3+x2, x=2, x=1 xx.


.

xx
f(x)=0. x3+x2=0 x2(x+1)=0 x=0 x=1.
f(x)=x3+x2 [2,1] f(x)0 x[2,1] f(x)0
x[1,1] (. 6.5).
, :
1
1
x 4 x3
x 4 x3
1
1 3
25 .
2
3
2
E()  ( x  x )dx  ( x  x )dx    

2
1
3 2 4
3 1 12
4
6.5.2.

 ,
N
f(x)=x2, g(x)=3x x=1, x=2.
.


f(x)g(x) [1,2]. :

g( x) x 2

f ( x)

3 x x 2  3 x 0 x( x  3) 0 x

0 x 3.

[1,2].
f(x)g(x) [1,2] 6.5.1 (. 6.5).
6.5.1

x
f(x) g(x)

1
4

0
+ 0

2
2

x[1,0] f(x)g(x) x[0,2] f(x)g(x).


,
E( )

1 3 x  x

dx

1( x

 3 x )dx  (3 x  x )dx
0

_ 1,5

_1

_ 0,5

_1
_2

. 6.5.

31
.
6

1
_2

x3
x 2 x3
x2
 3  3 
2 1 2
3 0
3

4
0,5

2
_1

_ 0,5

0,5

_3

_2

_4

_4

. 6.5.

1,5

330

6.5.1.  f, x = , x = , xx, :
1
) f(x)=ex, =1, =1
) f(x)=x2, =1, =3
) f ( x ) = , =1, =e
x
1
) f ( x ) = , =1, =e
) f ( x ) = x , =0, =4
) f(x)=x(x+2), =0, =3.
x

6.5.2.
f(x)=6x2 3x xx.


6.5.3.
f(x)=x3 + 1, g(x)=x2 + 1.

6.5.4.
f(x)=x2 2x+1 g(x)=x3 x2 x+1 x =2, x =3.

6.5.5. N
f(x) = 2x, g(x) = 2x x=0 x = 2.
6.5.6.  , 6.5.
C1:y=4x2+24x C2:y=5x2+30x.
20 /m2,
.

 f(x)=5x2.
6.5.7.
)
 Cf (1,5).
)
 Cf,
xx.

6.5.8.  6.5 . N ,
.
6.5.9. N

2
 x  2, x  1

f ( x)
2 x  1, x t 1
x=0, x=3 x.

0,8

C2

5
4

0,6

C1

0,4

2
1

. 6.5.

0,2
1

x
0,2

0,4

0,6

. 6.5.

0,8

331

6.5.10. N
 6.5.
6.5.11. f(x)=lnx.
)  f
x=1 x=e.
)  , Cf .
)  Cf, x
Cf A(e,1).
6.5.12.
 6.5.
)  f,
)  .
y=4 x

y=x
y

y=4

y=2 x
_1

15

. 6.5.

. 6.5.

6.6 . .
, ,
, .
f [,] , x = , x = xx (. 6.6). A
xx ,
(. 6.6).
Vf ( ):
Vf

( f ( x ))2 dx 

y
y
Cf

. 6.6.
. 6.6.

332

yy Vf
, f (
) [,].
f ( )

Vf

f ( )

( f 1( y))2 dy 

, :
f ( x ) = 2 x , 0x4
Cf x=0, x=4 xx (. 6.6).
xx, 6.6,
( ..).
:
4
x2
4
4
2
V f (2 x ) dx 4 xdx 4  = 32 .
0
0
2 0
f,
g x = x = f(x)g(x)0 x [, ],
:
Vf ,g

[( f ( x ))2  ( g ( x ))2 ]dx. 

, (. 6.6):
1
g ( x )
x

f ( x)

x .


1
0
x
x[1,2], :
x

Vf ,g

2
1

1
[( x )  ]dx
x
2

1
x  2 dx
x

2
1

x2 1

2 x 1

. 

y
A(4, 4)

y=2 x

g(x)= x

1
f(x)= x

. 6.6.

. 6.6.

. 6.6.

333

6.6.1.

6.6 xx, =() R. ,


, R.
.

(0,0) (, R) =R/.
:
f ( x)

R
x , 0 d x d .

E, o Vf
xx, :
Vf

R2
2

R
x dx
0

2
x dx
0

R2
2

x3

3 0

R 2 3
2 3

1 2
R . 
3

6.6.2.

, , R.
.
2
2
f ( x ) = R x , 0xR,

x=0 x=R xx (. 6.6).


xx,
Vf

( R  x
2

( R  x )dx

) dx

x3
R2 x 
3 0

( R 3 

R3
)
3

2 3
R . 
3

R ,
4 3
V 2V f
R .
3
y

R
y= x

y= R2 _ x2

x
_R

. 6.6.

. 6.6.

334

, Cf
[,]. '
y=f(x) (
[,]) lf (, ) lf (.
6.6).
f [,]
. y=f(x)
( ):

lf

A
O

. 6.6.

1  ( f c( x ))2 dx. 

lf

6.6.3.

1 x
x
f ( x ) = (e + e ) .
2
y=f(x) 0 1.
.

f :
1
1
f ( x ) = (e x + e x ) = (e x e x )
2
2

l f 1  ( f c( x ))2 dx , 

= 0, = 1
lf
1

1  (e x  e  x ) dx
2

1 x
(e  e  x )2 dx
4

1 1 x
(e  e  x ) dx

0
2

1
1  (e 2 x  e 2 x  2) dx
4
1 x1
[ e ]0  [ e  x ]10
2

1 2x
(e  e 2 x  2) dx 
4

1
(e  e 1 ). 
2

3
2,5

6.6.1.

xx y=3x2
x=1 xx yy
(. 6.6).
6.6.2.

xx

2
1,5
1
0,5
0

0,2

0,4

0,6

y=3x2

. 6.6.

0,8

335

y=2x x = 0, x = /4 xx
(. 6.6).
6.6.3.
 xx
f(x) = x2 g(x) = 3x.
6.6.4.

xx y = 2 x + 1 x=1, x=3
xx (. 6.6).
6.6.5.

xx y=3x2 x=1, x=3
xx.
6.6.6.


( x ) = x ,
xx, ff(x)
g(x)=1 x = 0 x = /3 (. 6.6).
6.6.7.
 x'x,
f(x) = 2x2 g(x) = 3x3.

 xx,
6.6.8. N
f(x) =xex
x=0 x=5 (. 6.6).
3

2,5

0,8

2
0,6

1,5

0,4

0,2

0,5
0

0,2

0,4

0,6

0,8

0,5

y=2x

1,5

2,5

y= 2 x+1

. 6.6.

. 6.6.
0,4

1
0,3
0,8
0,6

0,2

0,4
0,1
0,2
0

0,5

f(x)= x , g(x)=1

. 6.6.

1,5

f(x)=xe - x

. 6.6.

10

336

6.6.9.
 f ( x ) = 3 x 2 . , y = f(x)
[0,27].
6.6.10.
 f(x)=ex/2+ex/2. ,
y=f(x) [1,1].
1
6.6.11.
 f ( x ) = (e rx + e rx ) , r
2r
.
) 
1+( f (x))2= r 2 f 2(x).
)
 rf(x).
)  y=f(x) [,]
:
1
r
r
r
r
)]
l f = [([(e
e r ++e era ))(e
(e r++eera)].
2r

6.7 .
f .

F
F(x)=f(x), x.

f ( x)dx

f .

F ( x )  c, c R 

F f.
.

) f ( x )dx = f ( x )dx , R
) ( f ( x ) + g ( x ))dx = f ( x )dx + g ( x )dx

.
.
O f .

f ( x) g ( x)dx = f ( x) g ( x) f ( x) g( x)dx
f ( g ( x)) g ( x)dx = f (u)du , u=g(x)
du=g(x)dx.

f ( x)dx

G f [, ].

) f ( x )dx

G( )  G() [G( x )] 

) f ( x )dx

f ( x)dx  f ( x)dx 

f ( x )dx 

) [ f ( x )  g ( x )]dx

f ( x)dx  g ( x )dx 

) f(x)0 x [a,] f ( x )dx t 0 .

337

) f(x)g(x) x [a,]

f ( x )dx d g ( x )dx .

a f ( x)dx d a

f ( x ) dx 

f (t)dt

G f [, ].

f (ft()tdt)dt ' FFc(cx()x) f (f x()x) 


F( x)

xx

f (t)dt 

G( )  G() 

aa

f ( x) g c( x)dx

[ f ( x ) g ( x )]  f c( x ) g ( x )dx 

g ( )

f ( g ( x)) g c( x)dx g () f (u)du 


u = g(x), du=g(x)dx.
f [, ],
(a, ) ,

f ( )


f,
x=, x= xx.
E
f, g,
x=, x= xx.

V
f, g, f(x)g(x)0
x[,], x = x = .
y = f(x)
.

E( )

f ( x)dx 

E( )

V
Cf, x= ,
x= xx.

a f ( x)dx 

f ( x )  g ( x ) dx 

( f ( x ))2 dx 

Vf

[( f ( x ))2  ( g ( x ))2 ]dx 

Vf , g

lf

1  ( f c( x ))2 dx 

338

6.8 .
,
, .
1

1 x

 1 dx

0.

1.

2.

ln xdx xlnxx+ c.

3.

4.

 f(x)=g(x) x[2,2] f(1)=g(1)+1, x[2,2]


A
g(x)=f(x)1.

2 ln x
dx ln2x+ c.
x

5.

0 f (t)dt , f

F ( x )

6.8. F(2)=4.

. 6.8.

6.

7.


f

f ( x)dx = xf ( x) xf ( x)dx .


f,g [, ]. f(x)g(x)
x[, ],

f ( x)dx d g ( x)dx .

f, g [, ].

f(x)g(x) x[, ].

9.

8.

f ( x)dx

f ( x)dx d g ( x)dx ,

0 , f(x)=0 x[, ].

f, g [, ], :
10.

11.

12.

f ( x) g ( x)dx f ( x)dx g ( x)dx.

a

( x 4  2 x )dx d

a

( x 4  2 x  x10 e  x )dx, >0.

f [, ], [,], :

f c( x) xdx

[ f ( x ) x ]

 f ( x )dx. 

339

13.

f . F f
, G(x)=F(x)+ex f.

14.

f
, x0, x1 :
x
f(x1)= f(x0)+ 1 f c( x )dx .

15.
16.

17.

f, ' , :

f ( x)dx = f ( x) c , cR.

f ( x)dx

0 , f() = 0 (, ).

f(x) g(x) [, ],
f, g x =
x = E()

f ( x )dx  g ( x )dx  E()

g ( x )dx  f ( x )dx .

f(x)= _ x2

_1

18.


1

6.8 1 f ( x )dx

E( ).

_1

. 6.8.
y

19.

6.8
0

f ( x)dx  0 f ( x)dx .

. 6.8.

20.

21.

dx
( x 2  2 x )dx
0
f(x)=x22x x.
f [, ] .
y=f(x) :
lf

1  f c( x ) dx .

1.

To
) ln(lx)+c

1
dx :
1 x
) ln(xl)+c

) ln 1  x  c 

) ln x  c 

340

2.

f(x) = x f(0) = 1, f() :


) 1
) 2
) 0

) 1
y

3.

f 6.8. A 1, 2, 3 3,1,2 ,

) 4
A

4.

5.

6.

f ( x )dx  :

) 6

0 (4 f ( x)  3)dx

) f =

3
4

) 1

) 2

. 6.8.

0  f [0,2] :

) f =

3
8

) f =

4
3

) f=3

f(x) = c x[,] :
) ( +)/2

) c

) +

F, G f , :
) F G .
) F' = G G' = F.
) F' ^ G .
) F G f g.
A f, g f(x)=g(x) x[0,1] f(0)=g(0)+1, :

7.

8.

0 ( f ( x)  g ( x))dx

1 .

) f(x)=g(x)1 x[0,1].

) f(x)g(x) x[0,1].

) f(x)g(x) x[0,1].

f(x)=x2x, x[0,1] x :
)

( x 2  x ) dx 

0(x  x

) dx 

) 1

F f [, ], :
9.

)
)
 )
10.

f ( x)dx

f ( x)dx F () F ( ) 
 ) f ( x )dx f ( x )dx  f ( x )dx 

) f ( x )dx F( )  F () 
) f ( x )dx F ()  F ( ) 


)
dx  f ( x )dx 
) f ( x )dx f ( x )dx  f ( x )dx  ) f ( x )dx z f ( tx))dt

, ,, , :
)

F ()  F ( ) 

f( )

dx z
ff ((xx))dx
ff ((tx))dtdx  f ( x)dx  

)
)
 )

dx z f (ft)(dt
 
t)dt
ff( x)dx
f( a)

)
 )

dx  f ( xf)(dx
t)dt
 
ff((xx))dx
f ( a)

 )

f ( )

f ( x)dx  f ( x)dx 

)
)
)

))
 )

f ( t)dt 0 )


f ( t)dt 0 

f( )

f ( )

f ( x)dx z f(af)(tf)(dtt)dt 

f ( x)dx  f (af)( xf)(dxt)dt 

f ( x)dx  f ( x)dx 

f ( t)dt 0 

f ( t)dt 0 


 )

f ( x)dx  f ( x)dx

f ( t)dt 
341

 ) f ( x )dx  f ( x )dx f ( t)dt   )


) f ( x )dx  f ( tx))dt
dx f (f u( t))du
dt 


, , , , :

f ( x)dx f (t)dt  f (u)du 

 ) ( f ( x)D g ( x ))dx f ( x )dx g ( x )dx 

)
)) f ( x )dx 
)) f ( x )dx f ( t)dt  f (u)du   )
f(tf)(dtx) dx 

f ( x )dx g ( x )dx )


) ( f ( x ) D g ( x ))
( f ( x )Dgg((xx))))dx
f((xx))dx
dx
g ( x )dx  
dx

)
dx
f



)
( x )dx
 f ( tf)
(dtx )dx
dx
) f,gff
) f[,
( x )]
dx R,
f ( :
x )dx 

 )
)) ff((xx))dx
dx  f ( tx))dtdx f (fu()t)du
dt   )
))

11.





 )
)
)
12.
 )
)
 )
)


))
))dx
dx
ff((xx))dx
D gg((xx))
dx gg((xx))dx
dx )
( f ( x) 

f ( x ) dx 


f ( x )dx

))
ff ((xx))dx

dx f ( x )dx
(

g
(
x
))
f ( x )dx  g ( x )dx  )

f ( x )dx 
 g ( x))dx
f (x)dx
(ff ( x) dx
g ( x)dx 

(ff ( x)dx g ( x)) dx f (x)dxf ( x)dx  g ( x)dx 

 )
(
f
(
x
)

g
(
x
))
dx
f
(
x
)
dx

g
(
x
)
dx


)
(
f
(
x
)

g
(
x
))
dx
f
(
x
)
dx


) f ( x )dx  f (x)dx 

g ( x)dx 

f,

 x=, x= xx


 )
( x ))
E() ( f ( xf) (x )gdx
 dx f ( x )dx  g ( x )dx 
13.

) f(x)0 x [, ]. 
) () f.

) f(x)0 x [, ].
) f(x)0 x[, ].
1 2
2
) V x dx  x 2 dx 
0
1
Vf y
1 2
2 2
1 2 6.8)
2 2

(.
1
) V x dx  x dx
x dx x dx 
) )
V V x20dx
0
1
1

xx
:
0
14.)
)) V

11

) V 2 (x x 2xdx
x 22dx
2 
dx   x 2 dx
)  )V
00
1
0 )dx 

)
)) V

21

(xx2 dx
 x2 )dx 
00

) )) V

2
2  2 x 2 dx 
(
x

x
dx V f
1 ))
V0f

2
6)
)
V
V  x 2 dx 
)
V
xdx 

1 2
6


2 2
4 2
6 xdx
)
V

)
V

x
dx

) xx
V f (. 6.8)
xdx

V ff
x dx 
)
:
1
2
22 2
6 4
4 4 2
)
)2 V
) V f V f xdx
 )Vx 2 dx
15. )
f
)

xdx  x dx 
2
f
2 2 2
2 2
4 6
44 2
x )
dx V f) V
) )
)
2 f
xdxx 2dx  
) VVf f 2 2 2 xdx
2
2
2
6 2
4
6 2
)
V
x dx 
) V f) 2 Vxdx


x
dx

f
2
2 f
2

 ( x  xx22dx
)dx 
1

. 6.8.
y
B

y=x

4
2

A
2

. 6.8.

6 2

2 x dx 
6.9 .
)

Vf

6.9.1.
 , (
) A( t) = 0, 09 t .
480 ,
.
.

342

6.9.2. F ( x )

x t
e 2 tdt
0

 G( x )

) F(x)+G(x)=ex1, F ( x )  G( x )
) F(x) G(x).
)


6.9.3.

/2

e t 2 tdt,

x t

0 e

tdt , xR.

1 x
[ e ( 2 x  2 x )  1] .
5
/2
/3

e t 2 tdt .

(u + 1)(u + 3) du . -

:
x
dx 
)
x(x  2)

xe x

(e x 2  1)(e x 2  2) dx 

( x 2  2)( x 2  4) dx 

6.9.4. f(x)=x410x2+2 g(x)=3x212x+2.


) f(x)=g(x) 4 ,0, 1 3.
) h(x)=f(x)g(x).
) N
f g.
6.9.5. f(x)=x2x2.
a
) f ( x )dx .
1
) ,

1 f ( x)dx  .

6.9.6.
 ' xx xx x ,
F(x) (. 6.9).
x=a x = (
[, ])
Wf

F( x)dx.

)
 x cm
, k F(x) = x,
. 2Newton
1 cm , 5 cm.
)  x cm F(x) = /x2, . ,

. ,
3 cm
1 cm.

F
x

. 6.9


.
, .
. ,
. ,
. ,
.

7.1 .

7.2
 .
7.3 .

7.4  .
Bernoulli.
7.5  .
7.6 .
7.7 .

7.8 .

lx im

7.9 .

x0

344

7.1 .
, , ,
.
, ,
.
.
m, F
(. 7.1).

d 2 x ( t)
+ 2 x ( t) = 0
2
dt

(7.1.1)

F
x(t)

x(t)+2x(t)=0, x(t)
t >0 .

M
m

. 7.1

dx( t)
= kt
dt

(7.1.2)

x(t)+kt=0 x(t) t k .
(7.1.1), (7.1.2), x(t)
t
x ( t) =

dx( t)
d 2 x ( t)

x ( t) =
.
dt
dt 2

, y=y(x) y=y(x)
x :
y =

dy
dy( x )
, ( y( x ) =
)
dx
dx

x :
d 2 y( x )
d2 y

) ...,
y = 2 ,( y ( x ) =
dx 2
dx
, , .
x(t)
, t.
.
,
.
:

u u
+
= 0,
x y

u=u(x,y)

(7.1.3)

u u

,
x y

345

u=u(x,y) x y .
.

dy
+ 2 xy = e x ,
dx

dy
+ xy = 0,
dx

d2 y
dy
5 + 6 y = 0,
2
dx
dx

w w w
+
+
= 0,
x y z

y=y(x) w=w(x,y,z).
.
x, y=y(x) y, y,...,y(n1), y (n) y x.
:


F x , y, yc, ycc,!, y( n1) , y( n)

0,

n t 1,

dy d 2 y
d ( n1) y d ( n) y
F x , y, , 2 ,, ( n1) , n = 0,
dx dx
dx
dx

n 1,

y( x ) ,

(7.1.4)

y = y( x ).

(7.1.5)

F .
, .

.
,
d2 y
dy
+ 3x
+ x2 y + 3x 1 = 0
2
dx
dx
, y3y+y+1=0
....
. ,
.
y=e2x
y5y+6y=0, y=e2x :
y 5 y + 6 y = (e 2 x ) 5(e 2 x ) + 6 e 2 x = 4e 2 x 10e 2 x + 6 e 2 x = 0 .
y=x2 y2x=0.
y=x2 :
y 2 x = ( x 2 ) 2 x = 2 x 2 x = 0 .

346

:
y: R (7.1.4) :
) y n-
) y (7.1.4) x A, :

F x , y( x ), yc( x ), ycc( x ),!, y( n1) ( x ), y( n) ( x )

0,  x A.

A y: R (, ), [, ],
(, ], [, ) , R (a,+), [a,+), (,), [,), (, +).
: ;
y=x1/2, x(0,+), :
dy
= x 1/ 2 .
dx
y( x ) = 2 x + c , cR
c , :
dy
1
= (2 x + c) = 2
= x 1/ 2.
dx
2 x
,
cR.
(7.1.6)
y( x ) = 2 x + c , c R

. (7.1.4)
n1 .

( ). ,
( ) ....
(7.1.6) cR, .
c=1 :
y( x ) = 2 x + 1 .
, y(1)=2, (7.1.6) :
y(1) = 2 2 = 2 1 + c c = 0 .
,
y( x ) = 2 x .
:

y = (x, c1, ..., cn)

dy
= x 1/ 2 y(1)=2
dx

(7.1.7)

n c1, ..., cn (7.1.4), :

347

)  (c1, ..., cn) Rn (7.1.7)


, , (7.1.4)
)  (xo, yo, y1, ..., yn1)
F, (c1, ..., cn) , y y(xo) = yo,..., y(n1) (xo) = yn1.
,
, .

7.1.1

y+4y=0.
) .
) y=2x+2x .
)  y=c12x+c22x c1, c2R.
)

y(0)=1 y = 1 .
4

) ( y), 2.
) y=2x+2x x :

yc 2 2 x  2 2 x , ycc 4 2 x  4 2 x

4( 2 x  4 2 x ) 4 y. 

A .
) y=c12x+c22x x :
y=4c12x4c22x.
y y :
y + 4 y = 4c1 2 x 4c2 2 x + 4(c1 2 x + c2 2 x ) =
= 4c1 2 x 4c2 2 x + 4c1 2 x + 4c2 2 x = 0.
, y=c12x+c22x
c1, c2R.
) y(0)=1 y=c12x+c22x :

y(0) 1 c1 0  c2 0 1 c1 1  c2 0 1 c1 1. 

y = 1 c1, =1,
4
:
2
2

y = 1 c1 + c2 = 1 1
+ c2
= 1 c2 = 2 1.
2
2
4
4
4

348

, y = 2 x + ( 2 1) 2 x .
7.1.2.

y=xx xy=y+x2+y2.
.

y=xx ={xR:x0}.
y=xx x :
x
yc ( xx )c x cx  x(x )c x 
,
2 x
:
x
x2

.
xy = x x +
=
x

x
+

2 x
2 x

y=xx :
x2
y + x + y = xx + x + x x = xx + x (1 + x ) = xx +
.
2 x
2

y=xx
.
7.1.3.

, P(t)
t. q
(
).
)  P=P(t) P(t)=ec+qt, cR
.
)  P(t) t=10 sec
P(0).
) A t=1 sec 100,
.
.

dP
) P(t) P ( t) =
. ,
dt
dP
,
= qP , q .
dt
c+qt
P(t)=e , cR, , :

349

dP
= (e c + qt ) = e c (e qt ) = e c q e qt = qe c + qt = qP .
dt
) ()
P ( t) = e c + qt = e c e qt = P0 e qt ,
P0=ec.
t=0, :
P (0) = P0 e q0 P (0) = P0 .
t=10sec P (10) = P0 e q10 P (10) = P0 e10 q ,
P0=ec .
) P(1)=100, :
P (1) = 100 P0 e q = 100 P0 =

100
= 100e q .
q
e

7.1.1. :
) y+2xy=ex
)

d2 y
dy
5 + 6y = 0
2
dx
dx

) y(4)+4y+3y=x
)

d 4 y d 3 y d 2 y dy
+
+
+ + y =1
dx 4 dx 3 dx 2 dx

7.1.2.  .
) y=x2+c, y=2x
) y=cx2, xy=2y
) y2=e2x+c, yy=e2x
y2
1
2
y/x
=
) y=e3x+ex, y4y+3y=0,
) y =
,
y=2xy

)
y=ce
,
y
xy x 2
1 x2
2
2
2
2
) y =x cx, 2xyy=x +y
7.1.3. y=2xy, y>0.
) .
2
) y = 4e x .
2
)  y = c ex , cR .
) y(2) =1.
7.1.4.  y=c1exx+c2ex x
y+2y+2y=0. y(0) = 0
y'() = 0.
7.1.5.  ' xx
(t) t. S(t) , S(t)= (t).
) t.
)
 t=7sec,
(. t=0).
) , S(1)=10.

350

7.2  .
, , ..
y = f(x),
y' = f'(x) x. (7.1.4)
n=1, :
dy

(7.2.1)
F ( x , y, yc) 0, F x , y, . 0 
dx


, , 7.1, , .
, (7.2.1)
dy
,
dy
dx
= f ( x , y),
(7.2.2)
dx

f(x,y) .

y( x ) =

2y
,
x

x 0 .

(7.2.3)

y=cx2
(7.2.4)

y y', (7.2.3),
cR. (7.2.3) :
y=(cx2)=2cx,

y (7.2.3)
2 y 2c x 2
=
= 2c x .
x
x
,
y x .

y( x ) =

1 y2
.
xy

(7.2.5)

y,
xy=lny+c,
(7.2.6)

(7.2.5) cR,
y x. , x,
:
1
1 y2
1 y2
.
xy + y = xy =
y =
y
y
xy
y x. (7.2.6)
.

351

,
. :

dy
= f ( x ) g ((y),
x ),

dx
x y.

dy
P ( x ) M ( y) + Q( x ) N ( y) = 0
(7.2.7)
dx

.
N(y)0, P(x)0, (7.2.7) :
M ( y) dy
Q( x )
M ( y)
Q( x )
=

dy =
dx ,
N ( y) dx
P( x)
N ( y)
P( x)
x y (
). ,
, :
M ( y)

Q( x )

N ( y) dy = P( x) dx + c ,

cR.

(7.2.8)

y = N(y)=0, y =
(7.2.7).

7.2.3.

y=x1/2, x(0,+).
.

, :
dy
= x 1/ 2 dy = x 1/ 2 dx .
dx
dy = x

1/ 2

dx y = 2 x + c, c R.

7.2.4.

e y
.

dy
( x + x 3 ) = 0.
dx

:
e y dy = ( x + x 3 ) dx.

352

:
x2 x4
x2 x4
+
+ c y = ln( +
+ c) , cR.
2
4
2
4
x2 x4
y( x ) = ln( +
+ c) , cR.
2
4
y
y
3
e dy = ( x + x ) dx e =

7.2.5.

.
dy
dx

yc( x )

k(a  y)(  y),  y(0)=0,



, , k >>0, k>0.

(7.2.9)

:
dy
(a  y)(  y)

kdx 

:
dy

(a  y)(  y) k dx  c.

(7.2.10)


, :
1
1
1
1
1
(7.2.11)


(a  y)(  y )  a a  y a   y

(7.2.11) (7.2.10) :
1
a

(a y) dy + a ( y) dy = kx + c

1
1
ln a  y 
ln  y
a
a

kx  c1 , 

ya
e kx ( a ) e c ( a ) .
y
y, (7.2.9) :

a  c e( a ) kx
,
1  c e( a ) kx

(7.2.12)

c=ec1(a). (ay)(y)=0, y=a, y= [
(7.2.9)], .
, y(0)=0, (7.2.12) :
a  c e( a ) k 0
a
y(0) 0 0
c

( a  ) k 0

1 ce
y( x )

353

c (7.2.12) y( x )

a [1  e( a ) kx ]
.
 ae( a ) kx

7.2.6.

DVD player home cinema


y = y(x), x . y = y(x)
player.
) y = y(x).
) .
)
 home cinema Hi-End
DVD, 2599 , .
.

) DVD player , dy
= y( x ) .
y=y(x)
dx
)
dy
= dx.
y
:

dy
= dx ln y = x + c e ln y = e x + c e ln y = e c e x y = c e x ,
y

ec=c. y(x)=cex, cR.


) home cinema DVD player,
2599 , ' y(0)=2599.
, :
y(0) = 2599 2599 = c e 0 c = 2599.

y(x)=2599ex , player,
(x=1) :
y(1) = 2599 e 1 = 2599 / e 956 .

7.2.1. :
) y+6xy=0, y()=5

) eyy=(x+x3), y(0)=1

) y=logx, y(e)=0

7.2.2.

.
) y=xex, y(1)=3

) y=logx, y(e)=0

) y=2sinxcosx, y(0)=1

) y=2x2, y(0)=1

354

7.2.3. :
y
x +1
,
) y = 3
) y = 2 ,
x
y +3

) y=2x
) y=x3y2

) y =

x2y y
,
y +1

) xy=2y

7.2.4.

.
) y=xex, y(1)=3
) y=logx, y(e)=0
) y=2sinxcosx, y(0)=1

) y=2x2, y(0)=1

x y (4 y)
, y(0)=y0>0.
1+ x
) y=y(x) .
) .
) lim y( x ) .

7.2.5. y( x ) =

x +

7.2.6.
 o ,
t, y=y(t)
. 7 , .

7.3 .
,
.
f(x,y)=x2+xy, (x,y)IR2 f(x, y) 0. , :

f ( x , y) ( x )2  ( x )( y)

2 x 2  2 x y

2 ( x 2  x y)

2 f ( x , y) .

f ( x , y) 2 f ( x , y), z 0 . .
:
f(x, y) m 0

f ( x , y) m f ( x , y). 

(7.3.1)
x 2 + y 2 sin(x/y) 1 0 .
:
,
P ( x , y) + Q( x , y)

dy
= 0,
dx

(7.3.2)

P(x,y), Q(x,y)
x y.

355

z=y/x z=x/y.
(7.3.2)
. .

7.3.1.


x2y2+2xyy=0

(7.3.3)

.
.

(7.3.3) :
x 2 y 2 + 2 xy

dy
=0
dx

(7.3.4)

(7.3.2), P(x,y)=x2y2 Q(x,y)=2xy. (7.3.1), P(x,y) Q(x,y)
. , :

P ( x , y)

2 ( x 2  y 2 )

2 P ( x , y) Q( x , y)

2 Q( x , y).

P(x,y) Q(x,y) ,
z=y/x :
z=

y
dy y
dz
dy
dz dz
dy
dz
y = z x z == z
x +y =
x zx
= z +x x+ x .
=z+x
x
dx x
dx
dx
dx dx
dx
dx

(7.3.5)

(7.3.5) (7.3.4) :
x2 z2 x2 + 2 x z x ( x

dz
dz
+ z ) = 0 x 2 (1 z 2 ) + 2 x 2 z x + 2 x 2 z 2 = 0
dx
dx
x 2 (1 + z 2 ) dx + 2 x 3 z dz = 0 (1 + z 2 ) dx + 2 x z dz = 0,

.
2 x zdz = (1 + z 2 ) dx

, :
z

1 1

z
1
dz =
dx ,
2
2x
1+ z

1 + z 2 dz = 2 x dx log(1 + z

(7.3.6)

) = log x + c

c
log(1 + z 2 ) = log x + log c (c = log c) 1 + z 2 = , c R .
x

, z=y/x, (7.3.3.) x2+y2=cx, c R , y=y(x). , :

356

c c c
x 2 x + + y2 = 0 ,
2 2 2

(x_ 2c) +y =(2c) , cR


2

2 c
x + y = , cR .
2

2
x


K(c/2,0) c/2, c R (.
7.3).

. 7.3

7.3.2.

dy
x + y x = 0 dx
.
.

, (7.3.2)
P(x,y)=xy, Q(x,y)=x

P ( x , y) ( x )  ( y)

( x  y)

P ( x , y) , Q( x , y) ( x )

( x )

Q( x , y) .

P(x,y) Q(x,y)
, z=y/x y=zx,
(7.3.5). , :
dy
dz
dz
1

x + z x x z + x = 0 x + zx zx x 2
= 0 x = x2
dz = dx .
dx
dx
dx
x


:
1

dz = x dx z = ln x + c z = ln x + ln c z = ln c x ,

cR

c=lnc. z=y/x, y=x lncx, c R.

7.3.1.  , ,
.
x
2
y

3
2
2
y
) h( x , y) = x + y sin ) k( x , y) = xy + x
) f ( x , y) = y + xy ) g ( x , y) = x + y x e
x
7.3.2. .
y2

) y =
x

x
y

x
) 2 xy e x 2 + y 2 sin y = 0
y

) y =

x2 + y
x3

357

7.3.3. 
.
dy 1 2 y
x+ y
= +
) ( x 2 2 y 2 )dx + xydy = 0 )

) xy = y + 2 xe y / x
) y =
dx 3 3 x
xy
7.3.4. .
) y =

x 2 + y2
, y(1)=2
xy

) 2 x y y + x 2 3 y 2 = 0 , y(2)=1

) y = 1 +

y
x + xy

, y(2)=3

7.3.5.

.
2
2
) 2 xyy = x + 2 y

) x 2 y 3 xy 2 y 2 = 0

7.3.6.  :

dy
dx

) ( x + y) dx ( x y) dy = 0

ax  y 
F
, 
x  y 

. , x=zh, y=wk,
h,k,
z, w.
:
)

dy x + y + 4
=

dx x y 6

) y =

x + y +1
2x + 4 y + 3

7.4 . Bernoulli.
.
,
y=y(x). , y=3y+6

(7.4.1)

y3y=6

. (7.4.1)
Q( x ) = e

3 dx

= e 3 x ,

( ) :
d
yc e 3 x  3 y e 3 x 6 e 3 x
y e 3 x 6 e 3 x .
dx
x,
:
d
3 x
3 x
3 x
3 x
dx y e dx 6e dx y e 2 e  c, c R. 
d
3 x
3 x
3 x
3 x
dx y e dx 6e dx y e 2 e  c, c R. 
,
. :

358


y(x) +p(x)y= f(x),
(7.4.2)

p(x), f(x) ' R. f(x)=0, .
,
y( x ) + p( x ) y + q( x ) y = r( x ) ,
p(x), q(x) r(x) x.

(7.4.2). ,
Euler,
Q( x ) = e

p( x ) dx

(7.4.3)

x (. y). (7.4.2)
:

p( x ) dx
p( x ) dx
p( x ) dx
p( x ) dx
p( x ) dx
y e
+ p( x ) e
y = f ( x) e
y e
= f ( x) e

y e p( x ) dx dx = f ( x ) e p( x ) dx dx + c y e p( x ) dx = f ( x ) e p( x ) dx dx + c .

:
H (7.4.2) :
p( x ) dx
p( x ) dx
y( x ) = e
f ( x) e
dx + c , c R .

(7.4.4)

:
) A f(x)=0 ( ) y(x) + p(x)y=0,
:
p( x ) dx
y( x ) = c e
.
) p(x), f(x) , (7.4.2) 7.2.
) p(x)=0, y=f(x), , :
y = f ( x ) ydy = f ( x ) dx y = f ( x ) dx + c, c R.
7.4.1.

y+3y=e2x.

359

, (7.4.2) p(x)=3
f(x)=e2x. (7.4.4) :
3 dx
3 dx
y( x ) = e 3 x e5 x dx + c .
y( x ) = e e 2 x e dx + c y( x ) = e 3 x e 2 x e3 x dx + c

, :
1
1

y( x ) = e 3 x e5 x + c y( x ) = e 2 x + c e 3 x , c R.
5
5

7.4.2.

dy 1
+ y = 3 x , x0.
x0
dx x

p(x)=1/x f(x)=3x :
1
p( x ) dx
x dx

Q( x ) = e
=e
= e ln x =x.
, (7.4.4) :
1
1
y( x ) = e ln x 3 x e ln x dx + c y( x ) = 3 x x dx + c y( x ) = 3 x 2 dx + c ,

x
x
:

c
y( x ) = x 2 + ,
x

cR .

1
y = xy 2 , x
(7.4.2) y2 . (
) Bernoulli.
:
y +

y( x ) + p( x ) y = f ( x ) y m , m 0,1

(7.4.5)

p(x), f(x) x,
Bernoulli.
m=0, Bernoulli , m=1
. m ( m0,1)
z=ylm [
, z(x)]. .

360

7.4.5.

:
y +

3
y = x 2 y2 , x > 0
x

(7.4.6)

.
(7.4.6) Bernoulli
3
m=2 p( x ) = , f ( x ) = x 2 .
x
y0,
z = y1 2 z = y 1,
z=y2y

3 1
3
y = x 2 z z = x 2 .
x
x
. :
y 2 y

y( x ) =

1
, c ln x 0, x > 0
x (c ln x )
3

cR . y(x)=0, x>0
(7.4.6), .

7.4.1. y3y=6.
) .
) .
7.4.2. () .
3
4
4
3
) y y = x
) y + y = sin x
) y + y = x
x
x
7.4.3. .
3
) y + 2 xy = 2 x ,

) yc  y sin x ,

y(0) = 1

y( ) 1 

) y xy = x

) y + xy = x , y(0) = 4
2
y = 4, y(2) = 100
) y +
10 + 2 y

7.4.4. y+yexy2=0.

6
7.4.5. y + y = 3 y 3 ,
x

y(1) =

1
.
8

361

7.5 .

. :
n

n ( x ) y( n) ( x ) + n1( x ) y( n1) ( x ) + ... + 1( x ) y( x ) + 0 ( x ) y( x ) = f ( x ),

(7.5.1)

f(x) i (x), i=0,1,2,...,n


x.
f(x)=0, (7.5.1) , f(x)0
.
i, i = 0, 1, 2,..., ,

n y( n) ( x ) + n1 y( n1) ( x ) + ... + 2 y( x ) + 1 y ( x ) + 0 y( x ) = f ( x )
.
2

y( x ) + p y( x ) + q y( x ) = 0
(7.5.2)

p, q . ,
.
(7.5.2) y(x)=ex. :

yc( x )

e x ycc( x )

2 e x ,

(7.5.2), 2 e x + p e x + q e x = 0 e x ( 2 + p + q) = 0,
2 + p + q = 0 .
(7.5.3)


(7.5.2). 0 (7.5.3), y(x) = e0x
(7.5.2). , ,
y+3y4y=0 2+34=0 1=1 2 = 4,
y1(x) = e1x = ex y2 (x) = e2x = e4x .
, n

y( n) ( x )  n1 y( n1) ( x )  ...  2 ycc( x )  1 yc( x )  0 y( x ) 0 ,


n + n1 n1 + ... + 1 + 0 = 0 .

.. y( 4) 3 y + 2 y y = 0 4 3 3 + 2 2 1 ==00,

d5 x
d3 x
dx

+5
7 x = 0 5 3 3 + 5 7 = 0 ....
3
5
3
dt
dt
dt

362


(7.5.1). ,
.
, ,
.
:
(7.5.2) (7.5.3).
=p24q (7.5.3) 1,2 .
yo
(7.5.2) :
)
 >0, 12 ( (7.5.3)
), e1 x e2 x (7.5.2) :

y = c1 e1x + c2 e2 x
(7.5.4)
)  = 0, 1 = 2 = ( (7.5.3)
), ex x ex (7.5.2)
:
y = c1 e x + c2 x e x = (c1 + c2 x ) e x .

(7.5.5)
)  < 0, (7.5.3)
, 1 = a + i, 2 = a i , e( a i ) x  e( a i ) x


(7.5.2) :
y

d1 e( a i ) x  d2 e( a i ) x ,

y e ax (c1 x  c2 x ). 

(7.5.6)


.
7.5.1.

y + 3 y + 2 y = 0 .
.

2 + 3 + 2 = 0 .
, 1 = 2, 2 = 1 , (7.5.4),
y = c1 e 2 x + c2 e x .
7.5.2.

) y4y+4y=0.

363

) y(0)=y'(0)=1.
.

) 24+4=0 1= 2==2,
yo=c1e2x+ c2xe2x, c1,c2R.
) , , :
= 2c1 e 2 x + 2c2 x e 2 x + c2 e 2 x = e 2 x (2c1 + 2 xc2 + c2 ) .
y
y(0)=y(0)=1 :
(0) = 1 2c1 + c2 = 1 .
y (0) = 1 c1 + c2 = 1 , y
, c1=0, c2=1 y=xe2x.
7.5.3.

y+4y+5y=0.
.

2+4+5=0, = p24q=4245=4<0
, 1 = 2 + i, 2 = 2 i .
(7.5.6), y e 2 x (c1 x  c2 x ) .
(7.5.6), c1 = A ,
c2 = A (7.5.6) o o
y e ax ( A x  A x )

Ae ax ( x  x )

Ae ax ( x  ) 

( ( + ) = + ).
x , y Ae x ( x  )  ,
> 0 , < 0 .
,
7.1 .
.

7.5.4.

m,
F
(. 7.5).
.

F
x(t)

M
m

. 7.5

F=ma, a
F. , Hooke,

364

F=kx, k>0 , x M
O ( F x
). a=a(t)
x=x(t) ,
d2 x
a= 2 .
dt
:
k x

d2 x
d2 x k

 x
dt 2
dt 2 m

d2 x
 2 x 0 x cc  2 x
dt 2

0 ,

k
k
=
=().
2+2=0,
m
m
2 + 2 = 0 2 = 2 = i , , 1 = 0 + i, 2 = 0 i .
, (7.5.6), x e 0 x (c1 t  c2 t). 
c1 = A , c2 = A , x(t) = A(t+), 0.

O, .

2
=

,
, :
y( x ) + p y( x ) + q y( x ) = f ( x ),

p, q f(x) .
,

(7.5.7)

y 2 y + y = e ,
(7.5.8)

p=2, q=1 f(x)=e3x. (7.5.8),
y2y+y=0, 22+1=0 ( ,
=1),
3x

y = c1 e x + c2 x e x , c1 , c2 R.
(7.5.8)


(7.5.8)

1
y = e3 x ,
4
1
3
9
y = e3 x , y = e3 x y = e3 x
4
4
4

9
3
1
y 2 y + y = e3 x e3 x 2 e3 x + e3 x = e3 x e3 x = e3 x ,
4
4
4
.
y2y+y=0 ,

365

1
y = y + y = c1 e x + c2 x e x + e3 x ,
4
(7.5.8).
, :
y
(7.5.7) :
y=yo+ y,
yo
y( x ) + p y( x ) + q y( x ) = 0
y (7.5.7) (
).
, (7.5.7),
. , :
y;
, . ,
.

7.5.5.


(7.5.9)

y5y+6y=x1,

y5y+6y=0, 25+6=0 1=2, 2=3.


yo =c1e2x+ c2e3x.
(7.5.9). f(x)=x1 y = x + . y' = , y'' = 0
(7.5.9) :
0 5 + 6 x + 6 = x 1 6x + (6 5) = x 1.
x 6=1 65=1,
:
1
1
= , = .
6
36
1
1
x
. ,
6
36
(7.5.9)
(7.5.9) y =

y = y + y = c1 e 2 x + c2 e3 x +

1
1
x
.
6
36

366

(7.5.7)
f , y=P(x),
P(x) f.
(7.5.7)
f . f :
y (7.5.7) :
) f(x) Pn(x) n x. :
q0,
y

an x n  an1 x n1  ...  a1 x  a0 

q=0,
y

x (an x n  an1 x n1  ...  a1 x  a0 ), 

ai ,i=0,1,2,3,...,n .
)  f(x)=kex, k, .
y=ex,
.
)  f(x)=k1x+k2x, k1,k2 .
y=x+Bx, ,
.
(7.5.7)
(), () (). , f
() (), f(x)=exPn(x),
y

e x (an x n  an1 x n1  ...  a1 x  a0 ) 

ai, i = 0, 1, 2, 3,..., . f (), () () (, ), :

f ( x ) e x Pn ( x )x

f ( x ) e x Pn ( x )x , 

e x x (an x n  an1 x n1  ...  a1 x  a0 )  e x x ( n x n  n1 x n1  ...  1 x  0 )

e x x (an x n  an1 x n1  ...  a1 x  a0 )  e x x ( n x n  n1 x n1  ...  1 x  0 ) 

ai,i, i = 0, 1, 2, 3,..., n .

f
.
.

367

7.5.6.

yyy=4xe2x.
.

f () (), , f(x)=exP1(x),
P1(x)=4x =2.
y=e2x (x+), A,B .
y = e 2 x (2 Ax + A + 2 B),

y = e 2 x (4 Ax + 4 A + 4 B)

:
e 2 x (4 Ax + 4 A + 4 B) e 2 x (2 Ax + A + 2 B) e 2 x ( Ax + B) = 4 xe 2 x
e 2 x ( Ax + 3 A + B) = e 2 x 4 x Ax + 3 A + B = 4 x + 0 .
A=4 3+=0=12
y=e2x (4x12).
yyy=0,
21=0. 1 =

1 5

c1 e

1 5

 c2 e

1+ 5
1 5
,
, 2 =
5
5

.


1 5

y  y

c1 e

1 5

 c2 e

 e 2 x (4 x  12) .

7.5.7.

yy2y=2x.
.

, , yo=c1ex+c2e2x.
f(x)=2x y=2x+B2x.
:
y = 2 A 2 x 2 B 2 x ,

y = 4 A 2 x 2 B 2 x

(4 A 2 x  4 B 2 x )  (2 A 2 x  2 B 2 x )  2( A 2 x  B 2 x ) 2 x 

( 6 A  2 B) 2 x  (6 B  2 A) 2 x 1 2 x  0 2 x .
6A+2B=1 2A6B=0,
:
3
1
B =
A=
.
20
20

368

y =

3
1
2 x + 2 x :
20
20
3
1
y = y + y = c1 e x + c2 e 2 x 2 x + 2 x .
20
20

7.5.1.  .
) 2xy+x2y(x)y=2

) yy2y=0

) y2y=0

) yy+xy+y=x3

) 4y+4y+y=0

) 2y+4xy=x+1

;
7.5.2.  f(x)=ex g(x)=5ex
y2y+y=0. y=c1ex+c25ex, c1,c2R
.
7.5.3. :
) y6y=0
) y7y=0
) y+2y+5y=0

) y2yy=0

) y+10y+21y=0

7.5.4.

) y3y+4y=0

d2 I
dI
+ 20 + 200 I = 0 .
2
dt
dt

7.5.5.  f(x)=x2+4x+6
y2y+y=x2, .
7.5.6.  y= d1e(+i)x + d2e(i)x (7.5.2)
, y=eax (c1 x + c2 x).
7.5.7. .
) y + y = 0, y(2) = y(2) = 1

) y y 2 y = 0, y(0) = 2, y(0) = 1

) y 5 y + 4 y = 0, y(0) = 5, y(0) = 8

) y + 4 y = 0, y(0) = 0, y(0) = 2

7.5.8. :
) y y 2 y = e3 x
) y 2 y = 4 x 2 e x
2

) y y 2 y = 4 x 2

) y 3 y + 2 y = 2x

) y 7 y + y = e 2 x (3 x 5)

) y 3 y + 2 y = 8 x 1

7.5.9. .
) y 3 y + 2 y = 2x , y(1)=0, y(1)=1

x
) y + 8 y + 15 y = e ( x + 3) , y(0)=y(0)=0

) y + y = x 3 x + 1 , y(2)=3, y(2)=0

) y 2 y + y =

ex
, y(1)=0, y(1)=1
x

369

7.6 .
, , , , . -,
.
, .

7.6.1 .

. , ' ,
.


dR
= k R,
(7.6.1)
dt

k<0. (7.6.1) o

dR
dR
= k dt
= k dt ln R = k t + c R( t) = c e kt , ,
(7.6.2)
R
R

c=ec . (.
t=0 ), c (7.6.2)
R(0) = ce k0 c = R(0) .
, c R(0). k .

7.6.1.

14, ,
. , -14
(
). -14
-14, 5730 .
77,7%
.
.

, (7.6.1) R(t)=cekt, c=R(0) . , -14 5730


, R(5730)=0,5R(0) :
ln 0, 5
0, 5 R(0) = R(0) e5730 k ln 0, 5 = 5730 k k =
.
5730

370

77,7%
, t R(t)=0,777R(0). :
ln 0, 777
ln 0, 777
t=
2086.
0, 777 R(0) = R(0) e k t 0, 777 = e k t t =
ln 0, 5
k
5730
2086 .

7.6.2 .
, ,

. ,
.
.

7.6.2.

' 2 c ' 200 c. 30 min


16 c. 88 c,
.
.

=(t) ,
d

. , dt

d
(200  ) ,
dt
( ).
:
d
dt

(200  )

d
200 

dt

d
200 

dt  ln(200  )

t  c. 

:
t

1
( ln(200  )  c). 

(7.6.3)

t=0 2oC, t=30


1
16oC, (7.6.3) 0 = ( ln(200 2) c) c = ln198

1
ln198  ln184
30
( ln(200  16)  c) 30  c  ln184
.
30

371

c, (7.6.3)

ln198 ln112
t = 30
233 min .
ln198 ln184

7.6.3 .

. ,
.

.

7.6.3.

)

3000 , 3 .
. .
) ;
; .
.

) x=x(t) t. ,
dx
1
= 4 x (3000 x ),
(7.6.4)
dt 10

. (7.6.4) (
)
3000
x ( t) =
.
1 + c e 0,3 t
x(0)=3, c, :
3000
x(0) = 3
= 3 c = 999 .
1 + c e0

3000
x ( t) =

1 + 999 e 0,3 t

(7.6.5)


(7.6.5)
t = 2, :
3000
x(2) =
x(2) 5 .
1 + 999 e 0,6 t
) , 3000 ,
x(t)=3000,
3000
3000 =
e 0,3 t = 0 .
0,3 t
1 + 999 e

372

t. t+,
. , , .

7.6.1.  ,
:
dN
k N, 
dt
 N = N(t) k . 50 mg , 10%
, :
) M t.
) T 4 .
) T 50% .
7.6.2.  .
100 mg,
5% . :
) M t.
) T 10% .
7.6.3.  . 300 , 2
10%. :
) T t.
) o .
7.6.4.  0o F , 100o F.
10 min 25o F. :
) T 50o F.
) T 20 min.
7.6.5.
 50F 150o F.
10 min 75o F, ,
100o F.
7.6.6.  50 F 150 F.
10 min 75 F, ,
100 F.
7.6.7.  190 F, 72 F. 150 F.
:
) T 5 min.
) T 100 F.
7.6.8. ' 500 5

373

.
, N(t)

,

dN
= k N (500 N ) , k .
dt

7.7 .

n.

x, y=y(x)
y x, :

F x , y, yc, ycc,!, y( n1) , y( n)

0, n t 1 

' .

,
, .
dy
+ Q( x ) N ( y) = 0
dx

P ( x ) M ( y)

f(x,y),
m.

f(x,y)=mf(x,y) 0

y(x)+p(x)y=f(x),
p(x), f(x) .
P ( x , y) + Q( x , y)

dy
= 0,
dx

. P(x,y) Q(x,y)
x y.
Euler
y(x)+p(x)y=f(x).

y(x)+p(x)y=f(x).
Bernoulli.

Q( x ) = e

p( x ) dx

p( x ) dx
p( x ) dx
y( x ) = e
f ( x) e
dx + c , c R

y( x ) + p( x ) y = f ( x ) y m , m 0,1
p(x), f(x) .

374

n.

n y( n) ( x ) + n1 y( n1) ( x ) + ... + 12yy(x( )x )++0 y( x ) = f ( x )

y(x)+py(x)+qy(x)=0, (p,q )

y(x)+py(x)+qy(x)=0.

2+p+q=0

n.

n + n1 n1 + ... + 1 + 0 = 0
) yo = c1e1x + c1e2x > 0

yo ) yo = (c1 + c2 x) ex = 0

) yo = eax(c1x+ c2x) <0


y(x)+py(x)+qy(x)=0
p, q .

(=p24q 2+p+q=0)

y=y0+y
- y0

y(x)+py(x)+qy(x)=0
y(x)+py(x)+qy(x)=f(x).
y
y(x)+py(x)+qy(x)=f(x)

) y=anxn+...+a1x+a0, q0
y(x)+py(x)+qy(x)=Pn(x)
) y=x(anxn+...+a1x+a0 ), q=0
Pn(x) n x.

y(x)+py(x)+qy(x)=kex.

y(x)+py(x)+qy(x)=k1x+ k2x.

y=ex

y= x+B x

7.8 .
,
, .
1.
2.
3.

H y+3xy+4y=x2, .
dy
= x 2 .
dx
dy
= x 2 .
H
dx
H

375

w w
+
= 0, w = w( x , y) .
x y

4.

5.

y(x)=2ex+xex y+2y+y=0.

6.

y(x)=1 y+2y+y=x.

7.

y = 2x y+4y=0.

8.

y = 2x y+4y=0.

9.

y = 2x + c, cR y+4y=0.

10.

y(x)=2e3x y+y=0, y(3)=2.

11.

1 F(x,y,y)=0.

12.

y = (x, c),
c R.

13.


F(x,y,c)=0.

14.


F(x,y,c1,c2)=0, c1,c2 c R.

15.

H exdxydy=0 .

16.

17.

H f(x,y)=x4+2y4 4.

18.

x4 + 2 y
4.
H f ( x , y) =
xy 3

19.

dy
2 xy
= 2 2 .
dx x y

20.

dy
2 xy
x
= 2 2 z = .
dx x y
y

21.

H y+3y=x .

22.

H y

23.

y(x)=c1ex + c2e2x yy2y=0.

dy x 4 + 2 y 4
.
=
dx
x y3

3
y = x 4 y1/ 3 .
x

376

24.

ypy+qy=0 p24q<0 y=c1ex+c2xex.

25.

y+py+qy=0 p24q<0
y=eax(c1x+c2x).

26.

y+py+qy=0 p24q=0 y= c1ex+c2ex,


c1,c2R.

.
dy
= y 2 :
dx
1
) y =
) y = x
) y=x+1
x

dy

= 2 x y 1 :
dx

1.

2.

( x 2 + c)2
4

c
1
) y = + c
) y = 2
x
x

dy 2
y = x 4 , x > 0 :

dx x
) (x + c)2

3.

) y = x2

) 1 +

x5
x5
x ( x 5 + c)2
( x 5 + c)2
+ c x2 + 2x
+ c x2
) +
)
)
3
3
5
4
4



2
(1+x )y+2xy=1, y(0)=1 :
4
x +1
x +1
) y = 2
) y = x + 3
) y = 2
) y = 2
+c
x
x +1
x +1


) 1 +

4.

3
y = x 2 y 2 , x > 0 :
x
1
2
1
1
) y = 5 (c 2 ln x )
) y = 3 (c 2 ln x )
x
x

Bernoulli y +
5.

6.

) y = x 3 ln x + c

) y =

1
(c ln x )1
3
x


H f(x,y)=x3+xy2ex/y :
) 3

) 2

) 1

H y =
7.

1
2
3
) y = x ln x + 2cx ) y = 2 ln x + c
x

x2 + y 2
:
xy
2
2
3
) y = x ln x + 2cx

y =
8.

) 4

2
2
2
) y = 2 x ln x 2cx

x +y
, y(1)=2 :
xy
2

2
2
) y = 2 x ln x + 4 x

2
2
) y = + 2 x ln x + 4 x

) y 2 + x = 3 x 2 ln x + 4 x 2

2
2
2
2
) y + x = x + x ln x + 3 x

377

y =
9.

) y

x 2 y5
+ x=c
2
5

) y 3

x 2 y5
+ x=c
2
5

) ln( xy)

x +1
:
y4 + 1

x+ y
ln( x y) = c
) e

y =
10.
) y=x+3

x 2 y8
+ x=c
2
8

) y=(x+3)2

1
y2
, x > 0 (1, ) :
3
x
) y=(3+lnx)1

) y=x+(3lnx)1

y6y+25y=0 :
11.

3x
3x
) y = c1 e 4 x + c2 e 4 x

3x
) y e (c1 4x  c2 3x ) 

3x
3x
) y = c1 x e 4 x + c2 x e 4 x

3x
3x
) y = e 4 x + 2c2 e 4 x

y6y+25y=64ex.
:
12.

13.

3x
3x
x
) y = c1 e 4 x + c2 e 4 x + 4e

3x
3x
x
) y = c1 e 5x + c2 e 5x + 4e

) y e 2 x (c1 4 x  c2 e3 x 4 x )  2e  x 

) y e3 x (c1 4 x  c2 e3 x 4 x )  2e  x 

y=9x2+2x1 :
2
2
) y = + 1
) y = + 1
x
x

) y =

14.

x 2 x3 3x 4
+ +
2
3
4

) y =

yy2y=4x2 :
x
3x
2
) y = y0 + y = c1e + c2 e 2 x + 3 x 3

) y = y0 + y = c1e

2x + 2x 3

Bernoulli y +

15.

x 2 x3 3x 4
+ +
+ c x, c R
2
3
4

) y=t2+c

x
2x
2
) y = y0 + y = c1e + c2 e 2 x + 2 x 3
2x
2
) y = y0 + y = c2 e 2 x + 2 x 3

6
y = 3 y 4 / 3 , t > 0 . H :
t
) y

1
, c t 2  t z 0, t ! 0 
3
( c t  t)

) y

t2
, c t 2  t z 0, t ! 0 
2
3
( c t  t)


) y=(t2+c)1, t2+c0

378

7.9 .
7.9.1.  , :
) y(6)+4y2y+5y8=x

) y3yy+xy=0
n

d2 y
3
) 2 = y + 2
dx

d2 y
) 2
dx

3/ 2

=1+ x y

7.9.2. :
dy
= 2 x y
)
) y'=yx
dx

1

) 2 x

dy
= 1 y2
dx

dy
( x 1) y 5
=
)
dx x 2 (2 y 3 y)

dy
= (64 xy) 3
)
dx

7.9.3. c1 c2, y(x)=c1x+c2x

y 0, y 0 .
4
6

7.9.4.  y+4y=0,
y(0) = 0 y'(0) = 1.
7.9.5. :
) y 2 x y = 0,
) y + y = 2,

y(0) = 1

y(0) = 0

) x y + 2 y = 4 x 2 ,

y(1) = 4

1
y = 0, y(0) = 2
x2

x

y
+
y
=
e
,
y
(
0
)
=
1
)

3
) y 2 y = x , y(0) =
4

) y

7.9.6. :
3 x
) y + 3 y = 2 x e

) y +

3 x
) x y + ( x 2) y = 3 x e

2x 3
y = 4 x3
x

) y5y=ex

7.9.7. :
)

dy
= y ex ,
dx

x+ y
) y = e ,

y(0) = 2 e

y(0) = 1

) yy=x, y(0)=2
y
) ( y + e ) y = 1,

y(1) = 1

7.9.8. :
) y+y=xy3

) y+y=y1

7.9.9. :
dy
= y e x , y(0) = 2 e
)
) yy=x, y(0)=2
dx

2
) x ( x + 1) y y = 2 x ( x + 1)

379

7.9.10.  , ,
.
x2 + 2 y2
yx
) f ( x , y) =

) g ( x , y) =
xy
x
) h( x , y) =

y
x + xy

) k( x , y) =

x 4 + 3x 2 y2 + y4
x3 y

7.9.11. 
.
yx
x + 2y
) y =

) y =
x
x
) y c

y 2  2x

xy

) y(y2x2)=2xy

7.9.12.
y +

1
x
y=
.
x
x

) y=y(x).
) x0, lim y( x ) .
x 0

7.9.13. .
) y+6y'+9y=0
) y + y +

1
y =0
4

) y 20 y + 64 y = 0

) y + y + 2 y = 0

d2 y
dy
5 +7y = 0
)
2
dy
dy

d2 y
dy
18 + 81 y = 0
)
2
dy
dy

7.9.14.
d2 I
4 I = t2et .
2
dt
7.9.15. yy2y=0, y(0)=a, y(0)=2.
) .
)
 a, y=y(t) t+,
lim y( t) = 0 .
t +

7.9.16. y+3y+2y=0, y(0)=1, y(0)=1.


7.9.17. :
) y2y+y=x21

) y2y+y=3e2x

) y2y+y=xex

) y2y+y=3ex

) y2y+y=4x
y y
)
=3
y

7.9.18.  y = p(x) y2 +q(x) y + r(x) = 0, p(x), q(x), r(x) ' , Riccati.


Riccati
y' ex y2 + 3y 3ex = 0.

380

) y1(x) = ex .
1
) y(x) = ex +
:
u( x )
u' u + ex = 0.
) ().
) :
y( x ) = e x +

2
2ce + e x
x

7.9.19.  ,
0F . 20min 40 F 40min
20 F. .
7.9.20. 
dT
+ k T = 0, T = T ( t) ,
dt

k . 100 F,
t = 0 0 F. 20 min
50 F, :
) 20 F.
) 10 min.
7.9.21.
 RC 3002t( Volt), 150 hm 1/600 Farad.
5 Coulomb
)
 q =q(t) t.
)
 t = /2.
7.9.22.
 RL 4t ( Volt), 100
Ohm L=4 Henry.
L
, I=I(t) t.
7.9.23.
 RCL (. 7.9), 200 (100t) Volt, 5 Ohm, L = 0,05 enry
0,0004 Farad, .

, t.

E(t)

. 7.9.

LAPLACE

, ,
. , Laplace.
Laplace
, , .
Laplace,
.

8.1 O Laplace.

8.2 Laplace.
8.3  Laplace.
8.4 .
8.5 .
8.6 .

lx im
x0

382

8.1 Laplace.

6 f ( x ) dx  f
a
' [, ], , R. ,
f , . , :
f

f f ( x) dx

f ( x ) dx

f

f f ( x) dx . 

, . , f:[a,+)R
( G f),
G( x ) = f(x)dx
f ( x ) dx
:

f

f ( x ) dx

f ( x) dx
t of
lim

f

f ( x ) dx

f(x)dx

lim G( t)  G(a) .

t of

f
f

f

f ( x ) dx

t of t

f ( x ) dx

f

lim
a

G(a)  lim G( t) 

f ( x ) dx

f ( x ) dx 

t of

f
a

f ( x ) dx

lim G( t)  lim G( s).

t of

s of

,
. ,
.
.
8.1.1

:
f 4 x
f 1
)
)
e dx 
dx .
0
3
x 1
.
) f:[0,+)R f(x)=e4x , :

f 4 x
e
0

dx

lim

of

4 x
e
0

dx

e 4  1
lim

of
4

1
.
4

1 .
x 1
f [3,+)
1
1
f

dx

lim
dx.
f
(
x
)
dx


3
3 x 1
of 3 x  1
) f:[3,+)R f ( x ) =

383

1
dx
x 1

ln(  1)  ln 2 

lim ln( 1) = + . , .
+

,
,
Laplace f. , :
Laplace f:[0,+)R F

F ( s) L{ f ( x )}

f  sx

(8.1.1)

f ( x ) dx . 

F,
sR, (8.1.1).
s,
(8.1.1), x.
(8.1.1) sR,
f(x) Laplace.
8.1.2

Laplace f(x)=eax, x0.


.

(8.1.1), Laplace f(x)=eax, x0,


f  sx

F ( s) L{ f ( x )}

t ( a s ) x
e
0

f ( x ) dx

f  sx ax

0
t

dx

e( a  s ) x

a  s 0

e dx

f ( a s ) x

dx .

e( a s ) t
1


as as

as<0s>a :

t of
lim

t ( a s ) x
e
0

dx

e( a s ) t
1
lim

t of a  s

as

F ( s) = L{ f ( x )} =

1
,
sa

0

1
as

1
.
sa

s>a .

Laplace , 8.1.1. 8.1.5 -

384

Laplace x
x, Laplace f(x)=1
Laplace f(x)=eax a=0. ,
Laplace f(x)=xn, (. 8.1.6).
Laplace,
.
L1.
 Laplace. L{f1(x)}=F1(s), L{f2(x)}=F2(s)
c1 c2R , Laplace c1f1(x)+ c2f2(x) c1F1(s)+ c2F2(s). :
L {c1 f1( x ) + c2 f2 ( x )} = c1 L { f1( x )} + c2 L{ f2 ( x )} = c1 F1( s) + c2 F2 ( s) .
L2.  Laplace. L{f(x)}=F(s), aR
, :
L {e ax f ( x )} = F ( s a) .

Laplace 8.1.2.
L3.  L{f(x)}=F(s) Laplace f,
n, :
dn
L { x n f ( x )} = (1)n n ( F ( s)) .
ds
L4.  Laplace f. f, , .
, Laplace
f :
L{ f ( x )} = s L{ f ( x )} f (0)
L{ f ( x )} = s 2 L{ f ( x )} s f (0) f (0) .

8.1.1

f(x)

L{ f ( x )} F ( s)

1
,
sa

e ax
xn

(x)
(x)

n!
s n+1

1
,
s

f  s x
e
0

f ( x ) dx 

s>0

s ! a (a ! 0) 

8.1.2

f(x)

s > 0 (n )

, s!0
2
s  2
s
, s!0
2
s  2

L{ f ( x )} F ( s)

f  s x

f ( x ) dx 

e ax (x)

, s! a 
( s  a)2  2

e ax(x)

, s! a 
( s  a)2  2

385

8.1.3

Laplace f(x)=3+2x2.
.

Laplace
L1 :

^ `

L^ f ( x )` L{3}  L{2 x 2 } 3 L{1}  2 L x 2

L^ f ( x )`

3s 2  4
,
s3

1 2
3  2 3
s s

3s 2  4
,
s3

s ! 0. 

s ! 0 .

8.1.4
Laplace f(x)=e3x+223x.
.

E L1 L2 :
6 x  1
L^ f ( x )` L{e 3 x }  2 L{ 2 3 x} L{e3 x }  2 L{1  2 3 x} L{e3 x }  2 L 1 

2

1
1
s
L{e3 x }  L{1}  L^6 x`
  2
.
s  3 s s  36

1
1
s
L e3 x  2 2 3 x
  2
, s ! 3. 
s  3 s s  36

)
)
)
)

8.1.1. :
f
f 1
1
f
f 1 f 1
1 f 1
f)
 f)1 dxf 14 )
 1 dx
dx f 14 dx 
) 1 fdx  1) dxf
)
)
)

4
) 2 )
dx
) 3 )
2 
)
dx  2 x dx 
2 dx33  x  2 dx
xx  3232 dxx )
2 xx 4 2 x 4
3
x2
x  2 f
x  a x 2 fx 4  a x
f
f 2 x f 2 x
f
2 x
a x
f.
f
f e 2 .
 f.
, ae ! a0x  dx , a ! 0 
xfdxe,.
.
ee 2x0dx
ee af
0f
aax !dx
x dx 0f e 2 x dx .

. 0 )
dx

.
dxe,.
a !dx
0 , ae !
.
e
dx

f
.
e
dx

.
0  dx , a ! 0 
)

0
f
f
0
f
0


 1 
f
1 1 p
dx
a ,
p !p>1,
1, a !
0
8.1.2. a
a>0.
p 1
xp

f
f  x f  x
f)
f e  x)
)dx  f e xx dx 
ee  x dx
0 e )
0 ) dx
dx  0 e dx 
0
0
0
0
0
0
2x
0 )
0  e 2 x)
  0 e 22xx dx 
dx
ee22xx dx
dx e )
)dx f e dx 
f ) f
f
f
f

8.1.3. p,
8.1.4. Laplace :
) f(x)=2x23x+4
) g(x)=2x+3(2 x).

f px px

e e dx.dx 

8.1.5. f1(x)=(x) f2(x)=(x).


L{f1(x)}=F1(s), L{f2(x)}=F2(s) Laplace f1, f2
.
1 s2
) F1( s) = 2 F1( s), s >s >0.0

)  L{f(x)}=sL{f(x)}f(0) (. L4)
s
f1(x)=(x) F2 ( s) = F1( s), s >s >0.0

386

)  () ()
8.1.1 Laplace :
f(x)=(x) g(x)= (x).
) Laplace :
x
f1(x)=ex(x) f2(x)= e (x).

1
8.1.6.  8.1.1 L {1} = , s >s >0,0 L3, s
:
1!
2!
L { x} = 2 , s > 0 , L { x 2 } = 3 , s > 0 .
s
s
n!
n
, L { x } = n+1 , s > 0 .
s
, Laplace f(x)=[(x+1)ex]2.
8.1.7.
 8.1.1 Laplace, Laplace f(x),
8.1.3 F(s).
8.1.3

f(x)

F(s)
1
( s + a)2

xeax
(1 ax)eax

1
( s + a)2

eax et

a
(a z )
( s  a)( s  )

eaxx
eaxx
(a + x)
(a + x)

( s  a)2  2

( s  a)2  2
sa  a
s2  2
s a  a

s2  2

8.2 Laplace.
y= f(x) Laplace F(s) f.
Laplace Laplace. -

387

Laplace
, . Laplace
.
:
f1, f2 , Laplace,
:
L { f1( x )} = L { f2 ( x )} f1( x ) = f2 ( x ) x.
, Laplace F(s)= L{f(x)}
f(x), f(x) . f(x) Laplace F(s) f(x)=L1{F(s)}.
,
Laplace.
:
F(s). Laplace
F(s), f(x), L{f(x)}= F(s).
:

f(x)=L1{F(s)}.
(8.2.1)
n!
, f(x) L{ f ( x )} = n+1 , , 8.1.1
s
L{f(x)}= L{xn}, f(x)=xn=L1{F(s)}.
Laplace,
8.1.1 ,
.
() Laplace , . ( ,
).

8.2.1.

f(x), L{ f ( x )} =

1
.
( s + 1)( s + 2)

1
1
1
.
f ( x ) = L1
= L {F ( s)} , F ( s) =
(
1
)(
2
)
s
+
s
+
(
1
)(
2
)
s
+
s
+

F(s) , :

388

1
( s  1) ( s  2)


.
s 1 s  2
1
as  2a  s 

,
( s  1) ( s  2)
( s  1)( s  2)
+ = 0 2 + = 1.
F ( s)

= 1 = 1
F ( s) =

1
1
1
=

.
( s + 1) ( s + 2) s + 1 s + 2

8.1.1
L{ f ( x )} = F ( s) =

1
= L{e x },
s +1

1
= L{e 2 x } ,
s+2

1
1

= L{e x } L{e 2 x } = L{e x e 2 x } .


s +1 s + 2

f(x)=exe2x, Laplace F(s) :

x 2 x
1
f ( x ) = L1
= e e .
( s + 1)( s + 2)

Laplace ,
Laplace 8.1.
1.  Laplace. F1(s), F2(s)
Laplace f1(x), f2(x), c1, c2R , :
L 1{c1 F1( s) + c2 F2 ( s)} = c1 L1 { F1( s)} + c2 L1{F2 ( s)} = c1 f1( x ) + c2 f2 ( x ) .
2.  Laplace. f(x) Laplace F(s) (. L1{F(s)}=f(x))
aR , Laplace F(sa) :
L1{F(sa)}=eax f(x).

8.2.2.

Laplace F ( s) =

2s + 1 .
s + 6 s + 13
2

Laplace F(s), -

389

, '
8.1.1. F(s) (=16<0),

s 2 + 6 s + 13 = s 2 + 2 s 3 + 32 + 4 = ( s + 3)2 + 22 .
, 1
:
2 s + 6 5

5 1

s+3
2
L1 {F ( s)} = L1
= 2 L1
L
2
2
2
2
2
2 .
( s + 3) + 2
( s + 3) + 2 2
( s + 3) + 2
2,
5
L1 {F ( s)} = 2 L1 {F1( s + 3)} L1 {F2 ( s + 3)},
2

s
2
F1( s) 2
L{(2 x )},
F2 ( s) 2
L{(2 x )}.
2
s 2
s  22
:
5
L1 ^ F ( s)` 2 e 3 x (2 x )  e 3 x (2 x ) .
2
, Laplace F(s) 8.1.1. F(s)
, ( Laplace
Laplace) , .

8.2.1. Laplace, :
2
3
s
) F ( s) = 2
) F ( s) =
) F ( s) =
2
( s 1) + 4
s +9
( s 2)2 + 3
) F ( s) =

24

( s 5)5

) F ( s) =

1

s 1

) F ( s) =

s +1
( s + 1)2 + 7

) F ( s) =

s +1
s + 3s + 5

) F ( s) =

1
6
4
s+2 s


8.2.2. Laplace, :
1
12
2
) F ( s) = 2
) F ( s) =
) F ( s) = 2
s 2s + 2
3s + 9
s
) F ( s) =

s+2
s 3s + 4

8.2.3.  f(x), L{ f ( x )}
, , , .

1
,
( s  )( s  )( s  )

390

8.2.4. F ( s) =

1
.
( s + 1)( s 2 + 1)

) , , ,
1

s+
=
+ 2
.
2
( s + 1) ( s + 1) s + 1 s + 1

) Laplace f ( x ) = L1
.
2
( s + 1) ( s + 1)

8.3  Laplace.
Laplace
n
. n
:
n y( n) ( x ) + n1 y( n1) ( x ) + ... + 2 y( x ) + 1 y ( x ) + 0 y( x ) = f ( x ) ,
i, i=0,1,2,..., n . ,
, :
y(0) = c0 , y(0) = c1 , y(0) = c2 ,..., y( n1) (0) = cn1 .
, ,
n.
, 7 ,
,
.
Laplace, '
.

Laplace f. '
Laplace .
L4, Laplace :
L{f(x)}=F(s) Laplace f, :
L{ f c( x )} sF ( s)  f (0),


(8.3.1)
L{ f cc( x )} s 2 F ( s)  s f (0)  f c(0),
f
, Laplace .

391

n f(n) f,
f . ,
Laplace n f(n) f :
L { f ( n) ( x )} = s n F ( s) s( n1) f (0) s( n 2) f (0) ... s f ( n 2) (0) f ( n1) (0) .

(8.3.2)


, . :
y2y= e5x, y(0)=3.
Laplace y2y=e5x
L{y2y}=L{e5x} L{y} 2L{y}=L{e5x}. 8.1.1 (8.3.1)
:
1
, s>5
L{ y} = s L{ y( x )} y(0),
L{e5 x } =
s5
:
1
s L{ y( x )} y(0) 2 L{ y( x )} =
, s > 5.
s5
y(0)=3 :
1
s L{ y( x )} 3 2 L{ y( x )} =
,
s5
, Laplace L{y(x)}.
L{y(x)} :
L{ y( x )}( s 2) =

1
1 + 3s 15
3s 14
.
+ 3 L{ y( x )}( s 2) =
L{ y( x )} =
s5
s5
( s 2)( s 5)


Laplace Y ( s) =

3s 14
3s 14
, y( x ) = L1
.
( s 2)( s 5)
( s 2)( s 5)

Y(s) :
Y ( s) =

3s 14
8 1
1 1
=
+
, ( s 2) ( s 5) 3 s 2 3 s 5

Laplace :
3s 14 8 1 1 1 1 1 8 2 x 1 5 x
y( x ) = L1
= L
+ L
= e + e .
3
( s 2)( s 5) 3
( s 2) 3
( s 5) 3
8
1
, y( x ) = e 2 x + e5 x .
3
3

. , :

392

)  Laplace .
)  8.1.1 (8.3.1) ( (8.3.2),
)
L{y(x)}=Y(s).
) (), Y(s).
)  Laplace L{y(x)}=Y(s)
y(x)=L1{Y(s)}.
,
.

8.3.1.

yy2y=0 y(0)=1, y(0)=5.


.

Laplace :

L{ y y 2 y} = L{0} L{ y} L{ y } 2 L{ y} = L{0} .

(8.3.1) Laplace
y=f(x) y(0)=1,
y(0)=5,
s 2 L{ y( x )} y(0) s y(0) s L{ y( x )} y(0) 2 L{ y( x )} = 0
L{y(x)}=Y(s). :
L{ y( x )} ( s 2 s 2) 5 s 1 = 0 L{ y( x )} =

s+6
.
s s2
2

Laplace,
,
:
2 2x 5 x .
s+6
y( x ) = L1 2
= e + e
3
3
s s 2
.

8.3.1. Laplace.
) y+2y=0, y(0)=1

) y+2y=2, y(0)=1

) y+2y=ex, y(0)=1

) y+2y=0, y(1)=1

) y+5y=0, y(1)=0

) y5y=e5x, y(0)=2

1
8.3.2.  y+16y=2(4x), y(0)= , y'(0)=0,
2
Laplace .

393

8.3.3. 

dN
= 0, 05 N ,
dt

= (t) (0)=20. L{N(t)}=M(s) M(s) N(t).


8.3.4. 
y + 4 y + 8 y = x , y(0) = 1, y(0) = 0
Laplace .

8.4 .
Laplace
f:[0,+)R.
.

Laplace.
Laplace.
n-
Laplace.

F ( s) L{ f ( x )}
f

f

e  sx f ( x ) dx 
t

f ( x ) dx

lim

t of

f ( x) dx 
0

L {c1 f1( x ) + c2 f2 ( x )} = c1 L { f1( x )} + c2 L{ f2 ( x )}


L{eax f(x)}=F(sa) F(s)=L{f(x)}
L { x n f ( x )} = (1)n

dn
( F ( s)),
ds n

- L 1{c F ( s) + c F ( s)} = c L1 { F ( s)} + c L1{F ( s)}


1 1
2 2
1
1
2
2
Laplace.
Laplace.

Laplace.

L 1 {F ( s a)} = e ax f ( x ), a R
L { f1( x )} = L { f2 ( x )} f1( x ) = f2 ( x ), x > 0
L{ f ( x )} = sF ( s) f (0)

Laplace
f L{f(x)}=F(s).

L{ f cc( x )} s 2 F ( s)  s f (0)  f c(0) 


!
( n)
L{ f ( x )} = s n F ( s) s( n1) f (0) s( n 2) f (0)

... s f ( n 2) (0) f ( n1) (0)


Laplace.

)  Laplace
.
)
 L{y(x)}=Y(s).
)
 Y(s).
) 
Laplace Y(s).

394

8.5 .
,
, .
f

1.

2.

3.

4.

1
dx
x

f

.
f ( x ) dx


lim

t of

f

0
t

) dx  .
ef x( xdx

1 x dx

f .

 Laplace f(x)=1, x>0 F


5.

F ( s) L{ f ( x )}

f

e s x f ( x ) dx 

 sR, .
6.

Laplace f(x)=1, x>0


1
L{1} = , s < 0 .
s

7.

 Laplace f:[0,+)R f(x)=e


1
, s >1 .
L{e x } =
s 1

8.

L{ x 4  2e x  2 x} 4 L{ x}  2 L{e x }  2 L{x} .

9.

 Laplace F
.

10.

11.

 Laplace L 1 {F(s)}=f(x), :
L 1 {F(s 1)} = e x f(x), aR.
 F1(s), F2(s) Laplace f1(x), f2(x)

c1, c2 R, :
1

L {c1 F1(s) + c2 F2(s)} = c1 L {F1(s)} + c2 L {F2(s)}.


12.

 Laplace f, f ', f "


f(0)=f(0)=0, L{f(x)}=sL{f(x)}.

395

.

1.

2.

3.

) 2

3
4

f

) 1
2

f

1
:
dx

x2
) 0
( xdx
) dx

efsx
:

) s<0
) s=0
) s>0
) s=1
Laplace f(x) = x2 :
2
1
) 2s
) 3
) 3
) s
s
s



Laplace f(x)=xe4x :

4.

1
( s 4)2

105 9 / 2
s

16

1
( s 4)3

3
( s 4)2

1
( s 4)1/ 2




Laplace f(x)=x7/2 :
5.

6.

7.

8.

104 9 / 2 3
s

16

9 / 2

) s

Laplace F ( s) =
) ( 6 x )

) (2 6 x )

) 2 ( 6 x )

) e3x

) 4e3x+ex

) 4e3x ex

104 9 / 2
s
16

1
:
s +6
2

) ( 6 x )
3s + 7
Laplace F ( s) = 2
:
s 2s 3
) 4e3x ex

y5y=e5x, y(0)=0, :
) y(x)=4xe3x

) y(x)=5xe5x

) y(x)=xe5x

) y(x)=2xe5x

8.6 .
8.6.1.  , ,
f
(a)= tfa1( xe)dx
tdt, a .
0

) 
(a+1)=a(a), a>0.
) (1)=1.
)  () ()
(+1)=!. +
+
+
) x 5 e x dx , x 4 e 3 x dx , x 7 e x dx .
2

8.6.2.  8.6.1
n!
Laplace f(x)=xn n L{ x n } = n+1 , s >
s >0.0
s

396

8.6.3.  , .
)

+ 3
x dx
2

x 3 dx

xe x dx
2

+
1
dx
) x 2 e x dx
0
1 3x
8.6.4.  ,

3
dx
2 x 1

x
e ,
Laplace f ( x ) =
4,

x2

x > 2.
2,
8.6.5.  Laplace f ( x ) =
2,
8.6.

x 3

,
x > 3.

8.6.6. Laplace .
1
s+4
s +1
s
) F ( s) =
) F ( s) = 2
) F ( s) = 2
) F ( s) = 2
2
s 2s + 9
s 9
s + 4s + 8
( s 2) + 9
8.6.7.
F ( s)

4
s ( s  s  2)
3

,

 2 3

s s
s s  2 s 1

[: s 2 s 2 = ( s 2)( s + 1) ], , , , ,

1
).
Laplace F(s), f ( x ) = L 3 2
s ( s s 2)
dI
+ 50 I = 5 , I= I(t) I(0) = 0.
dt
L{I(t)}=J(s) J(s) , , I(t).
f (x)

8.6.8.


8.6.9. 
Laplace.
) y"+2y'3y=2x, y(0)=y(0)=0

) y"+y=x, y(0)=0, y(0)=2


) y"+4y+4y=0, y(0)=2, y(0)=2
)

d2 y
dy
8
2
dx
dx

8.6.11.
 , Laplace f,
8.6.

_2

. 8.6.

25 y 0, y( ) 0, yc( ) 6

8.6.10.  Laplace
y"+4y'+4y=x2,
y(0)=y'(0)=0 (:
8.6.7).

f(x)
6
5
4
3
2
1
1

. 8.6.

397


( )

9:
10: 

11:
12:

13:

lx im
x0

,
, ,
. . ,

, .

9.1 .
9.2 .

9.3 .

9.4

.
9.5 .

9.6 .

9.7 .
9.8 .

lx im
x0

400

9.1 .
3 .
, , , f:R(0,+) f(x)=x,
>0 ( 1), . , ,
(. 9.1) , g:(0,+)R g(x)=logx.
>1 0<<1
f(g(x))=logx=x g(f(x))=logx=x.
, .
Euler, e=2,718281828459... lux.

John Napier1. , , . , =10, logx
x,
. , 10,
.
()
(),
2,3026. , ln1,20, log1,20=0,0792,
ln1,20=0,0792 2,3026=0,1824 ( ). , , , 1
,
0,4343.
2,3026

y
g
f
(
4
3),
3
2
, h
.
1
, 1
2
3
4 x
4 3 2 1
, 1
. ,
2
,
3

, 4
,
.
. 9.1.

f(x)= x , >1
, 3,
h(x)=log x.
:
1. John Napier (1550-1617). .

401

log=1, log1=0, logx=x, logx=x,


.
9.1 , .
9.1
.
2 0 12
0

2 =1

log21=0

21=2

log22=1

log24=2

log28=3

2 =4
2 =8
4

log216=4

2 =32

log232=5

26=64

log264=6

2 =16

log2128=7

log2256=8

log2512=9

2 =128
2 =256
2 =512
10

2 =1024

log21024=10

211=2048

log22048=11

12

2 =4096

log24096=12

, log327=3 33=27, log216=4 24=16, log10000=4 104=10000.


9.1.1

64 128
, 9.1. :
64128=2627=26+7=213=8192.

2.
9.1.2

, 64:32
, 9.1. :
64:32=26:25=265=21=2.

2.

402

f(x)=logx
:
) ( f(x)=logx, x>0).
) , ( f(x)=logx ).
) ( f(x)=logx
) (. 9.1).
) , (x1<x2logx1<logx2) .
) x>1 logx>0 0<x<1, logx<0.

9.2 .
.
,
. (characteristic), (mantissa).
9.2.1

log78509=4,89492, 0,89492 4.
250, log250 =2,39794, 102,39794 =250.
250 25 2.500, 2.
101 =10, 102= 100, 103=1000, 102,55630
100 1000 .
1, 10 100,
101 = 10, 102 =100, .
9.2.2


:
log25.000 =4,39794
log25=1,39794
log2.500 =3,39794
log2,5=0,39794
log250 =2,39794
9.2.2, 10, , , .
, , 10. :
) , , .
) ,

, . 9.2.1

403

9.2.1
.

567
32,5
2,39

3
2
1


0
1
2

2
1
0

0,275
0,0197
0,00294

1
2
3

9.2.1 )  : ) 1.234, ) 3,12, ) 0,02, ) 21.



)  (c) : ) 567, c=5, ) 4.521, c=0, ) 4, c=1, ) 123,
c=3.
.

) : ) 3, ) 0, ) 2, ) 1.
) : ) 567.000, ) 4,521, ) 0,4, ) 0,00123.
,
, ,
. , ,
0,456 1 0,6590, 0,65901 = 0,3410.
log0,456=0,3410.
, .
, , ,
. 9.2.2 , , , 6, 7, 8. , ,
.
9.2.2
.

6.780
678
67,8
6,78
0,678
0,0678
0,00678

3
2
1
0
1
2
3

0,8312
0,8312
0,8312
0,8312
0,8312
0,8312
0,8312

3,8312
2,8312
1,8312
0,8312
0,83121=0,1688
0,83122=1,1688
0,83123=2,1688

404

9.2.3 0 100. ,
55 log55=1,74036. 550,
2, log55=1,74036. ,
. , , .
9.2.3
.
x

log(x)

log(x)

log(x)

log(x)

1
2
3
4
5
6
7
8
9
10
11
12
13
14
15
16
17
18
19
20
21
22
23
24
25

0,000000
0,301030
0,477121
0,602060
0,698970
0,778151
0,845098
0,903090
0,954243
1,000000
1,041393
1,079181
1,113943
1,146128
1,176091
1,204120
1,230449
1,255273
1,278754
1,301030
1,322219
1,342423
1,361728
1,380211
1,397940

26
27
28
29
30
31
32
33
34
35
36
37
38
39
40
41
42
43
44
45
46
47
48
49
50

1,414973
1,431364
1,447158
1,462398
1,477121
1,491362
1,505150
1,518514
1,531479
1,544068
1,556303
1,568202
1,579784
1,591065
1,602060
1,612784
1,623249
1,633468
1,643453
1,653213
1,662758
1,672098
1,681241
1,690196
1,698970

51
52
53
54
55
56
57
58
59
60
61
62
63
64
65
66
67
68
69
70
71
72
73
74
75

1,707570
1,716003
1,724276
1,732394
1,740363
1,748188
1,755875
1,763428
1,770852
1,778151
1,785330
1,792392
1,799341
1,806180
1,812913
1,819544
1,826075
1,832509
1,838849
1,845098
1,851258
1,857332
1,863323
1,869232
1,875061

76
77
78
79
80
81
82
83
84
85
86
87
88
89
90
91
92
93
94
95
96
97
98
99
100

1,880814
1,886491
1,892095
1,897627
1,903090
1,908485
1,913814
1,919078
1,924279
1,929419
1,934498
1,939519
1,944483
1,949390
1,954243
1,959041
1,963788
1,968483
1,973128
1,977724
1,982271
1,986772
1,991226
1,995635
2,000000

9.3 .
, , .
3,
, , , . ,
9.3.

405

9.3
.

/
/
/

1
11

log(xy)=logx+logy
1
log(xy)=logx+logy
log(xy)=logx+logy
log(xy)=logx+logy
log(xy)=logx+logy

4
44

2
22

x
log

log

2log
log
log
xx
xxx log
loglog
xx=xlog
x log

5
55

1
k
1
11 log

x
log
kk x
x

log
log
logxxk x log x
x kk log
log
3
k
k

6
66

3
33

/
/ /
/

x
log
x yy
xx
log
log
x
log
log
log

log
log
4 log y
xx
log
y log x log y
y
y
y
1
y 1
11

log
log

log
log
log
log yy log y
5 log y
y
y
y
log
log xxx
log x
log
log

x
log
x

log x

log x log
6

log

log
log

, logx1=logx2x1=x2, x1, x2R+ .


(6), ,
, .
xR+, ,>0, ,1 loglog=1 logx=loglogx.
log
, .

9.4 .
, , , .

9.4.1 : =

7
.
2 5
6

, ,
. ,
3

3
7
7
,
,

log
A
=log
6
6
2 5
2 5

log A = log 3 7 log 2 6 5 log A = log 3 7 log 2 log 6 5


1
1
1
1
log A = log 7 3 log 2 log 5 6 log A = log 7 log 2 log5.
3
6
. , log7=0,8451,
log2=0,3010 log5=0,69897. log =0,136, log 0,731=0,136. log =log 0,731,
=0,731.

406

9.5 .

x .
9.5.1
2

: 2x=32, 4x +2x6=16, (5x+1)3x=1.


.
x=, R*+, 1.
1 : . =k, x=k,
x=k.
2 : , , .
logx=log ' x log=log
x

log
.
log

f(x) =, R*+, 1, f(x) x.


1 : . =k, f(x)=k,
f(x)=k.
2 : , , .
.
f(x)= , logf(x)=log, f(x) log=log,
f ( x)

log
.
log

f(x)=g(x), R*+, 1, f(x) g(x) x.


f(x)=g(x).
f(x)=g(x), ,R*+, 1, f(x) g(x) x.
1 : . .
2 : . ,
:
logf(x)=logg(x), f(x)log=g(x)log, f(x)=g(x)log
f(x)=g(x), ,R*+, 1, f(x) g(x) x.

407

x=y y y
, y=x>0 xR.
f(x)=g(x), ,R*+, 1, 1, f(x) g(x) x.
x

=y, y

y , x>0 x>0 xR.

x=x, ,R*+, 1, 1, =1.


1
, y y
y , x>0 x>0 xR.

x=y x

f(x)g(x)=1, f(x) g(x) x f(x)>0.


:
) f(x)=1,
) g(x)=0 f(x)>0,
) f(x)=1 g(x) .
9.5.2

: 3x=6521. 38=6521.

3x=38x=8.
9.5.3

: 2,27x=5,42.

x 1
log2,27x=log5,42xlog2,27=log5,42 3x 1 2 1 = 5 x 1 1 + 4 3 x 1 5 =5 x 1 9 3 = 27

3
25
3
5 x 1 125

5 25

9.5.4
2

: 5x 7x+12=1.

2
5x 7x+12=50x27x+12=0x=3 x=4.
9.5.5
2

: 2x +8x=22x.

x2+8x=2xx2+6x=0x(x+6)=0x=0 x=6.

408

9.5.6

: 3x +x=91+x3x +x=(32)1+x.
2

3x +x=32+2xx2+x=2+2xx2x2=0x=1 x=2.
9.5.7

: 22x+292x+2=0.

2x2292x+2=04(2x)292x+2=0. 2x=y, : 4y29y+2=0.


=49
97
1
y1,2 =
y1 = 2 >0 y2 = >0.
8
4

1
2x=22x=21x=1 2x= 2x=22x=2.
4
9.5.8

: 23x13x2=5x2+45x3.

23x13x131=5x151+45x152.
3

x 1

1
3 x 1 27

1 4
3
x 1
x 1 5
x 1 9


+
3
=5
=
2
=
5

x 1
3
25
125
3
5

5 25
5

x 1

3
= .
5

3
y= >0, : yx1=y3x1=3x=4.
5

9.6 .

x x.
, ,
.
,
:
) logx= x=10,
) logx=log x= >0,
) logf(x)=log f(x)= >0,
) logf(x)=logg(x) f(x)=g(x) g(x)>0.
9.6.1

x: ) logx=4 ) log(x+3)=log2.

409

) log=xx=, : logx=4x=104 x =
) log(x+3)=log2. x+3>0x>3.

1
1
x=
x =0,0001.
4
10
10000
1

: log(x+3)=log2 log(x+3)=log21x+3=21x+3= x= >3,


2
2
.

9.7 .
.

(0,1)(1,+)

(0,1)(1,+)

f(x)=x

f(x)=logx

<x<+

0<x<+

0<y<+

<y<+

, (0,1)

, (1,+)

xy=x+y,

log(xy)=logx+logy,

x
x y ,
y

x
log log x log y,
y

(x)y=xy,

logx=xlog,

xx=()x

logx=x,

x

x

logx=x.

.
/

/
/

/
/

/ /

11

,,
1
log(xy)=logx+logy,
, 44
1 log(xy)=logx+logy
log(xy)=logx+logy
log(xy)=logx+logy

22

x
xx
log
2 x=xlog
log=xlog
=xlog
log
=xlog log

55

33

1
k
1
xx k x 1 log x
k log
log k3 xx
log log
3 log
k
k

66

xx
x y
log

log
log
log xxlog
log
log
4 y
y log x log y
y
y

1
1 log y 1
loglog
log y
y log y
5 log
y
y
log
log x
log xx
log xx
log log x
6 log
log
log

410

.
y=xy=0=1
y=xy=

x=exln

1
x

y=lnx x=ey
y=logx x=x=eyln

y=exy=e0=1

1
ln ln x
x

1
log log x
x

log x

log x

log

.
/

=, ,R+, 1.

f(x)=, ,R+, 1, f(x) x.

f(x)=g(x), ,R+, 1, f(x) g(x) x.

f(x)=g(x), ,R+, 1, f(x) g(x) x.

f(x)=g(x), ,R+, 1, f(x) g(x) x.

f(x)=g(x), ,R+, 1, 1, f(x) g(x) x.

x=x, ,R+, 1, 1 =1.

f(x)g(x)=1, f(x) g(x) x f(x)>0.

9.8 .
9.8.1. :
11
) 5x=625,
) 2x= ,
) 22x+11=8x,
13
9.8.2. :
2
) 4x +2x6=16, ) 32x6=1,
) 63x+4=100,

) 3x24=52x,

) 5x+1+85x=325.

) 82x9=2x.

9.8.3. :
) 4x+1+32x=1, ) 9x+332x=84x+22x, ) 29x+312x242x=0,

) 2x52x=4.

9.8.4. (x2+x+1)x 6x=1.


9.8.5. :
) 73x+2+4x+2=73x+4+4x+3,

) 2logx+25logx=12,

) 9x+13x+2=81(27x+93x1).

9.8.6. : log(8x+2)=log3+log(x23).
1
9.8.7. : logx+log x +10 =2+log x .+10
2
9.8.8. : log(4x)=log33+log(x25).

10


,
, .

10.1 .
10.2
 .

10.3 .
10.4  .
10.5 .
10.6 .

lx im
x0

412

10.1 .

AB
,

, , ,,
. , () , = , , , =
= .
(. 10.1) :

. 10.1.

: =

: , , (

) , ( ).
, . :
:

1
1
1
, =
, =
.


, , .
,
(. 10.1), x (, )
= 2 2 , , 2 = 2 + 2.

413

10.1
y

(, )

. 10.1.

sin
cos
tan
cot
sec
csc

10.1.
: . , . ,
.

90 (
2
).
, 10.1, ,
= 90 , , :
:

= = = (90 ),

= = = (90 )

= = = (90 ),

= = = (90 )

= = = (90 ),

= = = (90 ).

10.1.1
 25 25 = 0,4226, 25 = 0,9063 25 = 0,4666,
65.
.

: , ,
, :
1
1
1
1
2,3663, 25 =
1,1034,
25 =

0,4226
0,9063
25
25
25 , =

1
1

0,4666
25

2,1430.

414

25 65 (25+65=90), :

65 = 25 0,9063,

65 = 250,4226,

65 = 25 2,1430,

65 = 25 0,4666,

65 = 25 2,3663,

75 = 25 1,1034.

10.2 .
,
.

10.2.1 .
, , .
1
, 1, 60=1, 1=
(1). , 60
1
1, 60= 1 , 1=
(1). , 17 , 2
60
14 , 17 2 14.
,
.
.
(rad).
.
:
1

180

, 2 360 , 1
.
180

10.2.1 8 12 15 .
.

, :
81215 = 8+12+15= 8+12+

15
= 8+12+0,25 = 8+12,25 =
60

12, 25
8+0,204 8,204.
60

= 8+

10.2.2 42,14 , .
.

, :
42,14 = 42 + 0,14 1
= 42 + 0,14 60 = 42 + 8 + 0,4
= 42 + 8 + 0,4 1 = 42 + 8 + 0,4 60
= 42 + 8 + 24 = 42 8 24.

415

, Windows (. 10.2).
dms, --.
,
--, Inv + dms.

. 10.2.

10.2.2 30, 45 60.


(. 10.2.) 1
45. :
2 = 12 + 12, 2 = 2, = 2 .
:
1

45 =
= 1 =

45 = = 2 =

2
0, 7071
22 0, 7071
2

1
2

2 0, 7071
45 =
= 1=
45
= = 2 =2 0, 7071

2
2

2
45 = =
1

2
45 = =
= 2

45
B

1
45 =
= 1
1

45

. 10.2.

2 2 (. 10.2).
45 =
= 1. ,
2 ,
,

1
. :

45 = = 1 2 .
2 + 2 = 2.
30
2 + 12 = 22
2
2
2 + 1 = 4
60
2 = 3
B
1 1
= 3
. 10.2.

30
60 :
1

=
0, 5 , 60
= = 0, 8660
3 ,
130 =
3

30 =
30
, 2 =
= 0, 5 , 60
0, 8660,
=
060
, 8660
=
=
2

2
2
2

1
3
3

=30
=
= 0, 5 ,
=
30 =
0, 8660,
, 60
1
3 =
0, 8660
30 = = 2 0, 8660
=
= 0, 5 , 2
, 60
2

2
2
1
3

=
=
1, 7321 ,
130 =3

, 60
3
0, 5774
30 =
=

=
1, 7321
3 =
0, 5774
, 1
3 , 60
3

1
3
2 2 3

2 2

= 3 1, 7321,
1
3 1
=
3

31 , =
=030
=
= 0, 5 ,
==
0, 8660
30 =
60
,
=
0, 560
=
, 8660
,
,

1
7321
=

2 2
2

2
1

416

3
1 ,
3

3
1 =
3 1 =

3
0, 5774
60,
=
=
=
=3
1, 7321 ,
0
5774
,=
60
= =
30
,
=0, 57741,, 7321

1
3
3

3
=3 =
1 60 =
3 , 3
1 1, 7321,
=
30
= 0, 5774

1
3
3
2 2 3
2 2 32 2

2 2 3
3 =

2 22 3 2 3
= 1=
,1547
1,=
1547
=

, 60
1547

30 =30
,1547
, 3
60
= 1,=
=1 ,== 3 1, 1,1547 ,
3 = , 160
= 3 =30
3
3
3

33 0, 5774
3 =3

2 3 2 33
=
,
=
=
1,1547,
= =

3
3

3
3
2
2
2
30
,
60
30 = = 1 2 ,30
60= = 1 2 , 60 = = 1 2
2 2 3
=
, 45
1,1547
= =
10.2

30,

3
3
60.
10.2.

30 =

30

45

60

30

45

60

1
2

2
2

3
2

3
3

3
2

2
2

1
2

2 3
3

3
3

2 3
3

10.2.3  300m (. 10.2).


,
, , 30.
. (
.)
.

 .
, 30.
30 :
h

30 =
, h = 30030,
300

h = 300 3 h 173.21.
3
,
173,21 m.

10.2.4  100ft,
120ft. (.
10.2).

h
30o
300 m

. 10.2.

417
.

 (100ft, ), (120ft, ) .

. :

120
1, 2.
100
( ) 1,2
:
39,794297943.

100 ft

120 ft

, 40 .

. 10.2.

10.2.3 .
. , ,
,
.
.
2 x 2 x 1,
x

1
,
x

x
1

,
x x

1
,
x

1 2 x 2 x ,

x
1

,
x x
1 2 x 2 x.

, ,
. ,
.
.
(x)=x,

(x)=x,

(x)=x,

(x)=x,

(+x)=x,

(+x)=x,

(+x)=x, (x)=x,

(x)=x,

(+x)=x, (x)=x,
(x)=x,

(2+x)=x, (2+x)=x, (2+x)=x, (2+x)=x,


(2x)=x, (2x)=x,

(2x)=x, (2x)=x.

.
(+x)=+,

()=,

(+x)=,

()=+,

418


( ) ,
( ) 1 ,
1


( ) ,
( ) 1 ,
1

1
( ) 1 ,
( ) ,

1
( ) 1 .
( ) .

10.2.5 15 .
.

15 : 15 = 45 30, :
33
11

45 30
33 = 33 33..
15 (45 30 )
=

1 45 30
33 33+ 33
11+1
1
33

10.3 .
,
.
,
.

10.3.1 .
(. 10.3) ,,

180 , , , ,
:
.
++=180,

10.3.2 .

, , , .
, 10.3.

,
. ,
.
, . ,
.

10.3.3 .
(. 10.3). -

. 10.3.

. 10.3.

419

10.3
.

.
( , , . .)

.
( = = , ).

420

, h. , :

h
h .

, :

h
h .

, h = h = , :
= ,

,
.

, :

.


, :

1. .
:

, , . ,
.
: ,
, 0
180.
,
( , . . 10.3).
.

10.3.1  (. 10.3), =4,56, =43 =57. .


.

,
.
, :
=180(43+57)=180100= 80.

4,56

57

43

. 10.3.

421

4, 56
4, 56 80o

6, 58.
E
80o 43
43

4, 56
4, 56 57

5, 61.

57 43
43

10.3.2  , AWACS
S.O.S. , . 10.3 ,
, . ,
. ;
.

x ,
. :
=180(115+27)=180142=38.

, x:
500
500 115
x

x

115 38
38

736.

, 736 km,
.

10.3.4 .

( )

(. 10.3).
, h. , :
2=h2+2 h2 = 2 2.

x
y

(10.3.1)

115o

: = + ,
2=(+)2=2+2+2.

(10.3.2)

2=h2+2,
(10.3.1) (10.3.2), :
2

27 o
500 km

. 10.3.

= + + +2= + +2.

(10.3.3)


=, (10.3.3)

= .

= .

. 10.3.

422

, 2=2+22. , 2= 2+22.
,
:

2. .
:

2=2+22,
2=2+22,
2=2+22.


,
( , . . 10.3).

10.3.3  (. 10.3), =32, =48


=125,2. .
.

, , :
2=2+22
2=322+48223248125,2
2=1024+23043072(0,576)
2=1024+2304+1769,472=5097,472=71,369 71.

32
125,2o
48

. 10.3.

, ,
. , ,
.
, ,
, .
, .
2=2+22
322=712+48227148
1024=5041+230468166321=6816=0,9273
=21,9722.
, 180(125,2+22)180147,232,8.

10.3.4  (. 10.3), =3,5, =4,7 =2,8.


.
.

() ,

.
, . ,
.

4,7

2,8

3,5

. 10.3.

423

2=2+22
4,72=3,52+2,8223,52,8
22,09=12,25+7,8419,6 2=19,6
=0,10204 =95,856654 95,86.

, :
2=2+22
3,52=4,72+2,8224,72,8
12,25=22,09+7,8426,32 17,18=26,32
=0,652735 =49,25182951 49,25.

, :
180(+)180(95,86+49,25)180145,11.

10.3.5 .
AB (. 10.3), , , A,B,, , :

: =+

. 10.3.

=
2

( )( ) ,

( )

=
,

2
( )( )( ),

2=++ .

10.4 .
10.4.1 .
, (bearing) . ,
.
,
,
.
, , 0- 90

424

( ), (. 10.4).
0 360 (.
10.4). ,
, :
0: (N)
90: (E)
180: (S)
270: (W).

10.4.2 .
,
(d) 600
. .
10.4 , , . , .
d. ,
.
N

S
: N W

S
: N E

S
: S W

S
: SE

. 10.4.
N 0

N 0

52
W
270

E
90

W
270

N 0

130

E
90

N 0

W
270

E
90

210

W
270
345

E
90

d
Z

S 180
: 52

S 180
: 130

S 180
: 210

. 10.4.

S 180
: 345

. 10.4.

10.4.1  (. 10.4). 25
( ), N15oW. 20
N32. 2 , d ;
.

1 , 2

425

. , 1 50 , 2 40 .
10.4 1=50 2=40 ,
. , 4 10.3
, d=12. 1,2,
1+2 :
1+2=180(32+15)=18047=133.

:
2 1
50 40
1 50 40 2 1
1 10 133
2

2

2 1 50 40
2
50 40
2
2
90
2

2 1 10
1 10
1
1
66, 5 2
1, 7216 2
0,19128 2
10, 82873
2
90
2
90
2
2

1, 2 :
1 2 21, 65746

1 56 2 77 .

1 + 2 133

, :
1 1 2
d
50
50 47

d
d 37, 526 .
2

77 47
77

10.4.2  5,1km (. 10.4). , 7,2km .


N6510. ,
;
.

, 6510.
=906510=2450.

50
15

32 40

. 10.4.
15

32

5,1 km

6510

. 10.4.

7,2 km

. 10.4.

426

(24 50)

7, 2

5,1
0, 41998

7, 2 0, 41998
5,1

0, 59291 0, 593.

=36o21 =180o36o21=143o39. ,
=36o21. : =180o(24o50+36o21)= 180o61o11=118o49.
, :

(118 49)

5,1 0, 8762
0, 41998

(24 50)

0, 8762

5,1
0, 41998

10, 64 km.

,


, 10,64km.

I:

I:
10.5

.
I:

.

= =

==
= =
=

== =

= =

cos
= ===
==

cos

=== =

=
cos

=
=

cos
==

=
=

= =

tan
== ==

=
tan

== =

==
=

tan

tan

=
=

== =
=

=
=

cot


= =

=
=

= =
cot

= =
=

cotcot
== =

==
=

sec

=== =

sec

=
=

==
=

=
=

secsec

== =

=
=
csc
=

=
=

csc

= =

==
==

csccsc

= =
1
1
1

= 1 , = 1 , = 1

= , = , =
1
1 1

1= 1
=

= ,
=
,

=
,
=

180


1 = 180
1 =
180
1 =

2 = 360
2 = 360

2
= 360

1 =
1 = 180

1 = 180
180

sin
sin
sin
sin


= = = ( 90 )

= = = ( 90 )

427

= = = ( 90 )
.

= = = (90 )

= = = ( 90 )

= = = ( 90 )

1
x

x
= = == =
, ( 90 )
x x

1
x

x
= = == =
, ( 90 )

x
x

= = = ( 90 )

= = = ( 90 )

2
2

x+

= =x=1,
= ( 90 )

1
x
x = =
,
x=
x = ( 90 )
=

1
x
x =
=
,

x = ( 90 )
=x=
1
x =
,
x
1
(x)=x,
x
=
,
x
(x)=x,

x =

1
,
x
1+2x=2x,
x =

1+2x=2x.

(x)=x,
(x)=x,

(+x)=x,

(x)=x,

(+x)=x,

(+x)=x,

(x)=x,

(+x)=x,

(x)=x,

(x)=x,

(2+x)=x,

(2+x)=x,

(2+x)=x,

(2+x)=+x,

(2x)=x,
(2x)=x,

(2x)=x,
(2x)=x.

(+)=+

( )


,
1


,
1

(+)=+
( )

()=
( )

1
,
x

()=+
( )

1
,

1
.


()

428

2=2+22,

2=2+22.

2=2+22,

=+

( )( ) ,

=
2

) = ( )( ) , = ( ) ,

2 (

, )( )( ),
=
(2

2
2

.
..


. ( , ,
. .)
()

429

10.6 .
10.6.1  ,
:

) 843
) 6353
) 751036
) 654

) 493846
) 155
) 553
) 194723
10.6.2  , , :

) 18,7
) 38.022
) 72,81


) 22.3232
) 83.025
) 11,75


) 47.8268
) 0,9
10.6.3 65 :

650,9063, 650,4226, 652,1445,

650,4663, 652,3662, 651,1034.


25.

150 ft
o

20

. 10.6.

10.6.4  20 ,
150ft (.
10.6);
10.6.5 
12m.
, 70,
(. 10.6).
10.6.6  , 120ft
,
15. ;

70o

12 m

. 10.6.

430

10.6.7
 ,

20
40 , 75ft
(. 10.6).
10.6.8
 .
50
20ft ,
;

20o

40o
75 ft

. 10.6.

10.6.9
 60ft.
40ft,
40.
;

35 ft

10.6.10 
35ft 20ft (.
10.6);

20 ft

10.6.11 
12km .
, N63o20E.
(. 10.6);
10.6.12  V .
, ,
. , VB 54o.

VP
. VP
VB
. , ,
VP d (. 10.6).

)

2cm, VP;

)
 VP
3,93cm, ;

)  d
VP.

. 10.6.

12 km

6320

. 10.6.

A
d

V
P

. 10.6.

431

)  VP d.

10.6.13 
, ,

,
. Shasta 14.162ft.
, .
87o53.
, , ft (. 10.6);
10.6.14
 , , ,
30ft.
, , 10ft
. ,
40ft (. 10.6).
10.6.15  :

) x

2

) (4530),

) 27 16 +27 16 ,

= 87o53

) 165 .

R14.162 ft

10.6.16  ,
5 3
2.
2 ,
6 2
10.6.17 :

) = 38, =21, =24,

) =36,5, =24, =34,

) =2345, =2345, =124,67.
10.6.18 
, 295 .
,
, 45
045. 255.

(. 10.6);

. 10.6.

10 ft

40 ft

. 10.6.

10.6.19  . 150
km/h 320o.
200 km/h 200.
3
;
10.6.20 
14ft, 10ft

255

295

45

. 10.6.

45 mi

432

80. : ) . 16ft
) .
10.6.21 d (. 10.6):
10.6.22  , 5000 ft,
35 25. , 105
60.
(. 10.6);
105

d
11 in.

50
12 in.

. 10.6.

60
25
5000 ft

x
y

15 in.

35

. 10.6.

11


.
.
, .

. , .

11.1 .
11.2  .
11.3 .
11.4 .
11.5 .

lx im
x0

434

11.1 .

.
, ,
,
.
:
. .
, , , , .
, , . , .

11.1.1 .
, :
R3 R2, ,
.
,
.
JJJG
(. 11.1).

()

()

. 11.1.

, .
G
G
GG
, , , , ,...,u,v,....

JJJG
,
JJJG
JJJG , .
JJJG
1, .
JJJG
JJJ


G
.
JJJG


JJJG
JJJG (),
() //() (. 11.1).

()

. 11.1.

,
JJJG
JJG
JJJG JJG
. , = .
R3 ( R2)
. . ,

435

R3 ( R2),
R3 ( R2).

11.1.2 .

JJG

, O I, 1
JJG
x.
G
i xx. x
x, x x (. 11.1).

JJGG
JJJJ
G , xx

//

x
(
OM
)
i ,
JJJJG JJJJ
G
JJJJG
G
G
(OM ) =x, OM (OM ) x (. 11.1).
xx , x (x).
x

M(x)

. 11.1.

11.1.3 .
G

G
Gxx yy
G
G G
j
i
i
j =1. j , i
, . xx
yy .

Oxy (. 11.1).
Oxy (. 11.1). yy, xx M1, xx, yy M2.
x M1 xx y M2 yy,
x y . o
. , ,
(x,y)
.
G
,

JJJJJG (. G11.1)
JJJJJG
, ( K1 K 2 ) 1 , ( 1 2 ) 2 y

M(x,y)

M2
j
x

j
x

i
y

. 11.1.

x
O

i
y

. 11.1.

M1

436

G
G
=(1,2).
11.1
G
JJJG JJJG JJG JJJJJG JJJJJG
:
= + = 1 2 + 1 2 .
JJJJJG
JJJJJG
 1 2  1 2
JJJG
x y.
JJJJJG
JJJJJG
JJJJJG
G
 1 2  1 2  1 2 1 i
JJJJJ
JJJJJGG
GG
1122
121 jj .

JJJG

y
B

r2
r1

j
x

x
O

B = 1 i+2 j = 1 i+2 j.

K1

i
y

11.1.4 .

K2

. 11.1.

, . , :
xyz
G,
G
G G
G
G G
G G
i , j k, i = j = k = 1 , i , j k
. xx , yy
zz (. 11.1).

JJJG

JJJG xyz (. 11.1). JJJJG


OM
yz, xOz, xOy
, , . yz, xOz,
JJJG
xOy
. (O )=

JJJJG
JJJG
JJJG
OM , (O)= (O )=
.
JJJG
JJJJG
JJJJG
, , OM , OM =(,,)
JJJG
=(,,). : = i , = j , = k, :
= i+ j+ k = i+ j+ k .
z

r2

r1

k
i

. 11.1.

437

=(x1,y1,z1) R3 ,
G

(0,0,0) (x1,y1,z1) (. 11.1).
G

(x1,y1,z1).

R3 ( R2), G
=(x1,y1,z1) (.
G
G
=(x1,y1)), =(x1,y1,z1) (. =(x1,y1)).

=(x1 ,y1,z1)

. 11.1.

:
G
G
, , ,
(. 11.1).
R3.

11.1.5 .
R3 R2
:
G
G
=(x1,y1,z1) =(x2,y2,z2) R3 R,
:
) :
G G JJJJJG
+ = + =(x1+x2, y1+y2, z1+z2).
) 
:
G JJG
= =(x1,y1,z1,).
)
:
G G
=x
= 1x2+y1y2+z1z2.

G
G
G R3 0 =(0,0,0). R3
G
G
G G
+ = 0 .
11.1 G G
G
.
R2. , R2 , =(x1,y1)
G
=(x2,y2), :
G G JJJJJG
) + = + =(x1,y1)+(x2,y2)=(x1+x2, y1+y2).
G JJG
) = =(x1,y1)=(x1,y1), R.

G G
z
= 1x2+y1y2.
) =x
1
z

1
3

>1

()

<<0

<

()

. 11.1.

0<<1

()

438






. G
, =(x1,y1,z1)R3, :

2
2


x1
+ y12 +z

(11.1.1)
G =
1



G
G
G

=(x
,y
z
)
,
:
.
,

1
1,
1
G

=(x
, :
,

=(x
=(x
,y11,,y
z1,)z:
, :
.
.
,
,

1,y
1,z1)
1,):
=(x
.
,

1,y11,2z
1)1,
G
2
2G
2

=(x
,y
)R
,

:
2
2G G 2

=
x
+
y
+
z
(1)
2
2
2
2
2
2
1
1
G
1 (1)
1
1
2y y++
2
+ y
+
z1=
=
x2 x(1)
++
(1)
1
G
1z1 z1 z1 (1)
=
Gx1 1+1y1 1+
G G = (x1 ,y1 ) G :2
G

=
(x
,y )

=
(x
= 1(x,y1),y
)
:
: = x1 + y12
(11.1.2)
1 )
1

:
1 1 G = (x1 ,y
2 1
2
G
G
2
2

=
x
+
y
(2)
2
2
2
2
G (2)
JJJG
1
1
2y y (2)
+ y
==
1
1y1+
1 (2)
1 (2)
=
x1x2 1+x+

(x1,y1,z1) (x2,y2,z2) ,
JJJG
JJJG


=(x2x1, y2y1, z2z1). d JJJ
G
JJJ
JJJ
G
G JJJ
G
JJJ
G

,y
,z
)

(x
,y
,z
)

(x
1
1
1
2
2
2

,z1
)

(x

(x1,y

,z2,z,z),21):

(x
,z
,z) 2
)

,
,

(x
(x
JJJ
G2(x
2,z
11,y
1,y
1,y
22,y

,y
(x
,y
,z22,y

(x

1JJJ
1)11
2)22
JJG 1
JJJ
G
G

Gy z

=(x2x1,dy2
y1,
z2z1).



2
2
=(x
x1, JJJ
y2

2 d y

=(x
x,y,2y
y
y
zz2z
z12).z

2
1,=(x
2z
1).
JJJ
(11.1.3)
2x
1).
1).

d
( x
x
y1
)
+(
z

=(x
zJJJ

= GA B d=d
2x
12, 1y21y
1,2 1,
21,
2
1 ) +(
2
2 z1 )
JJJ
G
JJJ
G
G
G

, :
JJJ


, :
, :

, :

JJJG
JJJG
JJJG , :
2
2
2
JJJ
JJJ
G G ,
,y
)

(x
,y
),

(x

=(x2x1, y2y1), JJJ


G
2
2
2
d
=
=
(
x

x
)
+(
y

y
)
+(
z

z
)
(3)
2
2
2
2
2
1
1
2
2
2)
= d( x=

x
+(
y2(x
z2y+(
y2zy
)2
d2 =
d
=
==
( yx12)
x 2+(
)x1 )+(
y2)y11)(3)
+(
z2 zz2 2z
z )1 (3)(3)2 1
1 )
=
z+(
2 2 x
1 )1 +(
2 
1 )1 +(
2 
1 )1 1 (3)
JJJ
G
d( x

1y2

:
JJJG
JJJ
G ,y ), JJJ
GG JJJ
JJJ
G
G
JJJ
JJJ
G G =(x2x1, y2y1)
JJJ
JJJ
G

(x
,y
)

(x

1
1
2
2

,,

(x1,y1)

(x

=(x
x1, y2
y=(x

x, y, y

,
,

(x
(x
)
)
(x
(x
,y22,y),

=(x
y y) 1
)
2,y,y
2),,y
2
1)=(x
1
2),
22x

(x
(x

(11.1.4)
1,y11)11
2,y22),
12, 1y21y2 12)1
y 12)x
d
A
B :
( x2
x1 )2 +(
y2

=
=
d

:
:
d
JJJ
G :


2
JJJ
JJJ
G dG d
JJJ
G
2
2
=
=
(
x

x
)
+(
y
 y1 )2
(4)
2
2
2
2
2
1
2 (4)
2)
2)
= d( x=

x
)
+(
y

y
)
(4)
d2 =
d
=

=
=
(
x
(
x


x
x
)
+(
+(
y
y


y
y
)
(4)
1 =
1 y2 
2
1
( x2 2 2
x121 )1
+(
y21 )1
(4) :

:
:

:
G
G
G
3
G GG

R3 3
RR:
,
GG GGG,GG,G1.
R :

3 ,, ,
G
3 3R
,, 1.
, 1.

R
:
1.

,,
,,
,

R
R
:
:

1.

R R :
,, ,
2
) =
G G G 2 G1.GGGG GG 2G 2G 2 G G = G 2
1.
=
=
)
= G G

=
=
1.1.

G G G G G2.GGGG GG GGGG G G = G
2.
=
=
)
+)=
G G +G G G G
=
=

(
2.2.
G G G GG3.GG GG GG GG GG GG GGGGG G(
G+)=
GG G +

G
3.
( +)= (+
+)=

(
(
+)=
+

+
+

=

)
GG G ( ) G G
+)=
()=
3.3.

G
G
G
G
G
G
G
G
G
G G G G=GG ()= ( )
GG GG )
4.
G G
)
=
=
()=
(
(
) G

()=

0 (

)
=0
4.4.4. = ()=

(
=
()=
)
GG =0 G G
G =0
G G
G5.GGGGG G
G G GGGGGG GGt 0 G
G
G


=0 .
G
5.
t 0  
=0
. =0

0

=0

=0
=0
5.5.
=0
t t
0 0t

=0
=0
. . .

:
:

:
G G
G G
GG GG G 2. 3 ,,

R3, :
3
G
3 3R
3:
2.

,,

R
,
:

2.
2.

,,
,,

R
,
,
:
2. ,,

R ,R
:
,
, :
,

G
G
G G G d G (
G G G G G GGGG G1.
G
G
( Cauchy Schwarz),
)
G
1.d1. 1.
(
Schwarz),
Cauchy
(
(
Cauchy
Cauchy
Schwarz),
Schwarz),
d dd
(
Cauchy
Schwarz),
G
G
G
2
2
G
G
G
2G 2
G G 2 2 2 + 2 (
2G G
2 2G)
G G 2 G GGG2 G 2G 2.
G G2 +
G G 2G)
G+
()
),
2 2 =
G
2 2 
+2.
2.+

=
2

+
(
2. +
+
+
++
=
22+
++ (
(
(
)
)
+
   =
2=
)
G
G
G G G GG G G G 3.GG G G)G GGGr d G  (
()
).
r 3.
3.d3. 
(
)
)
r

ddd)
(
(
)
r(
r

:
:
:
:
:
:
1.


,
,
1.
1.

G
G
1.

,
,, :
GG
G G
G


G
G
G
G G G GGGG)
G
G
G
G
G
G
G
G
G
G
:
(

)
.
G
G
G
( :
):
:
z (
((
)
)
z z z
((
(
)
. .
G
G
. )
(
)
.)
)
G ,
G G G G GG
G GG GGG GGG G G GG=G G GG G z 0 ,
G
G
2.

2.


2. 2.

,
=
z=

z z0 z
0, ,0,

=
0,

G,
G G ,
G G G
G
G
G

= ..

=
=
. . .

=
10.1.6


.
G
10.1.6
10.1.6

.. G
10.1.6

3 G .
G G G
G G
G

(x
,y
31,z1) (x23,y32,z2) R ,
G
1
3 R


(x
,y
(x,y121,y,z)121,z)

R ,y
, 22,y

(x
(x

(x
(x
,z ,z)

1,z
1)
12)
2)
G

(x
(x
RR
, ,

G
1,y11,z
1
2,y22,z
2)22
G
G(.
.

10)
G

. .

(.
10)

.
.

(.

(.
10)
10)

(. 10)

439

11.1.6 .

G (x1,y1,z1) (x2,y2,z2) R ,

(. 11.1) ,

:
G
G

G G
G G
G G
G G
G G
G
G 
 (  )   (  1) 
.
. 11.1.
1
G
=(x,y,z), :

x1 x2
y y2
z z2
, y 1
z 1
1.
1
1
1

(11.1.5)

11.1.1


, (x1,y1,z1) (x2,y2,z2) R3,
G
G
. (11.1.5) = 1,
:
x x
y y
z z
x 1 2 , y 1 2 z 1 2 .
2
2
2

10.1.7

10.1.7
G
G
G

,
,...,
11.1.7

.
10.1.7

1
2
R

10.1.7 .
GG G
G G G
G
3G
G G G
G

R ,

,
1
1 +
+...+

,
1,
2 ,...,
2,,
R,
2 12

1 ,2 ,...,G

.
GR3R, 3
,

1 1 + 2 2 +...+ ,
G
G
G
G
G
G
G
3
G G
G
G
G ,,2,,
G
1R,
+ 2

1, .2. ,,R,

1 ,

1
2 +...+

2 ,...,
1
2 ,...,
1
,2 ,..., .

1 +
2 21+...+
R,
, R
1,2
,,

G
G
G
G
G
G
G10.1.7G1
G G1,
, ,..., .

,...,
+...+ ,
.
2,,R,
1 ,2
.
1 ,2 ,...,
G G
G 1 2 3
.
G G ,
G
3
1 ,2 G,...,

R3

R , .

G
G


1 ,
3
R
,

1,2,,
2 ,...,

,
,...,

G
G
G
G
1 ,,
2

G
G
G

,
R,

:
G
G
G
G
1
2

G
G
G
3

+...+

,
,,
R,

1 2
,
,

1,2,,
+ 2

0 .
1 1
2 2
,...,
R,
. G
R

1
,
2 +...+
1=
2

:

G 11
1,G2,,R,
1+
2 2G+...+ = 0 .
G,
G
G
G ,
G
G
G
G
G
G
G
G
G

,
,...,
.
1 1 +
2 2 +...+
= 0 .
.
1
,2
,...,
1

R,

,
:
,

1,

,
,...,
2 1
1 +
2 2 +...+
.
2,,
10.1.7

.
1
2

= 0
G
G
G
G
G
G
G
3
G .
G
G
G
G
G
G

,
,...,
3

,
,...,

1 ,
2 ,...,

+...+ =
0 .
1 ,
2

.

R , ,

.
-

1 ,2 ,...,

1
2

G
G
G
G
G
G
G
G
G

G :

1,2,,
G
R,

11 ==
+ 2 2 +.
1 G,1.

,
G
1=
=0.
2 1,...,
+ G22
+...+

,G

1
,2
,,.
R,

3 1=

1
1 +

0 2,==
G

2
=
G
G
2 2 +...+ G
=
G
10.1.7

.
G
G
G
G
G
G
1 G1 +G2 2 +...+
= 0 .
G

,
,...,

G
G
G
=
==
=0.

1
2

2 ,

3
1 ,2 ,...,
.


11 +
2 R
G 1

,2
,...,
==
=0.

G
2 +...+
G= 0 ,
G
1
.
G

,
1 ,2 ,...,
1=2G
G G ,
G
3
G G
G
G
G
1 G,
,

G
G 3G
G

,...,

3,

1 ,2 ,...,

2 ,...,
1
2

G
G

1 ,

,
,...,

==
=0.
1=

,
,...,

=
==
=0.
2

2 ,...,
G
1 , 2
1
2

G
G
G
1
2

+...+

,
,,
R,

G
G
G
1 2

G G 1 1G 2
2
G G
3

,
,...,


G ,

=
0
,
,,

1.
1
2

G
G G R
1
1+
2 2 +...+

1 ,

.
2 ,...,

8.:

2 ,...,

1,2,,
R,

G G8.

,
,...,
.
G
G
G
G
G
G
1
2

G ,...,
G
G 2
8. .
,
)G
R3

.
=
==
1
=0.
0.1.7

11
,

G
G
G

3 G

=
0
.
2+ 2 2 +...+
8.
G
G
G
G
G
G
1

3 1 ,32 3,..., R

3
G
G

R
,


.
G1 ,
,...,
1 ,2G,...,

3
2

.
.

G G1 G,23,...,
)
, G

R

,

G
G G
GG
G
R,

:
,

R
,

,...,

.
1,
2,,

G
3
1,,2

G

,
,...,

.
+

+...+

,
,,
R,

.
12 G2
.


G G
G
1G
,
1 1
G R
.

,G

G G.
G G 2 G2 .

0G,

G G
3 G

+18.
G )
1 + 2 2 +...+ = 0 ,

,22,,

1
1 1
2 +...+
=0 .
R,

R
1
2, ,...,
,
.

)
,
,

.
G
G
G
.

0
,
G
G
G
G
G
G
==
=0.

1=
3
G
G
G
G
3 G
2

,
,...,

3
3
G
R

,.

,
R

,
3)
).

G G 1,

)
, R

12,G, 2
,...,
3
.

)


G G.
G

G
G 31 ,2 ,...,
G G GG
3R , G
G


1
+
2
2
+...+
0

,...,
.
G
G
G
=
.

, 0 ,, ,2.
,

1:
,

,
, 1

1
,
,

2,,
R,
.

1 ,2 ,...,
GR
G
G
G
.

.
=2
==

1R,
=0.
, :

+...+

01,. 2,,
=.
.
1 1 + G
G G G G3,3 G , G R3 ,
G G G
G 2 2
8.
)

G
G
G
1,,2G ,...,

, G
RR

0 )
.
=
1 ,
,

2 ,...,
G
G
G R3
.
)

,

G G .
G
1 ,
,2 ,...,

G
G
G G .
1 ,2 ,...,

.
G

11 + G2 2 +...+
=
0,
.

0,
G
G
G

8.


11 + 2 2 +...+ = 0 ,
G
=2
==


=0.G

.
3
G 1 G
G
G

R
,
,


G 3 G
=2==
=0.
1

,
,...,

1
2

3 , , R3

,
G
G G
G ).
G G
G

0,

,
,...,

1
2
1 ,2 ,..., .

G G .
G

.
1 ,28.,...,
.
G
G G

440

:
G
G
) R3, G G
.
= R.
G
G G
) , , ,

G
G
G
G
. , = + , ,


.
R (.
11.1).
G G
G
G
3G G
G
G
G
2

,
,...,
1
2
R ,,
=(x1,y1)

=(x
,y
)

2
2
G
G
G
G
2 +...+
,
1,

2,,,
R,
G G
G

, . :

.
, .
G G
G
3
G
G
G
3

1 ,2
,...,
R ,
.
1 ,
G x1
2 ,..., R G
y1
(11.1.6)
/
/
=0 .

1,
R,

:
,
,
2,,


1,2,,R,
x
y
G
2
2
G
G

2 2 +...+ = 0 .
G G GG G
G

R3

1 ,2 ,...,
G G
G G G
1,2
,...,

= 0 .

G
G
G
G G
,2,,R,G

. 1

+...+

=
0
,

1 1

.
1=2==
=0.
G
G
G
1


,2 ,..., ,
R3

G G
G
G
G
G G

.

1
+
2 2 +...+

1 ,
2 ,...,

1
= 0 ,
G
3
R .
=0.
8.
G
G
G
G
G,
3
3
j
=(1,0,0),
=(0,1,0)

=(0,0,1),

k
i

,
R ,

3
G
R , :

G .
G G G G
G
G
G 0G,
i j = j k = k i = 0 i = j = k = 1,
.
G G G
3
3
G
G G
G
,

, ,
R

R ,

=(x1,
,y1,z1)

i , j k (.
,
G
G
G
G
G
.

= x i + y j +
z k. 0 ,
11.1),
1

G
R3 ,
z

z1

k
j

=
i

y1
y

x1

. 11.1.

. 11.1.

11.1.8 .
G
G
R3,
:
G JG
D
= G JG , [0 ,].
(11.1.7)


G

441
z

. 11.1.

()

. 11.1.

()

. 11.1.

G
G
G
G G
(. 11.1). ( , ).

:
G
G

) R3 (. 11.1),
G G
( )
A .
G
G
G
G
) , = , R (
), (11.1.7) :

G2
G JG
G G
G G

= G JG = G
G = G2= G2 = .

>0, =1, =0 [. 11.1()]. <0, =1, = [. 11.()].


(11.1.1),
(11.1.2) (11.1.7), :
G
G
x1 x2 + y1 y2
) =(x1,y1) =(x2,y2), :

(11.1.8)
=
x12 + y12 x22 + y22
G
G
) =(x1,y1,z1) =(x2,y2,z2), :

x1 x2 + y1 y2 + z1 z2
x + y12 + z12 x22 + y22 + z22
2
1

(11.1.9)

11.1.9 .
G

R3 1, 2, 3 , G
G G =(x,y,z)
i , j k (. 11.1).
z

x
G , 2

G G
i
G G
i
G G
j
G G
j

x
G , 2

y
G,

G G
i
G G
i
G G
j
G G
j
G G
k
G G
k

x
G , 2

y
G,

G G
j
G G
j
G G
k
G G
k

y
G,

z
G,

z
G,

(x,y,z)

z
G,


G
.

G G
k
G G
k

j
i

2
y

(x,y)

. 11.1.

442

: 2+2+2=1, :

x2
y2
z2
2 + 2 + 2 = G 2  G 2  G 2

1.

11.1.10 .
G G
G
-G
,
R3.
G
G
G
G G



G = + ,
G
G

(.
11.1).

G
G G
G
G
G
G= . ,  ,.
,
G G G
G G G G
G G
(  ) ( )  .
G G
G
G G
G
G G
=0, , ( ). :
G G
G
G G G
, = G 2 +. :

G G
G
G2

. 11.1.

G G

G G

G G

G2 .

(11.1.10)


G
G
G
G . :
G G
G
(11.1.11)


G
G
G G G
G G G
: , : ( ) .

11.1.1  G(x,y)
(x1,y1), (x2,y2) (x3,y3),
G (. 11.1).

, 11.1.1, :
x x3 y2 y3
M 2
,
.
2
2

. 11.1.


JJJJG , G
AM =2. (11.1.5), :
x

x2 x3
y y
y1 2 2 3
2 , y
2 , x x1 x2 x3 , y y1 y2 y3 ,
1 2
1 2
3
3

x1 2

x x x y y y
G 1 2 3 , 1 2 3 .
3
3

11.1.2  G , , (2,5) (1,1).

443
.

, G(0,0). (x,y) , :

2 1 x
5 1 y
, x=1 y= 6. (1, 6).
0 =
3
3
G
G
11.1.3 =(9,6,3) =(3,2,1) .
0=

G
G
G
G
G
,
=(9,6,3)=3(3,2,1)=3 . =3 =
G
.
G
11.1.4  G =(5,5)
G
=(2,1) =(3,4).

JG
G G G
G G G GG G G G
G
G
G
= 1//=
2
= 1  2 ,
2 , :
1  2 // . 1
G
G
G
1  (5,5)=(2,1)+(3,4)(5,5)=(23, 4)

23=5 4=5.
=3
JG
G
G
G
=7. 1 7 =(14,7) 2 3 =(9,12).
G
G
G
11.1.5 =(1,1), =(2,5) =(4,7).

) .
G
G
G

) .
.

) (11.1.6), ,
1 1
G G
1 5 1 2 3 0 , , .
2 5
G
1 1
G
G G

1 7 1 4 3 0 , , .
4 7
.
G
G
G
)  G , G
G
,R,  .
G
G
G
,  (4,7)=(1,1)+(2,5)(4,7)=(,)+(2,5).
4 2
,

, =2 =1.
G
G G
7 5
2  .

11.1.6  x, (3x+1, 2) (1, 45x)


10. yy,
.
.

10, :

2
2
= 10 [1 (3 x + 1)] + [(4 5 x ) 2] = 10 (3 x )2 + (2 5 x )2 = 10
9 x 2 + 4 20 x + 25 x 2 = 100 34 x 2 20 x 96 = 0 x = 2 x =

24
17

444

, x x=2. , (7, 2) (1,6).


JJJG
JJJ
yy,
G (0,y) , . :
2
2
2
2
49
49+ yy2244yy+44=11+ yy22+12
12yy+36
36
16
16yy=16
16
yy=11.
((0077))2 +((yy22))2 = ((00+11))2 +((yy+66))2

(0,1).
G
G G G G
11.1.7 i  j  k , .
.

G
G
=(1,1, 1) = 3 , :

x
x +y +z
2

1
, =
3

y
x +y +z
2

1
, =
3

z
x +y +z
2

1
.
3

G
G
11.1.8 =(3,2,1) =(2,1,4) .
.

G G

, =(3,2,1)(2,1,4)=32+(2)1+1(4)=624=0.
G
G
G G

, , A .
G
G
11.1.9 =(1,01) =(1,1,0). .
.

(11.1.9),

1 1 + 1 0 + 0 1
1
1
=
= . .
3
1+1+ 0 1+ 0 +1
2 2 2
x12 + y12 + z12 x22 + y22 + z22
G
G JG

G
11.1.10  , . =3 =4, 3
:
JG JG JG JG JG JG JG JGJG2 JG 2JG JG 2 JG JGJG JG JG JG

,,)
)2
, ) 2
. 2.  .
, )
), 2,)
) 2,)2.)

x1 x2 + y1 y2 + z1 z2

G JG G G

1
3 4
6,
) (11.1.7),
3
2
JG JG 2
JG JG
JG JG
JG JG
G2
G JG G 2
) , 2 
( 2  ) 2 2   2  4  4 


) 2 + =

4 9  4 6  16 76,

2
2 + = 76.

G
G
G

11.1.11
, :
G
G
G
G

) =(2,1) =(1,3), ) =(1,1,1) =(2,2,3).
.

(11.1.10) :

(2, 1) (1, 3)
2 (1) + (1) 3
) = 2 =
(2, 1) =
(2, 1) = (2,1),
2
5

22 + (1)2
G G
G G (1,1,1) (2, 2, 3)
1 (2)  1 2  1 3
)
(1,1,1)
(1,1,1) (1,1,1).
G 2
2
3
2
2
2

1 1 1

445

G
G
11.1.12 
=(5,5) =(2,1)
G
=(3,4).
G
G
G
1 , 1 A ,
.
JJG
G JG
G
G
G G
G G G G G
G
G GJJ
G GGG GA G0, , G
G GG G =(1,2).
G G 0 ,G,
GG GG G =(1,2).

1
A
,

,
1

=(1,2).
,

,
0

=(1,2).

=(1,2).

=(1,2).
1
1 A0
1
1
1
1
1 1 1 1
1
1
1 1 1
G
GJJG JG
JJG G
JJJJ
GG1 G11 G1 JJGJJGJJG 1JJJG 1 JJ1G GJJ1G JGJJG JG JGJJG JJ
JJ
G
JJ
G
J
G
JJ
G
G
JJ
G
G
G
G
G
G

,
, ,

, A

0,,

1  2

1 ee 1

=(4,3).


1
1
1

,

JJ1G,

JG

11GG,(4,3).

A,
1 =(1,2).
,

1 ,

1 A 1 (4,3).
,

A
(4,3).
A

A
(4,3).

1 (4,3).
1 (4,3).
GG1
GG1GGG
JJG
G G1GG1 1 GG1G G1 GG1G1GG G1GG GJJGJJ
GGG GJJ
GGGGG GGJJ
G
G
G
G
G
G
G
G
G
G
G
G
G
G
G
G
G
G
G
G
G
G
G
G
G
11 ee
1

2eeee2
,ee
eeee

2
.

12
,

1
ee//

.1ee.(4,3).
.
1
1
2ee
1


2

1211
G
.

.
1 ,1,
1
111
1ee
2//eeee

 21ee.2 21
1 ee
2 ee 1
.

ee

ee
11 .
1 ,
1 
1 1
21
11ee
1
1
1
1
1
1
2
1
2
2
1
1
1
1
2
JJG
G
JJGG
JJGGG G GG GGGJJG JJGJJGJJGG
JJ
G
G
G
G
G
G
G
G
G
G
G
G
G


2 ee 1
.
12
1 (5,5)=(1,2)+(4,3)(5,5)=(
2
1 ee


11

112
1, (5,5)=(1,2)+(4,3)(5,5)=(4,2+3).
1
1
111,
1
1 ,
1
2 2
1
1
1 ,
G2 G JJ1GG1
JJ
G G1 G1 2JJ
JJ
G
G
JJ
G
JJ
G
G
G
G
G
G
5,5)=(1,2)+(4,3)(5,5)=(4,2+3).
21
,

:
2+
1 1

(5,5)=(1,2)+(4,3)(5,5)=(4,2+3).
: 4=5
2+3=5,

4=5
=1
1 1 
1 (5,5)=(1,2)+(4,3)(5,5)=(4,2+3).
1
11
11
,

 1
(5,5)=(1,2)+(4,3)(5,5)=(4,2+3).
 1
(5,5)=(1,2)+(4,3)(5,5)=(4,2+3).
1
1 (5,5)=(1,2)+(4,3)(5,5)=(4,2+3).
=1
=1.

=1.
(5,5)=(1,2)+(4,3)(5,5)=(4,2+3).

4=5

:

2+3=5,
4=5

2+3=5,

:
4=5
2+3=5,

:
4=5

2+3=5,
2+3=5,

G
G
G
G 4=5
G
G :
G 4=5
G
G2+3=5,

=(1)(1,2)=(1,2)

:
1
2 =(1

:
=(1)(4,3)=(4,3).
=(1)(1,2)=(1,2)

ee

ee

ee

ee

.
=1

=1.
:
4=5
2+3=5,1

=1.
=1
=1
=1

=1.
=1.
=1.
2
1=1
1
1
2
1
G
G
G
GG G G 1
G
G
G
G G G
21=(1)(4,3)=(4,3).
=(1)(1,2)=(1,2)
=(1)(1,2)=(1,2)

G=(4,7)
:

:
2 =(1)(4,3)=(4,3).

: 1 =(1)(1,2)=(1,2)
11.1.13

1



1
1 =(1)(1,2)=(1,2)

2 =(1)(4,3)=(4,3).
2,
=(1)(1,2)=(1,2)

=(1)(4,3)=(4,3).

:
:

2 =(1)(4,3)=(4,3).
2 =(1)(4,3)=(4,3).
G1
G =(1)(1,2)=(1,2)
2

=(1)(4,3)=(4,3).
: 1 =(1)(1,2)=(1,2)
=(1,2)
(. 11.1).
.

G
G
G G

1=(2,1),
ee 1
J
G
G
G
JG 0. , , :
G
G
 (4,7)=(1,2)+(2,1). :

G G
2ee 1 .

. 11.1.

2 4

, =2 =3.
2 72

G G G GG
G
G
G
G
G
G
11 2 , 0 ,
,



1 ,

1 A
,
2
G
G
G
G
JJG
JJG JG
JJG
(6,3). , A (4,3).
1 (2,4) 2 ,
1
1
1
G G
G G
G G G
1  2 1 ee 1 2 ee 1 .
11.2 .
JJG
G
G
G
1 1 2 1 ,
JJG
G
G

R3 1 1  1 (5,5)=(1,2)+(4,3)(5,5)=(4,2+3).
, :
: 4=5 2+3=5,
=1 =1.
G
G

G

: 1 =(1)(1,2)=(1,2) 2 =(1)(4,3)=(4,3).
G
G
G G G
=
, =
(. 11.2)
:
)
G
G G
G
G G
k
, ( A
A ).).
j
) G, G

G G
G

, =

G G G
i
, ,

G G G

i, j , k ..
G G G G G
. 11.2.
) u ,
G G
, 0.

.
3
R ,

R,

R3

446 :
,
R3

:
,
,

:
R,
G G
,

) G,
,
G
G
G

11 + 2 2 +...+ G = 0 ,
= = 0.

G
,
)

,
,
=
G
G
G
G


11 +
2
2 +...+

= 0 , =0 = =0.

(.
11.2)

G

G
G
G G
G .
u

,
E , :
,
. 11.2.
J
G
J
G


.
,

E u .
G
0 ,
,


,
,
G


.

0,
,

:
JJGG
JJGG G
G
) u  u
GG u ( G r GG)
GG G GG GG
)
u ( Gr ) Guu rr uu G
G G G G
G G
\
) u u ( u ),, R.

:
G
GG G
G
3
=(x1,y1,z1) =(x2,yJG2,z2) R i, j , k ,
G
G
G
G
G
G
G
x1i  y1 j  z1k x2 i  y2 j  z2 k .
G
G
:
G JG
u

y1
y2

z1 G x1
i
z2
x2

z1 G x1
j
z2
x2

y1 G
k.
y2

,
, :
G G G
i
j k
G JG
u x1 y1 z1 .
x 2 y2 z 2

G
G
11.2.1 =(2,1,3) =(1,0,4).
.



1 3 2 3 2 1

= 2 1 3 =
i
j+
k = 4i 5 j + 1k =(4,5,1).
0 4
1 4
1 0
1 0 4

447

11.2.2 (1,2), (3,4), (4,7).


.

JJJG
JJJG
JJJG
, =(31,42)=(2,2)

JJJG
=(41,72)=(3,9). , ,
JJJG JJJG
. ,
1
: ( )
( ).
2
G G G
i
j k
JJJG JJJG
G
G
G
2 0G 2 0G 2 2 G
i
j
k 0i  0 j  24 k (0, 0, 24)
u 2 2 0
9 0
3 0
3 9
3 9 0
( z = 0).
JJJG JJJG
1
1
u
0  0  (24)2 24 ( )
( )
24 12.
2
2

11.3 .
11.1 11.2 ( ) ( ) .
, :
, , R3,
, , , G
G JG
u :
G JG G
GGG
(  ) ( u ) .

:
G
G
G
3

(x
,y
,z
),
(x
,y
,z

1 1 1
2 2 2), (x3,y3,z3) R
GG G
i, j , k , :

y z1
x z1
x y1
( ) = x3 1
y3 1
+ z3 1
.
y2 z 2
x 2 z2
x 2 y2
:

R3, =(x1,y1,z1),
G , ,
G
=(x2,y2,z2) =(x3,y3,z3), , , :
x1 y1 z1
G JG G
GGG
(  ) ( u ) x2 y2 z2 .
x3

y3

z3

,
, , , , .

11.3.1 .
, , . -

448

GG G
i, j , k , , (. 11.3).
,
,
G G
E u
G
G JG
 , u
G
. :
G JG G
G G G
GGG
V u ( u ) (  ).

. 11.3.

, , , ,
G

G JG
u  , G 2

G JG
, u  .
2
11.3.2 .
, ,
:
) , , , (
).
x1 y1 z1
GGG
(  ) x2 y2 z2 0.
x3

y3 z3
G G G G G G
GGG
) , , : (  ) ( u ) ( u ).



) R , , , : ( ) = ( ) = ( ) = ( ).
.

G
G
G
11.3.1  =(2,1,1), =(1,1,1) =(1,2,1) .
.

GGG
: (  )

2 1
1 1

1
1

1

1 1
1

2

1 1

1

1

1

 9  0.

449

11.4 .

+ = + = ( x1 + x2 , y1 + y2 , z1 + z2 )
z


= = ( x1 , y1 , z1 )

1
3

3
>1

<<0

0<<1

x2 , y1 y2 , z1 z2 )y
( x1

( x1 x2 , y1 y2 , z1 z2 )

<
x

:
:

d
:

( x , y , z )


1( x11, y11, z1)

= x1 x2 + y1 y2 + z1 z2
x1 x2 y1 y2 z1 z2
G 22 x1 x222 y12y2 2 z1z2
( x1, y1, z1)
= xx11 +y2y11 +z2z11
x1 y1 z12
( x1, y1, z1)
2
2
G x1 y212 22 (x1, y1)
= xx11 +yy11
( x1, y1)

d = AB = ( x2 x1 )2 +( y2 y1 )2 +( z2 z1 )2 ,
(x1,y1,z1) (x2,y2,z2)

d = AB = ( x2 x1 )2 +( y2 y 1 )2 , (x1,y1) (x2,y2)
2
=

Cauchy Schwarz


=


( + ) = +


= ( ) = ( )



0 = 0 = 0

d ( x2 x1) ( y2 y1) ,

(x1,y1) (x2,y2)
2

450


( )

( ) ( )


0 0 0

x1 x2
y y2
z z2
, y 1
, z 1
1
1
1

1 , 2 ,..., R3 , 1,2,...,R, ,
:


11 + 2 2 + ... + = 0.

1 , 2 ,..., ,
.


11 + 2 2 + ... + = 0., 1=2=...==0.

G G
, R3
,
. , = , R.

0 , .
x y1

/ / 1
=0
x 2 y2

, , R3 , .
G G

, G R3 , = 0.

G
0 ,
.



= [0, ]

x1 x2 + y1 y2
x12 + y12 x22 + y22
=

G
G
=(x1,y1), =(x2,y2)

x1 x2 + y1 y2 + z1 z2

x12 + y12 + z12 x22 + y22 + z22


G
G
=(x1,y1,z1), =(x2,y2,z2)



= 2

451
z


i
x
= 1 = =

i

j
y
= 2 = =

k
z
= 3 = =

(x,y,z)

j
i

2
y

(x,y)


= = , 0


E =

JG G
G JG
u  u
G G G G G G G
u ( r ) u r u
G G G G
G G
u u ( u ), R

y1
=
y2

y1
k , = x1
y2
x2
G
G
=(x1,y1,z1), =(x2,y2,z2)

z1 x1
i
z2
x2

z1 x1
j+
z2
x2



( )=( )

y1
y2

z1 ,
z2

452

x1


= ( ) = x2

y1
y2

z1
z2

x3

y3

z3

V = = ( ) = ( )

1. , , :

x1

( ) = x2

y1
y2

z1
z2 = 0

x3

y3

z3

2. , , :

( ) = ( ) = ( ).
3. R , , , :



( ) = ( ) = ( ) = ( ) .

11.5 .
G
G
11.5.1  =(2,1). ,
G
G
.
G
G

G
11.5.2 =(2,1) =(3,1), u , 2 u = .
G
G
11.5.3 
G u =(6,5) =(1,2)
=(2,3).

11.5.4 =(3,4,1).
11.5.5 A(2,5) B(10,3). :

) xx , .

) (): y=1 , .
19
11.5.6  A(4,13), B(10,1) G 4, , G 3
. .

453

G
G
G
11.5.7 u =(2,6) v =(2,1) w=(3,1).
G
G JG
G
G
G G G
G G
G
11.5.8  i  2 k , 2i  j  k i  2 j  2 k , :
GGGGJGJGJGJG
GGGGJGJGJGJGGGGG
GGGGJGJGJGJG GGGG GGGG JGJGJGJGGGGG
)
)

)
)
u
)
u
u
,))
)
(u(u
)
)
)
u
u
u
,,)
, ,)
)
((((u
u
u
)))),,,),)
))
((((u
u
u
))u)u)u
u
,,)
,)
u
u
((uuu
u
)).).). .

11.5.9  (4,4,3), (2,4,3) (8,6,6) , .

11.5.10 (4,4,3), (2,4,3), (8,6,6) (1,2,3) , .


G G
11.5.11 , :
JG
JG
JG JG
JG JG
JG JG
JG
JG

) 2 , 4  5 ) 2  5 10 2  5 2 51.. 2 , 5
JG JG JG JG JG JG
G JG
G JG G JGJG

1.

11.5.
12

2 , =2

(
, ()(, ,)..)
1. 1.

2,2, , 5 =5
5
.

JGJG GG  3GG .. GG
3 3 3 JGJG GG
22
33..
G GJG G JG JG

J
G
G
G
J
G
G
G
G
G
G
G

 3. 33. .

1.

:
1. 4433 00

:
:
11.5.13 

,,,
JJGG
GG
GG
GG
2
3
2
3
22


3

3. .G JG G
JG G
G JG
G JG G G
JG GG GG
G JG

2GGJGJG GG  0 . G

(, ) , ( , )
GG JGJG JGJG
:

 00.. .

((, ,)), ,(
(
,

: 
11.5.14
, ,
G JG 22

J
G
J
G
J
G
J
G
GG
GG
G G
i.

((,,)),,, ((,,))

, .
GG GG
J,G, .JG.
G
G
ii. / /
. x :
11.5.15 =(2,x) =(3,2),
J
G
J
G
G

G
G
G
iii.

)i. A ..
) // .
) (, )
.
JG JG
4
G
ii. / / .
G
G
G
JG
G
G JG
G JG
G u
11.5.16 

G
1. 2 ,
3 (G, )
. u 5  4 ,
G JG
uG . G 6 JG

G GJGG
G JGJG
G=(3,5,1)
JG
iii. (, JG) . G=(10,6,2).
J
G
JGG u(
1. 21., GG


5, )GG4GG .,
uJGJGJGJG5 
4 ,
JGJGJG (2, ) GG3G.GJGJ
GJG
GGGG
GG 43 JG
:

((6((,
.

5
4444,,
,

1.
=2,
222,2,, ,
3G333
uuu6u 555

11.5.17


1.1.1.

,,,)))), .
..

J
G
J
G
G
G
6
6
6
6

:
:
,
.
, G u:
GGGGGJGJGJGJG i.
G
G

G
i. G u .
)
GGGi.GGG uG . )
GG u G .GG.GG G GGG G GG
G
GGu
Gu
i.

u
.
G
i.

.
i.

.
i.

u
.
G

+
++

+
+
,

,
,

G
G

+,,+

u .
u . + + , , ,
GGGGG
11.5.18 

G
G
G
G
G
G
G

.
G GG GG
.G GG GGG GGG G G GG GGGGGGG GG G GG
G G GGGG GGG
GG G GG
u..G
G
G
uu
u
G ,, G,

(
,,((
,,=
(()),=

=
22=
,, 2,2,2=

()
=

=
=
=
))=
, , ,
,,

,(,
,=
=

,=

=
==
,

(
),=
=
)=

)((,)
=
()=(
,),)=
=
,==33=
3 33
1.11==
1 11
44 4 G44 G
G
GG
G G G G GG G G
G
+

+
, =(2,2,1)

2,

+ ++
,+

11.5.19

:
1).


,
+

=(1,
G
G
G
G
G GG GG GG GG GG G G
G
G
G
G
GG
G G G G
G G G
GG
, )(=
)=
(3,
=
, =
)((=
11.5.20

=2=

,
=
1 = 12
). = 1

==2(2,, 2,,=
=
)=
( 3,
),
=
=
=
3=
1
,()),=
, ,

)
=

, (,
3, )=
4
4 4
4
G
G
11.5.21
 G G ,
G
G
2 + , 2 .

G G G B(1,2) (1,6). xx,


11.5.22 
G G G


+ ++

+,

()=().
G

G G GG G G G G G
G
G G

G G

)(=
11.5.23
G
, ==

(
,
)
=1= 1, (, ) .
2=
=,=2
2,, 2
), =
,( ), =
G+,=

32

) 4=

+ 4++
4
G G GG G G G
G G
G G
)(=
11.5.24
) =
==23,, 2, =2

=3 (
3,
,1=
1
)= =
:

), =
,( ), =
=
=
6
4 4
G G

) , .



) u = 3 = 2 3 + .
2

= .
11.5.25 

2,

+ + = 6.

454

G
11.5.26
 u =(23,14)

GG G G
G GG,
G

=(5,4) =(16,4).
G

G
G
G
G
G
G
G
G
GG
GG
GGG

GG
GG
G G

, ,,

( ( ,(,
==1
2=22
11.5.27 

==2 323,

(
(,
,) )=
, ) )=== .
==
2, , =

1 =
464
: G
G
G
2 2

G
G
G


) . , ) 2 + 2 , ) ( +) , ) + , ) (2 +3 ) (4 5 ).
G
G
G
11.5.28 =(1,2,3), =(2,3,5) =(1,1,2), .

12


, .
,
(, , , ), .

12.1 .
12.2  .
12.3 .

12.4 ().
12.5 .
12.6 .
12.7 .
12.8 .
12.9 .
12.10 .
12.11 .

12.12 .
12.13 .

lx im

12.14 .
12.15 .

x0

456

12.1 .
A R2 f: AR . C{(x,y)A:
f(x,y)=0} xy. f , C ,
C . , , ,
.
, {(x,y) R2:x2+y21=0} [. 12.1()], , . , {(x,y)R2:2x+3y+6=0} [(. 12.1()], .
1,5

2 x3y6=0

x 2 y 2 1=0

0,5

1
0

0
1,5

1 0,5

0,5

1,5

0
6 5 4 3 2 1
1

0,5

1
()

()

1,5

. 12.1.

3
4

12.2 .

() u,

u0. , () (. 12.2), :

r = ro + MO M ,

r ro
, o. MO M u

, : MO M = t u , tR, r = rO + u, tR.
()

u, u0, :

(12.2.1)

r = rO + t u , t R .
1 2 ()



, () (. 12.2), r = r1 + M1 M M1 M2 = r2 r1 , r , r1

r = r1 + M1 M
r2 , 1 2 .



M1 M2, M1 M = t M1 M2 , tR. r = r1 + M1 M r = r1 + t (r2 r1 ),
tR.

() 1 2 r1 r2
, :
G G
G G
r r1  t ( r2  r1 ), t R .
(12.2.2)

G

r1=(x,y), ro =(xo,yo) u=(,), (12.2.1), :


r = rO + t u ( x , y) = ( xO , yO ) + t ( , ) ( x , y ) = ( x O + t , y O + t ).

457
y

y
o
ro

r1

r2
r

. 12.2.

. 12.2.

x xO t

y yO t

(12.2.3)

()

u .

, r1 =(x1,y1) r2 =(x2,y2), (12.2.2) :




r = r1 + t ( r2 r1 ) ( x , y) = ( x1 , y1 ) + t ( x2 x1 , y2 y1 ) ( x , y) = ( x1 + t ( x 2 x1 ), y1 + t ( y2 y1 ) ) .

, (), (x1,y1) (x2,y2), :


x x1  t ( x2  x1 )
(12.2.4)

, t R .
y y1  t ( y2  y1 )
(12.2.3) t, :
x  xO

x  xO
y  yO

y  yO

0.

, (12.2.4) t, :
x  x1
x2  x1

x
y  y1
x1
y2  y1
x2

y 1
y1 1

0.

y2 1

, (), :

) (xo,yo) u=(,), u0 :

x  xO

x  xO
y  yO

) (x1,y1) (x2,y2) :

x  x1
x2  x1

x
y  y1
x1
y2  y1
x2

y  yO

y 1
y1 1

0.

0.

(12.2.5)

(12.2.6)

y2 1

1(x1,y1), 2(x2,y2) 3(x3,y3)

458

, :

x
x1

y 1
y1 1

x2

y2 1

0.

12.3 .
() (xo,yo)
u (, ) :
x xO y yO
,
=

(12.3.1)

G
(xxo)(yyo) r =(x,y), ro =(xo,yo)
G
G
G G G
n =(,) ( r  ro ) n 0 .
(12.3.1) :
xy+yoxo=0.

x+By+=0

(12.3.2)

(12.3.2),
G
, n =(, )
(, ). , (xo,yo) (12.3.2),
xo+Byo+=0, =(xo+Byo). (12.3.2) :

(xxo)+B(yyo)=0
G
r =(x,y) ro =(xo,yo), :
G G G

( r  ro ) n 0

(12.3.3)

(12.3.4)
G
(12.3.3) n =(, ) (, ).
, G x+By+=0 :
) =(,)
G
) n =(,).

12.3.1 .
(12.3.2) :
=0, 0 0, (12.3.2) :

x ,  .

x= yy
.
0 ==0, (12.3.2) x=0 yy.
=0, 0 0, (12.3.2) :
x 

0 x 

x ,  .

x= xx
.
0 ==0, (12.3.2) y=0 xx.
y 

0 y =

459

0, 0 =0, (12.3.2) :

x y x ,  .

(0,0) (. 12.3).
0, 0 0, (12.3.2) :
x  y 0 y 

x
y






y

x y

1,

,  .

y=x, >0

x
y=x, <0

. 12.3.

12.3.1

x=4 x=4 (. 12.3).


y

x=4

x=4

. 12.3.

12.3.2

y=2 y = 2 (. 12.3).
y

y=2
x

. 12.3.

x
y=2

460

12.4 ().
x'x.
:
H xx
() ()
xx (. 12.4).
y

. 12.4

:
) ()//xx, =0.
) , 0<.
( ) ()
, z , () xx, :
2
=.
:

z ,
) () yy, =
2
.
) ()//xx, =0, .
(x1,y1)
B(x2,y2), x1x2 :
y2  y1
.

x2  x1
, ,
( 1//yy, 2//yy). ( ).
, , ,
1 2, 1
0 ( (1)//yy (2)//xx
). ( ).
12=1.

461

(): Ax+By+=0, 0, ,  .

12.5 .
.
) (x0,y0) .

(. 12.5), (12.2.5) :
0

(12.5.1)

yyo=(xxo).
) (x1,y1) B(x2,y2).
, x1x2, (. 12.5)
yyo=(xxo) :
y2  y1
y2  y1
y  y1
( x  x1 ) y  y2
( x  x2 ).
x2  x1
x2  x1

y2  y1

x2  x1

) (x0,y0) x.
(x0,y0) (. 12.5) (
) ,
xo :

x=x0
y

(12.5.2)

(x,y)

(x2 , y2)

(x0 , y0)

(x0 , y)
(x0 , y0)

(x1 , y1)

. 12.5.

x0

. 12.5.

. 12.5.

12.6 .
.

1) .
(1) (2), (1):A1x+B1y+1=0
(2):A2x+B2y+2=0. (1) (2) 22. () :
) () .
22 , 2,
. :
Dz0

1 1
2 2

z 0 1 2  2 1 z 0.

462

:
x

Dx
y
D

Dy
D

, Dx

 1

 2

Dy

A1

 1

A2

 2

, .
) () .
22 , D=0 Dx Dy
. :
A1 B1 1

.
A2 B2 2
(1) (2) .
) () .
22 , D=Dx=Dy=0, :
A1
A2

B1
B2

1
.
2

(1) (2) .

2) .
:
(1):A1x+B1y+1=0, (2):A2x+B2y+2=0, (3): A3x+B3y+3=0.
32.
,
,
. :
A2 1 1
A2 2 2 0.
A3 3 3

3) .
, .
:
, [. 12.6(, )] [. 12.6()].

, , .

463
1
2
1

()

()

()

. 12.6.

1.
(1): A1x+B1y+1=0 (2): A2x+B2y+2=0, ,

A1x+B1y+1+(A2x+B2y+2)=0, R

(12.6.1)

(1) (2) ,
(12.6.1).
:
) =0 (12.6.1) (1):A1x+B1y+1=0 .
) (12.6.1) (1):A1x+B1y+1=0 .
) (1) (2) , (12.6.1) (1) (2).
, (1), (2) (12.6.1) :
1

2
, 2 2 1
,
1
2
1 2

(1)//(2) 1=2, 

=  2
1
2


1
1

2
2

1  2 .
1  2

1 = 2 = , (12.6.1) (1) (2).


)

2
, 1 0 (12.6.1) :
1
1(A1x+B1y+1)+2(A2x+B2y+2)=0

4) .
xOy (): Ax+By+=0.
() .

464

x+By+>0 , , x+By+<0
.
( ) , : (xo ,yo) x+By+>0,
, .
12.6.1
y


():4x5y20=0.

, (0,0), ().
40+5020=20<0.
(I) , 12.6, (I)
.

(,0)
(0,)

. 12.6.

5) .
() , Ax+By+=0 M(x,y)
(. 12.6).

d 0 : d=

Axo + Byo +
2 + 2

6) .
, , .
A(x1,y1), B(x2,y2), (x3,y3), (. 12.6).
:
1 x2 x1 y2 y1
( AB ) =
.
2 x3 x1 y3 y1
(x3 , y3)

0(x0 , y0)

(x2 , y2)

n=(A, B)

1(x1, y1)
x

. 12.6.

(x1 , y1)

. 12.6.

465

12.7 .
,
(. 11.1.3), .
,
. ,
:
, ( )
.
r=()
= (. 12.7). r
( r0 0<2) =(r,) (r,).
r
.

(r, )
r

. 12.7.

, r,
(r,).

x .
(. 12.7),
0. :
xx

22
xx +yy22
x r

22
22
rr = xx + yy ,,

. .
yy
y r

xx22+yy22

(x,y) (r,)

y
r

12.7.1 .
r ()
r (),
[ ()].
(h,),

(r,) (. 12.7). ( )
2
=, :
( )

. 12.7.

( ) h
h r ( ).
( ) r

(), ,
: h=r(), h
.

(r, )
(h,)

. 12.7.

466

12.7.1 

(1,2) u = (1,3).
.

r = rO + t u = (1,2)+t(1,3), tR.
,

x 1 t
, t R.

y 2  3t

12.7.2  (3,4)

= (1,2).
.

:
.

x xO y yO
x 3 y 4

1
2

12.7.3 1(2,3) 2(4,5).


.

x x1
y y1
x2
y 3
x 2 y 3
x2
, :

y 3.
x2 x1 y2 y1
4 (2) 5 3
6
2
3

12.7.4 (,), (+, ) (+2, 2) .


.

6 (,), (+, ) (+2,


2) :

1
1
1

1

1
( )
2 1
2 1
2 2
2 2 1

=(2+)+(+2)+(+)(2)(+2)()=0.

, .

12.7.5  () (1,3), (6,9) (3,0).


.

=(3 (6), 9) = (9, 9), (). () :


x 1 y 3
, x+y+2=0.

9
9
3
12.7.6  ()
5
yy (0,2).

3
() y=x+, y x 2.
5

467

12.7.7 x 3 + y 2 2 = 0 x 6 3 y + 3 = 0 ..
.

3
6
3
6
:
2 =
. =:
= 1 =
3
2
3
2
3 6

= 1.
1 2 =
2 3
.

12.7.8  , (1):2x+3y4=0
(2):x+9y+=0 : ) ) .
.

)  12.4, (1) (2) , , 1=2. :


2 3
, =6 R.
9

)
 , (1) (2) : 12=1. :
1

A
2 2
A

2 .

3 3

2
2
27
1 2 1 1
R.
1,
3 9
27
2
12.7.9  3x4y+5=0 x+2y1=0.
, A(2,3).
.

(2,3) , . ,
1, 3x4y+5+(x+2y1)=0.
, :
1
3243+5+(2+231)=0612+5+(2+61)=01+7=0 .
7
, :

1
1
2
1
3 x 4 y 5 ( x 2 y 1) 0 3 x 4 y 5  x  y 
7
7
7
7

22
26
34
x
y 
0 11 x 13 y  17 0.
7
7
7
12.7.10 (1,2) (): 4x3y5=0.

(. 12.6 12.6.5) :
d( , ) =

Axo + Byo +
+
2

4 1 + 3 2 5
4 +3
2

5
25

5
= 1.
5

12.7.11 

468

(1):A1x+B1y+1=0 (2):A2x+B2y+2=0.
.


(1) (2), (x,y), .
(1) (2)
(. 12.7). (1) (2)
(1) (2) :
(x,y) (1). :

M ( 1 ) d( M , 1 ) = d( M , 2 )

= d( M , 2 )

A1 x + B1 y + 1
A1 + B1
2

A2 x + B2 y + 2
A2 + B2
2

A1 x + B1 y + 1
A1 + B1
2

A1 x + B1 y + 1
A1 + B1
2

A2 x + B2 y + 2
A2 2 + B2 2

A2 x + B2 y + 2
A2 + B2
2

A1 x + B1 y + 1
A1 + B1
2

.
,
(1) (2).
.
2

12.7.12 

(5, ) (. 12.7).
3

(r,) (), :
()=r. :
() 5

1
() 5 () 2, 5.
3
2

. 12.7.

: r=2,5.

(r, )

12.7.14 
(2,3)
12 (. 12.7).

(2,3)
: ():y3=(x2). x=0, y=32,
yy (0,32). , () xx y=0,
x=23 =0, -

. 12.7.

(5, )
3

(r,) (),
: ()=r()
r()+5=0. .

12.7.13 
5 (.
12.7).

(r,)

. 12.7.

A2 x +

A2

469

. 0, () xx
2 3
, 0 .

, :

10

8
6

1
2 1  3 )() 12 1 ( 2  3 )(3  2 ) 12
(
) 121
() 12 ()(
) 12
)(3  2 ) 12
( (
2 4
2
2 2

M(2, 3)
1
1 2  3
1
1
2


3
212
2
39 0 2 3  3 .
(
) )(12
)
(

)
12
(
)
(3
2

)
(





2 )(3
2

) 12
(
2

)
12

(









(2
3)
24
4
12
2
2 2(2 3) 24
2 4  12  9 0 2 3  2 .
2
2

3
2
2
3

 3)
 12
24
0
2 .3  .
24(2
4 2 12
9 40
2 93
4
6
8
6 4 2
0 2
2

2
2

()
. 12.7.
:
3
x y
() : y 3 ( x 2) 2 y 6 3 x 6 1.
2
4 6
3
x y
() : y 3 ( x 2) 2 y 6 3 x 6 1.
y
2
2
4 6
10

6
4

d( A, 1) =

Axo + Byo +
A +B
2

Axo + Byo +
A2 + B 2

1 0 + (1) 4 + 1
1 + (1)
2

3
2

1 0 + (1) 4 + 1
12 + (1)2

3 2
.
2

x
6

3 2
.
2

. 12.7.

9 2 18
= .
=
4
4

() 1, () ():y=x1
(1,4) 2,
() y4=1(x1), ():y=x+5. -

12.7.16  (1,4)
:y=x1 (. 12.7).

. :
3 2
= d 2 =
2

(2).
(2), x=0, y=4. (0,4)
(2).
d (1):
d( A, 1) =

12.7.15  ,
(1): x
y+1= 0 (2):xy+4=0 (. 12.7).

x
10

1
1

1
2

. 12.7.

M
5
6

x
7

470

():y=x1,
():y=x+5. : 2y=4, y=2 x=3. ,
(3,2).
(), . (,).
4
1
2 , =5 =0. (5,0).
:
3
2
2

12.8 .

. , 3 ..
. 2000 ,
,
.
1600, Kepler
. 80 , Newton
.
,
. , . .

, (. 12.8).
12.8, , . , ,
, ,
. , . ,
, .
,
.

. 12.8.

471

12.9 .
, :
,
.
, .

12.9.1 .
xy (xo ,yo)
R. M(x,y) //y //x (.
12.9), (=90) : ()2+()2=()2,
, : (xxo)2+(yyo)2=R2.
(xo,yo) R :
(xxo)2+(yyo)2=R2.
(. 12.9), xo=0 yo=0,
: x2+y2=R2.
y
y

(x,y)
R

R
(x0 , y0)

(0,0)

(x,y)
x

. 12.9.

. 12.9.

12.9.2 .
, .

2.
x2+y2+Ax+By+=0. ,
2+24>0,


K , R =
2 2

A2 + B2 4
.
2

.
:

x 2 2 x xo xo 2 y 2 2 y yo yo 2 R 2 ,

x 2  y 2  (2 xo ) x  (2 yo ) y  ( xo 2  yo 2  R 2 ) 0.

472

:
x2+y2+Ax+By+=0, A=2xo, B=2yo xo 2 yo 2 R2 .
, x2+y2+Ax+By+=0 ,
2

A2 2
B2 A2 B2
x

0.

4
4 4
4

A2+B24>0, :
2
2
A
B

x + + y + =
2
2

A2 + B2 4
.


K , R=
2 2

A2 + B2 4
.
2

:
) A2+B24=0, x2+y2+Ax+By+=0 ,

, .
2 2
) A2+B24<0, x2+y2+Ax+By+=0 , (x,y), .

12.9.3 .

KM (. 12.9)
x, :

x xo = KA = R y y o = = R ,
y
(x,y)
x=xo+R y=yo+R.

R
(x0 , y0)

,
(x,y) (xo,yo) R,
:

x xo R
, 0 2.
(12.9.1)

y yo R


,
xo=0 yo=0, (12.9.1)
x=R y=R.

. 12.9.
(r,)
R
(r0 ,0)

12.9.4 .
,
R (ro,o) (. 12.9). (r,)
, (. 10)
, :

r0

0
x

. 12.9.

473

R2

r 2  r 2  2r r (  ).

, (ro,o) R :
r 2  r 2  2r r (  ) R 2 .

(12.9.2)

:
) (R,0), =R (12.9.2) :
=2R.

) (R, ), (12.9.2) : =2R.


2
12.9.5 .
(xo,yo) R (xxo)2+
+(yyo)2=R2 ()


(x1,y1) (. 12.9).

(x,y)

(),

, .


, =(xx
=(x1xo, y1yo) :
1, yy
1)

= 0 ( x x1 ) ( x1 xo ) + ( y y1 )( y1 yo ) = 0
[( x xo ) ( x1 xo )] ( x1 xo ) + [( y yo ) ( y1 yo )] ( y1 yo ) = 0
( x xo )( x1 xo ) + ( y yo )( y1 yo ) = ( x1 xo )2 + ( y1 yo )2
( x xo )( x1 xo ) + ( y yo )( y1 yo ) = ( x1 xo )2 + ( y1 yo )2
( x xo )( x1 xo ) + ( y yo )( y1 yo ) = R 2 .

(x1,y1) ,
(x1xo)2+(y1yo)2=R2. ,
(xo, yo) R, (x1, y1) , :
(xxo)(x1xo)+(yyo)(y1yo)=R2.

(x,y)

(x1 , y1)
y
(x0 , y0)

(12.9.3)

,
xo=0 yo=0, (12.9.3) xx1+(yy1)=R2.

. 12.9.

12.9.1

(2,3) R=5.
.

(x2)2+(y3)2=52, , x2+y24x6y12=0.

12.9.2

: x2+y24x6y10=0.
.

4 6
= 4, = 6, = 10, K  ,  = (2,3)
2 2

4 2 + (6)2 4(10)
16 + 36 + 40
93
R=
=
=
= 23 .
2
2
2
12.9.3  (1,2), (5,4), (10, 5),

474

.
.

x2+y2+Ax+By+=0. , . :
+=5
5 +4+=41
10+5+=125.

= 18, = 6, =25, x2+y218x+6y+25=0,


K , K(9,3) R =
2 2

A2 + B2 4
= 65 .
2

12.9.4  C:x2+y22x4y18=0 (2,3).


C (. 12.9).

.


, 1,2 .
2 2

 B(x,y) (). , , = 0 .


=(x2,
=0
y3) (1,1)(x2, y3)=0

=(21,32)=(1,1)
(x2)+(y3)=0 x+y5=0.

: x+y5=0.
12.9.5.  , (): 2xy+1=0 (2,5) x+y=9 (. 12.9).
.

(), ,
2xy+1=0. 2xy+1=0 =2, ()
1
1
. , () : y 5 ( x 2), ,
2
2
():x+2y12=0. ,
x+y=9, x+2y=12. , (6,3)

xy=9

xy=1

(2,3)

(x,y)

(2,5)

(1,2)

. 12.9.

. 12.9.

475

.
():2xy+1=0. :
R = d( K , ) =

Axo + Byo +
A +B
2

2 6 1 3 + 1
2 +1
2

, : ( x 6)2 + ( y 3)2 = 2 5

10
= 2 5.
5

= 20.

12.10 .
,
, :
,
, ,
d(,) (. 12.10).
d(,) . 2 2.
,
d(M,E)+d(M,E)=2.
:
:
) d(,)<d(M,E)+d(M,E), 2<2, <.
) =0, ,
.

12.10.1 .
C .
Oxy x y ,
(,0), (,0) 2 (. 12.10). :
x 2 y2
2 1, =
222.2 .
2

Oxy x
y (. 12.10), C E(0,),
y

2
: x=

(0,)

. 12.10.

2
: x=

(x,y)

(,0) (,0)

d(,)=2
d(,)(,)=2

x
(0,)

. 12.10.

. 12.10.

476

(0,) 2, :

x 2 y2

1, =
222.2 .
2 2

:
) x y
(. 12.10) :
x 2 y2

1, =
222.2 .
(12.10.1)
2 2

. 12.10.

) x y (. 12.10) :

x 2 y2

1, =
22 2. 2 .
2 2

(12.10.2)

:
,
:
) (2) x2,
xx.
) (2) y2,
yy.

12.10.2 .
(12.10.1) (12.10.2) ,
, :
) xx yy .
) A, A, ,
.
) 2,
2.
) (0,0) .
) ,
, . 2d(M,N)2.
.
x 2 y2
12.10, C : 2 + 2 =
,
:
1

) A(,0), A(,0), (0,) (0,)


.
) x=, x=, y= y=.
2
2
) () : x
( ) : x

.
x 2 y2
12.10, C: 2 2 1,

) A(0,), A(0,), (,0) (,0)


.

477

) x=, x= , y= y=.
2
2
( ) : y

) () : y

12.10.3 .
, , .
,
, .
, :
C .

1.
2

, , = 2 2., :
2

2 2
2
2 2

2
=
1 = 1 .
,
2

:
:

(. 12.10).

) , 1 .

) , 0

) , , (. 12.10).

12.10.4 .
x 2 y2

1
2 2
(x,y), (. 12.10). C (x,y),
yy=(xx).

,
x2 y2

2 2 1

y y ( x x )
o
o

y , x.
, ,

. 12.10.
y
(x , y)

. 12.10.

478

, 0.
2 x
=0 2 :
y
xx
yy
x2 y2
2 x
y yo 2 ( x xo ) 2 2 2 2 1.
y

(x,y), , ,
x2 y2
: 2 2 1.

x 2 y2
2 2 1 (x,y)

:
xx yy

(12.10.3)
2 1.
2


x 2 y2

1 (x,y) :
2 2
xx yy

(12.10.4)

1.
2 2

12.10.5 .
, , ,
.
:

, , (. 12.10).

. 12.10.

12.11 .
.

, , d(,) (. 12.11).
d(,) .
2 2.
,
d(M,E)d(M,E)=2.

()=2
()()=2

. 12.11.

: d(M,E)d(M,E)<d(EE), 2<2, <.

479

12.11.1 .
. Oxy x y ,
(,0), (,0), 2 (. 12.11). :
x 2 y2
2 1, =
222.2 .
2

(12.11.1)

:
) xx (,0) (,0).
.
) 2.
xy y x (. 12.11), ,
(0,), (0,) 2, :

y2 x 2

1, =
222. 2 .
2 2

(12.11.2)

=,
:

(x,y)

(,0)

(,0) x

x2y2=2 y2x2=2.
. 12.11.

: , +
.
2.

y
(0,)

12.11.2 .

) C xx yy
(. 12.11). , ,
, ,
. .
) yy .
)
x= x=, (. 12.11).
, ,

(0,)

. 12.11.
y
3

4
x2
y2
x2

1 x 2 2 x 2 2 0 x x .
2
2
2

x=

x2
1 2 1 x 2 2 x 2 2 0 x x .

x 2 y2

1 , = 2 2 , :
2 2

2
x=

. 12.11.

480

x 2 y2
2 2 1

y2 x 2
2 2 1 .

yy, (0,) (0,),


xx. , ,
y= y=.

12.11.3 .
. 12.11 12.11,
. .
:
)

x 2 y2

1 , (. 12.11)
2 2

y2 x 2
) 2 2 1 , (. 12.11)

x , y x .

(12.11.3)

x , y x .

(12.11.4)

: : , ,
y.
y
(0,)

y
y=

y=

y= x

y= x

(0,)

. 12.11.

. 12.11.

12.11.4 .
, , . :

.

1.
2

,, = 2 + 2 , :

481
y

2
2
2 + 2 , 2 1 ,
2
= 1 .

2

:
)
,
(. 12.11).
) 1,
, , 0.
, , .
) = = 2 .

. 12.11.

0 (x0 , y0)

12.11.5 .

x
y
2 1 2

(xo,yo) (. 12.11), :

. 12.11.

xx yy
(12.11.5)
2 1.
2


2
2
y
x
2 2 1 ,

(xo,yo) :

yy xx

1.
2 2

(12.11.6)

12.11.6 .
. , .

. 12.11.


, E (. 12.11).
()

12.12 .

C()

, , .

()=()

,
, (. 12.12).

, .

E()

. 12.12.

482

,
.
, ,
d(M,E)=d(M,).

12.12.1 .
. Oxy
x
p
(. 12.12). , E , 0 2
p
() : x , y2=2px.
2

p , (. 12.12 12.12) p
.
Oxy,
p
y (. 12.12), 0,
p
2
(): y x2=2py.
2
Oxy:
p
p
) E , 0 () : x , (. 12.12) : y2=2px.
2
2
y

p
, 0
2

: x=

(x, y)

(x, y)

p<0

p>0

p
, 0
2

p
2
()

p>0

0,

()

: x=

p
2

()

. 12.12.

x 2=2py

: y=

p
2

x
0,

x
p
: y=
2

x 2=2py
p<0
()

. 12.12.

(12.12.1)

483

p
p
) 0, (): y , (. 12.12) x2=2py.
2
2

(12.12.2)

12.12.2 .
y2=2px (. 12.12), :
) xx.
) p x, ( x0) ,
yy, p>0, yy, p<0.
) .
x2=2py (. 12.12), :
) yy.
) p>0, xx, p<0 xx.
) .
: , .

12.12.3 .
y2=2px (xo,yo) (. 12.12)
:
(12.12.3)

yyo=p(x+xo)
x2=2py (xo,yo) :
xxo=p(y+yo)

(12.12.4)

12.12.4 .

.

,
, (. 12.12).
y
y

0 (x0 , y0)

0 (x0 , y0)

N(x0 ,0) 1 2
p

,0

. 12.12.
. 12.12.

484

12.13 .
,

[. 12.13 (), (), ()].
p=(EB), (r,) . M,
:
r
( ME)

.
( M ) p r
:
:
r
, p r .

p r
, :

r
r

.
p r
r p


, [. 12.13()].
, , :

r
, p r .
p r

, r, :
p
p
r
.
1
1
:
) , 0<<1.
) , =1.
) , >1.
r

(r, )

(r, )

r
E

()

(r, )
r

()

. 12.13.

(r, )

()

485

, , .
,

.
3

0, , ,
.
2
2
( , ).
,
Kepler, Kepler ,
.
.

12.13.1  , 1, 2 x ,
:

) =5, =3,

) (3,0) (3,0) (5,0),

) 8 6,
3

) 6 = ,
5
3

) 8 = ,
5

) 10, 6
.

x2 y2

1.
25 9
) =5, =3
x 2 y2

1. 2=222=16. , :
2 2
x2 y2

1.
25 16
)  8, 2=8, =4 6,
2=6 =3. 2=222=16192=7 :

) =5 =3,

x2 y2

1.
25 7
)  6, 2=6, =3
3
3
= , = =5. 2=222=2592=16. ,
5
5
:
x2 y2

1.
25 16
3
)
 8, 2=8, =4.
5
= 1 2 (. 12.10.3), :

2
16
9
16 16
1 2 2 1
2
2 16.
2
25
25

486

x2 y2

1.
25 16
) 6, 2=6, =3. ,
2
2
10 () : x
() : x
, :

2
2
10
10 2 30.

, :

, 2=222=3092=21. , :
x 2 y2

1.
30 21

12.13.2


x2+2y2=2 :
) y=2x+1.
) y=2x.
) M(2,0).
.

() x2+2y2=2 M(x1,y1) xx1+2yy1=2


x
1 . :
2 y1
) y=2x+1, :
x1
2 x1 4 y1 .
2 y1
M(x1,y1) , : x12 2 y12 2 , x1=4y1,
:
1
1
(4 y1 )2 2 y12 2 18 y12 2 y12 y1 .
9
3
4 1
4 1
, x1 , y1 , x1 , y1 , .
3 3
3 3

, , y=2x+1,
:
4
2
4
2
x + y = 2 x y = 2 .
3
3
3
3
x
) () y=2x, 1 2 1 ,
2 y1
x1=y1. , M(x1,y1) , ,
2
2
x12 + 2 x12 = 2 x12 = x1 =
= y1.
3
3

2 2
2
2
, ( x1 , y1 ) =
,
( x1 , y1 ) = ,
.
3
3 3
3

, , y=2x, :
2
2
2
2
x + 2 y = 2
x 2 y = 2.
3
3
3
3

) () M(2,0),

487

. , 2x1+20y1=2x1=1. M(x1,y1) , :
2
1
1
.
x12 + 2 y12 = 2 1 + 2 y12 = 2 y12 = y1 =
y1 =
2
2
2

2
2
) ( x1 , y1 ) = (1,
).
2
2
, M(2,0),
x+ 2 y=2 x 2 y=2.

, ( x1 , y1 ) = (1,

x 2 y2

1 . ,
52 32
y=x1 (.12.13).

12.13.3 
.

x 2 y2

1 =5 =3. 2=222=2592=16
52 32
=4, (4,0) (4,0).
xx :

x 2 y2

1, 2+2=42=16.
2 2

y=x1,

y x 1
2
y2
x
2 2 1

. , , :
x 2 ( x 1)2

1 2 x 2 2 ( x 1)2 2 2 ( 2 2 ) x 2 22 x 2 ( 2 1) 0.
2
2

,
=0, 220 44+42(2+1)(22)=0, 2(2+1)
(22)=0.
2(2+1)(22)=0 2+4+2222=0 2+12=0 22=1.

2 2 16

, 2 2 1 ,

2 17
2
.

2 15

4
2

, :
x 2 y2
2 x 2 2 y2

1.
17 15
17
15
2
2

12.13.4  ,

. 12.13.

488

: 16y225x2=400 (. 12.13).
.

400, :

y2 x 2
1.

25 16

=5 =4 2=2+2 2=25+16 2=41 = 41,


(0, 41) (0, 41). :

41

5
= =

, y x x y x x.

5
4
4

12
12.13.5  r 2
.
9 252
, .

r r = x 2 + y 2
r2 =

12
x2 + y2 =
2
9 25

12

9 25

x 2 + y 2
y

y
y
, = =
x
r

x2 + y2 =

x2 + y2

12
9 25

y
x + y2

. :

9 x 2 16 y 2 = 144.

x 2 y2
1.

16 9
12.13.6  , (0,0),
xx y=x+2 (. 12.13).

20
18
16
14

12
10
8
6
4
2

14 12 10 8 6 4 2 0 2
2
4
6
8
10
12
14
16

. 12.13.

8 10 12 14

8
7
6
5
4
3
2
1
9 8 7 6 5 4 3 2 1 0 1 2 3 4 5 6 7 8
1
2
3
4
5
6
7
8

. 12.13.

489
.

y2=2px.
:
y x 2
.
2
y 2 px

(x+2)2=2px x24x+4=2px x2(2p+4)x+4=0. =[(2p+4)2]414=4p2+16p=4p(p+4).


y=x+2, . =0
4p(p+4)=0 p=4, .
y2=8x.

12.14 .
.

()
(xo,yo)
G
G G
u =(,), u z 0
y
o
ro

()

G
r

G
G
rO  u, t R.

()

x
y

xO  t
, t R.
yO  t

()

x xO

y yO

() 1(x1, y1) 2(x2, y2) G


G
r1 r2
y

1
r1

r2
r

()

G G
G G
r = r1 +t ( r2 r1 ) , t R .

490

x = x1 +t ( x 2 x1 )
, t R.
y = y1 +t ( y2 y1 )

()

x
x1
x2

()

y 1
y1 1 0
y2 1

Ax+By+=0, +0

( )
()

(x1,y1) B(x2,y2), x1x2

y2 y1
x2 x1

1 = 2

12=1
y

(x,y)

(xo,yo)

yy0=(xx0)

(x0 , y0)

y
y

(x1,y1)
B(x2,y2)
y  y2

y2  y1
( x  x2 )
x2  x1

(x2 , y2)
(x1 , y1)

y
y

(x0 , y)

(xo,yo)

x=x0

(x0 , y0)

x
y

x0

yy
A(0,)
y=x+

(0, )

491
y

xOy yOx

1
y=x

y=x
o

135

y=x y=x

45o

(xo,yo)
xx

(x0, y0)

y0

y=y0
x

M(,0)
(0,)
x y

1

(0, )
(,0)

(r, )
(h,)

(),
,

h=r()

(1) (2), (1):A1x+B1y+1=0 (2):A2x+B2y+2=0,

D0


(x,y)

(1) (2)

1 1
2 2

0 1 2 2 1 0

D
Dx
y y ,
D
D

A 1

1
Dx 1
D y 1
A2 2
2 2
x

D=0 Dx Dy
.
A1 B1 1

A2 B2 2

492

D=Dx=Dy=0
A1 B1 1

A2 B2 2

(1) (2)


(1):A1x+B1y+1=0, (2):A2x+B2y+2=0,
(3):A3x+B3y+3=0

A2
A2
A3

d=

( AB ) =

1
2
3

1
2 0
3

Axo + Byo +
A2 + B 2

1 x2 x1
2 x3 x1

y2 y1
.
y3 y1

(x,y)
R


(xo,yo) R

(x0 , y0)

(xxo)2+(yyo)2=R2


(0,0) R

R
(0,0)

x2+y2=R2


x2+y2+Ax+By+=0

(x,y)
x

A2+B24>0,


K ,
2 2
R

2 2 4
2

493
y

(x,y)
R


(x,y) (xo,yo)
Rx=xo+R Ry=yo+R, 02.

(x0 , y0)

A
x

B
(r,)


R2

R
(r0 ,0)

r 2  r 2  2 r r (  )

r0

0
x
(x,y)

(x1 , y1)
y

(xo,yo)
R, (x1,y1)
(xxo) (x1xo)+(yyo)(y1yo)=R2

(x0 , y0)

(0,0)
R, (x1,y1)

xx1+yy1=R2

x
E y E, E(,,0) (,0)
2

x
y
 2
2

2
: x=

(,0) (,0)

x2 y2

2 2

: x=

(x,y)

1, =
222.2

x
E y
E, E(0,) (0,)

(0,)

222.2
1, =

(0,)

494
y

2
2
x  y 1 o(xo, yo)
2 2
xx yy

1
2 2

(x , y)

xx yy

1
2 2

x
y
2 1 o(xo, yo)
2

x
E y
E, E(,0) (,0)

x2 y2

2 2

(x,y)

(,0)

(,0) x

1, =
222 2
y
(0,)

x
E y
E, E(0,) (0, )

y2 x2

2 2

1, =

222 2

(0,)
y

x 2 y2
2  2

y
x, y  x

y= x

y= x
y

x2 y2
2  2

Mo(xo,yo)
xx yy
 2 1
2

0 (x0 , y0)

495
y
(0,)
y=

y2 x2
2  2

y
x, y  x

y=

(0,)

y2 x2
2  2

Mo(xo,yo)

yy xx
 2
2

p
p
E , 0 () : x  y2=2px
2
2
y

p>0

p
, 0
2

: x=

(x, y)

(x, y)

p<0

p
, 0
2

p
2

: x=

p
2

p
p
E 0, (): y  x2=2py
2
2
p>0

x 2=2py
0,

x
p
: y=
2

: y=

p
2

x
0,
x 2=2py
p<0

496
y


y2=2px Mo(xo,yo)
yyo=p(x+xo)

0 (x0 , y0)


x2=2py Mo(xo,yo)

r
p  r
(r, )

r
E

xxo=p(y+yo)

(r, )

, ,
:
r
r
p  r

(r, )
r

(r, )

12.15 .
12.15.1  (), xx :
2


) = , ) = , ) =.
3
3
12.15.2  xx (),
:

) (6, 2) (3, 7)

) (1, 3)
(2, 4)
12.15.3 (2, 3), (6, 1) (10, 1) .
12.15.4  (7, 5), (6, 7) (2, 3). .
12.15.5  (3, 2) :

497

) (2, 5).
) (0, 3).

12.15.6  , :
3x+4y11 =0 2x3y+21=0 :

) x+2y+1=0.

) 3xy+5=0.

) .

) xx.

) yy.
12.15.7  (): 2x3y12=0
12.
12.15.8  (1):(+2)x+y+31=0 (2):(1)x+y+5=0.
, (1)//(2).
12.15.9  (1): (+1)x+(+2)y=0 (2): x(3+2)y+7=0.

, (1) (2) .
2
12.15.10  ():2xy+1=0 (2): x+2y+3=0
(2, 1) ,
.
12.15.11  (1, 0) (3, 6) (4, ).
.
12.15.12




(1, 4) (1, 5).


) .
) .
) .
) 
.
)

.

12.15.13  x+y=1. (2,3)


().
12.15.14  1, 2 1(t1, t+2) 2 (3t, 3t1)
t (t>0).

) , .

) t .

) t=3.
12.15.15
 (1, 0)
3x+y+6=0 3x+y12=0.
12.15.16  y=x x2+y24x+1=0. :

) ,

)

) .
12.15.17  x2+y2=4
x+y=0.

498

12.15.18  (4, 5)
(3,3).
12.15.19





(0, 0) :
) x p=5.
) x (1,4).
) y (2,2).
) y (0, 4).
) (2,0) : x2=0.
) x y = 4x+1.

12.15.20  6 4
xx
.
12.15.21 :
x2
+y2=1,
) 4x2+9y2=36,
) 9x2+25y2=225.
4
12.15.22 (1, 2), (3,2) (1, 4). :

) .

) .

) .

) .

12.15.23
 1 (1,1), 2 (2,2) 3 (3,1) . :

) .

) .

) .
12.15.24  2x3y+20=0
3x+5y27=0 x+7y16=0.
.
12.15.25  (1,2) x3y+1=0 y1=0 . .
12.15.26  (, ) (,

). (0, 0) (0< ).
2
12.15.27  , (2,6) (1, 2+), R.

) ;

) , .

) ;
12.15.28  xy, . t
(2t40, t30), t0.

) ;

) ;

) ; ;

499

12.15.29  C:x2+y2+4y=0 (1,1).


, .
x 2 y2
12.15.30  () C: 2 2 1 1 (x1,y1),

2 y2
() 2 2 .

x 2 y2

1.
2
6
)  (1, 3 ) .
) .
)  (x0,y0), x02 + y02 =4 ()+()=2 6 (,
).

12.15.31 x2+y2=4


12.15.32  x2+y2=16 C
4
12.15, y x . :
3

) .

) .

) .

)  .
12.15.33  .

) x2+2y2=9, ) r=1, ) r2=2.
12.15.34  .
22 2
16
1616
222 .
, r)
, r(35)=9,
)
r(35)=9,

)
, ,)
)
r(35)=9,
)
)r)
)
,,,))
r(35)=9,
)
))
.. .
rr
rr
r)
rr

3
cos
111

55533

111cos

cos
12.15.35  9,3107 .
9,1107 . ,
.
12.15.36  300 (. 12.15).
200 .
S 200 microsec,
y

300 mi

200 mi

. 12.15
. 12.15

500

. 186.000 sec,
0,186 microsec. S T,
. (: ,
2).
12.15.37  .

. r ,
.
,
:


) r = 100.000.000 , = , ) r = 120.000.000 , = .
4
6

13

. .
,
: , , , .
, ,
. , ,
, .
,

. ,
,
.

13.1 .
13.2  .
13.3  .
13.4  Napier.
13.5 .
13.6 .
13.7 .
13.8 .
13.9 .
13.10 .
13.11 .

lx im
x0

502

13.1 .
, , .
, ,
.
.

1) .
,
.
, :
) ,
(.
13.1). : .1 2 3 (3).
) .
) , : .1 2 3 (. 13.1).
: .
) 1, 2 , 3, ,
.
) 1A2 , 2A3 ,..., A1 .
.
) 2A3, 3A1, 1A2 1, 2 , 3, .:
2A3 1, 3A1 2 , 1A2 3.
) .1 2 3 1, 2 , 3,
1, 2 , 3, .
) 2A3 1, 3A1 2 , 1A2 3, 1, 2 , 3
1, 2 , 3 .

2
3

4
2

. 13.1.

. 13.1.

2) .
, , :
) .1 2 3, ( )
.
) , , [. 13.1()].

503

) .1 2 3 , .
) ,
, [. 13.1()].
1

3
3
O

O
2

()

()

. 13.1.

3) (, , ).
(
) R ( ). (,R) (. 13.1).
,
R.

4) .
:
) , .
)
2R (. 13.1).
) (. 13.1).
( ).

5) .

. (. 13.1).
: .
() . :
) <R, .
) =R, , .
) >R, .
) , : R2=2+2 (. 13.1).

. 13.1.

. 13.1.

. 13.1.

()

504

6) (, R).
:
) , (. 13.1).
) , (. 13.1).
: .
, , , R (.
13.1). ,
. .

7) .
:
) .
) , , ( ), (. 13.1).
, ,
.

) ,
R
.
)


(). . 13.1.
.
(. 13.1).
) .
) ,
.

8) .
:
)
.
)
,
.
)

.

9) .

:

()

()

. 13.1.

. 13.1.

505

) , , (. 13.1).
) , o
( ), [. 13.1(), ()]. 13.1()
() PP.
) PP,
, P (. 13.1).

10) .

.
:
)
[. 13.1()].
)
(,R) P (PA)
(P) (. 13.1). ,
, ,
PA PB (. 13.1.1).
) , .
)

.
) (,)
() (. 13.1)
PA
.
)
90

. 13.1.
P

()

. 13.1.

()

11) .

,
(. 13.1).

, .
:
)


.
) xPy

. 13.1.

. 13.1.

506

, (.
13.1).
) , (. 13.1).
xPy== B=.

P
x

12) .

,
(. 13.1). ,
, .

. 13.1.

: .
) , , B, B
. 13.1 ,
.
) .
, .
) rad (180o).

. 13.1.

60

13) .

(. 13.1)
.
. 13.1.
, , ..
.
, , ,
, , . , .
,
, , , B .
a ,
, s ,
(.. ). R
( ):
s = R a.
: , , , (rad)

507

(, , ). , .

13.1.1

13.1,
. =a=60o.

14) .
13.1 , 13.1 .
13.1
.

(90).

90.

90.

90.

90.

90.

( )

90.

90

90

()

90

()

90
90

90

()

. 13.1.
.

()

508

15) .
(. 13.1), , . , ,
,
,
. , , .
,
(.
13.1) (. 13.1.2).

,
+=+=+=180 (=2 ),

. 13.1.

+=+=+=180 (=2 ).
:
. , , .
.

. 13.1.

16) .
:
) , , ,
360 (. 13.1), (. 13.1.3):
++<360.
) , , ,
180 540, (. 13.1.4) ,
180<++<540.
) , , ,
:
( )
+ + =
,
R2
() R .
++ ,
2S , . :
( )
2S =
.
R2
: (2S0),
(R), .
.
,
, 2
(. 13.1). :

509

2S =

. (. 13.1.5).
8

() , , 1/8
. :

90

90

90

1
( AB ) 2 S E,
8
2
. 13.1.
=4R , R 2S

( 2S 90 , , ).
) , ,
(. 13.2.4).
) (.
13.2.5).
) ,
.
.
13.1.2

= 120, = 145 = 70 .
2m, .
, .
120
145
70
A
o.,
. .
90
90
90
:
2S =
:

120 145 70
155 31
+
+
2 =
=
.
90
90 90
90 18

2
2
2
1
31 1
31
( AB ) 2 S R 2 10,821m .
2
18 2
9

,
.
.

13.1.1  (,R) P (PA)


(P) (. . 505).
.

13.1, , ,
.
, . PA PB .
P , ,

510

. :
(1) = ( )
(2) = ( )

, , PA PB .

13.1.2
 ,
(. 508).
.

90. (. 13.1),
, , 90.
, 90 ,
.

.
.

13.1.3  , , ,
360, (. 13.1) (. . 508): ++<360.
.

,
.
, :

=, = = ++= ++. (13.1.1)

, ,
, :

<, < <.


++<++=360.

(13.1.2)

(13.1.1) (13.1.2), ++<360.

13.1.4  , , ,
180 540, , 180<++<540 (. .
508).
.

, :
++<360 (180) + (180)+(180)<360
540(++)<360 180<++

(13.1.3)

, 0<<180, 0<<180 0<< 180,


++<540

(13.1.4)

(13.1.3) (13.1.4) .

13.1.5.  , (

511

) (. . 509). :
2S =
.

(. 13.1),
R=1rad. 1/8
, 8 . ,
, , ===

2

2S= . =4R2 R=1rad,


2
=4rad2. , :
( AB )

.
8

4 rad2 rad2 .

8
8
2

( AB ) 2

, : 2 S = .
2
8
1 2
R

13.2 .
, , , , , , ,
R (. 13.2).
x Ay
, B . x y

, , ,
, ,

13.2
, y

.
. 13.2.
. ( )
=+
=+

(13.2.1)
(13.2.2)

=+

(13.2.3)

. ( )


=
=

(13.2.4)

.
= (13.2.5) = (13.2.8)
= (13.2.6) = (13.2.9)
= (13.2.7) = (13.2.10)

512

. ( Gauss)
=+ (13.2.11)
=+ (13.2.12)

=+ (13.2.13)

.
2

( ) ( )
=
2
( )

(13.2.14)

2 = ( ) ( ) ( )
2
( )
(13.2.15)

2S
2S
( )

2
2
2 =

2
S
2 ( ) ( 2 S )
2
2

(13.2.16)

:


( )

,T
2 S
.
2
R2
2

Gauss Delambre.
+




2
2
2 =
2
=
=
(13.2.17) =
(13.2.18)
2
2 +
2 +
2
2 
2 
2
2
2
2

Napier.

+
+

+
2
2
=

2
2
2

2
2

2 =
2
=


+
+
2
2
2

2
2

(13.2.19)

(13.2.20)

.
=
=
=

(13.2.21)
(13.2.22)
(13.2.23)

= (13.2.24)
= (13.2.25)
= (13.2.26)

:
) (13.2.1), (13.2.2), (13.2.3)
, ,
.
) (13.2.4) ,
.
) (13.2.11), (13.2.12), (13.2.13) , ,
, , , . ,

+
+

+
+

+
+

,
,

= ==
,
,
= ==
,
,
= ==
. . .

513

) (13.2.14) (13.2.20) , ,
. ( ..)
) (13.2.21) (13.2.26) , (. 13.2), , .
) (13.2.21) (13.2.26),
=,
:
=.
, , , ,
. .
1 (versine) ,
1
( ver) =1. ,
, 2
(haversine), ( hav) :
=

1
.
2

:
=()() (13.2.27) =()+ (13.2.30)
=()() (13.2.28) =()+ (13.2.31)
=()() (13.2.29) =()+ (13.2.32)
: 1

,
2 0.
2
2
.

13.2.1  , , . =68 =80 16, ==6030,


(. 13.2).
.


=
= .

(13.2.4),

, ,
log = 9,9397
log = 9,9672
+ log = 0,0063
log = 9,9132

. 13.2.

514

=5458 =12502. , >, >, <90, =5458. ,


90 =905458=3502 .

13.2.2 (13.2.5):
=.
.

( 13.2.2).
=+.

=
=()
=2
=(12)
=2.

0<<180, 0, :
=.

13.2.3.  ( Gauss), ( 13.2.11).


= +.
.

. :
=+
(180)=(180)(180)+(180)(180)(180)
=()+()
=
=+.

13.2.4  , , , (. 509).
.

:
 =. =. =,
=. :
+=+
()+()=0
0+()=0
(1)
()=0.

0, 0 <<180 0 <<180 , (1) :


=0 =. =, 0<, <180.

515

:  =. =. =,
=. :
+=+
()+()=0
0+()=0
()=0
(1)

0, 0 <, <180 , (1) :


=0 = =, 0<, <180.

13.2.5 
, (. 509).
.

, , : +>.

=+
(1)

(+)=
(2)
(1), (2) :

 (1  ) (  ) 1  2 2  1
2

(  ) 2 2 ! 0,
2

>0, >0 0, 0<, , <180, :


2
(+)>0 >(+)
(3)

+180, +>, 0<<180.


+<180, (3) : <+.
( x, [0, ]).

13.2.6  (. 13.2), =40


=3848 =2532. ( ) .
.

30, : =()+ (1), =1316.


=x, , :
log+log+log=logx
:

log = log38 48 =9,797


log

= log25 32 =9,6345

log = log40 =9,0682



logx

=8,4997, x=0,0316.

x=0,0316 ()=0,0133, (1) :


=()+x=0,0133, =1314.

516

, :
=( )(),


7734.,
2

=6419 =38 46. :


log() =9,9548
log() =9,7967
log =0,6403
log =0,2030
log =0,5948, =0 22.

13.3 .
13.2. . 1 10 ,
,
.

1.
=90, :
1.() =
1. () =
2.() =
2. () =
3.() =
3. () =
4.() =
4. () =
5.() =
5. () =
.

=
.
() ( 13.2.4) :

90o
1

=
= .
= 90, :


2().
3. () ( 13.2.11) :
= + (=90)
= +90
= +0
=.
() ( 13.2.11) :
=+ (=90)
90=+
0=+
= (0 0)

=

= .

517

() ( 13.2.13) :
=+ (=90)
=90+90
=0+1
=.
5 ().
: .
13.2 .

13.4 Napier.
1 . , Napier ,
.

= 90 (. 13.4).
,
. ,
, , , 90 . ,
, 90 90 ,
13.4().
90
90

90

()

90

90

90

. 13.4.

()

, .
, 90 90
90 .
, Napier :

Napier.
.

Napier.
.

518

Napier 1, 13.4, .

13.4.1

13.4(), 90
, 90 90 .
1 Napier =(90) =.
2 Napier : =(90)(90) =.
: , ,
,
13.4() , .

13.5 .
,
. , ,
, ,
.
,
= ( 1),
(0, 180). , =0,5, (0, 180) , =30 =150. ,
, .

2.
,
, .
.
=90.
[. 13.4()].
2 Napier :

(90)=(90) =.

(13.5.1)

0<< 180, >0, (13.5.1)


.

0

0
(0 ,90 ) (90 ,180 )

(0
,90
)
(0
,180
)

519

180, .

3.
,
, ,
, .
.
.
=90 [. 13.4()].
, , , .
2 Napier :
=(90)(90)=.

(13.5.2)

0 < 1, (13.5.2) :

1 .

(13.5.3)

,
. :
) , . (13.5.3)
, f(x)=x
(0, ), , , .
2
) , .
(13.5.3) , f(x)=x
( , ).
2

4.
.
) , ,
.
) ,
(, ).
4 3.

13.6 .
,
.
( 1).

13.6.1 .
:
1:  ,

520

( ) , 13.4().
2 :  Napier
, ,
.
3: 
.
4:  ( 2 4), .
5: , . , ,
.

13.6.2 .
, .
3
, . 4
.

1.
.
=90,
, 0<, <180 .
1:  (. 13.6),
, .

2 : .
 , 90 90,
Napier :

=(90)(90) = = .
(13.6.1)


.
 , ,
Napier :

(90 ) = = =
.
(13.6.2)


.
 , 90 ,
Napier :

.
(13.6.3)

(90)=(90) = =

4:  , :
=1 (0,90) =2(90,180),
 2 4
.

5 : .
 , , .

521
90

13.6 90, 90 . Napier,


:

90
90

(90)=(90) =.

. 13.6.

13.6.1

, =90, =96 =97.


13.6.
.
(13.6.1) :

B=

97 0,9925
=
=
=0,998.
96 0,9945

( >90, ), =93,6234 =86,3757.


.
(13.6.2) :

96 0,1045

0, 8573, 30, 9852.


97 0,1219

.
(13.6.3) :
97

96
:

8,1443
9, 5144

0, 856,

31,1296.

(90)=(90) =
0,856=0,9980,8573 0,856=0,856, .

,
, .
13.6.2

, =90, =96 =95.


, , 13.6.
.
(13.6.2) :
=

96 0,1045
=
=
= 1,1984 > 1 , .
95 0, 0872

=96 =95.

522

:
. , , , .
.
, ,
(13.6.1), (13.6.2), (13.6.3),
.
, (13.6.1) : = log()=log()+log(),

(13.6.2) : = log()=log()+log(),

(13.6.3) : = log()=log()+log(),
:
= log()=log()+log().
.

= 97
= 96
=86,5233
= 30,9485
= 31,1304

log () = 9,9968
log () = 0,0024
log () = 9,9992

log () = 0,9141 log () = 0,9108


log () = 9,0192 log () = 9,0216
log () = 9,9333
log () = 9,9324

log () +log() = 9,9992 +9,9333 = 9,9325

( ).

2.
.
, =90, , 0<, <180.
1:  (. 13.6),
, .

90
90

90

. 13.6.

2 : .
 90 , 90 , Napier :





(90 )=(90) =

=
=
(13.6.4)

.
 , 90 90 ,
Napier :
=(90)(90) =
(13.6.5)
.
 , 90 90,
Napier :

523

(90)=(90)(90) =

=
=


4:

(13.6.5) :

(13.6.6)

=1(0, 90) =2(90, 180).


 ( 2) , , =.

5 : .
 , .
Napier, :

=(90) =
(13.6.7)

, (13.6.7).
13.6.3

, =90, = 39 =137.
13.6.
.
(13.6.4) :
%
=13739=0,73130,8098=0,5922,

=149,366, 180=30,634.
.
(13.6.5) :
==13739=0,6820,629=0,429.
=25,404 =18025,404=154,596. ( >90, ),
= 154,596.
.
(13.6.6) :
==39137=0,7771(0,9325)=0,7246
1

= 0, 7246
= 1,38
=0,7246
=1,38

180=54,071, =125,929.
:
(13.6.7) : = =.
:
154,596125,929=149,366 0,429( 1,38)=0,5922
0,59202= 0,5922
( ).

524

3.

90

.
,=90,
0<, <180.
1:  (. 13.6),
, .

2 : .

.

Napier :

90

90

(90)= =

= =

. 13.6.

(13.6.8)

.
Napier :
= (90 ) = =

(13.6.9)

.
Napier :
= (90 ) = =

= =

(13.6.10)

3: .

Napier :

(90)=(90)(90) =

(13.6.11).

13.6.4

, =90, =37 =121.


13.6.
.
(13.6.8) :
==37121=0,7986(0,515)=0,4113, =114,285.
.
(13.6.9) :

37
121

.
(13.6.10) :

0,7536
0,8572

0,8791, 41 ,319.

525

121
37

1,6643
0,6018

2, 7655, 180 

70 ,12

109 ,88.

:
(13.6.11) :
1
1
1
1

0,4113


0,8791 2,7655
0,4113 04113, .

4.
.
, =90,
0<, <180.
1:  (. 13.6),
, .

2 : .

.

Napier :

(90  ) (90  )






90
90
90

. 13.6.

(13.6.12)
.

.
Napier :
(90)=(90) =
.
Napier :
(90  )

(13.6.13)

(13.6.14)

5: .

Napier :

(90)=(90) =

(13.6.15)

13.6.5

, =90, =1 =52.
13.6.
.
1o
0,0175
(13.6.12)

0,0284, =1,6267.

52
0,6157

526

.
(13.6.13) :
=152=0,99980,788=0,7878, =38,0157.
.
(13.6.14) :
=152=0,01751,2799=0,0224 =1,2831.
:
(13.6.15) , :

0,7878 0,7887, .

1
1
0,7878 0,0224

0,284

( ).

5.
( ).
, =90,
0<, <180.
1:  (. 13.6),
, .

2 : .

.

Napier :

90
90

.
Napier :

(90  ) (90  )

. 13.6.

(90 ) (90 ) (90 )


1

.

(13.6.16)

(13.6.17)

.
Napier :

(90  ) (90  )

(13.6.18)

5: .

Napier :

(90)= =.

90

(13.6.19)

527

13.6.6

, =90, B=4215 =58 2112.


13.6.
.
21,2
15
B= 4215= 42+ =42,25 =5821+ 12 =5821,2 =58+
=58,353.
60
60
60
.
(13.6.16) :

1
1
1

0,6786, 47,2657.

42,25 58,353
0,9083 1,6225 1,4737

.
(13.6.17) :

58,363 0,5247

0,7803, 38,7119.
0,6724
42,25

.
(13.6.18) :

42,25 0,7402

0,8695, 29,5994.

58,353
0,8513

:
(13.6.19) , :
= 0,6786=0,78030,8695 0,6786=0,6785, .
( ).
13.6.7

, =90, =50 =150.


, , 13.6.
.
(13.6.16) :

1
1
1
1

1,4537 1,

50 150
1,1917 ( 0,5773) 0,6879

.
=50 =150.

6.
.
, =90,
0<, <180.

528

, , ,
(0, 90) , , (90, 180).
1:  (. 13.6),
, .

2 : .
.
Napier :

= (90 ) = =
.

90
90
90

. 13.6.

(13.6.20)

=1 (0, 90) =1801.

.
Napier :

(90 ) = (90 ) = =

=1 (0, 90) =1801.

.
Napier :

= (90 ) (90 ) = =

(13.6.21)

(13.6.22)


(13.6.22) : =1 (0, 90) =1801.
3:  13.5,
(, , ) .
, : =1 (0, 90) =1801 : =1 (0, 90) =1801.
, : (1,1), (1, 1801), (1801, 1)
(1801, 1801).
 (1, 1801) (1801, 1),
.

, (1,1) (1801, 1801).
 : =1 (0, 90), =1801,
:
(1,1,1), (1,1,1801), (1801, 1801, 1) (1801, 1801, 1801).
 (0, 90), (1, 1, 1)
(1801, 1801, 1801), , (0, 90), ,
(90, 180).

(90, 180), : (1801, 1801, 1), (1,1, 1801).

4 : .

Napier :

=(90)(90) =

(13.6.23)

529

13.6.8

, =90, =30 =80.


, < , , (0, 90).
> (=30, =80), =30 =80.
13.6.9

, =90, =45 =60. (


, (0, 90) <, ).
13.6.
.
(13.6.20) :
45
1
=
=
= 0,5773, =35,265

60
1,732
=18035,265=144,735.
.
(13.6.21) :
=

90
90

90

. 13.6.

60
0,5
=
= 0,7071, =45 =18045=135.

45
0,7071

.
(13.6.22) :
=

45 0,7071
=
= 0,8165, =54,7372 =18054,7372=125,2628.

60
0,8660

:
) =45, =35,265, , (0, 90) <.
= 45, ,
, =54,7372. ,
: =54,7372, =35,265, =45.
) =135, =144,735, , (90, 180) >.
=45, , , =125,2628.
: =125,2628, =144,735, =135.
:
(13.6.23) =.
:
) 35,265=54,7372450,5391=0,81650,70710,5391 0,5773 ( ).
) 144,735=125,26281353,6023=0,81650,7071.
.

530

13.7 .
90 .
,
=90,
=180=18090=90 ( 1).
, , ,
, 13.6.2,
.

13.7.1  , =90, ,
0<, <180.
.

, , : +>.
.
:

=180=18090=90, =180=180=1, =180=180=1.

, (. 13.6). [ , , (.
13.6.2, 3)]. :
.
Napier :

(90)= =.
.
Napier :
= (90 ) =
1

=
= =
.

.
Napier :

= (90 ) =
1

=
= =
.

.
Napier :

(13.7.1)

(90)=(90)(90) =.

(13.7.2)

(13.7.3)

(13.7.4)

: =90, , (
), =180, =180, =180.

531

13.7.1

, =90, =112 =48.


13.6.
.
:
=180=18090=90.
=180=180112=68.
=180=18048=132.
, , =90 =68 =132,
13.6.9. :
: =104,513.
: =73,288.
: =129,857.
, :
=90, =112, =48 ( )
=180=180104,513=75,487.
=180=18073,288=106,712.
=180=180129,857=50,143.

13.8 .
, .
.
,
, .
, . , , ,
13.6.
.
.

1.
.

(. 13.9).
, ,
, 0<, , <180.

1 .
,
, . x -

1 2

. 13.8.

532

z (. 13.8).
, .
:
) (. 13.8).
, =90 (. 13.8)
.
z.
Napier :

(90)=(90)z =z z=.

(13.8.1)

x.
Napier :
x=(90) (90) x=.

(13.8.2)

, (13.8.2) x.
xA.
1.
Napier :

(90 ) = (90 ) (90 1 ) = 1 1 =

1
. (13.8.3)

.
Napier :
x=z(901) x=z1.
, =90 (. 13.8) x z.
2.
Napier :
x = ( z) (90 2 ) x = ( z ) 2 2 =


.
Napier :

( z)
.
x

(90)=x(z) =x(z).

(13.8.4)

(13.8.5)

B.
Napier :

= ( z) = x (90 ) ( z ) = x =

90
90

90 2

2
x

x
.
( z)

90
90

. 13.8.

. 13.8.

(13.8.6)

533

.
Napier :
(90)=(90)(902) =2.
, ,
= 1+2.
) (. 13.8).
, =90 (. 13.8)
z, x 1, .
, , =90 (. 13.8) x z , 2 1.
, : , =180 1
=12.
2
, (. 13.8).

1 2 x

90 2

90

90 1
90
90

. 13.8.

. 13.8.

90 1

1
z

. 13.8.

.
:

=+.

. 13.8.

(13.8.7)

B.
, :

= + =

(13.8.8)

.
:


= + =
. (13.8.9)

3 .
( 13.2.30 13.2.32), .
.

=( )+

(13.8.10)

= x , , :

log +log+log=logx

(13.8.11)

534

(13.8.11) logx x.
( 13.2.4),
( 13.2.27 13.2.29).
13.8.1

=100, =38 =40.


, , (. 13.8).
z.
(13.8.1) :
z= z=4038 z=0,76600,7813 z=0,5985, z=30,1.
x.
(13.8.2) :
x= x=4038 x=0,64280,6157 x=0,3958.
x=23,32 x=156,68.

xA. x=23,32.
1.
(13.8.3) :
1

1
1

1
1
38 40

1
1
0, 788 0,839

1,2104, 1

50,44 .

Napier :
x=z(901) x=z1.
, , =90 (. 13.8) x=23,32 z=10030,1=69,9.
2.
(13.8.4) :
2

( z)
69,9
2
2 6,91, 2 81,77.
x
23,32

.
(13.8.5) :
=x(z) =23,3269,9 =0,920,34
=0,31, = 71,94.
B.
(13.8.6) :

0,43
x
23,32


0,46, 24,70.
0,94
( z)
69,9

535

.
Napier : =2.
, =71,94, =24,7
=1 + 2=50,44+81,77=132,21.

2.
.
, , 0<, , <180.
1 .
, . :
=180, =180 =180
1, .
2
, (. 13.8).
.
:

(13.8.12)

=+
.
, :

= + =

+
.

.
:
+

= + =
.

(13.8.13)

(13.8.14)

3.
.
(. 13.8), , ,
0<, , <180.
, , ,
, 1.

. 13.8.

. 13.8.

536

4.
.
(. 13.8), ,
0<, , < 180.
,
. :


=180


=180


=180

3.

5.

. 13.8.

.
, , ,
0<, , <180.
,
. :
.
= + =

(13.8.15)

(13.8.15) :
B.

(13.8.16)

(13.8.17)

6.

.
(. 13.8), , , 0 <, , <180.
1 .
,
. =180, =180, =180 5.
2 .
, .
.
+
. (13.8.18)


(13.8.18) :
= + =

+
.

(13.8.19)

537

.
=

+
.

(13.8.20)

13.9 .
, .

, . ,
,
( ).

13.9.1 .
, .

(rhumbline) (loxodrome) (. 13.9). ,
,
(. 13.9). .

40o

12

20

140 o

20o

0o
100 o

80

60

40

()

()

. 13.9.

,
.

13.9.2 .
. , , .
180 ,
.

538

,
. ,
, ,
, .
. . , ,
, 13.9.
, .
.

, ,
, (. 13.9). (
).
,
, . , .
:
) , 90 (
).
) , 90 (
), , 90+, .
) D0 ,

( ,
: ,
).
:
) , .
) , ,
.
) , .
,

. 13.9.
. 13.9.

90o
80o
70o
60o
50o

40o

30o

o
40

, ,
. Do

(. 13.9), :
) 90.
) 90,
, 90+, .
) .
, ,
(. 13.8 1),
.

539

20o

10o
0o

o
o
10o 20o 30o 40 50

. 13.9.

(. 13.9):
) (latitude)
, (
). ,
0 90 0 90
( , 0) (North N) (South S)
.
(same names),
(contrary names).
: = 10 15 30 ( ) latitude=101530.
:
(orth) .
.
) (longitude)
( )
. , , 0180 0180 [
, Greenwich, 0 (East E) (West W)
]. (same names),
(contrary names).
: =0101530 ( ) longitude=0101530E.

13.9.1

N , ...
:416,338N, :7123,467W,
:38 37,204N, :913,394W.

540

, (. 13.9),
:
, 90=90416,338=9041,1056=48,8944.
, 90 =903837,204=9038,62=51,38.
, ==913,3947123,467=
=9,223271,3911=62,1679.
.
:
()=()()+()()()=
=48,894451,38+48,894451,3862,1679=
=0,657448940,62415236+0,7534991370,7813026490,466882154=
=0,410348308+0,274858599=0,685206907.
()=46,74811855=46,7481185560=2804,88.
, D0
... : D0=2804,88 ...

:
( ) ( ) ( ) ( ) ( )
( ) ( ) ( ) 51,38 48,8944 46,7481

( ) ( )
48,8944 46,7481

0,62415236 0,657448894 0,685207143 0,173663682

0,316436932,
0,753499137 0,728348248
0,548809776

71,5524181 71 33,145.

, : =71 33,145.
13.9.2

, (Cape of Good Hope)


:3425,61S, :1825,9090 E Buenos Aires
:363,7610S, :5530,2290W (. 13.9).
.

)
.
, :
, o 90=903425,61'=
= 9034,42683333=55,5732.

541

, 90 =90363,7610'=
= 9036,06268333=53,9374.
, =+=
=1825,9090'+5530,2290'=18,43181666+55,50381666=73,93563332. I
.
:
( ) ( ) ( ) ( ) ( ) ( )
55,57316667 53,93731667 55,57316667 73,93563332

. 13.9.

0,565353366 0,588669989 0,824848816 0,808373455 0,276717073


0,332806559 0,184511039 0,517317598
( ) 58,84750734 58,84750734 60 3530,85.

, D0
(Cape of Good Hope) Buenos Aires :
D0=3530,85 ..
.
:
( ) ( ) ( ) ( ) ( ) ( )
( )

( ) ( ) ( )

( ) ( )

53,93731667 58,84750734 55,57316667

58,84750734 55,57316667

0,588669989 0,517317598 0,565353366


0,855793493 0,824848816

0,588669989 0,292467245 0,296202744

0,419609915
0,70590025
0,70590025

( ) 65,2 .

, :
=180+()=180+65,2=245,2.
)
.
, :
, 90+ =90+3425,61'=90+34,42683333=
= 124,42683333.
, 90+=90+363,7610'=90+36,06268333=
= 126,06268333.

542

, =+=18 25,9090+55 30,2290=


=18,43181666+55,50381666=73,93563332.
.
:

( ) ( ) ( ) ( ) ( ) ( )
124,42683333 126,06268333 124,42683333 126,06268333 73,93563332
0,565353366 ( 0,588669989) 0,824848816 0,808373455 0,276717073
0,332806559 0,184511039 0,517317598
( ) 58,84750734 58,84750734 60 3530,85.
, D0
Buenos Aires :
D0=3530,85 ..

:
( ) ( ) ( ) ( ) ( ) ( )
( )

( ) ( ) ( )

( ) ( )

126,06268333 58,84750734 124,42683333

58,84750734 124,42683333

0,588669989 0,517317598 0,565353366


0,855793493 0,824848816

0,588669989 0,292467245 0,296202744

0,419609915
0,70590025
0,70590025

() 114,8.

, :
=360()=360114,8=245,2.
: ,
Buenos Aires,
,
.
Buenos Aires ,
, D0 ,
D0=3530,85 .. .

543

.
:
( ) ( ) ( ) ( ) ( ) ( )
( )

( ) ( ) ( )

( ) ( )

124,42683333 58,84750734 126,06268333

58,84750734 126,06268333

0,565353366 0,517317598 ( 0,588669989)


0,855793493 0,808373455

0,565353366 0,304529344 0,260824021

0,377021886,
0,691800743
0,691800743

( ) 112,15.

, Buenos Aires
=112,15.

13.9.3 .
,
, (.. ,
, ..). , . , .
, .
, , (. 13.9)
:
) , 1 . 1 1.
) , 090 270 1
2 .
) 2 .

( 1 2 )



( )

. 13.9.

544

13.9.4 .
, ,
.
.
(. 13.9) ,
() , () ().

,
(. 13.9).
Z

90 o

90

o
Z
90
180o

. 13.9.

:
) , ( ),
, ,
90.
) ( ), (
) . P =90+ ( )
=90 ( ), (
, , ).
) ( ):
. , =90, .
:
) , .
) ,
.
) ,
, , HA .
, , . ,
:
) : (), () ()
() ().
) : (), () ()

545

() ().
) : (), ()
() () .

13.10 .

13.2

Napier
.

Napier

,
, .
.
()  ,
, .
()  ,
(, ).




.
) .
) .
) .

) .

) ( ).
) .

,
.

) .
) .
)  .
)  .

) .
) .

13.1

180 .

546

, , ,
.

, , , .

,
.

,
, ,
, .

,
() , ()
().
,
.

13.11 .
13.11.1  : =120, =30 =150,
, .
13.11.2  : = 45, =135 =30,
.
13.11.3  : =60 ==90,
.
13.11.4  , , , ,
, :

=
=
.

13.11.5  =, : = =180 =
=180.
13.11.6  : =+
.
13.11.7 
,
90 90 .

547

13.11.8
 , =90, :

) =1 =1 ) =0,001 =127.
13.11.9 , =90, =115 =64.
13.11.10 , =90, :

) =78 =29 ) =57 =110.
13.11.11 , =90, =60 =75.
13.11.12 , =90, :

) =139 =118 ) =65 =72.
13.11.13 , =90, :
) =121 =105,

) =53 =141,
) =56 =141.

) =41 =141,


) =149 =64 ,
13.11.14 , =49,3, =128,23 =74,2.
13.11.15 , =98, =122 =47.
13.11.16 , :

) =67,5 =108,25 =47,33,
) =33 =60 =110.


) =103 =62 =53 ,
13.11.17 , :

) =60 =45 =160, ) =132 =130 =127.

) =60 =45 =60,
13.11. 18  : =120, =30 =150,
, .
13.11.19  52S, 50,
23S. , .
13.11.20  (:3838, :914W) o 35. ,
400 ...
13.11.21.  ( , , , , )
:

) =60, =30, =120 ) =60, =135, =45.

13.11.22  = 1 22 ,
2

2


2
.
= 2
+
2
2
2

13.11.23 (=90), :

) 2 =(+)+() ) 2=2+222.
13.11.24  = =2 =
=2 .

, 90
45.
13.11.25 , =90, :

) =135 =52 ) =94 =72.

548

13.11.26 , =94, =102 =141.


13.11.27 , =56, =48 =112.
13.11.28 , :

) =96 =68 =87 ) =96 =84 =125.
13.11.29
 (: 38 38, : 9
14) (Perth) (:3206S, :11535).


( )

14:

lx im
x0

14

, ... ,
, , ...
(),
( ) . ,
, status,
. ,
2238 .. Yao, , .

, , .
, , ( ),
.


. , , .

14.1 .
14.2 
.
14.3 .

14.4 .
14.5 .

lx im

14.6 .
14.7 .
14.8 .

x0

14.9 .

552

14.1 .
, .
R.A. Fisher (1890-1962), :
:
) .
) .
) .
,
. H ,
,
. ,
, , .
,
' .
,
.
(, ..)
, .
. ( ), .
:
) , ,
,
. .
) GPS
, GPS .
GPS ( , GPS
).
) ( ) ( : , ,
), .
.

, .
(, ) . X,Y,Z,....
() .

553

, :
) , , .
18, 19, 20, .
) GPS [0,+).
) W 1, 2, V ,
, .
, : .
, (
)
. - . , W
. ,
( , , , 0), ( , ),
, ...
,
(.. , , ) , ()
(.. ,
, , , ,
).
, ,
(, , ..).
(.. 1, 2, 3, 4 1, 10, 100 , 2, 1, 0, 1, 2,) . , Y V () () .
, ,
( 1, 2,, 6), ,
...
,
, , , .
(), , ...
, .. (sec), (hour), (m),
(kg) ...
,
. ,
, .
, ,
, , , .
()

554

. .
, ,

. ()
10 ,
, , . ,
, , ( ),
( ) ... ,
,
() (),
. , (Eurostat)

.
( ) , , ' . ,
,
. , , ,
, . ,

. , ,

.
, , , ,
. ,
.
,
, ,

. , , ,
. ,
,
.
.

14.1.1. ;
) .

555

) .
) .
) .
) .
) .
) .
) .
) .
) .
) .
14.1.2.  14.1.1 ,
.
14.1.3.  200 . 20
, ;
14.1.4.  ,
. , ( )
.
) , 300 .
) , 30 .
) .
) .
14.1.5.  ,
. ,

' . ,
, 4
200 .
) , , .
) .
)  .
14.1.6.  , . :
) ;
) .
) .
) .
) .
)

;
) ;

, ( ) .

556

14.2 .
( ,

), ,
,
. ,
.
, 14.2.1
1970
(, , ).
,
, 20
14.2.2.
14.2.2 (, ) =20 . , ,
20. , , x1, x2,...,x
t1,t2,..., t, xi. ,
, :
x1= , x2= ,
x3= , x4= ,
x5= ,, x20= ,
=5,
t1= , t2= ,
t3= , t4= ,
t5= .

14.2.1

, ( ) xi, i = 1, 2,..., 20
1, 2, 1, 10, 1, 2, 3, 2, 3, 2, 2, 2, 4, 10, 1, 6, 2, 6, 6, = 6

t1 1, t 2

2, t3

3, t 4

4, t5

6, t6

269

1.278

11.360

12.907

10.

t1, t2 ,..., t ,
v. ti ti .
ti.
, :

1 +2 +...+ = .

(14.2.1)

,
(.. | ).
, 14.2.3 14.2.4.

557

14.2.2
20 .

( )

2000

5000

800

4000

10

1800

2450

2500

950

2100

10

3500

11

1350

12

1500

13

3100

14

2800

10

15

1700

16

1600

17

1950

18

2050

19

3050

20

2150

14.2.3
: .
ti


fi


fi %

|||| |||

0,40

40

||

0,10

10

0,05

|||

0,15

15

|||| |

0,30

30

20

1,00

100

558

14.2.4
: .
ti


fi


fi %

||||

0,20

20

|||| |||

0,40

40

||

0,10

10

0,05

10

|||

0,15

15

10

||

0,10

10

20

1,00

100

14.2.3 14.2.4 vi , fi ti, :


fi

i
,

i 1, 2,..., .

(14.2.2)

, fi , fi% (.
14.2.3 14.2.4). :
) 0fi1 i=1, 2,..., 0i.
) f1+f2+...+f=1.
0i, i=1, 2,..., , :
f1 + f2 + ... + f =

+ + ... +
1 2
+ + ... + = 1 2
= =1.

ti , i , fi ( 14.2.3
14.2.4) .
(ti , i) (ti , fi),
.
, i fi
Ni
Fi, ( )
ti .
t1,t2,...,t, , ti i=1+2+...+i.
, Fi=f1+f2+...+fi, i= 1, 2,..., .
:
1=N1, 2 = 2 1,..., = 1

f1=F1, f2 = F2 F1,..., f = F F1,

559

( )
.
14.2.5 14.2.2.

14.2.5
: M .
ti


fi


Ni


Fi

0,25

0,25

0,40

13

0,65

0,10

15

0,75

0,05

16

0,80

0,15

19

0,95

10

0,05

20

1,00

20

1,00

( )
.
( ) , ( ), .
,
. ,
. ,
, ,
,
.

14.2.6
.
Valid

Frequency

Percent

Cumulative percent

20,0

20,0

40,0

60,0

10,0

70,0

5,0

75,0

15,0

90,0

10

10,0

100,0

Total

20

100,0

560


, . 14.2.6
: .
14.2.1.

14.2.7 .
14.2.7
ti


fi


Ni

0,15

16

1
2
3
4
5


Fi

0,45
18

) .
) :
3,
4,
2 4 ( ).
.

) ( ) :
f2 =

v2
6
6
0,15 = v =
= 40 , N 2 = v1 + v2 16 = v1 + 6 v1 = 10 ,
v
v
0,15
v 10
= 0, 25 ,
N1 = v1 = 10 , f1 = F1 = 1 =
v 40

F3 = f1 + f2 + f3 = F1 + f2 + f3 0, 45 = 0, 25 + 0,15 + f3 f3 = 0, 05 ,
f3 =

v3
v
0, 05 = 3 v3 = 2 ,
v
40

v = v1 + v2 + v3 + v4 + v5 40 = 10 + 6 + 2 + v4 + 18 v4 = 4 .
14.2.8.
) :
f1 + f2 + f3 = F3 = 45% .
f4 + f5 = 0,10 + 0, 45 = 55% 1 F3 = 1 0, 45 = 55% .
f2 + f3 + f4 = 0,15 + 0, 05 + 0,10 = 0, 30 = 30% .

561

14.2.8
ti


fi


Ni


Fi

10

0,25

10

0,25

0,15

16

0,40

0,05

18

0,45

0,10

22

0,55

18

0,45

40

1,00

40

1,00


. ,
,
. ,
.
, .
. ,
, .
, ,
, .

. ,
, ,
, ,
.
. 14.2 : E
14.2.1.
, , (. 14.2). ' , ,
ti
. vi
fi, . (ti, vi) (ti , fi) ,
.
(piechart) ,
. , -

562
40

(%)

20

10

30

()

()

. 14.2.
() () :
14.2.2.
8

8
7
6

5
4
3

2
0

1
0

10

12


()

10


()

. 14.2.
() ()
: 14.2.2.

, ( , )
i fi. ,
i ti
i fi, :
i = i

360o
= 360o fi i=1,2,...,.

563

,
, .
14.2
14.2.2.

, , .
, . 14.2 ( )
. , 2003
, 2005 2006
.

6
10

. 14.2.

14.2.2.
1000

900

800

700

600

500

400
2000

2002

2004

. 14.2.

2006

2008

2010

564

14.2
'
A .
)

.
)

.

14.2.2.

16
14
12
8
6
2
0

10

12

14

16 17 18

. 14.2.

) ,
vi , i = 1, 2,... ( = 6) (
)
14.2.9.
14.2.9
ti


fi


Ni


Fi

10

16

0,275862

16

0,275862

12

14

0,241379

30

0,517241

14

0,103448

36

0,62069

16

12

0,206897

48

0,827586

17

0,137931

56

0,965517

18

0,034483

58

58

i
, Ni = v1 + v2 +...+vi, Fi = f1 + f2 +...+fi

i=1, 2,..., 6.
) ,

14.2.
fi =

58
56
48
30
16
10

12

14

16 17 18

. 14.2.

14.2.1.  14.2.10 , , .

565

120
.
)  ,
.
)  2, 5, 8 10
.
)  2, 5, 8 10 .
)  2 9 .

14.2.10

1
2
3
4
5
6
7
8
9
10
11
12

4
7
9
15
16
20
24
10
5
5
3
2

14.2.2.
 14.2.11 .
)  .
)  ,
4.
)  , 4
6.
14.2.11
ti


Ni


Fi

0,10

20

5
6


fi

20

0,85
18

14.2.3.  20 '
:
2 3 5 2 4 2 2 3 4 4
2 3 5 4 4 4 4 3 6 2
)
 , () , ,
.

566

) 3 .
) 4 .
14.2.4. 40 :
1

)

.
) 3.
) 2.
14.2.5.
 30
:
21 25 26 22 26 24 22 26 24 27
22 26 21 25 24 25 25 25 24 25
24 22 24 23 27 27 23 24 25 24.
)
 30 .
)  30 .
) 
30 .
14.2.6. :
15

18

25

20

10

15

80

. 14.2.

60

100

120

 14.2.7.
14.2 180
, .
)  .
)  .

)
 .
)
 ' .

567

) .
14.2.8.
 ' . 40 14.2.12.
)  (
).
) .
) .
14.2.12

21

10

40

14.2.9.
 14.2.13 ( )
2000 2007.
14.2.13

2000

2001

2002

2003

2004

2005

2006

2007

560

550

610

8200

1220

1500

1950

2100

' .
14.2.10.
 400
:
:
:
:
:
.
25% .
18.
.
) .
) .

568

14.2.11.  20002010
14.2.14.
) .
) .
)

.
14.2.14

2000

180

720

2001

126

526

2002

256

564

2003

366

362

2004

278

444

2005

412

272

2006

492

264

2007

514

536

2008

686

608

2009

602

232

2010

522

280

14.3 .

, .
, ,
,
. ,
14.2.2, 20
1, 5%
14.3
.
4%
,
3%
,
2%

1%
, .
0%
1000
2000
3000
4000
5000
0


.
. 14.3

569

,
() ,
, .
.
, :
) ,
. , , (0,5 1,5 2,5 ..).
) ( ),
( ), [,)
( ), ( ), (,].
,
. ()
, .

:
1.  .
2.  ,
.
3.  () .
4.  ()
.
5. 
.
6.  ( ).
, 14.3.1, 40 .
14.3.1
40 .
252

255

263

276

265

276

275

279

265

256

264

277

285

277

277

281

277

256

265

277

241

289

265

265

247

258

267

287

265

253

277

292

300

262

270

264

272

273

278

277

min{x1,x2,...,x}
max{x1,x2,...,x}. 14.3.1
300 241 1.
2, (Range - R)

570

.
R = 300 241 = 59.
3, Sturges,
:
= 1 + 3,32 log = 1 + 1,44 ln
. ,
. 14.3.1 =40, :
= 1 + 1,44 ln40 6,31
= 6 .
4 () , .
. , c
R , :
R
c= .

( ),
,
. 14.3.1 = 6 R = 59,
c=

R 59
=
99,83
.83
6

, c=10 (
).
, () . ( )
c, .
c ,
.
240 c=10
, 240 240+10 = 250.
,
() .
:
240, 250, 260, 270, 280, 290, 300.
( )
( ), ( , ],
14.3.2 (5 6). i
i . , i = 1, 2,..., i .
-

571

14.3.2
14.3.1.

1,4

fi %

Ni

Fi %

240 2 50

245

0,8

250 2 60

255

15

20

0,6

260 2 70

265

12

30

20

50

270 2 80

275

14

35

34

85

280 2 90

285

10

38

95

290 3 00

295

40

100

40

100

. '

, ,
,
. 14.3

14.3.1, 14.3.2.

, ,
,
.

,

(. 14.3).
,
, (frequency polygon).


, . 14.3

14.3.1.
, .


3=12

1,2
1

0,4
0,2
0
240 250 260 270 280 290 300

. 14.3.

0,035


f3=0,30

0,030
0,025
0,020
0,015
0,010
0,005
0
240 250 260 270 280 290 300

. 14.3.

.
1,4
1,2
1
0,8
0,6
0,4
0,2
0
240 250 260 270 280 290 300

. 14.3.

.

572

, 1
(100%).
, ,
,
( . ,
). 14.3
14.3.1, () .
( )
, .

, .
, , . 14.3

14.3.1. , '
14

12
10
8
,
6
. ,
4

.
2
0

240 250 260 270 280 290 300

-- (stem
and leaf plots). . 14.3.

.
' ,
Fi 1
0,9
.
0,8
--
0,7
0,6

0,5
,
0,4
0,3
.
0,2
0,1

0
.
240 250 260 174 180 186 192


:
. 14.3.
) .

.
) / , -

573

,
.
,
. 25 (
) :
39 45 50 41 46

47 40 47 59 59

50 51 34 35 35

42 43 27 32 34

23 38 39 41 41


:
23 27 32 34 34 35 35 38 39 39 40 41
41 41 42 43 45 46 47 47 50 50 51 59 59
:
)
) , .
, ( 23)
(stem: ) 2 (leaf: ) 3.
:
Stem ( = 10) Leaf ( = 1)
2

, ( 59) :
Stem ( = 10)

Leaf ( = 1)


( ) .
:
Stem ( = 10)

Leaf ( = 1)

3 7

2 4 4 5 5 8 9 9

0 1 1 1 2 3 5 6 7 7

0 0 1 9 9

574


. , 1234 ,
, 1234 1234 1234.
,
,
() 2, 8, 10 5 () .

[20, 30), [30, 40), [40, 50), [50, 60).
.

14.3.1.
 50 .
104

109

126

152

130

152

150

158

143

143

130

112

128

154

176

154

154

162

154

112

154

112

125

154

82

178

129

128

201

135

94

116

134

174

127

106

154

184

132

165

205

124

140

128

144

146

156

154

219

217

) 5 .
)  , .
)  ,
.
14.3.2.
 ( ) 50 ( )
:
39

137

72

19

18

43

44

34

44

32

49

41

19

29

41

36

80

74

132

44

38

102

25

49

38

92

44

121

95

39

19

40

33

109

124

128

34

33

46

64

36

92

33

43

135

35

69

37

75

87

) .
)
 , .
)
 .
14.3.3.
 400 ' .
[ , )

20 30

3040

4050

5060

6070

7080

20

60

200

70

40

10

575

)  ,
.
)  .
)  , .
)  40
25 55 ;
14.3.4.  20
01000
724

738

647

583

883

999

829

803

782

755

811

756

756

953

738

638

738

592

988

811

) .
)
 100,
600 .
) .
14.3.5.  14.3.3 160 ,
180 , .
)
 .
)
 ,
.
14.3.3
i


( mm Hg)

9599

100104

18

12

105109

16

14

110114

24

20

115119

32

32

120124

20

26

125129

14

24

130134

12

22

135139

12

140144

145149

160

180

14.3.6.  () -

576

1995 :
) 
1995.
) 
, , 1995 .

5054

10

5559

10

6064

17

21

6569

36

57

7074

44

61

7579

73

109

8084

117

162

8589

123

195

14.3.7. 14.3.4 .
) .
) .
) .
14.3.4.

[)

fi


Fi %

15

20

59

50

913

85

1317

95

1721

2
1

 100 .
14.3.8.
14.3.5.
) 15 ;
)  , 35 ,
12,5 ;
)  , 35

577

, ;
14.3.5


fi %

[ )
0 5

10

5 10

15

10 15

12

15 20

15

20 25

18

25 30

18

30 35

12

14.4 .
, () . ,
,
.
14.2 14.3.
, . ,
,
, .
,
.
.
, .
.

14.4.1 ( x ) .

.

.
, x1, x2,..., x () , x :

x=

x1 + x2 + ... + x
=

xi
i =1

1
xi .
i =1

578

xi

i =1

x1+x2+...+x.

xi , ,

, .
, ( 40
) 14.3.1
x=

x1 + x2 + ... + x40 252 + 255 + 263 + ... + 277 10800


=
=
= 270 .
40
40
40

,
40 , , 270 ( ).
,
:

t11  t2 2  ...  t
1  2  ... 

ti i
i 1

1
ti i
i1

i 1

x = ti
i =1

i
= ti fi ,
i =1

t1,t2,...,t 1,2,..., f1,f2,...,f. t1,t2,...,t, x


ti i fi .
, .

14.2.2 :
x=

x1 + x2 + ... + x20 1 + 2 + 1 + 10 + ... + 6 + 2 68


=
=
= 3, 4
20
20
20

, 14.2.4, :
x

t11  t2 2  ...  t6 6
1  2  ...  6

1 4  2 8  3 2  4 1  6 3  10 2
20

68
20

3, 4

( ).

i
x
=
t
=

i ti fi
i =1
i =1
, ti .
, 14.3.2,

579

14.3.1 :
x

t11  ...  t6 6
1  ...  6

245 2  255 6  265 12  275 14  285 4  195 2


40

10780
40

269, 5 .

14.4.1, .
14.4.1
i

ti

tii

245

490

255

1530

265

12

3180

275

14

3850

285

1140

295

590

= 40

tii = 10780


( x =270) .
,

ti.
( ).
:
1.  , x1 = x2 = ...
= x = c, c.
2. 
.
3. 
, :

( x1 x ) + ( x2 x ) + ... + ( x x ) = ( xi x ) = 0 .
i =1

4. 
, c,
c, x1,x2,...,x x ,
y1 = x1 + c, y2 = x2 + c,..., y = x + c y = x + c.

580

5.  , , x1,x2,...,x
x , o y1=cx1, y2=cx2, , y=cx
y = cx.
6.  n , m1
x1, m2
x2 ... mn
x n . , m=m1+m2+...+mn,
:
x

m1 x1  m2 x2  ...  mn x n
m1  m2  ...  mn

1 n
mi xi . 
mi 1

14.4.2 () .
o
, 50% 50% .
() ,
,
() , .
:
a)  , .
b)

1
2 , ,

,
.
2

g)  () , 0, , 0 0+1 , .
, = 7 :
90
55
50
75
:

57

80

60,

50
55
57
75
60
:
+1 7 +1 8
0 =
=
= =4
2
2
2

80

90

581

. = 60.
= 8
90
55
50
75
57
:
50

55

60

57

75

80

60

95,

80

90

95,

4 5 ,
8
0 = = = 4 . :
2 2
60 + 75
= 67, 5.
2
(67,5)
.
,
() , .
,
. ,
95 9500, = 8 , 70,25 (
) 1184!
, 50%
50% ,
pa, a=1,2,...,99 %
pa (100 a)% .
p25, p50, p75 , 4 , Q1, Q2, Q3, (. 14.4).
=

50%

Q1

min
25%

25%

Q3
25%

max
25%

. 14.4.

Q2= p50=. , Q1 Q3 ,
, .
= 40 14.3.1,
' .
:
241, 247, 252, 253, 255, 256, 256, 258, 262, 263,
264, 264, 265, 265, 265, 265, 265, 265, 267, 270,
272, 273, 275, 276, 276, 277, 277, 277, 277, 277,
277, 277, 278, 279, 281, 285, 287, 289, 292, 300.
,

582

20 21 , :
270 + 272
= Q2 = p50 =
= 271 .
2
, 20 :
241, 247, 252, 253, 255, 256, 256, 258, 262, 263,
264, 264, 265, 265, 265, 265, 265, 265, 267, 270
, 10 11 . :
263 + 264
Q1 = p25 =
= 263, 5 .
2
, , 20
272, 273, 275, 276, 276, 277, 277, 277, 277, 277,
277, 277, 278, 279, 281, 285, 287, 289, 292, 300

277 + 277
Q3 = p75 =
= 277 .
2
p10 p90
10% 90% v = 40 , 0,0140=4 0,0940=36. p10
, 4 404=36
, 4 5
, :
253 + 255
= 254 .
p10 =
2
, p90 , 36
4036=4 , 36
37 , :
285 + 287
= 286 .
2
(. . 14.3),
,
. ,
, .
(
)
(
).
p90 =

:
)  .

583

)  , , , .
i Li ( )
, Fi Fi1
' ,
.
) () :
= Li +

0, 5 Fi 1
0, 5 Fi 1
c = Li +
c,
Fi Fi 1
fi

c , (
, c
).
Pa

a/100 0,5
. 14.4,
.
100
90
80
70
60
50
40
30
20

Fi%

10
0

x
p10 Q1

Q3

p90

. 14.4.

14.4.3 (0).
0 , ' .
:
) (, ,
)
) , , ,
.
14.2.2
8 ( 40%), 14.2.3.
14.2.2
t2=8 8 ( 40%), 14.2.4.

584

c :
)  .
)  , . i Li
( ) , fi
fi1 fi+1 .
)
 M0 () :
D
M0 = Li +
c,
D+E
D= fifi1 E= fifi+1.

14.4 .
,
.
B

fi

fi+1
fi_1

c
Li

M0

. 14.4.

14.4.1.

14.4.2 ( min)
100
. ,
.
25%
.
.


14.4.3. , :

14.4.2
i
1
2
3
4
5
6
7

ti
30
35
45
50
60
75
90

vi
6
14
20
36
10
12
2

585

t11  t2 2  ...  t7 7
1  2  ...  7

5050
100

50, 5.

=100 ,
50 51 .
, ,
50 + 50
= 50 min .
50, =
2
,
M0=50min ( 4=36).
25%
, Q1 . /4=100/4=25, 25 26 . ,
21 40 45, Q1=45.

14.4.3
i

ti

vi

ti vi

1
2
3
4
5
6
7

30
35
45
50
60
75
90

6 180
14 490
20 900
36 1800
10 600
12 900
2 180

100 5050

14.4.2.

14.3.2 ( ),
= 40 14.3.1,
.
.

M0 ,
(14), L4=270.
f4=14, f3=12, f5=4, c=10, :
D = fi fi 1 = f4 f3 = 14 12 = 2 , E = fi fi +1 = f4 f5 = 12 4 = 8
M0 = Li +
M0 = L4 +

D
c :
D+E

D
2
10 = 272 .
c = 270 +
2+8
D+E

14.4.1.  , ( )
:
) 4 0 2 1 4 3 2 3 3 4 4 111 5 4 7 2
) 20, 0, 7, 2, 7, 1, 90, 4
) 2, 3, 1, 5, 2, 7, 1, 2, 4, 1, 5, 9
14.4.2.
 ( ) 9 : 3, 5, ,
36, 6, 7, 4, 7, 8 x = 9 .
) 3 .
) .

586

) .
14.4.3.
 ( )
.
( 100 km) 9, 6, 7, 9, 9, 8, 8, 10, 7, 8, 12.
) .
) .
) 
, ( )
;
, ( ) ;
14.4.4.  15 , 8 7
. ( ) 8 :
1350,

1450,

1470,

1370,

1410,

1390,

1430,

1410,

7 :
1390,

1150,

1310,

1510,

1230,

1470,

1390

) 
.
)

.
)  .
14.4.5.
 , .
10 8.
14.4.6.
 10
18. :
) 17 20;
) , 16;
) , 16
18;
14.4.7.  40 , 36
. :
) 20 16 ;
) ;
)  ;
14.4.8.
 10 () 1200,
20 1500 30 .
45 1700,
.
14.4.9. :
5,

3,

6,

9,

8,

1,

17.

587

,
, :
) 7,
5,
8, 11, 10, 3, 19
) 10,
6, 12, 18, 16, 2, 34
) 17, 11, 20, 29, 26, 5, 53
14.4.10.
 10 ,
50, 25, 25, 65, 70, 15, 25, 95, 37, 33 ( ).
) , .
)  (),
, 19%;
)  3, ,
;
14.4.11.
 14.4.4 . :
) .
) .
)  .
5 ;
14.4.12. 14.4.5 .
) .
)  .
) .
14.4.4

( )

( )

020
2040
4060
6080
80100

8
12
17
6
1

14.4.5

xi

1
2
3

fi

Ni

xii

10
35
=50

14.5 .

. ,
,
() .
, , .
(R),
. , 14.3.1,

588

r=300241=59. ,
,
(
).
Q,
Q1 Q3, :
Q= Q3 Q1
[ (Q3 Q1)/2 , ,
Q ].
Q ,
, 50% . ,
,
. 14.3.1
14.4 Q1=263,5, Q3=277 Q= Q3Q1=277263,5=13,5.
, (),
x . , x1, x2,..., x
() ,
s 2X , , s2 1
1
s 2 = ( x i x )2 .
i =1
,

2
1 2 1
1 2
2
2
2
s = x i ( x ) s = xi xi .
i =1
v i =1
i =1

,
s2 :
s

1
( ti  x )2 i ,

i1

1
s = ti2 vi ( x )2 ,
v i =1

s = ( ti x ) fi ,
2

i =1

s =
2

i =1

ti2 fi

(x) ,
2

1
1
s = ti2 vi ti vi
v i =1
v i =1

s =
2

i =1

ti2 fi

ti fi ,
i =1

t1, t2,..., t 1,2,...,


f1, f2,..., f.
,

1.  , , 1 , s2 (
).
1.

589

14.5.1
i

t i

ti i

ti 2 i

245

490

120050

255

1530

390150

265

12

3180

842700

275

14

3850

1058750

285

1140

324900

295

590

174050

=40

tii = 10780

tii = 2910600

t1, t2,..., t .
, (
).
, 14.3.1 ( x = 269)
s2 =

1 40
(252 270)2 + (255 270)2 + ... + (277 270)2 6066
2
x
x
(

)
=
=
= 151, 65 .
i
40 i =1
40
40

14.3.2,
14.3.1, 14.5.1. :

1 6
1 6
s = ti2 vi ti vi
v i =1
40 i =1

107802 5390
1
= 134, 75 .
= 2910600
=
40
40
40

:
1.  , x1 = x2 = ...
= x = c, .
2. 
s2, :

( x1 x )2 + ( x2 x )2 + ... + ( x x )2 = ( xi x )2 = vs 2 .
i =1

3.  , c, , x1, x2,..., x s 2X , y1 = x1 + c, y2 = x2 + c,..., y = x + c sY2 = s 2X .

590

4  ,
, x1, x2,..., x
s 2X , y1 = cx1 , y2 = cx2 ..., y = cx
sY2 = c 2 s 2X .
5.  n , m1
s12 , m2 s22 , ...
mn sn2 . s2 ( m = m1 + m2 + ... + mm ) :
s2 =

m1s12 + m2 s22 + ... + mn sn2 1 n


= mi si2 .
m1 + m2 + ... + mn
m i =1

s2 ,
. , m, m2, sec, sec2
....
,
(
).
sX s, s = s 2 .
, 14.3.1 s = 151, 65 12, 3 ,
s = 134, 75 11, 6 , 14.3.2.

14.5.1.

14.4.1.
( min) 100
. , ,
.
.

90 30,
:
R=9030=60min.
Q = Q3 Q1
Q1 Q3. 14.4.1
Q1=45. , 3/4=3100/4=75, Q3
75 76 .
, 31 76 50, Q3=50. :
Q = Q3 Q1 = 50 45 = 5.

591

1
1
s = ti2 vi ti vi
v i =1
v i =1

2
,s = s .

1 7 2
1 7
s =
ti vi ti vi
v i =1
100 i =1

ti 2 i

ti

ti i

30

180

5400

2 35
14
490
50502 17725
1
, 25
3 =45
20= 177900
=
272750
100 100
100
4 50
36 1800

17150

14.5.2 :
2

14.5.2

2
7
1
1 7
50502 17725
2
= 177, 25 .
ti vi ti vi =
272750
=
v i =1 100
100 100
i =1

2
, s = 177, 25
2
177, ,25
25==13
13, ,31
31. .
ss== ss2 == 177

40500
90000

60

10

600

36000

75

12

900

67500

90

180

16200

100

5050

272750

14.5.1.
 , , 14.4.2.
() ;
14.5.2. :
10, 30, 50, 60, 70, 90, 110
10, 59, 59, 60, 61, 61, 110
10, 11, 22, 60, 98, 109, 110.
) .
) 
.
) ;
) .
) ;
14.5.3.
 , 0
1.
.
14.5.2. ;
14.5.4.  , ,
14.4.7. , , , ,
() ().
14.5.5.
 , ,

592

14.4.14.
5 ;
14.5.6.
 , ,
14.4.10.
cm ( m);
14.5.7.
 ,
, , 5, 7, 8 6 2.
14.5.8.
 ( )
14.5.3.
) .
)  14.5.3
.
) 

i
)  [ )
02
5
.
24
15
14.5.9.
 14.5.4
46
20
50
68
35
. 810
40
:
) .
)
 .
14.5.4
)  .

)  ,
[ )

i
25% .
8
165
162168
)
 ,
75% .
18
171
168174
)
 ,
20
177
174180
90% .
7
183
180186
) 
3
189
186192
.
) 
14.5.5
.
) 

.
xi
i
14.5.10.
 50 0
4
,
1
16
14.5.5 .
2
20
) 
3
?
50
4
?
1,8.

50
) 

593

50 .
) .
) .
)

50 .
)  50 .
) 
(. 14.5.5).

14.6 .

(), ,
. :
)
.
) , ,
.
) ,
, .
) .
) .
.
,

.

. 1885
Galton (1822-1911) Regression Towards Mediocrity in Hereditary Stature ,
, .
Galton,
.
:
. , ,
(.. , ,
' ) ,
(.. , ). , , (..
, - , ).

594

, ,
14.6.1

,
X
.

=10 .

,
i
xi
yi
X Y,
1
4
225
X.
2
2
156
.
3
10
390
14.6.1 ( 4
14
516
) =10
5
6
267
( )
6
8
330
.
7
10
411
(xi,yi)
8
4
213
,
9
12
450
14.6 .

10
10
402

. , (xi,yi), i = 1, 2,..., 10
, X Y ( ). X
Y, ( ) Y X.
14.6 14.6.1
10 .
, , ,
, . (
) .
500

500

400

400

Y
300

300

200

200

2,5

7,5

10

12,5

X
. 14.6.

14.6.1.

2,5

7,5

10

12,5

. 14.6.

14.6.1 .

595

yi

y = + x.

y=+x

yy, 2
( ) .

1

,
xi
x
x1 x3
. 14.6.
()

X Y. i .

, '
.

, 1805
Legendre, (1752-1833) Gauss, (1777-1855).
' , , ,

i=yixi
(xi,yi) y = + x (. 14.6). ,

12  22  ...  2

AT

i 1

i 1

i2 ( yi   xi )2 . 

(14.6.1)

, (14.6.1),  E  . 1 :

i 1

Q xi yi  xi yi
i 1
2
Q
Q

Q xi2  xi
i 1
i1
1


, D

y
Q
i 1

1
 E x i . 
Q

(14.6.2)

i 1

y D  E x  ()
.

1
1
x = xi ,
y = yi
i =1
i =1
x1,x2,...,x y1,y2,...,y ,

1.  , .

596

y  E x .

D
,

y  E x  E x y  y

y  E x 

E ( x  x ),

y D  E x 
( x , y ) .
14.6.1 14.6.2 (14.6.2) :

Q xi yi  xi yi
i

1
1
3360  30 80
10
10

10 30960  80 3360
10 776  80 2

i 1
2

2
Q xi  xi
i 1
i1

i 1

1
yi  E xi
Q

i 1

i 1

30 

96 .

14.6.2
i

xi

yi

xi 2

xi yi

225

16

900

156

312

10

390

100

3900

14

516

196

7224

267

36

1602

330

64

2640

10

411

100

4110

213

16

852

12

450

144

5400

10

10

402

100

4020

xi = 80

yi = 3360

xi = 776

xi yi = 30960

E = 30 = 96
:

y D  E x

96  30 x 

(. 14.6). ' , ( -

597

) (x=0)
:
y 96  30 0 96 .
, y  x 
(
, yy), (, )
x=0. , = 0,
.

E  ,
E  y  x ,
x1 x2=x1+1 . :

y 2  y1
y 2

y=96,00+30,00*x

500

400

y
300

200

2,5

7,5

10

12,5

. 14.6.

i .

(D  E x2 )  (D  E x1 ) D  E ( x1  1)  (D  E x1 )

E .

y1  E  :

E 
. , x ,
y  E  E > 0 E 
E < 0.
, y D  E x 96  30 x 
14.6.1,
1 (1.000.000 ),
30 (30.000 ).

(coefficient of determination) R2, :
Q

R2

1

Hi2
Q

i 1

(y
i 1

 y)

1

(y
i 1

(y
i 1

 y i ) 2
 y)

,

y i

D  E xii, Hi

yi  y i  i=1,2,...,.

0 1 (0R21) . R2 1
, 0
( ).

598

14.6.3
i

xi

yi

1
2
3
4
5
6
7
8
9
10

4
2
10
14
6
8
10
4
12
10

225
156
390
516
267
330
411
213
450
402

yi y

Hi

yi  y i  Hi2

( yi  y i ) 2 

( yi y ) 2

y i 

111
180
54
180
69
6
75
123
114
66

12321
32400
2916
32400
4761
36
5625
15129
12996
4356

216
156
396
516
276
336
396
216
456
396

9
0
6
0
9
6
15
3
6
6

81
0
36
0
81
36
225
9
36
36

122940

3360

540

14.6.3.
14.6.1 :
Q

R2

1

H
Q

i 1

(y
i 1

2
i

 y)

1

540
3360

0,9956. 

(99,6%)

.

14.6.1.

(xi,yi) i=1,2 ... . y = x


,
.
) y = x.
) E  .
i
xi
yi
.

1
30
75

2
20
52

3
60
120

4
80
170

5
40
86

6
50
110

7
70
153

8
90
194

) E 

599

i2

12  22  ...  2

AT

( yi  xi )2 .

i 1

i 1

,
v

f ( )

i2
i 1

( yi  xi )2

( y1  x1 )2  ( y2  x2 )2  ...  ( yv  xv )2

i 1

, 4, f()
f '() = 0.

f ( ) ( y1  x1 )2  ( y2  x2 )2  ...  ( yv  xv )2
( y12  y22  ...  yv2 )  2 ( x1 y1  x2 y2  ...  xv yv )  2 ( x12  x22  ...  xv2 )
:

f c( ) 2( x1 y1  x2 y2  ...  xv yv )  2 ( x12  x22  ...  xv2 )

,
v

i 1

i 1

f c( ) 2 xi yi  2 xi2

i 1

i 1

2( xi2  xi yi ) .

f '() = 0 :
v

i =1

xi2

x i yi ) = 0 =
i =1

x i yi
i =1
v

i =1

.
xi2

, E 
n

xxi yyi

i =1
i

i 1
n

ii=11

.

2
xxi2

) 14.6.4.
14.6.4
i

xi

30

20

60

80

40

50

70

90

yi

75

52

120

170

86

110

153

194

xi2

900

400

3600

6400

1600

2500

4900

8100

28400

xiyi

2250

1040

7200

13600

3440

5500

10710

17460

61200

61200
28400

2,155  y

E x

2,155 x .

600

14.6.5

14.6.1.
 14.6.5
,
1500 gr ().
)  .
)  ( ) ,
.
)  ( ) ,
.
) 
;
)

().
) 
(). ;

( cm)

( gr)

41

1450

40

1490

38

1400

38

1410

38

1380

32

1100

33

1150

38

1420

30

1000

34

1150

32

1100

39

1450

14.6.2.  14.6.5
, .
)  ,
.
)  ( ) ,
.
)  ( ) ,
.
) 
()
' ().
)
 ,
;
)  40 cm;
40 cm;

38

1400

39

1450

37

1350

39

1450

38

1380

42

1490

39

1480

38

1450

 ,
14.6.3.
.
14.6.6
( )
10 .
)  ( , )

;

14.6.6

60

230

40

161

120

365

160

515

80

263

100

335

100

335

140

464

180

587

70

245

601

) .
) 
.
) 
.
, ' .
)  x, .
) .
) 
85 ;
14.6.4. 14.6.3.
)  14.6.1,
y = x ( ),
.
)
 14.6.3.
)

85 ;

14.7 .
O ti

i ti.
(i = 1, 2,..., ).
fi ti.

fi

i
, i i= 11,, 22,...,
,...,

) 0fi1 i =1, 2,...,


) f1+f2+...+f=1

i.

i=v1+v2+...+i i = 1, 2,...,

Fi.

Fi=f1+f2+...+fi i = 1, 2,...,

.
.
(,
), .
.
Sturges.

=1+3,32log=1 + 1,44 ln

) .
) .
) .

602

1
xi
i1

(x )

1
ti i
i1

().

o ,
50%
50%
.

pa, a = 1, 2,..., 99.

, a% pa (100a)%
.

Q1, Q2, Q3.

p25, p50, p75.

.
(0).
.

= Li +

0, 5 Fi 1
0, 5 Fi 1
c = Li +
c
Fi Fi 1
fi

.
D
c
D+E

M0 = Li +

D=fifi1 E=fifi+1

) .
) .
) .

(R).


.
s2

1
( x i  x )2

i1

1
( ti  x )2 i

i1

s2.

1
1
s = xi2 xi
i =1
v i =1

2

1
1 2
= ti vi ti vi
v i =1
v i =1

s = s2

Q xi yi  xi yi
i

i 1
2

2
Q xi  xi
i 1
i1

i 1

y
Q
i 1

1
 E x i 
Q

i 1

,

603


() .

y D  E x 
Q

R2

1

Hi2
Q

i 1

(y
i 1

 y)

1

(y
i 1

(y
i 1

 y i ) 2
 y)

i Hiyi yiy
y i y iD DExEi,x
y i 
,2,...,
1,2Q
,...,

ii = i11,2,...,
..Q .
i ,, H
i 

14.8 .
,
, .
1.

, .

2.

i
ti i, i=360o i i = 1, 2,..., .

3.

4.

' .

5.

, .

6.

,
.

7.

1 8 8 2 4 6 8 1 888 88
2 4 6 88.

8.

x1, x2, ..., x : s 2 = ( xi x )2.

9.

s2 ,
.

10.

,
.

11.

5 ,
5.

12.

4, 2.

y D  E x  13. ,
.

i =1

604

14.

, .

15.

Q1 Q3
16.
, .

17. 0 .

.
fi ti :
1.

2.

) fi = ^ i

) fi =

) fi =

) fi =

1
i

:
) f1+...+f=1

) f1+...+f=0

) 1fi1

) 1fi2.

:
3.

) . ) .
) . )

4.
5.
6.

8 8 2 4 6 8 :
) 0 = 8

) 0 = 2

) 0 = 4

8 8 2 4 6 8 :
) = 3
) = 7
) = 2

) 0 =6.
) = 4.

;
)

;
7.

) )

:
8.

9.

) T .

) T .

) T .

) T .

:
) T 0 1 .

) K .

) T 1.

) A .

605

10.

,
:
) H .
) H .

) T .
) H .

:
11.

12.

) M 1, 2,, 6.
) M 1, 2,, 6.
) M 1, 2,, 6.
) M 0, 1,, 6.

, :
) .
) .
) .
) .
:
) 0 1, 0 R2 1.

13.

) 1, R2 > 1.
) 1 0, 1 R2 0.
) , R2 0.

14.9 .
14.9.1. O
:
s
CV
,
x


. 10%,
. .
) 3, 4, 5, 6, 7, 8, 9.
) 103, 104, 105, 106, 107, 108, 109.
) 130, 140, 150, 160, 170, 180, 190.
14.9.2.  12 , ( ) , , :
: 20 18 20 17 18 17 16 17 16 10
: 18 16 17 15 16 12 16 17 20 22
) , .
)
 -

606

.
)

, ,
5 ,
10%. .
14.9.3. T
 o 80 11 kg,
26 kg. 8
14 kg, 30% 17 kg, 48
20 kg 15%
23 kg.
) ' .
)  20 kg, . 80
.
)  .
) .
) .
) .
14.9.4.

. : 5 (0 ), 25 1 , 15 2 , 5 3 .
) .
)  .
)  .
) .
) .
) .
14.9.5.
 14.9.1 80
. 4 .
)  ,
9.9.1
.
( kg)
) 
[ )
Fi
.
4555
0,2
) 
.
5565
0,5
14.9.6.  x1,x2,...,x
() , , gx -

6575
7585

607

- ,
g x x1 x2 " x .
(
) , , ( )
...
) 
.
) 1, 3, 5, 10, 15.
)  (),
2, 22, 23, 24.
14.9.7.  n , .
x1  s12 ,
x2  s22 ...
xn  sn2 n- .
) :
x

x1  x2  ...  xn
n

1 n
xi . 
ni 1

) s2 :
s12  s22  ...  sn2
n
) ;

1 n 2
sn . 
ni 1

s2

14.9.8.  , y = a + x.
)  = 5 y1, y2, y3, y4 y5 x1=2, x2=1, x3=0, x4=1 x5=2 , :
a

1
( y1  y 2  y 3  y 4  y 5 ) , E
5

1
(2 y1  y 2  y 4  2 y 5 ).
10

)  = 6 y1, y2, y3, y4 y5 x1=2, x2=2, x3=0, x4=0, x5=1 x6=1, , :


a

1
(9 y1  9 y 2  15 y 3  15 y 4  18 y 5  18 y 6 ) , E
84

1
(5 y1  5 y 2  y 3  y 4  4 y 5  4 y 6 ) .
28

 14.9.8 14.9.9.

xi

yi

3,5

7,2

10,4

12,6

15,9

608

14.6.10.
 yi, i = 1, 2, 3, 4, 5 :
y1 = a 3 + 1, y2 = a 5+ 2, y3 = a + 3, y4 = a + 5 + 4, y5 = a + 3 + 5.
:
1
1
a
( y1  y 2  y3  y 4  y5 ) , E
(3 y1  5 y 2  5 y 4  3 y5 ).
5
68
14.9.11.  ,
' . 20
( ), (
1), ( 2), ( 3)
( 4) . 14.9.2.
X1, X2, X3, X4 Y.
R2.
14.9.2

1
2
3
4
5
6
7
8
9
10
11
12
13
14
15
16
17
18
19
20

82
99
123
231
53
85
37
90
120
313
51
81
181
151
121
88
213
54
111
77

1
5
7,4
6,8
6
8,4
8
8,9
7,7
7,8
6,1
10
8,2
5,8
5,7
6,2
6,6
6,3
8,98
8,86
9,9

2
8
9
10
12
7
8,4
6,6
6,8
7
9,9
5,5
7,6
7,3
7,1
6
5
11
9
9,2
6,1

3
10
11
12
13
4
6
3
9
9,9
13,1
6,4
8,8
9,8
8,7
5,4
4,3
11,1
3,3
10,6
6,1

4
2
3
4
5
1
5
6,6
4,4
2,2
6
7,1
4,1
4
5,1
5,2
3,4
4
2,1
2,4
3



( /)

1.1
1.2
1.3
1.4
1.5
1.6
1.7

. . . . . . . . . . . . . . . . . . . . . . . . . . . . . . 12
. . . . . . . . . . . . . . . . . . . . . . . . . . . . . . . 19
. . . . . . . . . . . . . . . . . . . . . . . . . . . . . . . . . . . . . . . . . . . . . . . . . . . . . . . . . . . . . . . . 25
. . . . . . . . . . . . . . . . . . . . . . . . . . . . . . . . . . . . . . . . . . . . . . . . . . . . . . . . . . . . . . . . 34
. . . . . . . . . . . . . . . . . . . . . . . . . . . . . . . . . . . . . . . . . . . . . . . . . . . . . . . . . . . 38
. . . . . . . . . . . . . . . . . . . . . . . . . . . . . . . . . . . . . . . . . . . . . . . . . . . . . . . . . . . . . . 39
. . . . . . . . . . . . . . . . . . . . . . . . . . . . . . . . . . . . . . . . . . . . . . . . . . . . . . . . . . . . . . . . . . . . 42

2.1 . . . . . . . . . . . . . . . . . . . . . . . . . . . . . . . . . . . . . . . . . . . . . . . . . . . . . . . . . . . . . . . 44
2.2 . . . . . . . . . . . . . . . . . . . . . . . . . . . . . . . . . . . . . . . . . . . . . . . . . . . . . . . . 52
2.3 I . . . . . . . . . . . . . . . . . . . . . . . . . . . . . . . . . . . . . . . . . . . . . . . . . . . . . . . . . . . . . . . . . . 58
2.4 . . . . . . . . . . . . . . . . . . . . . . . . . . . . . . . . . 67
2.5 . . . . . . . . . . . . . . . . . . . . . . . . . . . . . . . . . . . . . . . . . . . . . . . . . . . . . . . 72
2.6 . . . . . . . . . . . . . . . . . . . . . . . . . . . . . . . . . . . . . . . . . . . . . . . . . . . . . . . . . . . . . . . . 81
2.7 . . . . . . . . . . . . . . . . . . . . . . . . . . . . 88
2.8 . . . . . . . . . . . . . . . . . . . . . . . . . . . . . . . . . . . . . . . . . . . . . . . 96
2.9 . . . . . . . . . . . . . . . . . . . . . . . . . . . . . . . . . . . . . . . . . . . . . . . . . . . . . . . . . . . 98
2.10 . . . . . . . . . . . . . . . . . . . . . . . . . . . . . . . . . . . . . . . . . . . . . . . . . . . . . . . . . . . . . . 99
2.11 . . . . . . . . . . . . . . . . . . . . . . . . . . . . . . . . . . . . . . . . . . . . . . . . . . . . . . . . . . . . . . . . . . . 101
T

3.1
3.2
3.3
3.4
3.5
3.6

. . . . . . . . . . . . . . . . . . . . . . . . . . . . . .106
, . . . . . . . . . . . . . . . . . . . . . . . . . . . . . . . . . . . . . . . . . . . . . . . . 115
. . . . . . . . . . . . . . . . . . . . . . . . . . . . . . . . . . . . . . . . . . . . . . . . . 128
. . . . . . . . . . . . . . . . . . . . . . . . . . . . . . . . . . . . . . . . . 134
' x0 R. . . . . . . . . . . . . . 145
+ . . . . . . . . . . . . . . . . . . . . . . . . . . . . . . 158

610
3.7 . . . . . . . . . . . . . . . . . . . . . . . . . . . . . . . . . . . . . . . . . . . . . . . . . . . . . . . . . . . . . . 176
3.8 Bolzano . . . . . . . . . . . . . . . . . . . . . . . . . . . . . . . . . . . . 183
3.9 . . . . . . . . . . . . . . . . . . . . . . . . . . . . . . . . . . . . . . . . . . . . . . . . . . . . . . . . . 190
3.10 . . . . . . . . . . . . . . . . . . . . . . . . . . . . . . . . . . . . . . . . . . . . . . . . . . . . . . . . . . . . 192
3.11 . . . . . . . . . . . . . . . . . . . . . . . . . . . . . . . . . . . . . . . . . . . . . . . . . . . . . . . . . . . . . . . . . . 196
TETA

4.1
4.2
4.3
4.4
4.5
4.6
4.7
4.8
4.9
4.10

. . . . . . . . . . . . . . . . . . . . . . . . . . . . . . . . . . . . . . . . . . . . . . . . . . . . . . . . . . . 200
. . . . . . . . . . . . . . . . . . . . . . . . . . . . . . . . . . . . . . . 205
. . . . . . . . . . . . . . . . . . . . . . . . . 216
. . . . . . . . . . . . . . . . . . . . . . . . . . . . . . . . . . . . 223
L Hospital. . . . . . . . . . . . . . . . . . . . . . . . . . . . . . . . . . . . . . . . . . . . . . . . . . . . . . . . . . . . 236
. . . . . . . . . . . . . . . . . . . . . . . . . . . . . . . . . . . . . . . . . . . . . . . . . . . . . . . . . 240
M . . . . . . . . . . . . . . . . . . . . . . . . . . . . . . . . . . . . . . . . . . . . . . . . . . . . . . . . . . . . . . . . 246
. . . . . . . . . . . . . . . . . . . . . . . . . . . . . . . . . . . . . . . . . . . . . . . . . . . . . . . . 253
. . . . . . . . . . . . . . . . . . . . . . . . . . . . . . . . . . . . . . . . . . . . . . . . . . . . . . . . . . . . 255
. . . . . . . . . . . . . . . . . . . . . . . . . . . . . . . . . . . . . . . . . . . . . . . . . . . . . . . . . . . . . . . . . . 259


( )



5.1
5.2
5.3
5.4
5.5
5.6
5.7
5.8
5.9
5.10

. . . . . . . . . . . . . . . . . . . . 266
. . . . . . . . . . . . . . . . . . . . . . . . . . . . . . . . . . . . . . . . . . . 269
. . . . . . . . . . . . . . . . . . . . . . . . . . 275
. . . . . . . . . . . . . . . . . . . . . . . . . . . . . . . . . . . . . . . . . . . . 281
. . . . . . . . . . . . . . . . . . . . . . . . . . 286
De Moivre. . . . . . . . . . . . . . . . . . . . . . . . . . . . . . . . . . . . . . . . . . . . . . . . . . . . . . . . . . . . . . . . . 290
. . . . 293
. . . . . . . . . . . . . . . . . . . . . . . . . . . . . . . . . . . . . . . . . . . . . . . . . . . . . . . . . 299
. . . . . . . . . . . . . . . . . . . . . . . . . . . . . . . . . . . . . . . . . . . . . . . . . . . . . . . . . . . . 301
. . . . . . . . . . . . . . . . . . . . . . . . . . . . . . . . . . . . . . . . . . . . . . . . . . . . . . . . . . . . . . . . . . 304

6.1
6.2
6.3
6.4
6.5
6.6
6.7

. . . . . . . . . . . . . . . . . . . . . . . . . . . . . . . . . 306
. . . . . . . . . . . . . . . . . . . . . . . . . . . . . . . . . . . . . . . 313
. . . . . . . . . . . . . . . . . . . . . . . . . . . . . . . . 317
. . . . . . . . 322
. . . . . . . . . . . . . . . . . . . . . . . . . . . . . . . . . . . . . . . . . . . . . . . . . . . . . . . . . 327
. . . . . . . . . . . . . . . . . . . . . . . . . . . . . . . . . . . . . . . . . . . . . . . . 331
. . . . . . . . . . . . . . . . . . . . . . . . . . . . . . . . . . . . . . . . . . . . . . . . . . . . . . . . . 336

611
6.8 . . . . . . . . . . . . . . . . . . . . . . . . . . . . . . . . . . . . . . . . . . . . . . . . . . . . . . . . . . . . 338
6.9 . . . . . . . . . . . . . . . . . . . . . . . . . . . . . . . . . . . . . . . . . . . . . . . . . . . . . . . . . . . . . . . . . . 341


7.1 . . . . . . . . . . . . . . . . . . . . . . . . . . . . . . . . . . . . . . . . . . . . . . . . . . . 344
7.2  . . . . . 350
7.3 . . . . . . . . . . . . . . . . . . . . . . . . . . . . . . . . . . . . . . . . . . . . . . . . . . . . . . 354
7.4 . Bernoulli. . . . . . . . . . . . . . . . . . . . 357
7.5 . . . . . . . . . . . . . . . . . . . . . . . . . . . . . . . 361
7.6 . . . . . . . . . . . . . . . . . . . . . . . . . . . . . . . . . . . . . . . . . . . . . . . . . . . . . . . . . . . . . . . . . . . . . . . . . 369

7.6.1 . . . . . . . . . . . . . . . . . . . . . . . . . . . . . . . . . . . . . . . . . . . . . . . . . . . . . . . . . 369

7.6.2 . . . . . . . . . . . . . . . . . . . . . . . . . . . . . . . . . . . . . . . . . . . . . . . . . . . . . . . . . . . . . 370

7.6.3 . . . . . . . . . . . . . . . . . . . . . . . . . . . . . . . . . . . . . . . . . . . . . . . . . . . . . . . . . . . . 371
7.7 . . . . . . . . . . . . . . . . . . . . . . . . . . . . . . . . . . . . . . . . . . . . . . . . . . . . . . . . . 373
7.8 . . . . . . . . . . . . . . . . . . . . . . . . . . . . . . . . . . . . . . . . . . . . . . . . . . . . . . . . . . . . . 374
7.9 . . . . . . . . . . . . . . . . . . . . . . . . . . . . . . . . . . . . . . . . . . . . . . . . . . . . . . . . . . . . . . . . . . . 378

LAPLACE
8.1 Laplace. . . . . . . . . . . . . . . . . . . . . . . . . . . . . . . . . . . . . . . . . . . . . . . . . . . . . . . . . . 382
8.2 Laplace. . . . . . . . . . . . . . . . . . . . . . . . . . . . . . . . . . . . . . . . . . . . . . . 386
8.3  Laplace. . . . . . . . . . . . . . . . . . . . . . 390
8.4 . . . . . . . . . . . . . . . . . . . . . . . . . . . . . . . . . . . . . . . . . . . . . . . . . . . . . . . . . 393
8.5 . . . . . . . . . . . . . . . . . . . . . . . . . . . . . . . . . . . . . . . . . . . . . . . . . . . . . . . . . . . . . 394
8.6 . . . . . . . . . . . . . . . . . . . . . . . . . . . . . . . . . . . . . . . . . . . . . . . . . . . . . . . . . . . . . . . . . . . 395


( )

9.1
9.2
9.3
9.4
9.5
9.6
9.7
9.8

. . . . . . . . . . . . . . . . . . . . . . . . . . . . . . . . . . . . . . . . . . . . . . . . . . . . . . . . . . . . . . . . . . . . . . . . . 400
. . . . . . . . . . . . . . . . . . . . . . . . . . . . . . . . . . . . . . . . . . . . . . . . . . . . . . . . . 402
. . . . . . . . . . . . . . . . . . . . . . . . . . . . . . . . . . . . . . . . . . . . . . . . . . . . . . . . . . . 404
. . . . . . . . . . . . . . . . . . . . . . . . 405
. . . . . . . . . . . . . . . . . . . . . . . . . . . . . . . . . . . . . . . . . . . . . . . . . . . . . . . . . . . . . . . . 406
. . . . . . . . . . . . . . . . . . . . . . . . . . . . . . . . . . . . . . . . . . . . . . . . . . . . . . . . . . . . . 408
. . . . . . . . . . . . . . . . . . . . . . . . . . . . . . . . . . . . . . . . . . . . . . . . . . . . . . . . . 409
. . . . . . . . . . . . . . . . . . . . . . . . . . . . . . . . . . . . . . . . . . . . . . . . . . . . . . . . . . . . . . . . . . 410

10.1 . . . . . . . . . . . . . . . . . . . . . . . . . . . . . . . . . . . . . . . . . . . . . . . . . . . . . . . . . . . . 412

612
10.2 . . . . . . . . . . . . . . . . . . . . . . . . . . . . . . . . . . . . 414

10.2.1 . . . . . . . . . . . . . . . . . . . . . . . . . . . . . . . . . . . . . . . . . . . . . . . . . . . . 414

10.2.2 30, 45 60. . . . . . . . . . . . . . . . . . . . . . . . . . . . . . . . . . . . . . 415

10.2.3 . . . . . . . . . . . . . . . . . . . . . . . . . . . . . . . . . . . . . . . . . . . . . . . . . . . . 417
10.3 . . . . . . . . . . . . . . . . . . . . . . . . . . . . . . . . . . . . . . . . . . . . . . . . . . . . . . . . . 418

10.3.1 . . . . . . . . . . . . . . . . . . . . . . . . . . . . . . . . . . . . . . . . . . . . . . . . . . . . . . . . . . . . . . 418

10.3.2 . . . . . . . . . . . . . . . . . . . . . . . . . . . . . . . . . . . . . . . . . . . . 418

10.3.3 . . . . . . . . . . . . . . . . . . . . . . . . . . . . . . . . . . . . . . . . . . . . . . . . . . . . . . . . . . . . . 418

10.3.4 . . . . . . . . . . . . . . . . . . . . . . . . . . . . . . . . . . . . . . . . . . . . . . . . . . . . . . . . . . . 421

10.3.5 . . . . . . . . . . . . . . . . . . . . . . . . . 423
10.4 . . . . . . . . . . . . . . . . . . . . . . . . . . . . . . . . . . . . . . . . . . . . . . 423

10.4.1 . . . . . . . . . . . . . . . . . . . . . . . . . . . . . . . . . . . . . . . . . . . . . . . . . . . . . . . . . . . . . . . . . . . 423

10.4.2 . . . . . . . . . . . . . . . . . . . . . . . . . . . . . . . . . . . . . . . . . . . . . . . . . . . . . . . . . . . . 424
10.5 . . . . . . . . . . . . . . . . . . . . . . . . . . . . . . . . . . . . . . . . . . . . . . . . . . . . . . . . . 426
10.6 . . . . . . . . . . . . . . . . . . . . . . . . . . . . . . . . . . . . . . . . . . . . . . . . . . . . . . . . . . . . . . . . . . . 429


11.1 . . . . . . . . . . . . . . . . . . . . . . . . . . . . . . . . . . . . . . . . . . . . . . . . . . . . . . . . . . . 434

11.1.1 . . . . . . . . . . . . . . . . . . . . . . . . . . . . . . . . . . . . . . . . . . . . . . . . . . . . . . . . . . 434

11.1.2 . . . . . . . . . . . . . . . . . . . . . . . . . . . . . . . . . . . . . . . . . . . . . . . . . . . . . . . . 435

11.1.3 . . . . . . . . . . . . . . . . . . . . . . . . . . . . . . . . . . . . . . . . . . . . . . . . . . . . . . . 435

11.1.4 . . . . . . . . . . . . . . . . . . . . . . . . . . . . . . . . . . . . . . . . . . . . 436

11.1.5 . . . . . . . . . . . . . . . . . . . . . . . . . . . . . . . . . . . . . . . . . . . . . . 437

11.1.6 . . . . . . . . . . . . . . . . . . . . 439

11.1.7 . . . . . . . . . . . . . . . . . . . . . . . . . . . . . . . . . . 439

11.1.8 . . . . . . . . . . . . . . . . . . . . . . . . . . . . . . . . . . . . . . . . . . . . . . . . . 440

11.1.9 . . . . . . . . . . . . . . . . . . . . . . . . . . . . . . . . . . . . . . . . . . 441

11.1.10 . . . . . . . . . . . . . . . . . . . . . . . . . . . . . . . . . . . . . . . . . . . . . . . . . . . . . . . . 442
11.2 . . . . . . . . . . . . . . . . . . . . . . . . . . . . . . . . . . . . . . . . . . . . . . . . . . . . . 445
11.3 . . . . . . . . . . . . . . . . . . . . . . . . . . . . . . . . . . . . . . . . . . . . . . . . . . . . . . . . 447

11.3.1 . . . . . . . . . . . . . . . . . . . . . . . . . . . . . . 447

11.3.2 . . . . . . . . . . . . . . . . . . . . . . . . . . . . . . . . . . . . . . . . 448
11.4 . . . . . . . . . . . . . . . . . . . . . . . . . . . . . . . . . . . . . . . . . . . . . . . . . . . . . . . . . 449
11.5 . . . . . . . . . . . . . . . . . . . . . . . . . . . . . . . . . . . . . . . . . . . . . . . . . . . . . . . . . . . . . . . . . . . 452


12.1 . . . . . . . . . . . . . . . . . . . . . . . . . . . . . . . . . . . . . . . . . . . . . . . . . . . . . . . . . . . . . . 456
12.2 . . . . . . . . . . . . . . . . . . . . . . . . . . . . . . . . . . . . . . . . . . . . . . . . . . . . . . . 456
12.3 . . . . . . . . . . . . . . . . . . . . . . . . . . . . . . . . . . . . . . . . . . . . . . . . . . . . . 458

12.3.1 . . . . . . . . . . . . . . . . . . . . . . . . . . . . . . . . . . . . . . . . . . . . . 458
12.4 (). . . . . . . . . . . . . . . . . . . . . . . . . . . . . . . . . . . . . . . . . . . . . 460
12.5 . . . . . . . . . . . . . . . . . . . . . . . . . . . . . . . . . . . . . . . . . . . . . . . . . . . 461
12.6 . . . . . . . . . . . . . . . . . . . . . . . . . . . . . . . . . . . . . . . . . . . . . . . . . . . . . . . . . . . 461
12.7 . . . . . . . . . . . . . . . . . . . . . . . . . . . . . . . . . . . . . . . . . . . . . . . . . . . . . . . . . . . . . . 465

613

12.7.1 . . . . . . . . . . . . . . . . . . . . . . . . . . . . . . . . . . . . . . . 465
12.8 . . . . . . . . . . . . . . . . . . . . . . . . . . . . . . . . . . . . . . . . . . . . . . . . . . . . . . . . . . . . . . . . . . . . . 470
12.9 . . . . . . . . . . . . . . . . . . . . . . . . . . . . . . . . . . . . . . . . . . . . . . . . . . . . . . . . . . . . . . . . . . . . . . . . . . . 471

12.9.1 . . . . . . . . . . . . . . . . . . . . . . . . . . . . . . . . . . . . . . . . . . . . . . . . . . . . . 471

12.9.2 . . . . . . . . . . . . . . . . . . . . . . . . . . . . . . . . . . . . . . . . . . . . . . . . . . . . . . . . 471

12.9.3 . . . . . . . . . . . . . . . . . . . . . . . . . . . . . . . . . . . . . . . . . . . . . . . . 472

12.9.4 . . . . . . . . . . . . . . . . . . . . . . . . . . . . . . . . . . . . . . . 472

12.9.5 . . . . . . . . . . . . . . . . . . . . . . . . . . . . . . . . . . . . . . . . . . . . . . . . . . 473
12.10 . . . . . . . . . . . . . . . . . . . . . . . . . . . . . . . . . . . . . . . . . . . . . . . . . . . . . . . . . . . . . . . . . . . . . . . . . . 475

12.10.1 . . . . . . . . . . . . . . . . . . . . . . . . . . . . . . . . . . . . . . . . . . . . . . . . . 475

12.10.2 . . . . . . . . . . . . . . . . . . . . . . . . . . . . . . . . . . . . . . . . . . . . . . . . 476

12.10.3 . . . . . . . . . . . . . . . . . . . . . . . . . . . . . . . . . . . . . . . . . . . . . . . . . . . . . 477

12.10.4 . . . . . . . . . . . . . . . . . . . . . . . . . . . . . . . . . . . . . . . . . . . 477

12.10.5 . . . . . . . . . . . . . . . . . . . . . . . . . . . . . . . . . . . . . . . . . . . . 478
12.11 . . . . . . . . . . . . . . . . . . . . . . . . . . . . . . . . . . . . . . . . . . . . . . . . . . . . . . . . . . . . . . . . . . . . . . . . . 478

12.11.1 . . . . . . . . . . . . . . . . . . . . . . . . . . . . . . . . . . . . . . . . . . . . . . . . . 479

12.11.2 . . . . . . . . . . . . . . . . . . . . . . . . . . . . . . . . . . . . . . . . . . . . . . . . 479

12.11.3 . . . . . . . . . . . . . . . . . . . . . . . . . . . . . . . . . . . . . . . . . . . . . . . . . . . . 480

12.11.4 . . . . . . . . . . . . . . . . . . . . . . . . . . . . . . . . . . . . . . . . . . . . . . . . . . . . . 480

12.11.5 . . . . . . . . . . . . . . . . . . . . . . . . . . . . . . . . . . . . . . . . . . . 481

12.11.6 . . . . . . . . . . . . . . . . . . . . . . . . . . . . . . . . . . . . . . . . . . . 481
12.12 . . . . . . . . . . . . . . . . . . . . . . . . . . . . . . . . . . . . . . . . . . . . . . . . . . . . . . . . . . . . . . . . . . . . . . . . . 481

12.12.1 . . . . . . . . . . . . . . . . . . . . . . . . . . . . . . . . . . . . . . . . . . . . . . . . 482

12.12.2 . . . . . . . . . . . . . . . . . . . . . . . . . . . . . . . . . . . . . . . . . . . . . . . . 483

12.12.3 . . . . . . . . . . . . . . . . . . . . . . . . . . . . . . . . . . . . . . . . . . . 483

12.12.4 . . . . . . . . . . . . . . . . . . . . . . . . . . . . . . . . . . . . . . . . . . . 483
12.13 . . . . . . . . . . . . . . . . . . . . . . . . . . . . . . . . . . . . . . . . . . . . . . . . . . . . 484
12.14 . . . . . . . . . . . . . . . . . . . . . . . . . . . . . . . . . . . . . . . . . . . . . . . . . . . . . . . . . 489
12.15 . . . . . . . . . . . . . . . . . . . . . . . . . . . . . . . . . . . . . . . . . . . . . . . . . . . . . . . . . . . . . . . . . . . 496


13.1 . . . . . . . . . . . . . . . . . . . . . . . . . . . . . . . . . . . . . . . . . . . . . . . . . . . . . . . . . . . . . . . . . . . . . . 502
13.2 . . . . . . . . . . . . . . . . . . . . . . . . . . . . . . . . . . . . . . . . . . . . . 511
13.3 . . . . . . . . . . . . . . . . . . . . . . . . . . . . . . . . . . . 516
13.4 Napier. . . . . . . . . . . . . . . . . . . . . . . . . . . . . . . . . . . . . . . . . . . . . . . . . . . . . . . . . . . . . . . . . 517
13.5 . . . . . . . . . . . . . . . . . . . . . . . . . . . . . . . . . . . . . . . . . . . . . . . . . . . . . . . 518
13.6 . . . . . . . . . . . . . . . . . . . . . . . . . . . . . . . . . . . . . . . . . . . . . 519

13.6.1 . . . . . . . . . . . . . . . . . . . . . . . 519

13.6.2 . . . . . . . . . . . . . . . . . . . . . . . . . . . . 520
13.7 . . . . . . . . . . . . . . . . . . . . . . . . . . . . . . . . . . . . . . . . . . . . . . . . . . . . . . . 529
13.8 . . . . . . . . . . . . . . . . . . . . . . . . . . . . . . . . . . . . . . . . . . . . . . . . . . . . . . . . . 531
13.9 . . . . . . . . . . . . . . . . . . . . . . . . . . . . . . . . . . . . . 537

13.9.1 . . . . . . . . . . . . . . . . . . . . . . . . . . . . . . . . . . . . . . . . . . . . . . . . . . . . . . . . . . 537

13.9.2 . . . . . . . . . . . . . . . . . . . . . . . . . . . . . . . . . . . . . . . . . . . . . . . . . . . . . . . . . . 537

13.9.3 . . . . . . . . . . . . . . . . . . . . . . . . . . . . . . . . . . . . . . . . . . . . . . . . . . . . . . 543

614

13.9.4 . . . . . . . . . . . . . . . . . . . . . . . . . . . . . . . . . . . . . . . . . . 544
13.10 . . . . . . . . . . . . . . . . . . . . . . . . . . . . . . . . . . . . . . . . . . . . . . . . . . . . . . . . . 545
13.11 . . . . . . . . . . . . . . . . . . . . . . . . . . . . . . . . . . . . . . . . . . . . . . . . . . . . . . . . . . . . . . . . . . . 546


( /)


TATI
14.1 . . . . . . . . . . . . . . . . . . . . . . . . . . . . . . . . . . . . . . . . . . . . . . . . . . . . 552
14.2 . . . . . . . . . . . . . 556
14.3 . . . . . . . . . . . . . . . . . . . . . . . . . . . . . . . . . . . . . . . . . . . . . . . . . . . . . . . . 568
14.4 . . . . . . . . . . . . . . . . . . . . . . . . . . . . . . . . . . . . . . . . . . . . . . . . . . . . . . . . . . . . . . . . . . . . . 577

14.4.1 ( x ). . . . . . . . . . . . . . . . . . . . . . . . . . . . . . . . . . . . . . . . . . . . . . . 577

14.4.2 () . . . . . . . . . . . . . . . . . . . . . . . . . . . . . . . . . . . . . . . . . . . . . . . . 580

14.4.3 (0). . . . . . . . . . . . . . . . . . . . . . . . . . . . . . . . . . . . . . . . . . . . . . . . 583
14.5 . . . . . . . . . . . . . . . . . . . . . . . . . . . . . . . . . . . . . . . . . . . . . . . . . . . . . . . . . . . . . . . 587
14.6 . . . . . . . . . . . . . . . . . . . . . . . . . . . . . . . . . . . . . . . . . . . . . . . . . . . . . . . . . . . . . 593
14.7 . . . . . . . . . . . . . . . . . . . . . . . . . . . . . . . . . . . . . . . . . . . . . . . . . . . . . . . . . 601
14.8 . . . . . . . . . . . . . . . . . . . . . . . . . . . . . . . . . . . . . . . . . . . . . . . . . . . . . . . . . . . . . 603
14.9 . . . . . . . . . . . . . . . . . . . . . . . . . . . . . . . . . . . . . . . . . . . . . . . . . . . . . . . . . . . . . . . . . . 605
. . . . . . . . . . . . . . . . . . . . . . . . . . . . . . . . . . . . . . . . . . . . . . . . . . . . . . . . . . . . . . . . . . . . . . . 609

Anda mungkin juga menyukai